SCHWARTZ’S PRINCIPLESOFSURGERY ABSITEand Board Review
NOTICE
Medicineisan ever-changingscience.Asnewresearch and clinicalexperiencebroaden our knowledge, changes in treatment and drug therapy are required. Te authors and the publisher o this work have checked with sources believed to be reliable in their e orts to provide in ormation that is complete and generally in accord with the standards accepted at the time o publication. However, in view o the possibility o human error or changesin medicalsciences,neither theauthorsnor thepublisher nor any other party who has been involved in the preparation or publication o this work warrantsthatthein ormation containedherein isin everyrespectaccurateorcomplete, andtheydisclaimallresponsibility oranyerrorsoromissionsor ortheresultsobtained rom useo thein ormation containedin thiswork.Readersareencouragedtocon rm the in ormation contained herein with other sources. For example and in particular, readers are advised to check the product in ormation sheet included in the package o each drugtheyplan to administer to be certain that the in ormation contained in this workisaccurateand that changeshavenot been madein therecommended doseor in the contraindications or administration. Tis recommendation is o particular importancein connection with newor in requentlyused drugs.
PRINCIPLESOFSURGERY ABSITEand Board Review
10th Edition
Edited by
F.Charles Brunicardi,MD,FACS
MossFoundation Chairin Gastrointestinaland Personalized Surgery
Pro essorandVice ChairSurgicalServices
Chie o GeneralSurgery,UCLASanta Monica MedicalCenter
Department o Surgery
David Gefen Schoolo Medicine at UCLA
LosAngeles,Cali ornia
Associate Editors
DanaK.Andersen,MD,FACS
Program Director
Division o Digestive Diseasesand Nutrition
NationalInstitute o Diabetesand Digestive and KidneyDiseases
NationalInstituteso Health
Bethesda,Maryland
TimothyR.Billiar,MD,FACS
GeorgeVance FosterPro essorand Chairman
Department o Surgery
Universityo Pittsburgh Schoolo Medicine
Pittsburgh,Pennsylvania
David L.Dunn,MD,PhD,FACS
ExecutiveVice President orHealth Afairs
Pro essoro Surgery,Microbiologyand Immunology
Universityo Louisville
Louisville,Kentucky
JohnG.Hunter,MD,FACS
Mackenzie Pro essorand Chair
Department o Surgery
Oregon Health &Science University
Portland,Oregon
JefreyB.Matthews,MD,FACS
Surgeon-in-Chie and Chairman
Department o Surgery
DallasB.PhemisterPro essoro Surgery
Chicago,Illinois
RaphaelE.Pollock,MD,PhD,FACS
Pro essorand Director
Division o SurgicalOncologyChie o SurgicalServices
The JamesComprehensive CancerCenter
The Ohio State UniversityWexnerMedicalCenter
Columbus,Ohio
James X.Wu,MD GeneralSurgeryResident
Department o Surgery
UCLADavid Gefen Schoolo Medicine
LosAngeles,Cali ornia
SCHWARTZ’S
NewYork Chicago San Francisco Athens London Madrid MexicoCity Milan NewDelhi Singapore Sydney oronto
Schwartz’sPrinciplesofSurgery:ABSITEandBoardReview, 10thEdition
Copyright ©2016byMcGraw-HillEducation.Allrightsreserved.Printed in theUnited Stateso America.Except aspermitted under theUnited StatesCopyright Act o 1976,no part o thispublication maybereproduced or distributed in any orm or byanymeans,or stored in a data baseor retrievalsystem,without theprior written permission o the publisher.
Previouseditionscopyright 2011,2007byTeMcGraw-HillCompanies,Inc.
1 2 3 4 5 6 7 8 9 0 ROV/ROV 20 19 18 17 16
ISBN 978-0-07-183891-7
MHID 0-07-183891-0
Tisbookwasset in Minion Pro byCenveo®Publisher Services. TeeditorswereBrian Belvaland ChristieNaglieri Teproduction supervisor wasCatherineSaggese Project management wasprovided by anyaPunj,Cenveo Publisher Services. RRDonnelleywastheprinter and binder.
Tisbookisprinted on acid- reepaper.
LibraryofCongressCataloging-in-Publication Data
Schwartz’sprincipleso surgery.ABSI Eand board review/ edited by F.CharlesBrunicardi;associateeditors,Dana K.Andersen, imothyR.Billiar, David L.Dunn,John G.Hunter,RaphaelE.Pollock,Je reyB.Matthews.— enth edition. p.;cm.
Principleso surgery.ABSI Eand board review
Should beused in conjunction with thetenth edition o Schwartz’sprinciples o surgery.
Includesindex.
ISBN 978-0-07-183891-7(pbk.,alk.paper)—ISBN 0-07-183891-0
I. Brunicardi,F.Charles,editor. II. Schwartz’sprincipleso surgery. enth edition. Complemented by(expression): III. itle:Principleso surgery.ABSI Eand board review
[DNLM: 1. SurgicalProcedures,Operative—Examination Questions. WO 18.2]
RD31
617.0076—dc23
2015020914
InternationalEdition. ISBN 978-1-259-25124-5; MHID 1-259-25124-1. Copyright 2016 byMcGraw-HillEducation. ExclusiverightsbyMcGraw-HillEducation or manu acture and export.Tisbookcannot bere-exported rom thecountryto which it isconsigned by McGraw-HillEducation.TeInternationalEdition isnot availablein North America.
McGraw-HillEducation booksareavailableat specialquantitydiscountsto useas premiumsand salespromotionsor or usein corporatetrainingprograms. o contact a representative,pleasevisit theContact Uspagesat www.mhpro essional.com
v Contributors vii Introduction ix Acknowledgments xi PARTI BasicConsiderations 1 1 Leadership 3 2 SystemicResponse to Injuryand MetabolicSupport 5 3 Fluid and Electrolyte Management o the SurgicalPatient 9 4 Hemostasis,SurgicalBleeding,and Trans usion 17 5 Shock 25 6 SurgicalIn ections 33 7 Trauma 41 8 Burns 55 9 Wound Healing 61 10 Oncology 69 11 Transplantation 73 12 Patient Sa ety 81 13 PhysiologicMonitoring o the SurgicalPatient 87 14 MinimallyInvasive Surgery 93 15 Molecularand GenomicSurgery 97
SpecifcConsiderations 105 16 The Skin and SubcutaneousTissue 107 17 Breast 115 18 Disorderso the Head and Neck 121 19 Chest Wall,Pleura,and Mediastinum 125 20 CongenitalHeart Disease 135 21 Acquired Heart Disease 143 22 ThoracicAneurysmsand AorticDissection 151 23 ArterialDisease 159 24 Venousand LymphaticDisease 179 25 Esophagusand DiaphragmaticHernia 189 26 Stomach 203 27 The SurgicalManagement o Obesity 215 28 SmallIntestine 219 29 Colon,Rectum,and Anus 225 30 The Appendix 237 31 Liver 241 32 The Gallbladderand Extrahepatic BiliarySystem 253 33 Pancreas 261 34 Spleen 275 35 AbdominalWall,Omentum,Mesentery, and Retroperitoneum 283 CONTENTS
PARTII
vi 36 So tTissue Sarcomas 287 37 InguinalHernias 289 38 Thyroid,Parathyroid,and Adrenal 299 39 PediatricSurgery 305 40 Urology 315 41 Gynecology 319 42 Neurosurgery 323 43 OrthopedicSurgery 331 44 Surgeryo the Hand and Wrist 335 45 Plasticand Reconstructive Surgery 341 46 Anesthesia orthe SurgicalPatient 343 47 SurgicalConsiderationsin the Elderly 347 48 Ethics,Palliative Care,and Care at the End o Li e 351 49 GlobalSurgery 353 Index 357
DanaK.Andersen,MD,FACS
Program Director
Division o DigestiveDiseasesand Nutrition
NationalInstituteo Diabetesand Digestiveand KidneyDiseases
NationalInstituteso Health
Bethesda,Maryland
TimothyR.Billiar,MD,FACS
GeorgeVanceFoster Pro essor and Chairman
Department o Surgery
Universityo Pittsburgh Schoolo Medicine
Pittsburgh,Pennsylvania
F.Charles Brunicardi,MD,FACS
MossFoundation Chair in Gastrointestinal and Personalized Surgery
Pro essor and ViceChair SurgicalServices
Chie o GeneralSurgery,UCLASantaMonica MedicalCenter
Department o Surgery
David Ge en Schoolo Medicineat UCLA
LosAngeles,Cali ornia
MaryCondron,MD Resident
Department o Surgery
Oregon Health &ScienceUniversity
Portland,Oregon
ChristopherConnelly,MD Resident
Department o Surgery
Oregon Health &ScienceUniversity
Portland,Oregon
Mackenzie Cook,MD
GeneralSurgeryResident
Department o Surgery
Oregon Health &ScienceUniversity
Portland,Oregon
DavidL.Dunn,MD,PhD,FACS
ExecutiveVicePresident or Health A airs
Pro essor o Surgery,Microbiologyand Immunology Universityo Louisville
Louisville,Kentucky
John G.Hunter,MD,FACS
MackenziePro essor and Chair
Department o Surgery
Oregon Health &ScienceUniversity
Portland,Oregon
ScottLouis,MD
SurgeryResident
Oregon Health &ScienceUniversity
Portland,Oregon
JefreyB.Matthews,MD,FACS
Surgeon-in-Chie and Chairman
Department o Surgery
DallasB.Phemister Pro essor o Surgery
Chicago,Illinois
Fernando Mier,MD
Advanced GIand BariatricSurgeryFellow
Department o Generaland GastrointestinalSurgery
Oregon Health and ScienceUniversity
Portland,Oregon
Alexis Moren,MD GeneralSurgeryResident
Oregon Health &ScienceUniversity
Portland,Oregon
RaphaelE.Pollock,MD,PhD,FACS
Pro essor and Director
Division o SurgicalOncologyChie o SurgicalServices
TeJamesComprehensiveCancer Center
TeOhioStateUniversityWexner MedicalCenter
Columbus,Ohio
TanaLynnRepella,MD,PhD
GeneralSurgeryResident
Department o Surgery
Oregon Health &ScienceUniversity
Portland,Oregon
JuliaC.Swanson,MD
CongenitalCardiacSurgeryFellow
Department o Surgery
exasChildren’sHospital
Houston, exas
vii
CONTRIBUTORS
EricaSwenson,DO
GeneralSurgeryResident
Department o Surgery
Oregon Health &ScienceUniversity
Portland,Oregon
PatrickR.Varley,MD Resident
Department o GeneralSurgery
Universityo Pittsburgh MedicalCenter
Pittsburgh,Pennsylvania
James X.Wu,MD
GeneralSurgeryResident
Department o Surgery
UCLADavid Ge en Schoolo Medicine
LosAngeles,Cali ornia
EstinYang,MD,MPH Resident Physician
Department o Surgery
Oregon Health &ScienceUniversity
Portland,Oregon
viii
Tis 10th edition o Schwartz’s Principles of Surgery: ABSITE and Board Reviewmarks a milestone o excellence in surgical education or the betterment o cra , quality o care, and the edi cation o surgical students and colleagues alike. With 842 questions spanning the 49 updated chapterso thisedition,includingtwo newchapters, Fundamental Principles o Leadership raining in Surgery and Global Surgery, this is the comprehensive companion text or reviewingand assessingthein ormation compiled in themain book and or preparation or the American Board o SurgeryIn- raining Examination (ABSI E).
Contributorso theprimarybookhaveupdated thequestions or each chapter sincethelast edition in an e ort to continueto provide
a high level o review on the most up-to-date in ormation and techniques currently taught and employed in the operating theater. We have maintained the proven ormat o providing the answerbearing portion o the text immediately ollowing the question and answer asan efcient method or rein orcement and recall.Teuser mayread thequestion ollowed bytheanswer asa orm o review,or by covering the right-hand column o the page, the user can complete the questionsin a more authentic test ormat and uncover theanswers or review/scoring.
ix INTRODUCTION
ACKNOWLEDGMENTS
o Brian Belval, Christie Naglieri, and all at McGraw-Hill, we are thank ul or the continued belie in and support o this book. We
wish to thank Katie Elsbury or her dedication to the organization and editingo thisbook.
F.CharlesBrunicardi,MD,FACS
xi
PARTI
BasicConsiderations
1. Te undamentalprincipleso leadership are
A. Vision and willingness
B. Command and control
C. imemanagement and mentoring
D. Coaching,pacesetting,and democratic
2. E ective communication is a key component o leadership, given that miscommunication is a leading cause o medicalerrors.Which o the ollowingstatementsisFALSE regardingcommunication?
A. To Err Is Human, a publication by the US Institute o Medicine,identi ed medicalerrors asthe eighth leadingcauseo death in theUnitedStates,causing100,000 deathsannually.
B. E ectivecommunication thatensuresallteammembers understanddailygoalso care oranICUpatientcansigni cantlydecreasepatient’slengtho stayin theICU.
C. Communication errors are o en simply due to negligenceand ailuretotransmit in ormation.
D. In ormation trans er and communication errorscause delays in patient care and can cause serious adverse events.
E. Improved communication in OR among the cardiac surgery patients is associated with decreased adverse outcomes.
3. DanielGoleman o theHarvardBusinessReviewdescribed six key leadership styles. Which o the ollowing statementsisFALSEregardingleadership styles?
A. Te coercive style o leadership isantiquated and isno longer e ectivein surgery.
B. Democraticleadership isuse ul or buildingteam consensus and minimizing confict, but may rustrate team membersi thereisnoclear,uni yingvision.
C. Te pacesetter leads by example and sets high standards or his team,but typicallytakes over the tasks o something allingbehind instead o buildingthem up.
D. Teauthoritativeleader iso en themost e ective,and ocusesondirectingtheteamtowardacommonvision, allowing team members to give room or innovation and experimentation,and supportingtheir e orts.
Answer:A
Vision and willingness are the two undamental principles o leadership.Command and controlisa colloquialterm or the previously predominant style o leadership in surgery based on ear and intimidation. ime management and mentorship are key leadership skills. Coaching, pacesetting, and democraticareleadership styles.(SeeSchwartz10th ed.,p.3.)
Answer:C
Communication errors are o ten caused by miscommunication due to hierarchical di erences, concerns with upward in luence, con licting roles and role ambiguity, and interpersonalcon lict.(SeeSchwartz10th ed.,p.6.)
Answer:A hough it isnolonger appropriateasthepredominant leadership style, during times o duress when a clear, single leader is needed, a coercive leadership style may be prudent. his is most appropriate in emergency trauma settings. Excessive coerciveleadershipcan erodeteam members’senseo responsibility, motivation, sense o participation in a shared vision, and ultimately,per ormance.(SeeSchwartz10th ed.,p.9.)
3
1
Leadership
CHAPTER
4. Which is FALSE regarding modern confict resolution techniques?
A. Based upon objectivityand willingnesstolisten.
B. Should seek a solution that bene ts all involved and which isbased upon corevalueso theorganization.
C. raditional command-and-control technique based on ear and intimidation can lead to sanctions and lawsuits.
D. Confict resolution ismore success ulwhen both sides can admit theysharesome ault.
Answer:D
Modern con lict resolution techniques are based upon objectivity, willingness to listen, and pursuit o principle-based solutions.For example,an e ectivestyleo con lict resolution is the utilization o the “abundance mentality” model, which attemptsto achieve a solution that bene itsallinvolved and is based upon corevalueso theorganization,asopposed to the utilization o the traditional ault- indingmodel,which identi ies sides as right or wrong. Application o the abundance mentality in surgery elevates the con lict above the a ected parties and ocuses on the higher uni ying goal o improved patient care. Morbidity and mortality (M&M) con erences are managed in this style and have the purpose o practice improvement and improvingoverallqualityo carewithin the system, as opposed to placing guilt or blame on the surgeon or surgical trainees or the complication being reviewed. he traditional style o command-and-control technique based on ear and intimidation is no longer welcome in any health care system and can lead to sanctions, lawsuits, and removal o hospital privileges or position o leadership. (See Schwartz 10th ed.,p.7.)
4
SystemicResponse to Injury and MetabolicSupport
1. C-reactiveprotein (CRP)
A. Is secreted in a circadian rhythm with higher levels in themorning
B. Increasesa er eatingalargemeal
C. Doesnot increasein responseto stressin patientswith liver ailure
D. Is less sensitive than erythrocyte sedimentation rate as amarker o in ammation
Answer:C
heacutephaseproteinsarenonspeci icbiochemicalmarkers produced by hepatocytes in response to tissue injury, in ection, or in lammation. Interleukin (IL)-6 is a potent inducer o acute phase proteins that can include proteinase inhibitors, coagulation and complement proteins, and transport proteins. Clinically, only C-reactive protein (CRP) has been consistently used as a marker o injury response due to its dynamic re lection o in lammation. Importantly, CRP levels donotshowdiurnalvariationsandarenota ectedby eeding. Only pre-existing liver ailure will impair CRP production. here ore, it has become a use ul biomarker o in lammation as well as response to treatment. Its accuracy surpasses that o theerythrocytesedimentation rate.(SeeSchwartz10th ed., p.17.)
Answer:C
2. Which o the ollowingistrueregardingthein ammatory response ollowingtraumaticinjury?
A. Tereisan acuteproin ammatoryresponsecaused by stimulation o theadaptiveimmunesystem.
B. Tere is an anti-in ammatoryresponse that leads to a return tohomeostasisaccompanied suppression o the innateimmunesystem.
C. Te degree o in ammation is proportional to injury severity.
D. Systemic in ammation ollowing trauma is related to theimmuneresponsetomicrobes.
he degree o the systemic in lammatory response ollowing trauma is proportional to injury severity and is an independent predictor o subsequent organ dys unction and resultant mortality. Recent work has provided insight into the mechanismsbywhich immuneactivation in thissettingistriggered. heclinical eatureso theinjury-mediated systemicin lammatory response, characterized by increased body temperature, heart rate, respirations, and white blood cell count, are similar to those observed with in ection. While signi icant e orts have been devoted to establishing a microbial etiology or this response,it isnowwidelyaccepted that systemicin lammation ollowingtraumaissterile.(SeeSchwartz10th ed.,p.14.)
5
2
CHAPTER
3. High-mobilitygroup protein B1(HMGB1)
A. Is associated with the best-characterized damageassociated molecular pattern (DAMP), detectable in thecirculation within 30minuteso trauma
B. Isaprotein secreted byimmune-competent cellsstimulated by pathogen-associated molecular patterns (PAMPs)or in ammatorycytokines
C. Is also secreted by endothelial cells, platelets, and also asapart o celldeath
D. Allo theabove
4. Temost abundant aminoacid in thehuman bodyis
A. Carnitine
B. Arginine
C. Glutamine
D. Methionine
5. What is the role o mitochondrial DAMPs in the injurymediated in ammatoryresponse?
A. MitochondrialDNAinducesproduction o HMGB1.
B. MitochondrialDNAandpeptides rom damaged mitochondriaactivatethemacrophagein ammasome.
C. Mitochondrial DNA and peptides modulate the antiin ammatory response that suppresses the adaptive immunesystem.
D. Mitochondrial DNA is directly toxic to the liver and lungin high amounts.
Answer:D
he best-characterized DAMP in the context o the injuryassociated in lammatory response is high-mobility group protein B1 (HMGB1), which is rapidly released into the circulation within 30 minutes ollowing trauma. Subsequent studieshaveproven,however,thatHMGB1isactivelysecreted rom immune-competent cells stimulated by PAMPs (eg, endotoxin) or by in lammatory cytokines (eg, tumor necrosis actor and interleukin-1). Stressed nonimmune cells, such asendothelialcells,and platelet also activelysecreteHMGB1. Finally, passive release o HMGB1 can occur ollowing cell death, whether it is programmed or uncontrolled (necrosis). he diverse proin lammatorybiological responses that result rom HMGB1 signaling include: (1) the release o cytokines and chemokines rom macrophage/monocytes and dendritic cells;(2) neutrophilactivation and chemotaxis;(3) alterations in epithelial barrier unction, including increased permeability; and (4) increased procoagulant activity on platelet suraces;amongothers.(SeeSchwartz10th ed.,p.15.)
Answer:C
Glutamine is the most abundant amino acid in the human body, comprising nearly two-thirds o the ree intracellular aminoacid pool.(SeeSchwartz10th ed.,p.53.)
Answer:B
Mitochondrial proteins and/or DNA can act as DAMPs by triggering an in lammatory response to necrosis and cellular stress. Speci ically, the release o mitochondrial DNA (mtDNA) and ormyl peptides rom damaged or dys unctional mitochondria has been implicated in activation o the macrophage in lammasome, a cytosolic signaling complex that responds to cellular stress. With stress or tissue injury, mtDNA and peptides are released rom damaged mitochondria where they can contribute to a sterile in lammatory response. From an evolutionary perspective, given that eukaryotic mitochondria derive rom bacterial origin, it would make sense that they retain bacterial eatures capable o elicitingastrongresponsethat istypicallyassociated with a pathogen trigger.(SeeSchwartz10th ed.,p.16.)
6. Which is FALSE regarding the hypothalamic-pituitaryadrenal(HPA)axisand injury-associated stress?
A. Te HPA is initiated by the hypothalamus producing corticotropin-releasing hormone (CRH) in response toin ammatorycytokines.
B. CRH actson theanteriorpituitarytostimulateadrenocorticotropin hormone(AC H)secretion.
C. CRH simulates the zona asciculata o the adrenal gland tosynthesizeand secreteglucocorticoids.
D. Insufcient cortisol in response to critical illness can lead to tachycardia, hypotension, weakness, hypoglycemia,hyponatremia,and hyperkalemia.
Answer:C
CRH acts on the anterior pituitary to stimulate the secretion o adrenocorticotropin hormone (AC H) into the systemic circulation. AC H acts on the zona asciculata o the adrenal glands to synthesize and secrete glucocorticoids. Cortisol is the major glucocorticoid in humans and is essential or survival during signi icant physiologic stress. he resulting increasein cortisollevels ollowingtraumahasseveralimportant anti-in lammatory actions. Adrenal insu iciency represents a clinical syndrome highlighted largely by inadequate amounts o circulating cortisol and aldosterone. Classically, adrenal insu iciency is described in patients with atrophic adrenal glands caused by exogenous steroid administration who undergo a stressor such as surgery. hese patients subsequently mani est signs and symptoms such as tachycardia, hypotension, weakness, nausea, vomiting, and ever. (SeeSchwartz10th ed.,p.20.)
6
7. Nutritional ormulasused totreat pulmonary ailuretypically increase the at intake o a patient’s total caloric intaketo
A. 50%
B. 20%
C. 80%
D. 10%
Answer:A
In pulmonary ailure ormulas, atcontentisusuallyincreased to 50% o the total calories, with a corresponding reduction in carbohydratecontent. hegoalisto reducecarbon dioxide production and alleviate ventilation burden or ailing lungs. (SeeSchwartz10th ed.,p.54.)
7
Fluid and Electrolyte Management ofthe SurgicalPatient
1. Metab li a id sis with a n rmal ani n gap (AG) urs with
A. Diabeti a id sis
B. Renal ailure
C. Severediarrhea
D. Starvati n
2. All are p ssible auses p st perative hyp natremia EXCEP
A. Ex essin usi n n rmalsalineintra peratively
B. Administrati n antipsy h ti medi ati n.
C. ransient de rease in antidiureti h rm ne (ADH) se reti n.
D. Ex ess ralwater intake.
3. Whi h the ll wingisan earlysign hyperkalemia?
A. Peaked waves
B. Peaked Pwaves
C. Peaked (sh rtened)QRS mplex
D. Peaked Uwaves
Answer:C
Metab li a id siswith an rmalani n gap (AG)results r m either a id administrati n (HCl r NH4 +) r a l ss bi arb nate r m gastr intestinal (GI) l sses, su h as diarrhea, istulas (enteri , pan reati , r biliary), ureter sigm id st my, r r m renall ss. hebi arb natel ssisa mpanied by a gain hl ride, thus the AG remains un hanged. (See S hwartz10th ed.,p.74.)
Answer:C
Hyp natremia is aused by ex ess ree water (diluti n) r de reased s dium (depleti n). hus, ex essive intake ree water ( ral r IV) an lead t hyp natremia. Als , medi ati ns an ause water retenti n and subsequent hyp natremia, espe ially in lder patients. Primary renal disease, diureti use,andse reti n antidiureti h rm ne(ADH)are mm n auses s dium depleti n. ADH an be released transientlyp st peratively, r less requently, in syndr me inappr priate ADH se reti n. Lastly, pseud hyp natremia an be seen n lab rat ry testing when high serum glu se, lipid, r pr tein levels mpr mise s dium measurements. (SeeS hwartz10th ed.,p.69.)
Answer:A
Sympt ms hyperkalemia are primarilyGI,neur mus ular, and ardi vas ular. GI sympt ms in lude nausea, v miting, intestinal li ,anddiarrhea;neur mus ular sympt msrange r m weakness t as ending paralysis t respirat ry ailure; while ardi vas ular mani estati ns range r m ele tr ardi gram (ECG) hanges t ardia arrhythmias and arrest. ECG hangesthat maybeseen with hyperkalemiain lude
Peaked waves(early hange)
Flattened Pwave
Pr l nged PRinterval( irst-degreebl k)
Widened QRS mplex
Sinewave rmati n
Ventri ular ibrillati n
(SeeS hwartz10th ed.,p.71.)
9
3
CHAPTER
4. Hyp al emiamay ausewhi h the ll wing?
A. C ngestiveheart ailure
B. Atrial brillati n
C. Pan reatitis
D. Hyp parathyr idism
5. Te next m st appr priate test t rder in a patient with a pH 7.1,Pco2 40,s dium 132,p tassium 4.2,and hl ride 105is
A. Serum bi arb nate
B. Serum magnesium
C. Serum ethan l
D. Serum sali ylate
6. Whi h the ll wing is FALSE regarding hypert ni saline?
A. Isan arteri lar vas dilat r and mayin reasebleeding
B. Sh uld beav ided in l sed head injury
C. Sh uld n t beused r initialresus itati n
D. In reases erebralper usi n
7. N rmalsalineis
A. 135mEqNaCl/L
B. 145mEqNaCl/L
C. 148mEqNaCl/L
D. 154mEqNaCl/L
8. Fluid resus itati n usingalbumin
A. Isass iated with agul pathy
B. Isavailableas1% r 5%s luti ns
C. Can lead t pulm naryedema
D. De reased a t r XIII
9. Water nstituteswhat per entage t talb dyweight?
A. 30–40%
B. 40–50%
C. 50–60%
D. 60–70%
Answer:A Mild hyp al emia an present with mus le ramping r digital/peri ral paresthesias. Severe hyp al emia leads t de reased ardia ntra tilityandheart ailure.ECG hanges hyp al emia in lude pr l nged Q interval, -wave inversi n, heart bl k, and ventri ular ibrillati n. Hypparathyr idism and severepan reatitisarep tential auses hyp al emia.(SeeS hwartz10th ed.,p.72.)
Answer:A
Metab li a id sis results r m an in reased intake a ids, an in reased generati n a ids, r an in reased l ss bi arb nate. In evaluating a patient with a l w serum bi arb nate leveland metab li a id sis, irst measuretheAG,an index unmeasured ani ns.
AG=[Na]– [Cl+HCO3]
Metab li a id sis with an in reased AG urs r m either ex gen usa id ingesti n (ethylenegly l,sali ylate, r methan l) r end gen usa id pr du ti n β-hydr xybutyrateand a et a etatein ket a id sis,la tateinla ti a id sis, r rgani a idsin renalinsu i ien y.(SeeS hwartz10thed.,p.74.)
Answer:B
Hypert ni saline(7.5%)hasbeen used asatreatment m dalityin patients with l sed head injuries. It has been sh wn t in rease erebralper usi nandde reaseintra ranialpressure, thus de reasing brain edema. H wever, there als have been n erns in reasedbleedingbe ausehypert ni salineisan arteri lar vas dilat r.(SeeS hwartz10th ed.,p.76.)
Answer:D
S dium hl ride is mildly hypert ni , ntaining 154 mEq s dium that is balan ed by 154 mEq hl ride. he high hl ride n entrati n imp ses a signi i ant hl ride l ad up n thekidneysandmayleadt ahyper hl remi metab li a id sis.Itisan ideals luti n,h wever, r rre tingv lume de i its ass iated with hyp natremia, hyp hl remia, and metab li alkal sis.(SeeS hwartz10th ed.,p.76.)
Answer:C
Albumin is available as 5% ( sm lality 300 mOsm/L) r 25% ( sm lality 1500 mOsm/L). Due t in reased intravas ular n ti pressure, luid is drawn int the intravas ular spa e,leadingt pulm naryedema when albumin isused r resus itati n r hyp v lemi sh k.Hydr xyethyl star h s luti ns are ass iated with p st perative bleeding in ardia and neur surgerypatients.(SeeS hwartz10th ed.,p.77.)
Answer:C
Water nstitutes appr ximately 50 t 60% t tal b dy weight. he relati nship between t tal b dy weight and t tal b dy water ( BW) is relatively nstant r an individual and is primarilya re le ti n b dy at. Lean tissues, su h as mus le and s lid rgans, have higher water ntent than at and b ne. As a result, y ung, lean men have a higher pr p rti n b dyweight aswater than elderly r beseindividuals. An average y ung adult male will have 60% his t tal b dy
10
10. I apatient’sserum glu sein reasesby180mg/dL,what is the in rease in serum sm lality, assuming all ther lab rat ryvaluesremain nstant?
A D esn t hange
B. 8
C 10
D 12
11. What is the a tual p tassium a patient with pH 7.8 and serum p tassium 2.2?
A 2 2
B. 2.8
C 3 2
D 3 4
12. Te ree water de it a 70kgman with serum s dium 154is
A 0 1L
B. 0.7L
C 1L
D 7L
13. Apatientwithserum al ium 6.8andalbumin 1.2has a rre ted al ium
A 7 7
B 8 0
C. 8.6
D 9 2
14. All the ll wing treatments r hyperkalemia redu e serum p tassium EXCEP
A Bi arb nate
B. Kayexalate
C. Glu sein usi n with insulin
D Cal ium
weight as BW, while an average y ung adult emale’s will be 50%. hel wer per entage BW in w men rrelateswith a higher per entage adip se tissue and l wer per entage mus le mass in m st. Estimates BW sh uld be adjusted d wn appr ximately10t 20%in beseindividualsand up by 10%in maln urished individuals. he highest per entage BW is und in newb rns, with appr ximately 80% their t talb dyweight mp sed water. hisde reasest ab ut 65% by 1 year and therea ter remains airly nstant. (See S hwartz10th ed.,p.65.)
Answer:C
Osm ti pressure is measured in units sm les ( sm) r milli sm les (mOsm) that re er t the a tual number sm ti allya tiveparti les.F r example,1millim le(mm l) s dium hl ride ntributest 2mOsm ( ne r m s dium and ne r m hl ride). heprin ipaldeterminants smlality are the n entrati ns s dium, glu se, and urea (bl d ureanitr gen [BUN]):
Cal ulatedserum sm lality=2s dium+glu se/18+BUN/2.8
(SeeS hwartz10th ed.,p.67.)
Answer:D
he hangein p tassium ass iated with alkal sis an be alulated bythe ll wing rmula:
P tassium de reasesby0.3mEq/L r every0.1in rease in pH ab ven rmal
(SeeS hwartz10th ed.,p.71.)
Answer:D
his is the rmula used t estimate the am unt water required t rre t hypernatremia
= × Water deficit L serum sodium 140 140 TBW
Estimate BW (t talb dywater) as50% lean b dymassin men and 40%in w men.(SeeS hwartz10th ed.,p.69.)
Answer:D
When measuringt talserum al ium levels,thealbumin nentrati n must betaken int nsiderati n. Adjust t tal serum al ium d wn by 0.8 mg/dL r every 1g/dLde reasein albumin.(SeeS hwartz10th ed.,p.72.)
Answer:D
When ECG hanges are present, al ium hl ride r al ium glu nate (5–10 mL 10% s luti n) sh uld be administered immediately t untera t the my ardial e e ts hyperkalemia. Cal ium in usi n sh uld be used auti usly in patients re eivingdigitalis,be ausedigitalist xi itymaybepre ipitated. Glu seandbi arb nateshi tp tassiumintra ellularly.Kayexalateisa ati n ex hangeresin that bindsp tassium,either given enterally rasanenema.(SeeS hwarz10thed.,p.77.)
11
15. An al h li patient with serum albumin 3.9, K 3.1, Mg 2.4, Ca 7.8, and PO4 3.2 re eives three
b luses IVp tassium and hasserum p tassium 3.3.
Y u sh uld
A C ntinue t b lus p tassium until the serum level is >3.6.
B GiveMgSO4 IV.
C Che kthei nized al ium
D. Che ktheBUN and reatinine.
16. Cal ulatethedailymaintenan e uidsneeded r a60-kg emale
A 2060
B 2100
C. 2160
D 2400
17. Apatient wh hasspasmsin thehand when abl d pressure u isbl wn up m st likelyhas
A Hyper al emia
B Hyp al emia
C. Hypermagnesemia
D Hyp magnesemia
18. Te a tual AG a hr ni al h li with Na 133, K 4, Cl–101,HCO3 – 22,albumin 2.5mg/dLis
A 6
B 10
C. 14
D 15
ntr
Answer:B
Magnesium depleti n is a mm n pr blem in h spitalized patients, parti ularly in the ICU. he kidney is primarily resp nsible r magnesium h me stasis thr ugh regulati n by al ium/magnesium re ept rs n renal tubular ells that senseserummagnesium levels.Hyp magnesemiaresults r m a variety eti l gies ranging r m p r intake (starvati n, al h lism, pr l nged use IV luids, and t tal parenteral nutriti n with inadequate supplementati n magnesium), in reasedrenalex reti n (al h l,m stdiureti s,andamphteri in B), GI l sses (diarrhea), malabs rpti n, a ute panreatitis, diabeti ket a id sis, and primary ald ster nism. Hyp magnesemiaisimp rtantn t nly r itsdire te e ts n the nerv us system but als be ause it an pr du e hyp alemiaand lead t persistent hyp kalemia.When hyp kalemia r hyp al emia exist with hyp magnesemia, magnesium sh uldbeaggressivelyrepla edt assistin rest ringp tassium r al ium h me stasis.(SeeS hwartz10th ed.,p.73.)
Answer:B
A60-kg emalew uld re eiveat tal 2100mL luid daily: 1000mL r the irst10kg b dyweight(10kg×100mL/kg/ day), 500 mL r the next 20 kg (10 kg × 50 mL/kg/day), and 80mL r thelast40kg(40kg×20mL/kg/day).(SeeS hwartz 10th ed.,p.78.)
Answer:B
Asympt mati hyp al emia may ur with hyp pr teinemia (n rmal i nized al ium), but sympt ms an devel p withalkal sis(de reasedi nized al ium).In general,sympt ms d n t ur until the i nized ra ti n alls bel w 2.5mg/dL,andareneur mus ularand ardia in rigin,in luding paresthesias the a e and extremities, mus le ramps, arp pedal spasm, strid r, tetany, and seizures. Patients will dem nstrate hyperre lexia and p sitive Chv stek sign (spasm resulting r m tapping ver the a ial nerve) and r usseau sign (spasm resulting r m pressure applied t the nerves and vessels theupper extremity,aswhen btaininga bl d pressure).De reased ardia ntra tilityand heart ailure an als a mpanyhyp al emia.(SeeS hwartz10thed.,p.72.)
Answer:D
he n rmal AG is <12 mm l/L and is due primarily t the albumin e e t,s that theestimated AG must beadjusted r albumin (hyp albuminemiaredu estheAG).
C rre ted AG= a tualAG+[2.5(4.5–albumin)]
(SeeS hwartz10th ed.,p.74.)
Answer:D
A Bi arb nate
B Chl ridei n
C. P tassium i n
D Pr tein
he diss lved pr tein in plasma d es n t pass thr ugh the semipermeable ell membrane, and this a t is resp nsible r the e e tive r ll id sm ti pressure. (See S hwartz 10th ed.,p.66.)
12
19. Te e e tive sm ti pressure between the plasma and interstitial uid mpartmentsisprimarily
lled by
20. Te metab li derangement m st mm nly seen in patientswith pr usev miting
A. Hyp hl remi ,hyp kalemi metab li alkal sis
B. Hyp hl remi ,hyp kalemi metab li a id sis
C. Hyp hl remi ,hyperkalemi metab li alkal sis
D. Hyp hl remi ,hyperkalemi metab li a id sis
Answer:B
Hyp hl remi , hyp kalemi metab li alkal sis an ur r m is lated l ss gastri ntents in in ants with pyl ri sten sis r in adultswith du denalul er disease.Unlikev mitingass iated with an pen pyl rus,whi h inv lvesal ss gastri as well as pan reati , biliary, and intestinal se reti ns, v miting with an bstru ted pyl rus results nly in the l ss gastri luid, whi h is high in hl ride and hydr gen, and there re results in a hyp hl remi alkal sis. Initially the urinarybi arb natelevelishighin mpensati n r thealkal sis.Hydr gen i n reabs rpti n als ensues,with an a mpanied p tassium i n ex reti n.In resp nset theass iated v lume de i it, ald ster ne-mediated s dium reabs rpti n in reases p tassium ex reti n. he resulting hyp kalemia leads t the ex reti n hydr gen i ns in the a e alkalsis, a parad xi a iduria. reatment in ludes repla ement the v lume de i it with is t ni saline and then p tassium repla ement n e adequate urine utput is a hieved. (See S hwartz10th ed.,p.74.)
21. Sympt ms and signs extra ellular uid v lume de it in ludeall the ll wingEXCEP
A An rexia
B Apathy
C. De reased b dytemperature
D High pulsepressure
22. A l w urinary [NH4 +] with a hyper hl remi a id sis indi ateswhat ause?
A. Ex essivev miting
B Enter utane us stula
C. Chr ni diarrhea
D. Renaltubular a id sis
23. When la ti a id is pr du ed in resp nse t injury, the b dyminimizespH hangeby
A De reasing pr du ti n s dium bi arb nate in tissues
B Ex reting arb n di xidethr ugh thelungs
C. Ex retingla ti a id thr ugh thekidneys
D. Metab lizingthela ti a id in theliver
24. What is the best determinant whether a patient has a metab li a id sisversusalkal sis?
A. ArterialpH
B. Serum bi arb nate
C Pco2
D. Serum CO2 level
Answer:D
High pulse pressure urs with extra ellular luid v lume ex ess,but the ther sympt msand signsare hara teristi m derateextra ellular v lumede i it.(SeeS hwartz10th ed., p.68.)
Answer:D
Metab li a id sis with a n rmal AG results either r m ex gen us a id administrati n (HCl r NH4 +), r m l ss bi arb nate due t GI dis rders su h as diarrhea and istulas r ureter sigm id st my, r r m renal l sses. In these settings, the bi arb nate l ss is a mpanied by a gain hlride; thus, the AG remains un hanged. determine i the l ss bi arb nate has a renal ause, the urinary [NH4 +] an bemeasured.Al wurinary[NH4 +]in the a e hyper hl remi a id sis w uld indi ate that the kidneyis the site l ss, and evaluati n r renal tubular a id sis sh uld be undertaken. Pr ximal renal tubular a id sis results r m de reased tubular reabs rpti n HCO3 –, whereas distal renal tubular a id sis results r m de reased a id ex reti n. he arb ni anhydrase inhibit r a etaz lamide als auses bi arb nate l ss r m thekidneys.(SeeS hwartz10th ed.,p.74.)
Answer:B
La ti a id rea ts with base bi arb nate t pr du e arb ni a id. he arb ni a id is br ken d wn int water and arb n di xide that is ex reted by the lungs. Any diminuti n in pulm nary un ti n je pardizes this rea ti n. (See S hwartz 10th ed.,p.73.)
Answer:A
While bi arb nate, Pco2, and patient hist ry ten an suggest the m st likely metab li derangement, nly the measurement arterial pH n irms a id sis versus alkal sis. (SeeS hwartz10th ed.,p.74.)
13
25. I a patient’s arterial Pco2 is und t be 25 mm Hg, the arterialpH willbeappr ximately
A. 7.52
B 7 40
C 7 32
D. 7.28
26. Whi h the ll wing are NO hara teristi ndings a uterenal ailure?
A BUN >100mg/dL
B. Hyp kalemia
C Severea id sis
D Uremi peri arditis
E. Uremi en ephal pathy
27. An elderlydiabeti patient wh has a ute h le ystitis is und t have a serum s dium level 122 mEq/Land a bl d glu se 600mg/dL.Afer rre tingtheglu se n entrati n t 100 mg/dL with insulin, the serum s dium n entrati n w uld
A De reasesigni antlyunlessthepatient als re eived
3%saline
B De rease transiently but return t appr ximately 122mEq/Lwith ut spe i therapy
C. Remain essentiallyun hanged
D. In reaset then rmalrangewith ut spe i therapy
28. Ex essiveadministrati n n rmalsaline r uid resusitati n an lead t what metab li derangement?
A. Metab li alkal sis
B. Metab li a id sis
C Respirat ryalkal sis
D Respirat rya id sis
29. Te rst step in the management a ute hyper al emia sh uld be
A. C rre ti n de it extra ellular uid v lume
B Hem dialysis
C Administrati n ur semide
D. Administrati n mithramy in.
Answer:D
Al wPaco2 indi atesex esseliminati n arb n di xideby thelungs,and theb dypH will all.Within reas nablephysil gi rangesa 15mm Hg allin Paco2 sh uld pr du ea 0.12 hange r m the n rmal b dy pH 7.4. (See S hwartz 10th ed.,p.74.)
Answer:A Hyperkalemia, severe a id sis, uremi en ephal pathy, and uremi peri arditisareallindi ati ns li e-threateningpr blems, and urgent rre ti n is mandat ry. Elevati n BUN is mm nly seen as well, but is n t itsel an indi ati n r dialysis.(SeeS hwartz10th ed.,p.81.)
Answer:D
A rise in the extra ellular luid n entrati n a substan e that d esn t di usepassivelya r ss ellmembranes(eg,gluse r urea) ausesan in reasein e e tive sm ti pressure, a trans er water r m ells, and diluti nal hyp natremia. F r ea h 100 mg/dLrise in bl d glu se ab ve n rmal, the serum s dium level alls appr ximately t 3 mEq/L. Alternatively, the serum s dium level w uld in rease by ab ut 15mEq/Li thebl d glu selevel ell r m 600t 100mg/dL. (SeeS hwartz10th ed.,p.69.)
Answer:B
S dium hl ride is mildlyhypert ni , ntaining154 mEq s diumthatisbalan edby154mEq hl ride. hehigh hlride n entrati n imp ses a signi i ant hl ride l ad n the kidneysand maylead t a hyper hl remi metab li a id sis. S dium hl ride is an ideal s luti n, h wever, r rre ting v lumede i itsass iatedwithhyp natremia,hyp hl remia, andmetab li alkal sis.(SeeS hwartz10th ed.,p.74.)
Answer:A
Patients with a ute hyper al emia usually have either a ute hyperparathyr idism r metastati breast ar in ma with multipleb nymetastases. hesepatientsdevel p severeheada hes, b ne pain, thirst, emesis, and p lyuria. Unless treatment is instituted pr mptly, the sympt ms may be rapidly atal. Immediate rre ti n the ass iated de i it extraellular luid v lume is the m st imp rtant step in treatment. When e e tive, this results in the l wering the serum al ium level bydiluti n. On e extra ellular luid v lume has been repla ed, ur semideise e tivetreatment.Hem dialysis may als be empl yed, but its e e t is less rapid. Mithramyin isveryuse ulin ntr llingmetastati b nedisease,but its e e tissl w,andit ann tbedependedup nwhen thepatient hasa utehyper al emia.(SeeS hwartz10th ed.,p.72.)
30. A vi tim a m t r vehi le a ident arrives in hem rrhagi sh k. His arterial bl d gases are pH, 7.25; Po2, 95 mm Hg; Pco2, 25 mm Hg; HCO3 –, 15 mEq/L. Te patient’smetab li a id sisw uld betreated best with
A Ampule s dium bi arb nate
B. S dium bi arb natein usi n
C. La tated Ringer s luti n
D Hyperventilati n
Answer:C
In patients su ering r m hem rrhagi sh k, the presen e a metab li a id sis earlyin the p stresus itative peri d is indi ative tissue hyp xia due t persistent inadequate tissue per usi n. Attempts t rre t this pr blem by administering an alkalizing agent will n t s lve the basi pr blem.
14
31. Treedaysafer surgery r gastri ar in ma,a50-yearldal h li maleexhibitsdelirium,mus letrem rs,and hypera tive tend n re exes. Magnesium de ien y is suspe ted. All the ll wing statements regarding this situati n aretrueEXCEP
A. Ade isi n t administer magnesium sh uld bebased n theserum magnesium level.
B. Adequate ellular repla ement magnesium will require1t 3weeks.
C. A n mitant al iumde ien ysh uldbesuspe ted.
D Cal ium is a spe i antag nist the my ardial e e ts magnesium.
32. Re eeding syndr me an be ass iated with all the ll wingEXCEP
A. Respirat ry ailure
B Hyperkalemia
C C n usi n
D. Cardia arrhythmias
H wever,pr per v lumerepla ement bymeans abalan ed salt s luti n su h as la tated Ringer s luti n will rest re perusi n and rre t themetab li a id sisbyendinganaer bi metab lism.(SeeS hwartz10th ed.,p.79.)
Answer:A Magnesiumde i ien ysh uldbesuspe tedinanymaln urished patient wh exhibits disturbed neur mus ular r erebral a tivity in the p st perative peri d. Lab rat ry n irmati n ten is n t reliable, and the syndr me may exist in the presen e a n rmal serum magnesium level. Hyp al emia ten exists, parti ularlyin patients wh have lini al signs tetany. Intraven us magnesium an be administered sa ely t a well-hydrated patient r initial treatment a severe de i it, but n mitant ele tr ardi graphi m nit ring is essential. he ele tr ardi graphi hanges ass iated with a ute hypermagnesemia resemble th se hyperkalemia, and al ium hl ride r glu nate sh uld be readilyavailable t untera t any adverse my ardial e e ts ex ess magnesium i ns. Partial r mplete relie sympt ms may ll w the initial in usi n magnesium, alth ugh ntinued repla ement r aperi d 1t 3weeksisne essaryt replenish ellular st res.(SeeS hwartz10th ed.,p.78.)
Answer:B
Withre eeding,ashi tin metab lism r m att arb hydrate substratestimulatesinsulin release,whi h resultsin the ellular uptake ele tr lytes,parti ularlyph sphate,magnesium, p tassium,and al ium.H wever,severe hypergly emia may result r m blunted basal insulin se reti n. (See S hwartz 10th ed.,p.81.)
15
Hemostasis,SurgicalBleeding,and Transfusion
1. Which o the ollowingisNO oneo the our major physiologiceventso hemostasis?
A. Fibrinolysis
B. Vasodilatation
C. Platelet plug ormation
D. Fibrin production
2. Which is required or platelet adherence to injured endothelium?
A. TromboxaneA2
B. Glycoprotein (GP)IIb/IIIa
C. Adenosinediphosphate(ADP)
D. Von Willebrand actor (vWF)
Answer:B
Hemostasis is a complex process and its unction is to limit blood loss rom an injured vessel. Four major physiologic events participate in the hemostatic process: vascular constriction, platelet plug ormation, ibrin ormation, and ibrinolysis. hough each tend to be activated in order, the our processes are interrelated so that there is a continuum and multiplerein orcements.(SeeSchwartz10th ed.,p.85.)
Answer:D
Platelets do not normally adhere to each other or to the vessel wall but can orm a plug that aids in cessation o bleeding when vascular disruption occurs. Injury to the intimal layer in thevascular wallexposessubendothelialcollagen to which platelets adhere. his process requires von Willebrand actor (vWF), a protein in the subendothelium that is lacking in patientswith von Willebrand disease.vWFbindstoglycoprotein (GP) I/IX/V on the platelet membrane. Following adhesion, platelets initiate a release reaction that recruits other platelets rom the circulating blood to seal the disrupted vessel. Up to this point, this process is known as primaryhemostasis. Platelet aggregation is reversible and is not associated with secretion. Additionally, heparin does not inter ere with this reaction and thus hemostasis can occur in the heparinized patient.Adenosinediphosphate(ADP)and serotonin are theprincipalmediatorsin platelet aggregation.(SeeSchwartz 10th ed.,p.85.)
3. Which o the ollowing clotting actors is the rst actor common toboth intrinsicand extrinsicpathways?
A. Factor I( brinogen)
B. Factor IX(Christmas actor)
C. Factor X(Stuart-Prower actor)
D. Factor XI(plasmathromboplasmaantecedent)
Answer:C
he intrinsic pathway begins with the activation o actor XII that subsequently activates actors XI, IX, and VII. In this pathway, each o the primary actors is “intrinsic” to the circulatingplasma,wherebyno sur aceisrequired to initiatethe process.In theextrinsicpathway,tissue actor ( F) isreleased or exposed on the sur ace o the endothelium, bindingto circulating actor VII, acilitating its activation to VIIa. Each o these pathways continues on to a common sequence that beginswith theactivation o actor XtoXa(in thepresenceo VIIIa). Subsequently, Xa (with the help o actor Va) converts actor II (prothrombin) to thrombin and then actor I ( ibrinogen) to ibrin. Clot ormation occurs a ter ibrin monomers arecross-linked topolymerswith theassistanceo actor XIII. (SeeSchwartz10th ed.,p.87.)
17
4
CHAPTER
4. Which congenital actor de ciency is associated with delayed bleedinga er initialhemostasis?
A. Factor VII
B. Factor IX
C. Factor XI
D. Factor XIII
5. In a previously unexposed patient, when does the platelet count allin heparin-induced thrombocytopenia(HI )?
A. <24hours
B. 24–28hours
C. 3–4days
D. 5–7days
6. Which isNO an acquired platelet hemostaticde ect?
A. Massiveblood trans usion ollowingtrauma
B. Acuterenal ailure
C. Disseminated intravascular coagulation (DIC)
D. Polycythemiavera
7. What istrueabout coagulopathyrelated totrauma?
A. Acutecoagulopathyo trauma ismechanisticallysimilar toDIC.
B. Coagulopathy can develop in trauma patients ollowingacidosis,hypothermia,and dilution o coagulation actors,though coagulation isnormalupon admission.
C. Acute coagulopathy o trauma is caused by shock and tissueinjury.
D. Acute coagulopathy o trauma is mainly a dilutional coagulopathy.
Answer:D
Congenital actor XIII (FXIII) de iciency, originally recognized by Duckert in 1960, is a rare autosomal recessive disease usually associated with a severe bleeding diathesis. he male-to- emale ratio is 1:1. Although acquired FXIII de iciencyhas been described in association with hepatic ailure, in lammatory bowel disease, and myeloid leukemia, the only signi icant association with bleeding in children is the inherited de iciency. Bleeding is typically delayed because clots orm normally but are susceptible to ibrinolysis. Umbilical stump bleeding is characteristic, and there is a high risk o intracranial bleeding. Spontaneous abortion is usual in women with FXIIIde iciencyunlesstheyreceivereplacement therapy. Replacement can be accomplished with resh rozen plasma(FFP), cryoprecipitate, or a FXIII concentrate. Levels o 1 to 2%are usuallyadequate or hemostasis.(See Schwartz 10th ed.,p.89.)
Answer:D
Heparin-induced thrombocytopenia(HI ) isa orm o druginduced immune thrombocytopenia (I P). It is an immunological event in which antibodies against platelet actor-4 (PF4) ormed during exposure to heparin, a ecting platelet activation and endothelial unction with resultant thrombocytopenia and intravascular thrombosis. he platelet count typically begins to all 5 to 7 days a ter heparin has been started, but i it is a re-exposure, the decrease in count may occur within 1to2days.(SeeSchwartz10th ed.,p.90.)
Answer:C
Impaired platelet unction o ten accompanies thrombocytopenia but mayalso occur in the presence o a normal platelet count. he importance o this is obvious when one considers that 80%o overallstrength isrelated toplatelet unction. he li e span o platelets ranges rom 7 to 10 days, placing them at increased risk or impairment by medical disorders, prescription, and over-the-counter medications. Impairment o ADP-stimulated aggregation occurswith massivetrans usion o blood products. Uremia may be associated with increased bleeding time and impaired aggregation. De ective aggregation and platelet dys unction is also seen in patients with thrombocythemia, polycythemia vera, and myelo ibrosis. DIC is an acquired syndrome characterized by systemic activation o coagulation pathwaysthat result in excessivethrombin generation and the di use ormation o microthrombi. (SeeSchwartz10th ed.,p.92.)
Answer:C
raditional teaching regarding trauma-related coagulopathy attributeditsdevelopment toacidosis,hypothermia,and dilution o coagulation actors.Recent data,however,haveshown that over one-third o injured patientshasevidence o coagulopathy at the time o admission. More importantly, patients arriving with coagulopathy are at a signi icantly higher risk o mortality,especiallyin the irst 24hoursa ter injury.Acute Coagulopathy o trauma is not a simple dilutional coagulopathy but a complex problem with multiple mechanisms Whereasmultiplecontributing actorsexist,thekeyinitiators
18
8. What is the best laboratory test or determine degree o anticoagulation with dabigatran and rivaroxaban?
A. Prothrombin time/international normalized ratio (P /INR)
B. partialthromboplastin time(P )
C. Bleedingtime
D. Noneo theabove
9. A ullyheparinized patient developsacondition requiring emergency surgery. A er stopping the heparin, what else should bedonetopreparethepatient?
A. Nothing,i thesurgerycan bedelayed or 2to3hours.
B. Immediateadministration o protamine5mg or every 100unitso heparin most recentlyadministered.
C. Immediateadministration o FFP.
D. rans usion o 10unitso platelets.
to theprocesso ACo areshock and tissueinjury.ACo isa separate and distinct process rom DIC with its own speci ic components o hemostatic ailure. (See Schwartz 10th ed., p.93.)
Answer:D
Newer anticoagulants, such as dabigatran and rivaroxaban, haveno readilyavailablemethod o detection o thedegreeo anticoagulation.More concerningis the absence o anyavailablereversalagent.Unlikewar arin,thenonreversiblecoagulopathyassociated with dabigatran and rivaroxaban iso great concern to those providing emergent care to these patients. (SeeSchwartz10th ed.,p.94.)
Answer:A
Certain surgical procedures should not be per ormed in concert with anticoagulation. In particular, cases where even minor bleeding can cause great morbiditysuch as the central nervous system and the eye. Emergency operations are occasionally necessary in patients who have been heparinized. he irst step in these patients is to discontinue heparin. For more rapid reversal, protamine sul ate is e ective. However, signi icantadversereactions,especiallyin patientswithsevere ish allergies, may be encountered when administering protamine.Symptomsincludehypotension, lushing,bradycardia, nausea, and vomiting. Prolongation o the activated partial thromboplastin time(aP )a ter heparin neutralization with protamine may also be a result o the anticoagulant e ect o protamine. In the elective surgical patient who is receiving coumarin-derivative therapy su icient to e ect anticoagulation, the drugcan be discontinued several days be ore operation and the prothrombin concentration then checked (level greater than 50%is considered sa e). (See Schwartz 10th ed., p.94.)
10. PrimaryI P
A. Occurs more o en in children than adults, but has a similar clinicalcourse.
B. IncludesHI asasubtypeo drug-induced I P.
C. Is also known as thrombotic thrombocytopenic purpura( P).
D. Isadiseaseo impaired platelet production,unknown cause.
Answer:B
Primary immune thrombocytopenia is also known as idiopathicthrombocytopenicpurpura(I P).In children it isusually acute in onset, short-lived, and typically ollows a viral illness. In contrast, I P in adults is gradual in onset, chronic in nature, and has no identi iable cause. Because the circulating platelets in I P are young and unctional, bleeding is less or a given platelet count than when there is ailure o platelet production. hepathophysiologyo I Pisbelieved toinvolve both impaired platelet production and cell-mediated platelet destruction.
reatment o drug-induced I P may simply entail withdrawal o the o ending drug, but corticosteroids, gamma globulin,and anti-D immunoglobulin mayhasten recoveryo thecount.HI isa orm o drug-inducedI P.Itisan immunological event duringwhich antibodies against platelet actor-4 (PF4) ormed during exposure to heparin a ect platelet activation and endothelial unction with resultant thrombocytopenia and intravascular thrombosis. (See Schwartz 10th ed., p.90.)
19
11. Which o the ollowing is the most common intrinsic platelet de ect?
A. Trombasthenia
B. Bernard-Soulier syndrome
C. Cyclooxygenasede ciency
D. Storagepooldisease
12. Which ndingisnot consistent with P?
A. Microangiopathichemolyticanemia
B. Schistocyteson peripheralblood smear
C. Fever
D. Splenomegaly
13. What is FALSE regarding coagulation during cardiopulmonarybypass(CPB)?
A. Contact with circuit tubingand membranesactivates infammatorycascades,and causesabnormalplatelet and clotting actor unction.
B. Coagulopathyiscompounded bysheer stress.
C. Followingbypass,platelets’morphologyandabilityto aggregateareirreversiblyaltered.
D. Coagulopathy is compounded by hypothermia and hemodilution.
Answer:D hemost common intrinsicplateletde ect isknown asstorage pooldisease.Itmayinvolvelosso densegranules(storagesites or adenosine 5′-diphosphate [ADP], adenosine triphosphate [A P],Ca2+,andinorganicphosphate)andα-granules(storage sites or alargenumber o proteins,someo which arespeci ic toplatelets[eg,PF4and β-thromboglobulin],whileothersare presentin bothplateletα-granulesandplasma[eg, ibrinogen, vWF, and albumin]). Dense granule de iciency is the most prevalent o these. It may be an isolated de ect or occur with partial albinism in the Hermansky-Pudlak syndrome. Bleeding is variable; depending on how severe the granule de ect is. Bleeding is primarily caused by the decreased release o ADP rom theseplatelets.An isolated de ect o theα-granules is known as grayplatelet syndromebecause o the appearance o the platelets on Wright’s stain. Bleeding is usually mild with this syndrome. A ew patients have been reported who havedecreased numberso both denseand α-granules. hese patients have a more severe bleeding disorder. Patients with mild bleedingas a consequence o a orm o storage pool disease mayhave decreased bleedingi given DDAVP. It is likely that thehigh levelso vWFin theplasmaa ter DDAVP somehow compensate or the intrinsic platelet de ect. With more severebleeding,platelettrans usion isrequired.(SeeSchwartz 10th ed.,p.89.)
Answer:D
In P, large vWF molecules interact with platelets, leading to activation. hese large molecules result rom inhibition o a metalloproteinase enzyme, ADAMtS13, which cleaves the largevon Willebrand actor molecules. Pisclassicallycharacterized by thrombocytopenia, microangiopathic hemolytic anemia, ever, and renal and neurologic signs or symptoms. he indingo schistocytesonaperipheralbloodsmearaidsin thediagnosis.Plasmaexchangewithreplacemento FFPisthe treatment or acute P.Additionally,rituximab,a monoclonal antibodyagainst the CD20 protein on Blymphocytes has shown promise as an immunomodulatory therapy directed against patients with acquired P, o which the majorityare autoimmune-mediated.(SeeSchwartz10th ed.,p.91.)
Answer:C
Under normal conditions, homeostasis o the coagulation system is maintained by complex interactions between the endothelium, platelets, and coagulation actors. In patients undergoing cardiopulmonary bypass (CPB), contact with circuit tubing and membranes results in abnormal platelet and clotting actor activation, as well as activation o in lammatory cascades, that ultimately result in excessive ibrinolysis and a combination o both quantitative and qualitative platelet de ects. Platelets undergo reversible alterations in morphology and their ability to aggregate, which causes sequestration in the ilter, partially degranulated platelets, and platelet ragments. his multi actorial coagulopathy is compounded by the e ects o shear stress in the system, induced hypothermia, hemodilution, and anticoagulation. (SeeSchwartz10th ed.,p.95.)
20
14. Following a recent abdominal surgery, your patient is in theICU with septicshock.Belowwhat levelo hemoglobin would ablood trans usion beindicated?
A. <12g/dL
B. <10g/dL
C. <8g/dL
D. <7g/dL
15. Less than 0.5% o trans usions result in a serious trans usion-related complication. What is the leading causeo trans usion-related deaths?
A. rans usion-related acutelunginjury
B. ABO hemolytictrans usion reactions
C. Bacterialcontamination o platelets
D. IatrogenichepatitisCin ection
16. Allergicreactionsdonot occur with
A. Packed RBCs
B. FFP
C. Cryoprecipitate
D. Noneo theabove
17. What is the risk o Hepatitis C and HIV-1 transmission with blood trans usion?
A. 1:10,000,000
B. 1:1,000,000
C. 1:500,000
D. 1:100,000
18. What is NO a cause o bleeding due to massive trans usion?
A. Dilutionalcoagulopathy
B. Hypo brinogenemia
C. Hypothermia
D. 2,3-DPGtoxicity
Answer:D
A 1988 National Institutes o Health Consensus Report challenged the dictum that a hemoglobin value o less than 10 g/dL or a hematocrit level less than 30% indicates a need or preoperative red blood cell (RBC) trans usion. his was veri ied in a prospective randomized controlled trial in critically ill patients that compared a restrictive trans usion threshold to a more liberal strategy and demonstrated that maintaining hemoglobin levelsbetween 7and 9g/dLhad noadversee ect on mortality. In act, patients with APACHE II scores o ≤20 or patients<55yearsactuallyhad alower mortality.
Despite these results, change in daily clinical practice has been slow. Critically ill patients still requently receive transusions, with the pretrans usion hemoglobin approaching 9 g/dL in a recent large observational study. his outdated approach unnecessarilyexposespatientstoincreased riskand littlebene it.(SeeSchwartz10th ed.,p.98.)
Answer:A
rans usion-related complications are primarily related to blood-induced proin lammatory responses. rans usionrelated eventsareestimated to occur in approximately10%o all trans usions, but onlyless than 0.5%are serious in nature. rans usion-related deaths, though rare, do occur and are related primarily to trans usion-related acute lung injury ( RALI) (16–22%), ABO hemolytic trans usion reactions (12–15%),and bacterialcontamination o platelets(11–18%). (SeeSchwartz10th ed.,p.100.)
Answer:D
Allergic reactions are relatively requent, occurring in about 1%o all trans usions. Reactions are usuallymild and consist o rash, urticaria, and lushing. In rare instances, anaphylactic shock develops. Allergic reactions are caused by the trans usion o antibodies rom hypersensitive donors or the trans usion o antigens to which the recipient is hypersensitive. Allergic reactions can occur a ter the administration o anyblood product but arecommonlyassociated with FFPand platelets. reatment and prophylaxis consists o the administration o antihistamines. In more serious cases, epinephrine or steroidsmaybeindicated.(SeeSchwartz10th ed.,p.100.)
Answer:B
ransmission o hepatitis C and HIV-1 has been dramatically minimized by the introduction o better antibody and nucleic acid screening or these pathogens. he residual risk among allogeneic donations is now estimated to be less than 1per 1,000,000donationsand hepatitisBapproximately1per 300,000donations.(SeeSchwartz10th ed.,p.102.)
Answer:D
Massiveblood trans usion isawell-known causeo thrombocytopenia. Bleeding ollowing massive trans usion can occur due to hypothermia, dilutional coagulopathy, platelet dysunction, ibrinolysis, or hypo ibrinogenemia. Another cause o hemostatic ailurerelatedtotheadministration o bloodisa hemolytictrans usion reaction. he irst sign o a trans usion reaction may be di use bleeding. he pathogenesis o this
21
19. Temost common cause or atrans usion reaction is
A. Air embolism
B. Contaminated blood
C. Human error
D. Unusualcirculatingantibodies
20. Frozen plasma prepared rom reshly donated blood is necessarywhen apatient requires
A. Fibrinogen
B. Prothrombin
C. Antihemophilic actor
D. Christmas actor
E. Hageman actor
21. Te most common clinical mani estation o a hemolytic trans usion reaction is
A. Flankpain
B. Jaundice
C. Oliguria
D. Ashakingchill
22. What typeo bacterialsepsiscan lead tothrombocytopeniaand hemorrhagicdisorder?
A. Gram-negative
B. Gram-positive
C. A&B
D. Encapsulated bacteria
23. A er tissueinjury,the rst step in coagulation is
A. Bindingo actor XIItosubendothelialcollagen
B. Cleavageo actor XItoactive actor IX
C. Complexingo actor IXwith actor VIII in the presence o ionized calcium conversion o prothrombin tothrombin
D. Formation o brin rom brinogen
24. What aretheuseso thromboelastography( EG)?
A. Predicting need or li esaving interventions a er arrival or trauma
B. Predicting 24-hour and 30-day mortality ollowing trauma
C. Predictingearlytrans usion o RBC,plasma,platelets, and cryoprecipitate
D. Allo theabove
bleeding is thought to be related to the release o ADP rom hemolyzed RBCs, resulting in di use platelet aggregation, a ter which theplatelet clumpsareremoved out o thecirculation.(SeeSchwartz10th ed.,p.104.)
Answer:C
Although contaminated or outdated blood may cause a reaction, the most common cause is human error—blood drawn or typing rom the wrong patient, blood incorrectly crossmatched in thelaboratory,blood unitsmislabeled in thelaboratory, blood administered to the wrong patient. Most blood banking programs have instituted elaborate checks and balancestominimizetheseerrors.(SeeSchwartz10thed.,p.101.)
Answer:C
Frozen plasma is required or the trans usion o antihemophilic actor( actorVIII)or proaccelerin ( actorV). heother actors are present in banked preparations. (See Schwartz 10th ed.,p.99.)
Answer:C
All o the mani estations listed can occur with a hemolytic trans usion reaction. In a large series, oliguria (58%) and hemoglobinuria (56%) were themost common indings.(See Schwartz10th ed.,p.101.)
Answer:A
Lastly, severe hemorrhagic disorders due to thrombocytopenia have occurred as a result o gram-negative sepsis. he pathogenesis o endotoxin-induced thrombocytopenia has been suggested that alabile actor Visnecessary or thisinteraction.(SeeSchwartz10th ed.,p.104.)
Answer:A
Allthelisted stepsarepart o thecascadeinvolved in establishing a irm clot. he process begins with binding o Hageman actor( actorXII)tosubendothelialcollagenandendswiththe conversion o ibrinogen to ibrin. he ibrin orms an insoluble addition that stabilizes the platelet plug. (See Schwartz 10th ed.,p.87.)
Answer:D
hromboelastography ( EG) is the only test measuring all dynamic steps o clot ormation until eventual clot lysis or retraction. EGhasalso been shown to identi y,patientswho are likely to develop thromboembolic complications postinjuryand postoperatively.
Recent trauma data have shown EG to be use ul in predicting early trans usion o RBCs, plasma, platelets, and cryoprecipitate. EG can also predict the need or li esaving interventions shortlya ter arrival and to predict 24-hour and 30-daymortality.Lastly, EG can beuse ulto guideadministration o tranexamicacid to injured patientswith hyper ibrinolysis.(SeeSchwartz10th ed.,p.103.)
22
25. Bank blood is appropriate or replacing each o the ollowingEXCEP
A. Factor I( brinogen)
B. Factor II(prothrombin)
C. Factor VII(proconvertin)
D. Factor VIII(antihemophilic actor)
Answer:D Factor VIII is labile, and 60 to 80%o activity is gone 1 week a ter collection. he other actors listed are stable in banked blood.(SeeSchwartz10th ed.,p.99.)
23
1. Shock caused by a large tension pneumothorax is categorized as
A. raumashock
B. Vasodilatoryshock
C. Cardiogenicshock
D. Obstructiveshock
2. What istrueabout baroreceptors?
A. Volumereceptorscan beactivated in hemorrhagewith reduction in le atrialpressure.
B. Receptors in the aortic arch and carotid bodies inhibit theautonomicnervoussystem (ANS)when stretched.
C. When baroreceptors are stretched, they induced increased ANS output and produce constriction o peripheralvessels.
D. Noneo theabove.
Answer:D
In 1934, Blalock proposed our categories o shock: hypovolemic, vasogenic, cardiogenic, and neurogenic. Hypovolemic shock, the most common type, results rom losso circulating bloodvolume. hismayresult rom losso wholeblood(hemorrhagicshock),plasma,interstitial luid (bowelobstruction), or a combination. Vasogenic shock results rom decreased resistance within capacitance vessels, usually seen in sepsis. Neurogenic shock is a orm o vasogenic shock in which spinal cord injury or spinal anesthesia causes vasodilation due to acute loss o sympathetic vascular tone. Cardiogenic shock results rom ailure o the heart as a pump, as in arrhythmias or acutemyocardialin arction (MI).
Inrecentclinicalpractice, urtherclassi icationhasdescribed six types o shock: hypovolemic, septic (vasodilatory), neurogenic, cardiogenic, obstructive, and traumatic shock. Obstructive shock is a orm o cardiogenic shock that results rom mechanicalimpedimenttocirculationleadingtodepressedcardiac output rather than primary cardiac ailure. his includes etiologiessuchaspulmonaryembolismortensionpneumothorax. In traumaticshock, so t tissue and bony injury lead to the activation o in lammatory cells and the release o circulating actors,suchascytokinesandintracellularmoleculesthatmodulatetheimmuneresponse.Recentinvestigationshaverevealed that the in lammatory mediators released in response to tissueinjury(damage-associated molecular patterns[DAMPs]) are recognized by many o the same cellular receptors (pattern recognition receptors [PRRs]) and activate similar signalingpathwaysasdo bacterialproductselaborated in sepsis (pathogen-associatedmolecularpatterns[PAMPs]),suchaslipopolysaccharide. hesee ectso tissueinjuryarecombined with the e ects o hemorrhage, creating a more complex and ampliieddeviation romhomeostasis.(SeeSchwartz10thed.,p.109.)
Answer:B
Baroreceptorsalsoarean importanta erentpathwayin initiation o adaptive responses to shock. Volume receptors, sensitiveto changesin both chamber pressureand wallstretch,are present within the atria o the heart. hey become activated with low volume hemorrhage or mild reductions in right atrial pressure. Receptors in the aortic arch and carotid bodies respond to alterations in pressure or stretch o the arterial wall, respondingto larger reductions in intravascular volume
25
5
Shock
CHAPTER
3. Chemoreceptors in the aorta and carotid bodies do NO sensewhich o the ollowing?
A. Changesin O2 tension
B. H+ ion concentration
C. HCO3 – concentration
D. Carbon dioxide(CO2)levels
4. Neurogenicshockischaracterized bythepresenceo
A. Cool,moist skin
B. Increased cardiacoutput
C. Decreased peripheralvascular resistance
D. Decreased blood volume
5. When a patient with hemorrhagic shock is resuscitated using an intravenous colloid solution rather than lactated Ringer solution, all o the ollowing statements are true
EXCEP
A. Circulatinglevelso immunoglobulinsaredecreased.
B. Colloid solutions may bind to the ionized raction o serum calcium.
C. Endogenousproduction o albumin isdecreased.
D. Extracellular uid volumede cit isrestored.
6. In hemorrhage,larger arterioles vasoconstrict in response to the sympathetic nervous system. Which categories o shockareassociated with vasodilation o larger arterioles?
A. Septicshock
B. Cardiogenicshock
C. Neurogenicshock
D. A&C
or pressure. hese receptors normally inhibit induction o the autonomic nervous system (ANS). When activated, these baroreceptors diminish their output, thus disinhibiting the e ect o the ANS. he ANS then increases its output, principallyvia sympathetic activation at the vasomotor centers o the brain stem, producing centrally mediated constriction o peripheralvessels.(SeeSchwartz10th ed.,p.112.)
Answer:C
Chemoreceptors in the aorta and carotid bodies are sensitive to changes in O2 tension, H+ ion concentration, and carbon dioxide (CO2) levels. Stimulation o the chemoreceptors results in vasodilation o the coronaryarteries,slowingo the heart rate, and vasoconstriction o the splanchnic and skeletal circulation. In addition, a varietyo protein and nonprotein mediators are produced at the site o injuryaspart o the in lammatory response, and they act as a erent impulses to induceahost response.(SeeSchwartz10th ed.,p.112.)
Answer:C
Neurogenic shock is caused by loss o arteriolar and venular tone in response to paralysis (such as occurs with high spinal anesthesia), acute gastric dilatation, or sudden pain, or unpleasant sights;as such, it is characterized bya decrease in peripheralvascular resistance. A ected patients usuallypresent with warm, dry skin, a pulse rate that is slower than normal, and hypotension. A normovolemic state usually exists, and urine output is generally well maintained. Although blood volume measurements indicate a normal intravascular volume, because o the greatlyincreased reservoir capacityo thearteriolesandvenules,thereisadecreasein cardiacoutput secondaryto decreased venous return to the right side o the heart.(SeeSchwartz10th ed.,p.129.)
Answer:D
Because o higher osmotic pressure, colloid solutions draw extracellular luid into the vascular space, increasing the extracellular luid de icit. In addition, the ionized raction o serum calcium is decreased, circulating levels o immunoglobulin drop, and reaction to tetanus toxoid given to the patient su ering rom major trauma is decreased. Endogenousproduction o albumin alsodecreases.Colloid resuscitation is no more e ective than crystalloid resuscitation, and it ismoreexpensive.(SeeSchwartz10th ed.,p.122.)
Answer:D
he microvascular circulation plays an integral role in regulating cellular per usion and is signi icantly in luenced in response to shock. he microvascular bed is innervated by the sympathetic nervous system and has a pro ound e ect on the larger arterioles. Following hemorrhage, larger arterioles vasoconstrict; however, in the setting o sepsis or neurogenic shock, these vessels vasodilate. Additionally, a host o other vasoactiveproteins,includingvasopressin,angiotensin II,and endothelin-1,also lead to vasoconstriction to limit organ perusion toorganssuchasskin,skeletalmuscle,kidneys,andthe gastrointestinal (GI) tract to preserve per usion o the myocardium and central nervous system (CNS). (See Schwartz 10th ed.,p.114.)
26
7. Which o the ollowingistrueabout antidiuretichormone (ADH)production in injured patients?
A. ADH actsasapotent mesentericvasoconstrictor.
B. ADH levels allto normalwithin 2to 3dayso theinitialinsult.
C. ADH decreaseshepaticgluconeogenesis.
D. ADH secretion is mediated by the renin-angiotensin system.
Answer:A he pituitary also releases vasopressin or antidiuretic hormone (ADH) in response to hypovolemia, changes in circulating blood volume sensed by baroreceptors and le t atrial stretch receptors, and increased plasma osmolality detected byhypothalamicosmoreceptors.Epinephrine,angiotensin II, pain, and hyperglycemia increase production o ADH. ADH levelsremain elevated or about 1week a ter the initialinsult, depending on the severity and persistence o the hemodynamic abnormalities. ADH acts on the distal tubule and collecting duct o the nephron to increase water permeability, decrease water and sodium losses,and preserve intravascular volume. Also known as arginine vasopressin, ADH acts as a potent mesentericvasoconstrictor,shuntingcirculatingblood away rom the splanchnic organs during hypovolemia. his maycontributeto intestinalischemiaand predisposeto intestinalmucosalbarrier dys unction in shockstates.Vasopressin also increases hepatic gluconeogenesis and increases hepatic glycolysis.(SeeSchwartz10th ed.,p.113)
8. Which o ollowing occur as a result o epinephrine and norepinephrine?
A. Hepaticglycogenolysis
B. Hypoglycemia
C. Insulin sensitivity
D. Lipogenesis
9. A patient has a blood pressure o 70/50 mm Hg and a serum lactate level o 30 mg/100 mL(normal: 6–16). His cardiac output is 1.9 L/min, and his central venous pressureis2cm H2O.Temost likelydiagnosisis
A. Congestiveheart ailure
B. Cardiactamponade
C. Hypovolemicshock
D. Septicshock
Answer:A Epinephrine and norepinephrine have a pro ound impact on cellular metabolism. Hepatic glycogenolysis, gluconeogenesis,ketogenesis,skeletalmuscleprotein breakdown,and adiposetissuelipolysisareincreasedbycatecholamines.Cortisol, glucagon, and ADH also contribute to the catabolism during shock.Epinephrineinduces urther releaseo glucagon,while inhibiting the pancreatic β-cell release o insulin. he result is a catabolic state with glucose mobilization, hyperglycemia, protein breakdown, negative nitrogen balance, lipolysis, and insulin resistance during shock and injury. he relative underuse o glucose by peripheral tissues preserves it or the glucose-dependent organs such as the heart and brain. (See Schwartz10th ed.,p.115.)
Answer:C he indings given in the question are characteristic o hypovolemic shock, which can be de ined as inadequate tissue per usion secondary to an extracellular luid loss. he high lactate level is a result o anaerobic metabolism due to decreased blood lowto tissues. he hemodynamic measurements indicate both low blood low and low venous return. he total combination is most consistent with a diagnosis o hypovolemic shock. Pulmonary embolus, congestive heart ailure, and cardiac tamponade are all associated with a high central venous pressure. Septic shock, particularlyin its early phases, is usuallyhyperdynamic, and a ected patients have a greater-than-normalcardiacoutput.Completehemodynamic monitoringisvitalin hypovolemicshocksothat prompt diagnosis and rational therapy can be expeditiously carried out. (SeeSchwartz10th ed.,p.119.)
27
10. Which cytokine is anti-in ammatoryand increases a er shockand trauma?
A. Interleukin (IL)-1
B. IL-2
C. IL-6
D. IL-10
11. umor necrosis actor-alpha( NF-α)
A. Can be released as a response to bacteria or endotoxin
B. Increased morein traumathan septicpatients
C. Induces procoagulant activity and peripheral vasoconstriction
D. Contributestoanemiao chronicillness
Answer:D Interleukin (IL)-10 is considered an anti-in lammatory cytokine that may have immunosuppressive properties. Its production is increased a ter shock and trauma, and it has been associated with depressed immune unction clinically,aswell as an increased susceptibility to in ection. IL-10 is secreted by cells, monocytes, and macrophages, and inhibits proin lammatory cytokine secretion, O2 radical production by phagocytes, adhesion molecule expression, and lymphocyte activation. Administration o IL-10 depresses cytokine production and improves some aspects o immune unction in experimentalmodelso shockandsepsis.(SeeSchwartz10thed., p.118.)
Answer:A umor necrosis actor-alpha ( NF-α) was one o the irst cytokines to be described, and is one o the earliest cytokines released in response to injurious stimuli. Monocytes, macrophages, and cells release this potent proin lammatorycytokine. NF-α levelspeakwithin 90minuteso stimulation and return requentlyto baseline levels within 4 hours. Release o NF-α may be induced by bacteria or endotoxin, and leads to the development o shock and hypoper usion, most commonly observed in septic shock. Production o NF-α also may be induced ollowing other insults, such as hemorrhage and ischemia. NF-α levels correlate with mortality in animalmodelso hemorrhage.In contrast,theincreasein serum
NF-α levels reported in trauma patients is ar less than that seen in septic patients. Once released, NF-α can produce peripheralvasodilation,activatethereleaseo othercytokines, induce procoagulant activity, and stimulate a wide array o cellular metabolicchanges.Duringthestressresponse, NF-α contributes to the muscle protein breakdown and cachexia. (SeeSchwartz10th ed.,p.116.)
12. A 70-kg male patient presents to ED ollowing a stab wound to the abdomen. He is hypotensive, markedly tachycardic, and appears con used. What percent o blood volumehashelost?
A. 5%
B. 15%
C. 35%
D. 55%
Answer:D
heclinicalsignso shockmaybeevidencedbyagitation,cool clammy extremities, tachycardia, weak or absent peripheral pulses, and hypotension. Such apparent clinical shock results rom at least 25 to 30% loss o the blood volume. However, substantial volumes o blood may be lost be ore the classic clinical mani estations o shock are evident. hus, when a patient is signi icantlytachycardic or hypotensive, this represents both signi icant blood loss and physiologic decompensation. he clinical and physiologic response to hemorrhage hasbeen classi ied accordingtothemagnitudeo volumeloss. Loss o up to 15%o the circulatingvolume (700–750 mL or a 70-kgpatient) mayproducelittlein termso obvioussymptoms,whilelosso up to 30%o thecirculatingvolume(1.5L) mayresult in mild tachycardia,tachypnea,and anxiety.Hypotension, marked tachycardia (ie, pulse greater than 110–120 beats per minute [bpm]), and con usion may not be evident untilmorethan 30%o theblood volumehasbeen lost;losso 40%o circulating volume (2 L) is immediately li e threatening,and generallyrequiresoperativecontrolo bleeding.(See Schwartz10th ed.,p.119.)
28
13. Vasodilatoryshock
A. Is characterized by ailure o vascular smooth muscle toconstrict duetolowlevelso catecholamines
B. Leadstosuppression o therenin-angiotensin system
C. Can alsobecaused bycarbon monoxidepoisoning
D. Issimilar toearlycardiogenicshock
Answer:C
In theperipheralcirculation,pro oundvasoconstriction isthe typical physiologic response to the decreased arterial pressure and tissue per usion with hemorrhage, hypovolemia, or acute heart ailure. his is not the characteristic response in vasodilatory shock. Vasodilatory shock is the result o dysunction o the endothelium and vasculature secondary to circulatingin lammatorymediatorsand cellsor asa response to prolonged and severehypoper usion. hus,in vasodilatory shock,hypotension results rom ailureo thevascularsmooth muscle to constrict appropriately. Vasodilatoryshock is characterized by peripheral vasodilation with resultant hypotension and resistance to treatment with vasopressors. Despite the hypotension, plasma catecholamine levels are elevated, and the renin-angiotensin system is activated in vasodilatory shock. he most requently encountered orm o vasodilatoryshockissepticshock.Other causeso vasodilatoryshock include hypoxic lactic acidosis, carbon monoxide poisoning, decompensated and irreversiblehemorrhagicshock,terminal cardiogenic shock, and postcardiotomy shock. hus, vasodilatoryshockseemstorepresentthe inalcommon pathway or pro ound and prolonged shock o anyetiology.(SeeSchwartz 10th ed.,p.124.)
14. A patient in septic shock remains hypotensive despite adequate uidresuscitation andinitiation o norepinephrine. What is o en given to patients with hypotension re ractorytonorepinephrine?
A. Dopamine
B. Argininevasopressin
C. Dobutamine
D. Milrinone
15. ight glucose management in critically ill and septic patients
A. Requiresinsulin tokeep serum glucose<140
B. Hasnoe ect on mortality
C. Hasnoe ect on ventilator support
D. Decreaseslength o antibiotictherapy
Answer:B
A ter irst-line therapy o the septic patient with antibiotics, IV luids, and intubation i necessary, vasopressors may be necessaryto treat patients with septic shock. Catecholamines are the vasopressors used most o ten, with norepinephrine being the irst-line agent ollowed by epinephrine. Occasionally,patients with septic shock willdevelop arterialresistance to catecholamines. Arginine vasopressin, a potent vasoconstrictor,iso ten e icaciousin thissettingandiso ten addedto norepinephrine.(SeeSchwartz10th ed.,p.125.)
Answer:D
Hyperglycemia and insulin resistance are typical in critically ill and septic patients, including patients without underlying diabetes mellitus. A recent study reported signi icant positive impact o tight glucose management on outcome in critically illpatients. hetwo treatment groupsin thisrandomized,prospectivestudywereassignedtoreceiveintensiveinsulintherapy (maintenance o blood glucose between 80and 110mg/dL) or conventional treatment (in usion o insulin only i the blood glucose level exceeded 215 mg/dL, with a goal between 180 and 200 mg/dL). he mean morning glucose level was signi icantly higher in the conventional treatment as compared with the intensive insulin therapygroup (153 vs 103 mg/dL). Mortalityin theintensiveinsulin treatment group (4.6%) was signi icantly lower than in the conventional treatment group (8.0%),representinga42%reduction in mortality. hisreduction in mortality was most notable in the patients requiring longer than 5 days in the ICU. Furthermore, intensive insulin therapy reduced episodes o septicemia by 46%, reduced duration o antibiotic therapy, and decreased the need or prolonged ventilatorysupport and renalreplacement therapy. (SeeSchwartz10th ed.,p.125.)
29
16. Cardiogenicshock
A. Ismost commonlycaused byexacerbation o congestiveheart ailure.
B. Cardiogenic shock ollowing an acute myocardial in arction istypicallypresent on admission.
C. Cardiogenicshockoccursin 5to10%o acuteMIs.
D. Is characterized by hypotension, reduced cardiac index,andreducedpulmonaryarterywedgepressure.
Answer:C
Cardiogenic shock is de ined clinically as circulatory pump ailureleadingtodiminished orward lowandsubsequenttissue hypoxia, in the setting o adequate intravascular volume. Hemodynamic criteria include sustained hypotension (ie, systolic blood pressure [SBP] <90mm Hg or at least 30minutes), reduced cardiac index (<2.2 L/min per square meter), and elevated pulmonaryarterywedgepressure(>15mm Hg). Mortality rates or cardiogenic shock are 50 to 80%. Acute, extensiveMI isthemost common causeo cardiogenicshock; a smaller in arction in a patient with existing le t ventricular dys unction also may precipitate shock. Cardiogenic shock complicates 5 to 10% o acute MIs. Conversely, cardiogenic shockisthemost common causeo death in patientshospitalized with acute MI. Although shock may develop early a ter MI, it typically is not ound on admission. Seventy- ive percento patientswhohavecardiogenicshockcomplicatingacute MIs develop signs o cardiogenic shock within 24 hours a ter onset o in arction (average 7 hours). (See Schwartz 10th ed., p.126.)
17. Allo the ollowingresult rom theplacement o an intraaortic balloon pump in a patient with acute myocardial ailureEXCEP
A. Reduction o systolica erload
B. Increased cardiacoutput
C. Increased myocardialO2 demand
D. Increased diastolicper usion pressure
18. Which constellation o clinical ndings is suggestive o cardiactamponade?
A. Hypotension,widepulsepressure,tachycardia
B. achycardia,hypotension,jugular venousdistension
C. Hypotension, wide pulse pressure, jugular venous distension
D. Hypotension, mu ed heart tones, jugular venous distension
Answer:C
Intra-aortic balloon pumping increases cardiac output and improves coronary blood low by reduction o systolic a terload and augmentation o diastolicper usion pressure.Unlike vasopressor agents, these bene icial e ects occur without an increase in myocardial O2 demand. An intra-aortic balloon pump can be inserted at the bedside in the ICU via the emoralarterythrough either acutdown or usingthepercutaneous approach.(SeeSchwartz10th ed.,p.127.)
Answer:D
Cardiac tamponade also may be associated with dyspnea, orthopnea, cough, peripheral edema, chest pain, tachycardia, mu led heart tones, jugular venous distention, and elevated central venous pressure. Beck’s triad consists o hypotension, mu led heart tones, and neck vein distention. Un ortunately, absence o these clinical indings may not be su icient to exclude cardiac injuryand cardiac tamponade.Mu led heart tones may be di icult to appreciate in a busy trauma center and jugular venous distention and central venous pressure maybediminishedbycoexistentbleeding. here ore,patients atrisk or cardiactamponadewhosehemodynamicstatuspermits additional diagnostic tests requently require additional diagnostic maneuvers to con irm cardiac injury or tamponade.(SeeSchwartz10th ed.,p.128.)
19. A 43-year-old man is struck by a motor vehicle while crossingthestreet;hearrivesin theED hypotensive,bradycardic,and unabletomovehisextremities.What isthe most likelycauseo hishypotension?
A. Hypovolemicshock
B. Obstructiveshock
C. Neurogenicshock
D. Vasodilatoryshock
Answer:A
In asubset o patientswith spinalcord injuries rom penetratingwounds,most o thepatientswith hypotension had blood loss as the etiology(74%) rather than neurogenic causes, and ew (7%) had the classic indings o neurogenic shock. In the multiply injured patient, other causes o hypotension including hemorrhage, tension pneumothorax, and cardiogenic shock must be sought and excluded. (See Schwartz 10th ed., p.129.)
30
20. Corticosteroidsin thetreatment o septicshock
A. Improvesrateso shock reversalin patientsrequiring vasopressors
B. Improves mortality in patients with relative adrenal insufciency
C. Is contraindicated in patients with positive bacterial blood cultures
D. Noneo theabove
Answer:B he use o corticosteroids in the treatment o sepsis and septicshock hasbeen controversial or decades. heobservation thatseveresepsiso tenisassociatedwithadrenalinsu iciency or glucocorticoid receptor resistance has generated renewed interest in therapy or septic shock with corticosteroids. A single IV dose o 50 mg o hydrocortisone improved mean arterialblood pressure response relationships to norepinephrineand phenylephrinein patientswith septicshock,and was most notable in patients with relative adrenal insu iciency. A more recent study evaluated therapy with hydrocortisone (50 mg IV every 6 hours) and ludrocortisone (50 µg orally once daily) versus placebo or 1 week in patients with septic shock.Asin earlier studies,theauthorsper ormed corticotropin tests on these patients to document and strati y patients byrelativeadrenalinsu iciency.In thisstudy,the7-daytreatment with low doses o hydrocortisone and ludrocortisone signi icantly and sa ely lowered the risk o death in patients with septic shock and relative adrenal insu iciency. In an international,multicenter,randomized trialo corticosteroids in sepsis(COR ICUSstudy;499analyzablepatients),steroids showed no bene it in intent to treat mortalityor shock reversal. his study suggested that hydrocortisone therapy cannot be recommended as routine adjuvant therapy or septic shock.However,i SBP remains lessthan 90mm Hgdespite appropriate luid and vasopressor therapy, hydrocortisone at 200mg/day or 7daysin our divided dosesor bycontinuous in usion shouldbeconsidered.(SeeSchwartz10thed.,p.126.)
21. What isFALSEabout serum lactate?
A. Generated rom pyruvatein thesettingo insufcient O2
B. Metabolized bytheliver and kidneys.
C. Is an indirect measure o the magnitude and severity o shock.
D. Tetimetopeaklactate rom admission predictsrates o survival.
Answer:D
Lactate is generated by conversion o pyruvate to lactate by lactatedehydrogenasein thesettingo insu icient O2.Lactate isreleased into thecirculation and ispredominantlytaken up and metabolized by the liver and kidneys. he liver accounts or approximately 50% and the kidney or about 30% o whole body lactate uptake. Elevated serum lactate is an indirect measure o the O2 debt, and there ore an approximation o the magnitude and duration o the severity o shock. he admission lactate level, highest lactate level, and time interval to normalize the serum lactate are important prognostic indicators or survival.For example,in a studyo 76consecutive patients, 100%survival was observed amongthe patients with normalization o lactate within 24 hours, 78% survival when lactate normalized between 24 and 48 hours, and only 14%survivorship i it took longer than 48hoursto normalize the serum lactate. In contrast, individual variabilityo lactate maybe too great to permit accurate prediction o outcome in anyindividualcase.Basede icit and volumeo blood trans usion required in the irst 24hourso resuscitation maybebetter predictors o mortalitythan the plasma lactate alone. (See Schwartz10th ed.,p.130.)
31
1. rans errin playsarolein host de enseby
A. Sequestering iron, which is necessary or microbial growth
B. Increasingtheabilityo brinogen totrap microbes
C. Direct injurytothebacterialcellmembrane
D. Direct injurytothebacterialmitochondria
SurgicalInfections
Answer:A
Once microbes enter a sterile body compartment (eg, pleural or peritoneal cavity) or tissue, additional host de enses act to limit and/or eliminate these pathogens. Initially, several primitive and relatively nonspeci ic host de enses act to contain theniduso in ection,which mayincludemicrobesas well as debris, devitalized tissue, and oreign bodies, depending on the nature o the injury. hese de enses include the physical barrier o the tissue itsel , as well as the capacity o proteins, such as lacto errin and trans errin, to sequester the critical microbial growth actor iron, thereby limiting microbial growth. In addition, ibrinogen within the in lammatory luid has the abilityto trap large numbers o microbes during the process in which it polymerizes into ibrin. Within the peritoneal cavity, unique host de enses exist, including a diaphragmatic pumping mechanism whereby particles including microbes within peritoneal luid are expunged rom the abdominal cavity via specialized structures on the undersur ace o the diaphragm. Concurrently, containment by the omentum, the so-called “gatekeeper” o the abdomen and intestinalileus,servestowallo in ection.However,thelatter processes and ibrin trapping have a high likelihood o contributingtothe ormation o an intra-abdominalabscess.(See Schwartz10th ed.,p.138.)
2. Which is NO a component o systemic in ammatory responsesyndrome(SIRS)?
A. emperature
B. Whiteblood cell(WBC)count
C. Blood pressure
D. Heart rate
3. Tebestmethod or hair removal rom an operative eldis
A. Shavingthenight be ore
B. Depilatingthenight be oresurgery
C. Shavingin theoperatingroom
D. Usinghair clippersin theoperatingroom
Answer:C
Infection isde ined bythepresenceo microorganismsin host tissue or the bloodstream. At the site o in ection the classic indingso rubor,calor,and dolor in areas such asthe skin or subcutaneous tissue are common. Most in ections in normal individualswith intact host de ensesareassociated with these local mani estations, plus systemic mani estations such as elevated temperature,elevated whiteblood cell(WBC) count, tachycardia,or tachypnea. hesystemicmani estationsnoted above comprise the systemicinflammatoryresponsesyndrome (SIRS).(SeeSchwartz10th ed.,p.138.)
Answer:D
Hair removal should take place using a clipper rather than a razor; the latter promotes overgrowth o skin microbes in smallnicksand cuts.Dedicated useo thesemodalitiesclearly has been shown to diminish the quantity o skin micro lora. (SeeSchwartz10th ed.,p.141.)
33
6
CHAPTER
4. A patient with necrotizing pancreatitis undergoes computed tomography (C )-guided aspiration, which results in growth o Escherichia coli on culture. Te most appropriatetreatment is
A. Culture-appropriateantibiotictherapy
B. Endoscopic retrograde cholangiopancreatographywith sphincterotomy
C. C -guided placement o drain(s)
D. Exploratorylaparotomy
5. Which actor doesNO in uencethedevelopment o surgicalsitein ections(SSIs)?
A. Duration o procedure
B. Degreeo microbialcontamination o thewound
C. Malnutrition
D. Generalanesthesia
6. Duringalaparoscopicappendectomy,alargebowelinjury was caused during trochar placement with spillage o bowel contents into the abdomen. What class o surgical wound isthis?
A. ClassI(clean)
B. ClassII(clean/contaminated)
C. ClassIII(contaminated)
D. ClassIV(dirty)
Answer:D
he primary precept o surgical in ectious disease therapy consists o drainage o all purulent material, debridement o all in ected, devitalized tissue, and debris, and/or removal o oreign bodies at the site o in ection, plus remediation o the underlyingcause o in ection. Adiscrete, walled-o purulent luid collection (ie, an abscess) requires drainage via percutaneous drain insertion or an operative approach in which incision and drainage take place. An ongoing source o contamination (eg, bowel per oration) or the presence o an aggressive, rapidly spreading in ection (eg, necrotizing so t tissue in ection) invariably requires expedient, aggressive operativeintervention,bothtoremovecontaminatedmaterial and in ected tissue (eg, radical debridement or amputation) and to remove the initial cause o in ection (eg, bowel resection).(SeeSchwartz10th ed.,p.141.)
Answer:D
Surgical site in ections (SSIs) are in ections o the tissues, organs,or spacesexposed bysurgeonsduringper ormanceo an invasive procedure. SSIs are classi ied into incisional and organ/space in ections, and the ormer are urther subclassiied into super icial(limited to skin and subcutaneoustissue) and deep incisional categories. he development o SSIs is related to three actors: (1) the degree o microbial contamination o the wound during surgery, (2) the duration o the procedure,and (3)host actorssuch asdiabetes,malnutrition, obesity,immunesuppression,and anumber o other underlyingdiseasestates.(SeeSchwartz10th ed.,p.147.)
Answer:C
Surgicalwoundsareclassi ied based on thepresumed magnitudeo thebacterialload at thetimeo surgery.Clean wounds (Class I) include those in which no in ection is present; only skin micro lora potentially contaminate the wound, and no hollow viscus that contains microbes is entered. Class ID wounds are similar except that a prosthetic device (eg, mesh or valve) is inserted. Clean/contaminated wounds (Class II) includethosein whichahollowviscus,suchastherespiratory, alimentary, or genitourinary tracts, with indigenous bacterial lora is opened under controlled circumstances without signi icant spillage o contents. Contaminated wounds (ClassIII) includeopen accidentalwoundsencountered early a ter injury, those with extensive introduction o bacteria into a normally sterile area o the body due to major breaks in sterile technique (eg, open cardiac massage), gross spillage o viscus contents such as rom the intestine, or incision through in lamed, albeit nonpurulent, tissue. Dirty wounds (Class IV) include traumatic wounds in which a signi icant delay in treatment has occurred and in which necrotic tissue is present, those created in the presence o overt in ection as evidenced by the presence o purulent material, and those created to access a per orated viscus accompanied by a high degreeo contamination.(SeeSchwartz10th ed.,p.147.)
34
7. he most appropriate treatment o a 4-cm hepatic abscess is
A. Antibiotictherapyalone
B. Aspiration or cultureand antibiotictherapy
C. Percutaneousdrainageand antibiotictherapy
D. Operative exploration, open drainage o the abscess, and antibiotictherapy
Answer:C
Hepatic abscesses are rare, currently accounting or approximately15per 100,000hospitaladmissionsin theUnitedStates. Pyogenic abscesses account or approximately 80% o cases, theremaining20%beingequallydivided amongparasiticand ungal orms. Formerly, pyogenic liver abscesses were caused bypylephlebitisdueto neglected appendicitisor diverticulitis. oday,manipulation o thebiliarytract totreat avarietyo diseases has become a more common cause, although in nearly 50% o patients no cause is identi ied. he most common aerobicbacteriaidenti ied in recent seriesincludeE.coli,Klebsiella pneumoniae, and other enteric bacilli, enterococci, and Pseudomonas spp., while the most common anaerobic bacteria are Bacteroides spp., anaerobic streptococci, and Fusobacteriumspp.Candida albicansandother similar yeastscausethe majority o ungal hepatic abscesses. Small (<1 cm), multiple abscesses should be sampled and treated with a 4- to 6-week course o antibiotics. Larger abscesses invariably are amenable to percutaneous drainage, with parameters or antibiotic therapy and drain removal similar to those mentioned above. Splenic abscesses are extremelyrare and are treated in a similar ashion.Recurrent hepaticor splenicabscessesmayrequire operative intervention—unroo ing and marsupialization or splenectomy,respectively.(SeeSchwartz10th ed.,p.150.)
8. Postoperativeurinarytract in ections(U Is)
A. Are usually treated with a 7- to 10-day course o antibiotics.
B. Initial therapy should be directed by results o urine culture.
C. Are established by >104 CFU/mL o bacteria in urine culturein asymptomaticpatients.
D. Can be reduced by irrigating indwelling Foley cathetersdaily.
Answer:B
he presence o a postoperative U I should be considered based on urinalysis demonstratingWBCs or bacteria, a positive test or leukocyte esterase, or a combination o these elements. he diagnosis is established a ter >104 CFU/mL o microbesareidenti ied byculturetechniquesin symptomatic patients,or>105 CFU/mLin asymptomaticindividuals. reatment or 3 to 5 days with a single antibiotic directed against themost common organisms(eg,E.Coli,K.pneumoniae)that achieves high levels in the urine is appropriate. Initial therapy is directed by Gram’s stain results and is re ined as culture results become available. Postoperative surgical patients should have indwelling urinary catheters removed as quickly as possible, typically within 1 to 2 days, as long as they are mobile, to avoid the development o a U I. (See Schwartz 10th ed.,p.152.)
9. Te rst step in the evaluation and treatment o a patient with an in ected bug bite on the leg with cellulitis, bullae, thin grayish uid draining rom the wound, and pain out o proportion tothephysical ndingsis
A. Obtain C-reactiveprotein
B. C scan o theleg
C. Magneticresonanceimaging(MRI)o theleg
D. Operativeexploration
Answer:D
he diagnosis o necrotizing in ection is established solely upon a constellation o clinical indings, not all o which are present in every patient. Not surprisingly, patients o ten develop sepsis syndrome or septic shock without an obvious cause. he extremities, perineum, trunk, and torso are most commonly a ected, in that order. Care ul examination should be undertaken or an entry site such as a small break or sinus in the skin rom which grayish, turbid semipurulent material (“dishwater pus”) can be expressed, as well as or the presence o skin changes (bronze hue or brawnyinduration), blebs, or crepitus. he patient o ten develops pain at the site o in ection that appears to be out o proportion to any o the physical mani estations. Any o these indings mandates immediate surgical intervention, which should consist o exposureand direct visualization o potentiallyin ected tissue (includingdeepso ttissue, ascia,andunderlyingmuscle)and
35
10. What is FALSE regarding intravascular catheter in ections?
A. Selected low-virulence in ections can be treated with aprolonged courseo antibiotics.
B. In high-riskpatients,prophylacticantibioticsin used through the catheter can reduce rate o catheter in ections.
C. Bacteremia with gram-negative bacteria or ungi should prompt catheter removal.
D. Many patients with intravascular catheter in ections areasymptomatic.
radical resection o a ected areas. Radiologic studies should be undertaken only in patients in whom the diagnosis is not seriously considered, as they delay surgical intervention and requently provide con using in ormation. Un ortunately, surgicalextirpation o in ectedtissue requentlyentailsamputation and/or dis iguring procedures; however, incomplete procedures are associated with higher rates o morbidity and mortality.(SeeSchwartz10th ed.,p.151.)
Answer:B
Many patients who develop intravascular catheter in ections are asymptomatic, o ten exhibiting solely an elevation in the WBC count. Blood cultures obtained rom a peripheral site and drawn through thecatheter that revealthepresenceo the same organism increase the index o suspicion or the presenceo acatheter in ection.Obviouspurulenceat theexit site o the skin tunnel, severe sepsis syndrome due to any type o organism when other potentialcauseshavebeen excluded,or bacteremiaduetogram-negativeaerobesor ungishould lead to catheter removal. Selected catheter in ections due to lowvirulence microbes such as Staphylococcus epidermidis can be e ectively treated in approximately 50 to 60% o patients with a 14- to 21-day course o an antibiotic, which should be considered when no other vascular access site exists. Use o systemicantibacterialor anti ungalagentsto prevent catheter in ection iso no utilityand iscontraindicated.(SeeSchwartz 10th ed.,p.154.)
11. Patientswith apenicillin allergyareLEAS likelyto have across-reaction with
A. Syntheticpenicillins
B. Carbapenems
C. Cephalosporins
D. Monobactams
12. What is the estimated risk o transmission o human immunode ciency virus (HIV) rom a needlestick rom asourcewith HIV-in ected blood?
A. <0.5%
B. 1%
C. 5%
D. 10%
Answer:D
Allergy to antimicrobial agents must be considered prior to prescribing them. First, it is important to ascertain whether a patient has had any type o allergic reaction in association with administration o a particular antibiotic. However, one should take care to ensure that the purported reaction consists o true allergic symptoms and signs, such as urticaria, bronchospasm, or other similar mani estations, rather than indigestion or nausea.Penicillin allergyisquitecommon,the reported incidenceranging rom 0.7to10%.Although avoidingthe use o anybeta-lactam drugisappropriatein patients who mani est signi icant allergic reactions to penicillins, the incidence o cross-reactivity appears low or all related agents, with 1% cross-reactivity or carbapenems, 5 to 7% cross-reactivity or cephalosporins, and extremely small or nonexistent cross-reactivity or monobactams.(SeeSchwartz 10th ed.,p.146.)
Answer:A
While alarming to contemplate, the risk o human immunode iciency virus (HIV) transmission rom patient to surgeon is low. As o May 2011, there had been six cases o surgeons with HIV seroconversion rom a possible occupational exposure, with no new cases reported since 1999. O the numbers o health care workers with likely occupationally acquired HIV in ection (n = 200), surgeons were one o the lower risk groups (compared to nurses at 60casesand nonsurgeon physicians at 19 cases). he estimated risk o transmission rom a needlestick rom a source with HIV-in ected blood is estimated at 0.3%.(SeeSchwartz10th ed.,p.156.)
36
13. Closure o an appendectomy wound in a patient with per orated appendicitis who is receiving appropriate antibiotics will result in a wound in ection in what percentageo patients?
A. 3–4%
B. 8–12%
C. 15–18%
D. 22–25%
14. A chronic carrier state occurs with hepatitis C in ection in what percentageo patients?
A. 90–99%
B. 75–80%
C. 50–60%
D. 10–30%
15. Possible exposure to anthrax should be initially treated with
A. Colistin
B. Cipro oxacin or doxycycline
C. Amoxicillin
D. Observation
Answer:A Surgical management o the wound is also a critical determinant o the propensityto develop an SSI.In healthyindividuals, class I and II wounds maybe closed primarily, while skin closureo classIIIand IVwoundsisassociatedwith high rates o incisional SSIs (~25–50%). he super icial aspects o these latter types o wounds should be packed open and allowed to healbysecondaryintention,although selectiveuseo delayed primaryclosure has been associated with a reduction in incisionalSSIrates.It remainstobedetermined whether National Nosocomial In ections Surveillance (NNIS) system type strati ication schemescan beemployed prospectivelyin order to target speci ic subgroups o patients who will bene it rom theuseo prophylacticantibioticand/or speci icwound management techniques.Oneclear examplebased on cogent data rom clinical trials is that class III wounds in healthypatients undergoing appendectomy or per orated or gangrenous appendicitiscan beprimarilyclosed aslongasantibiotictherapy directed against aerobes and anaerobes is administered. hispractice leads to SSI rateso approximately3to 4%.(See Schwartz10th ed.,p.149.)
Answer:B
Hepatitis C virus (HCV), previouslyknown as non-A, non-B hepatitis,isanRNA lavivirus irstidenti iedspeci icallyinthe late1980s. hisvirusiscon inedtohumansandchimpanzees. Achronic carrier state develops in 75 to 80%o patients with the in ection, with chronic liver disease occurring in threeourths o patients who develop chronic in ection. he number o newin ectionsper year hasdeclinedsincethe1980sdue to routine testing o blood donors or this virus. Fortunately, HCV is not transmitted e iciently through occupational exposures to blood, with the seroconversion rate a ter accidentalneedlestickapproximately1.8%.(SeeSchwartz10thed., p.156.)
Answer:B
Inhalational anthrax develops a ter a 1- to 6-day incubation period, with nonspeci ic symptoms including malaise, myalgia, and ever. Over a short period o time, these symptoms worsen, with development o respiratorydistress, chest pain,and diaphoresis.Characteristicchest roentgenographic indings include a widened mediastinum and pleural e usions. A key aspect in establishing the diagnosis is eliciting an exposure history. Rapid antigen tests are currently under development or identi ication o this gram-positive rod. Postexposureprophylaxisconsistso administration o either cipro loxacin or doxycycline. I an isolate is demonstrated to be penicillin-sensitive, the patient should be switched to amoxicillin. Inhalational exposure ollowed by the development o symptoms is associated with a high mortality rate. reatment options include combination therapy with ciproloxacin, clindamycin, and ri ampin; clindamycin added to blocks production o toxin, while ri ampin penetrates into the central nervous system and intracellular locations. (See Schwartz10th ed.,p.156.)
37
16. Temost e ectivepostexposureprophylaxis or asurgeon stuck with a needle while operating on an HIV-positive patient is
A. None(noe ectivetreatment isknown).
B. wo- or three-drug therapy started within hours o exposure.
C. Single drug therapy started within 24 hours o exposure.
D. riple drug therapy started within 24 hours o exposure.
Answer:B
Postexposure prophylaxis or HIV has signi icantlydecreased the risk o seroconversion or health care workers with occupational exposure to HIV. Steps to initiate postexposure prophylaxis should be initiated within hours rather than days or the most e ective preventive therapy. Postexposure prophylaxiswith atwo- or three-drugregimen should beinitiated or health care workers with signi icant exposure to patients with an HIV-positive status. I a patient’s HIV status is unknown, it may be advisable to begin postexposure prophylaxis while testing is carried out, particularly i the patient is at high risk or in ection duetoHIV(eg,intravenousnarcoticuse).Generally,postexposureprophylaxisisnotwarranted or exposureto sourceswithunknown status,suchasdeceasedpersonsorneedles rom asharpscontainer.(SeeSchwartz10th ed.,p.156.)
17. What isNO an earlygoalin treatment o severesepsis?
A. Mean arterialpressure>65mm Hg
B. Centralvenouspressure8to2mm Hg
C. Urineoutput >0.5cc/kg/h
D. Serum lactate<2mmol/L
18. Apatient in theICU hasbeen on ventilator support or 3 weeks.Hehasnewonset elevated WBC count, ever,and consolidation seen on chestX-ray.Whatisan appropriate next step?
A. Exchange endotracheal tube and change respiratory circuit.
B. Obtain bronchoalveolar lavage.
C. Start treatment with empiricpenicillin G.
D. Obtain chest C .
19. Patients with severe, necrotizing pancreatitis should be treated with
A. NoantibioticsunlessC -guidedaspiration o thearea yieldspositivecultures
B. Empiricce oxitin or ce otetan
C. Empiricce uroximeplusgentamicin
D. Empiriccarbapenemsor uoroquinolones
Answer:D
Patients presenting with severe sepsis should receive resuscitation luidsto achieveacentralvenouspressuretarget o 8to 12mm Hg,withagoalo mean arterialpressureo >65mm Hg and urine output o >0.5 cc/kg/h. Delaying this resuscitative step or as little as 3 hours until arrival in the ICU has been shown to result in poor outcome. ypically this goal necessitatesearlyplacemento centralvenouscatheter.(SeeSchwartz 10th ed.,p.154.)
Answer:B
Prolonged mechanical ventilation is associated with nosocomial pneumonia. hese patients present with more severe disease, are more likely to be in ected with drug-resistant pathogens, and su er increased mortality compared with patients who develop community-acquired pneumonia. he diagnosis o pneumonia is established bypresence o a purulent sputum, elevated leukocyte count, ever, and new chest X-ray abnormalities such as consolidation. he presence o two o the clinical indings, plus chest X-ray indings, signi icantly increases the likelihood o pneumonia. Consideration should be given to per orming bronchoalveolar lavage to obtain samples or Gram stain and culture. Some authors advocatequantitativeculturesasameansto identi yathreshold or diagnosis. Surgical patients should be weaned rom mechanical ventilation as soon as easible, based on oxygenation and inspiratory e ort, as prolonged mechanical ventilation increases the risk o nosocomial pneumonia. (See Schwartz10th ed.,p.153.)
Answer:D
Current care o patients with severe acute pancreatitis includes staging with dynamic, contrast-enhanced helical C scan with 3-mm tomographs to determine the extent o pancreatic necrosis, coupled with the use o one o several prognostic scoring systems. Patients who exhibit signi icant pancreatic necrosis should be care ullymonitored in the ICU and undergo ollow-up C examination. he weight o current evidence also avors administration o empiric antibiotic therapy to reduce the incidence and severity o secondary pancreatic in ection, which typically occurs several weeks a ter the initial episode o pancreatitis. Several randomized,
38
20. A patient with a localized wound in ection afer surgery should betreated with
A. Antibioticsand warm soakstothewound
B. Antibioticsalone
C. Antibioticsand openingthewound
D. Incision and drainagealone
21. Which areas likely do NO contain resident microorganisms?
A. erminalileum
B. Oropharynx
C. Main pancreaticduct
D. Nares
prospective trials have demonstrated a decrease in the rate o in ection and mortality using agents such as carbapenems or luoroquinolones that achieve high pancreatic tissue levels. (SeeSchwartz10th ed.,p.150.)
Answer:D
E ective therapy or incisional SSIs consists solelyo incision and drainage without the addition o antibiotics. Antibiotic therapy is reserved or patients in whom evidence o severe cellulitis is present, or who mani est concurrent sepsis syndrome. heopen wound o ten isallowed tohealbysecondary intention, with dressings being changed twice a day. he use o topical antibiotics and antiseptics, to urther wound healing, remains unproven, although anecdotal studies indicate their potentialutilityin complexwoundsthatdonothealwith routinemeasures.(SeeSchwartz10th ed.,p.149.)
Answer:C
he urogenital, biliary, pancreatic ductal, and distal respiratory tracts do not possess resident micro lora in healthy individuals, although microbes may be present i these barriers are a ected by disease (eg, malignancy, in lammation, calculi,or oreign body),or i microorganismsareintroduced rom an external source (eg, urinary catheter or pulmonary aspiration). In contrast, signi icant numbers o microbes are encountered in many portions o the gastrointestinal tract, with vast numbers being ound within the oropharynx and distalcolorectum,although thespeci icorganismsdi er.(See Schwartz10th ed.,p.137.)
39
1. Cricothyroidotomy
A. Should not be per ormed in children younger than 12 years
B. Shouldonlybeper ormedin patientswhoarenotgood candidates or atracheostomy
C. Requiresthe use o an endotrachealtubesmaller than 4mm in diameter
D. Is pre erable to the use o percutaneous transtracheal ventilation
Answer:A
Patients in whom attempts at intubation have ailed or are precluded rom intubation due to extensive acial injuries require a surgicalairway. Cricothyroidotomy(Fig. 7-1) and percutaneoustranstrachealventilation arepre erred over tracheostomyin mostemergencysituationsbecauseo theirsimplicityandsa ety. Onedisadvantageo cricothyroidotomyistheinabilitytoplacea tubegreater than 6mm in diameter dueto thelimited aperture o the cricothyroid space. Cricothyroidotomy is also relatively contraindicatedin patientsyounger than 12yearsbecauseo the risko damagetothecricoidcartilageandthesubsequentrisko subglotticstenosis.(SeeSchwartz10thed.,p.163.)
FIG.7-1. A&B:Cricothyroidotomyisrecommended oremergent surgicalestablishment o a patent airway.Averticalskin incision avoids injuryto the anteriorjugularveins,which are located just lateralto the midline.Hemorrhage rom these vesselsobscuresvision and prolongs the procedure.When a transverse incision ismade in the cricothyroid membrane,the blade o the knie should be angled in eriorlyto avoid injuryto the vocalcords.A.Use o a tracheostomyhookstabilizesthe thyroid cartilage and acilitatestube insertion.B.A6.0-endotracheal tube isinserted a terdigitalcon irmation o airwayaccess.
2. Which o the ollowing is NO a sign o tension pneumothorax?
A. rachealdeviation
B. Decreased breath sounds
C. Respiratorydistresswith hypertension
D. Distended neckveins
Answer:C
he diagnosis o tension pneumothorax is presumed in any patient mani esting respiratory distress and hypotension in combination with anyo the ollowingphysicalsigns:tracheal deviation away rom the a ected side, lack o or decreased breath soundson thea ected side,and subcutaneousemphysema on the a ected side. Patients may have distended neck veins due to impedance o venous return, but the neck veins may be lat due to concurrent systemic hypovolemia. ension pneumothorax and simple pneumothorax have similar signs, symptoms, and examination indings, but hypotension quali ies the pneumothorax as a tension pneumothorax. (See Schwartz10th ed.,p.163.)
41
7
Trauma
A B
CHAPTER
3. Which o the ollowingisacauseo cardiogenicshockin a traumapatient?
A. Hemothorax
B. Penetratinginjurytotheaorta
C. Air embolism
D. Iatrogenicincreased a erload duetopressors
Answer:C
In trauma patients the di erential diagnosis o cardiogenic shock consists o a short list: (1) tension pneumothorax, (2) pericardial tamponade, (3) myocardial contusion or in arction,and (4)air embolism.
ension pneumothorax is the most requent cause o cardiac ailure. raumatic pericardial tamponade is most o ten associated with penetrating injury to the heart. As blood leaks out o the injured heart, it accumulates in the pericardial sac. Because the pericardium is not acutely distensible, the pressure in the pericardial sac rises to match that o the injured chamber. Since this pressure is usually greater than that o the right atrium, right atrial illing is impaired and right ventricular preload is reduced. his leads to decreased right ventricular output and increased central venous pressure (CVP). Increased intrapericardial pressure also impedes myocardial blood low, which leads to subendocardial ischemia and a urther reduction in cardiac output. his vicious cycle may progress insidiously with injury o the vena cava or atria, or precipitously with injury o either ventricle. With acutetamponade,aslittleas100mLo blood within thepericardial sac can produce li e-threatening hemodynamic compromise. Patients usuallypresent with a penetratinginjuryin proximitytotheheart,and theyarehypotensiveand havedistended neck veins or an elevated CVP. he classic indings o Beck’s triad (hypotension, distended neck, and mu led heart sounds) and pulsus paradoxus are not reliable indicators o acute tamponade. Ultrasonography (US) in the emergency department (ED) using a subxiphoid or parasternal view is extremelyhelp uli the indingsareclearlypositive(Fig.7-2); however, equivocal indings are common. Earlyin the course o tamponade, blood pressure (BP) and cardiac output will transiently improve with luid administration. his may lead the surgeon to question the diagnosis or be lulled into a alse sense o security. (See Schwartz 10th ed., Figure 7-5, pp. 165 and 171.)
42
A B
FIG.7-2. A.Admission chest ilm maynot show the ullextent o the patient’spulmonaryparenchymalinjury.B.Thispatient’sle t pulmonary contusion blossomed 12hourslater,and itsassociated opacityisnoted on repeat chest radiograph.
4. Atraumapatient arrives ollowingastab wound to thele chest with systolicblood pressure(SBP) 85mm Hg,which improvesslightlywithintravenous(IV) uidresuscitation.
Chest X-ray demonstrates clear lung elds. What is the most appropriatenext step?
A. Computed tomography(C )scan o thechest
B. PelvicX-ray
C. Focused abdominal sonography or trauma (FAS ) examination
D. ubethoracostomyo thele chest
5. Primaryrepair o thetracheashould becarried out with
A. Wiresuture
B. Absorbablemono lament suture
C. Nonabsorbablemono lament suture
D. Absorbablebraided suture
6. In which patient is emergency department thoracotomy contraindicated?
A. Motorvehicleaccidentvictim,cardiactamponadeseen on ultrasound,SBPdecreasingto50mm Hg.
B. Motorvehicleaccidentvictim,becameasystolicduring transport with 5 minutes o cardiopulmonaryresuscitation (CPR)with nosignso li e.
C. Patient with chest stab wound, SBP decreasing to 50 mm Hg.
D. Patientwith chest stabwound,becameasystolicduring transport with 20minuteso CPRwith nosignso li e.
Answer:C
Duringthecirculation section o theprimarysurvey, our li ethreatening injuries must be identi ied promptly:(1) massive hemothorax, (2) cardiac tamponade, (3) massive hemoperitoneum, and (4) mechanically unstable pelvic ractures with bleeding.
In this patient hemothorax is unlikely given normal chest X-ray;thus,hemoperitoneum and cardiactamponadeshould be suspected. Cardiac tamponade occurs most commonly a ter penetrating thoracic wounds, although occasionally blunt rupture o the heart, particularly the atrial appendage, isseen.Acutely,<100mLo pericardialblood maycausepericardial tamponade. he classic Beck’s triad—dilated neck veins,mu led heart tones,and adeclinein arterialpressure— is usually not appreciated in the trauma bay because o the noisy environment and associated hypovolemia. Diagnosis is best achieved by bedside ultrasound o the pericardium, which is one o the our views o the FAS examination. (See Schwartz10th ed.,p.166.)
Answer:B
Injurieso thetracheaarerepaired with arunning3-0absorbable mono ilament suture. racheostomy is not required in most patients. Esophageal injuries are repaired in a similar ashion.I an esophagealwound islargeor i tissueismissing, a sternocleidomastoid muscle pedicle lap is warranted, and a closed suction drain is a reasonable precaution. he drain should be near but not in contact with the esophageal or any other sutureline.Itcan beremovedin 7to10daysi thesuture line remains secure. Care must be taken when exploring the tracheaand esophagusto avoid iatrogenicinjuryto therecurrent laryngealnerve.(SeeSchwartz10th ed.,p.202.)
Answer:D
heutilityo resuscitativethoracotomy(R )hasbeendebated or decades.Current indicationsarebased on 30yearso prospective data, supported by a recent multicenter prospective study.R isassociated with thehighest survivalratea ter isolated cardiac injury; 35%o patients presenting in shock and 20% without vital signs (ie, no pulse or obtainable BP) are salvaged a ter isolated penetrating injury to the heart. For all penetrating wounds, survival rate is 15%. Conversely, patient outcome is poor when R is done or blunt trauma, with 2% survival among patients in shock and <1% survival among those with no vital signs. hus, patients undergoing cardiopulmonaryresuscitation (CPR)upon arrivaltotheEDshould undergo R selectively based on injury and transport time. (SeeSchwartz10th ed.,p.167.)
7. Apatient with spontaneous eye opening, who is con used and localizespain hasaGlasgowComaScore(GCS)o
A. 9
B. 11
C. 13
D. 15
Answer:C
he Glasgow Coma Score (GCS) should be determined or all injured patients ( able 7-1). It is calculated by adding the scores o the best motor response, best verbal response, and eyeopening.Scoresrange rom 3(thelowest) to 15(normal). Scores o 13 to 15 indicate mild head injury, 9 to 12 moderate injury,and less than 9 severe injury. he GCSis use ul or both triage and prognosis. (See Schwartz 10th ed., able 7-3, pp.168and 170.)
43
8. Neckinjuries
A. Lessthan 15%penetratinginjuriesrequireneckexploration,amajoritycan bemanaged conservatively.
B. Dividedintothreezones,withzoneIabovetheangleo the mandible, zone II between the thoracic outlet and angle o mandible, and zone III in erior to the clavicles.
C. Allpatientswith neck injuryshould receive computed tomographyangiogram (C A)o theneck.
D. Patients with dysphagia, hoarseness, hematoma, venousbleeding,hemoptysis,or subcutaneousemphysemashould undergoneckexploration.
Answer:A
Zone I is in erior to the clavicles encompassing the thoracic outletstructures,zoneIIisbetween thethoracicoutletandthe angle o the mandible, and zone III is above the angle o the mandible. Patients with symptomatic zone I and III injuries should ideally undergo diagnostic imaging be ore operation i they remain hemodynamically stable. Speci ic symptoms which indicate urther imaging include dysphagia, hoarseness, hematoma, venous bleeding, minor hemoptysis, and subcutaneous emphysema. Symptomatic patients should undergo C A with urther evaluation or operation based upon theimaging indings;lessthan 15%o penetratingcervicaltrauma requires neck exploration. Asymptomatic patients are typically observed or 6 to 12 hours. he one caveat is asymptomatic patients with a transcervical gunshot wound; these patients should undergo C A to determine the track o the bullet. C A o the neck and chest determines trajectory o the injury tract; urther studies are per ormed based on proximity to major structures. Angiographic diagnosis, particularlyo zoneIII injuries,can then bemanaged byselective angioembolization.(SeeSchwartz10th ed.,p.177.)
9. Appropriatesurgicalmanagemento athrough-and-through gunshot wound to the lung with minimal bleeding and someair leakis
A. Chest tubeonly
B. Oversewing entrance and exit wounds to decrease the air leak
C. Pulmonary tractotomy with a stapler and oversewing o vesselsor bronchi
D. Wedgeresection o theinjured lung
Answer:C
Pulmonary injuries requiring operative intervention usually result rom penetrating injury. Formerly the entrance and exit wounds were oversewn to control hemorrhage. his set thestage or air embolism,which occasionallycaused sudden death in the operating room or in the immediate postoperative period. A recent development, pulmonary tractotomy, hasbeen employed to reducethisproblem aswellasthe need or pulmonary resection. Linear stapling devices are inserted directlyinto the injurytract and positioned to cause the least degree o devascularization. wo staple lines are created and the lung is divided between. his allows direct access to the bleedingvesselsandleakingbronchi.Noe ortismadetoclose thede ect.Lobectomyor pneumonectomyisrarelynecessary. Lobectomy is only indicated or a completely devascularized
44
TABLE7-1
Adults Infants/Children Eyeopening 4 Spontaneous Spontaneous 3 Tovoice Tovoice 2 Topain Topain 1 None None Verbal 5 Oriented Alert,normalvocalization 4 Conused Cries,butconsolable 3 Inappropriatewords Persistentlyirritable 2 Incomprehensiblewords Restless,agitated,moaning 1 None None Motorresponse 6 Obeyscommands Spontaneous,purposeul 5 Localizespain Localizespain 4 Withdraws Withdraws 3 Abnormal exion Abnormal exion 2 Abnormalextension Abnormalextension 1 None None aScore is
scoreso the best motorresponse,best verbalresponse,and eye opening.Scoresrange rom 3(the lowest)to 15(normal).
Glasgow Coma Scalea
calculated byadding the
10. What istrueregardingtheevaluation o blunt abdominal trauma?
A. Patients with abdominal wall rigidity and negative abdominalC should undergo diagnostic peritoneal lavage(DPL)toruleout smallbowelinjury.
B. I FAS examination is negative in a hemodynamically unstable patient then DPL is indicated to rule out abdominalbleeding.
C. FAS examination cannot detect intraperitoneal uid i thetotalvolumeis<1000mL.
D. Bowel injury can be ruled out in hemodynamically stablepatientswith abdominalC scanning.
or destroyed lobe. Parenchymal injuries severe enough to requirepneumonectomyarerarelysurvivable,and major pulmonaryhilar injuries necessitatingpneumonectomyare usuallylethalin the ield.(SeeSchwartz10th ed.,p.202.)
Answer:B he presence o abdominal rigidity and hemodynamic compromise is an undisputed indication or prompt surgical exploration. Blunt abdominal trauma is evaluated initiallyby FAS examination in most major traumacenters,and thishas largely supplanted diagnostic peritoneal lavage (DPL). FAS is not 100%sensitive, however, so diagnostic peritoneal aspiration is warranted in hemodynamically unstable patients without a de ined source o blood loss to rule out abdominal hemorrhage. his method is exquisitely sensitive or detecting intraperitoneal luid o >250 mL. Patients with luid on FAS examination, considered a “positive FAS ,”who do not have immediate indications or laparotomy and are hemodynamicallystable undergo C scanning to quanti ytheir injuries.C also isindicated or hemodynamicallystablepatients or whom the physical examination is unreliable. Despite the increasing diagnostic accuracy o multidetector C scanners, identi ication o intestinal injuries remains a limitation. Patients with ree intra-abdominal luid without solid organ injury are closely monitored or evolving signs o peritonitis; i patients have a signi icant closed head injury or cannot be seriallyexamined,DPLshouldbeper ormedtoexcludebowel injury. A ter placement o the catheter, a 10-mL syringe is connected and the abdominal contents aspirated (termed a diagnostic peritoneal aspiration). he aspirate is considered to show positive indings i >10 mL o blood is aspirated. I <10 mLis withdrawn, a liter o normal saline is instilled. he e luent iswithdrawn viasiphoningand sent to thelaboratory or redbloodcell(RBC)count,whitebloodcell(WBC)count, and determination o amylase, bilirubin, and alkaline phosphatase levels. See able 7-2 or values representing positive indings.(SeeSchwartz10th ed., able7-6,pp.179and 181.)
45
TABLE7-2
Abdominal Trauma Thoracoabdominal StabWounds Redbloodcellcount >100,000/mL >10,000/mL Whitebloodcellcount >500/mL >500/mL Amylaselevel >19IU/L >19IU/L Alkalinephosphatase level >2IU/L >2IU/L Bilirubinlevel >0.01mg/dL >0.01mg/dL
Criteria or“positive”fnding on diagnostic peritoneallavage
11. A er an automobile accident, a 30-year-old woman is discovered to have a posterior pelvic racture. Hypotension and tachycardia respond marginally to volume replacement.Onceit isevident that her major problem is ree intraperitoneal bleeding and a pelvic hematoma in association with the racture, appropriate management would be
A. Application o medical antishock trousers with in ation o theextremityand abdominalsections.
B. Arterialembolization o thepelvicvessels.
C. Celiotomy and ligation o the internal iliac arteries bilaterally.
D. Celiotomyand pelvicpacking.
E. External xation application tostabilizethepelvis.
12. Which istrueo vascular injurieso theextremities?
A. In the absence o hard signs o vascular injury, i the diference between SBP in an injured limb is within 15% o the uninjured limb, no urther evaluation is needed.
B. Occult pro unda emoris injuries can result in compartment syndromeand limbloss.
C. Allpatientswith signi cant hematomashould besurgicallyexplored.
D. Vascular injury repair should be per ormed prior to realignment o bony racturesor dislocations.
Answer:D
Severe pelvic bleeding is a major problem in the trauma patient. Neither external ixation nor the use o medical antishock trousers control ree intra-abdominal hemorrhage regardless o its source. In the unstable patient, celiotomy is mandatory. I there is a ruptured retroperitoneal hematoma bleeding into the peritoneal cavity, control is a major problem. Internal iliac artery ligation has been abandoned as it is rarely e ective. Angiography and arterial embolization may bee ectivewith an arterialbleedingproblem,but most severe pelvichemorrhageisvenousin origin.I thehematomaisstable,itisbesttoleaveitundisturbed.However,i thehematoma has ruptured into the peritoneal cavity, pelvic packing o ers thebest hopeo control.(SeeSchwartz10th ed.,p.181.)
Answer:B
Physical examination o ten identi ies arterial injuries, and indings are classi ied as either hard signs or so t signs o vascular injury ( able 7-3). In general, hard signs constitute indications or operative exploration, whereas so t signs are indications or urther testing or observation. Bony ractures or kneedislocationsshould berealigned be orede initivevascular examination. In management o vascular trauma, controversy exists regarding the treatment o patients with so t signs o injury, particularly those with injuries in proximity to major vessels. It is known that some o these patients will have arterial injuries that require repair. he most common approach hasbeen tomeasureSBPusingDoppler ultrasonographyand comparethevalue or theinjured sidewith that or the uninjured side, termed the A-Aindex. I the pressures are within 10%o each other, a signi icant injury is unlikely and no urther evaluation is per ormed. I the di erence is >10%, C A or arteriography is indicated. Others argue that there are occult injuries, such as pseudoaneurysms or injuries o the pro unda emoris or peroneal arteries, which may not be detectedwiththistechnique.I hemorrhageoccurs rom these injuries, compartment syndrome and limb loss may occur. Although busy trauma centers continue to debate this issue, the surgeon who is obliged to treat the occasional injured patient may be better served by per orming C A in selected patients with so t signs. (See Schwartz 10th ed., able 7-8, pp.181and 185.)
TABLE7-3
HardSigns(Operation Mandatory)
Pulsatilehemorrhage
Absentpulses
SoftSigns(FurtherEvaluation Indicated)
Proximitytovasculature
Signifcanthematoma
Acuteischemia Associatednerveinjury
A-Aindexo<0.9
Thrillorbruit
46
Signsand symptomso peripheralarterial injury
A-Aindex= systolicblood pressure on the injured side compared with that on the uninjured side.
13. Which o the ollowing statements about blunt carotid injuriesistrue?
A. Magneticresonanceimagingisthediagnosticmodalityo choicein patientsat risk.
B. Approximately 50% o patients have a delayed diagnosis.
C. Te mechanism o injury is usually cervical exion and rotation.
D. Such injuries are always treated operatively when identi ed.
Answer:B
Blunt injuryto thecarotid or vertebralarteriesmaycausedissection,thrombosis,or pseudoaneurysm.More than one hal o patients have a delayed diagnosis. Facial contact resulting in hypertension and rotation appears to be the mechanism. o reduce delayed recognition, the authors employ C A in patients at risk, to identi y these injuries be ore neurologic symptomsdevelop. heinjuries requentlyoccur at or extend into the base o the skull and are usuallynot surgicallyaccessible. Currently accepted treatment or thrombosis and dissection is anticoagulation with heparin ollowed by war arin or 3 months. Pseudoaneurysms also occur near the base o the skull. I they are small, they can be ollowed with repeat angiography. I enlargement occurs, consideration should be given to the placement by an interventional radiologist o a stent across the aneurysm. Another possibilityis to approach theintracranialportion o thecarotid byremovingtheoverlyingboneandper ormingadirectrepair. hismethodhasonly recently been described and has been per ormed in a limited number o patients.(SeeSchwartz10th ed.,p.198.)
14. Massivetrans usion protocols
A. Should include trans usion o plasma and platelets in addition topacked RBCs
B. Should onlybeinitiated a er blood typing,but crossmatch isnot needed
C. Should be initiated in patients with tachycardia despiteadministration o 3.5Lo crystalloid uids
D. Should include testing or coagulopathies, present in 5%o patientsrequiringmassivetrans usion
Answer:A
In the critically injured patient requiring large amounts o blood component therapy, a massive trans usion protocol should be ollowed. his approach calls or administration o various components in a speci ic ratio during trans usion to achieve restoration o blood volume and correction o coagulopathy. Although the optimal ratio is yet to be determined,current scienti icevidenceindicatesapresumptive1:2 RBC: plasma ratio in patients at risk or massive trans usion. Because complete typing and cross-matching takes up to 45 minutes, patients requiring emergent trans usions are given type O,type-speci ic,or biologicallycompatible RBCs.Blood typing, and to a lesser extent cross-matching, is essential to avoid li e-threatening intravascular hemolytic trans usion reactions(Fig.7-3).
Injured patients with li e-threatening hemorrhage develop an acutecoagulopathyo trauma(ACO ).Activated protein C is a key element, although the complete mechanism remains to be elucidated. Fibrinolysis is an important component o the ACO ; present in only 5% o injured patients requiring hospitalization, but 20% in those requiring massive trans usion.(SeeSchwartz10th ed.,Figure7-32,pp.184and 186.)
47
Massive Transfusion Protocol
Trigger: Uncontrolled hemorrhage
• eg, SBP <90 mm Hg Despite 3½ LCrystalloid (50 mL/kg)
• eg, EBL>150 mL/min
• eg, pH<7.1; body temperature <34°C; ISS >25
Surgery &Anesthesia Response
Continued TreatmentofShock
Hemorrhage Control
Correct Hypothermia
Correct Acidosis
Normalize Ca++
Check labs q30 min as needed
Ongoing ComponentTherapy
PT, PTT >1.5 control 2 units thawed plasma
rapidTEG-ACT >110 sec 2 units thawed plasma
Platelet count <50,000/mcL 1 unit ofapheresis platelets
rapidTEG-MA<55mm 1 unit ofapheresis platelets
Fibrinogen <100 mg/dL 10 units pooled cryoprecipitate
rapidTEG-angle <63 degrees 10 units pooled cryoprecipitate
rapidTEG EPL>15% 5 g amicar
Blood BankResponse
Shipments are delivered every 30 min untilMassive Transfusion Protocolis terminated. Each shipment’s quantity can be doubled at the request ofSurgery or Anesthesia. Shipments >4 are determined by patient’s clinicalcourse and lab values.
DenverHealth MedicalCenter’sMassive Trans usion Protocol.ACT= activated clotting time;Cryo = cryoprecipitate;EPL= estimated percent lysis;FFP= resh rozen plasma;INR= internationalnormalized ratio;MA= maximum amplitude;PRBCs= packed red blood cells; PT= prothrombin time;PTT= partialthromboplastin time;SBP= systolicblood pressure;TEG= thromboelastography.
FIG.
15. Te most appropriate treatment or a duodenal hematomathat occurs rom blunt traumais
A. Exploratory laparotomy and bypass o the duodenum.
B. Exploratory laparotomy and evacuation o the hematoma.
C. Exploratory laparotomy to rule out associated injuries.
D. Observation.
Answer:D
Duodenal hematomas are caused by a direct blow to the abdomen and occur more o ten in children than adults. Blood accumulates between the seromuscular and submucosal layers, eventuallycausingobstruction. he diagnosis is suspected by the onset o vomiting ollowing blunt abdominal trauma; barium examination o the duodenum reveals either the coiled spring sign or obstruction. Most duodenal hematomas in children can be managed nonoperativelywith nasogastric suction and parenteral nutrition. Resolution o the obstruction occurs in the majority o patients i this therapyis continued or 7 to 14 days. I surgical intervention becomes necessary, evacuation o the hematoma is associated with equal success but ewer complications than bypass procedures. Despite ew existing data on adults, there is no reason to believe that their hematomas should be treated di erently rom those o children. A new approach is laparoscopic evacuation i the obstruction persists more than 7 days.(SeeSchwartz10th ed.,p.207.)
48
Shipment PRBCs FFP Platelets Cryo 1 4 2 3 4 4 4 4 2 2 2 2 1 1 10 10
7-3.
16. Damagecontrolsurgery(DCS)
A. Limits enteric spillage byrapid repair o partialsmall bowelinjurieswithwhipstitch,andcompletetransection with aGIAstaplingdevice.
B. Aims to control surgical bleeding and identi y injuriesthat can bemanaged conservativelyor with interventionalradiology.
C. Is indicated when patients develop intraoperative re ractory hypothermia, serum pH >7.6, or re ractorycoagulopathy.
D. Abdominal wall should be closed with penetrating towelclips.
Answer:A hegoalo damagecontrolsurgery(DCS) isto controlsurgicalbleedingand limit gastrointestinal(GI) spillage. heoperativetechniquesused aretemporarymeasures,with de initive repair o injuries delayed until the patient is physiologically replete. Small GI injuries (stomach, duodenum, small intestine, and colon) may be controlled using a rapid whipstitch o 2-0 polypropylene. Complete transection o the bowel or segmental damage is controlled using a GIA stapler, o ten with resection o the injured segment. Be ore the patient is returned to the surgical intensive care unit (SICU), the abdomen mustbetemporarilyclosed.Originally,penetratingtowel clipswereused to approximatetheskin;however,theensuing bowel edema o ten produces a delayed abdominal compartment syndrome. Instead, the bowel is covered with a enestrated sub ascial sterile drape (45 × 60 cm Steri-Drape 3M Health Care), and two Jackson-Pratt drains are placed along the ascial edges; this is then covered using an Ioban drape, which allows closed suction to control reper usion-related ascitic luid egress while providing adequate space or bowel expansion to prevent abdominal compartment syndrome. (SeeSchwartz10th ed.,p.193.)
17. Terapy or increased intracranial pressure (ICP) in a patient with a closed head injury is instituted when the ICPisgreater than
A. 10
B. 20
C. 30
D. 40
18. Cerebralper usion pressure(CPP)
A. EqualstheSBPminusICP
B. Should betargeted tobegreater than 100mm Hg
C. Is lowered with sedation, osmotic diuresis, paralysis, ventricular drainage,and barbituratecoma
D. Can be increased by lowering ICP and avoiding hypotension
Answer:B
In patients with abnormal indings on C scans and GCS scoreso ≤8,intracranialpressure(ICP) should bemonitored using iberopticintraparenchymaldevicesor intraventricular catheters.Although an ICP o 10mm Hgisbelieved to be the upper limit o normal, therapy generally is not initiated until ICP is >20 mm Hg. Indications or operative intervention to remove space-occupying hematomas are based on the clot volume, amount o midline shi t, location o the clot, GCS score, and ICP. A shi t o >5 mm typically is considered an indication or evacuation,but thisisnot an absoluterule.(See Schwartz10th ed.,p.195.)
Answer:D
he goal o resuscitation and management in patients with head injuries is to avoid hypotension (SBP o <100 mm Hg) andhypoxia(partialpressureo arterialoxygen o <60or arterialoxygen saturation o <90).Attention,there ore,is ocused on maintaining cerebral per usion rather than merely lowering ICP. Resuscitation e orts aim or a euvolemic state and an SBP o >100 mm Hg. Cerebral per usion pressure (CPP) is equal to the mean arterial pressure minus the ICP, with a target range o >50 mm Hg. CPP can be increased by either lowering ICP or raising mean arterial pressure. Sedation, osmoticdiuresis,paralysis,ventricular drainage,and barbiturate coma are used in sequence, with coma induction being thelast resort.(SeeSchwartz10th ed.,p.195.)
49
19. An 18-year-old man is admitted to the ED shortly a er beinginvolvedin an automobileaccident.Heisin acoma (GCS = 7). His pulse is barely palpable at a rate o 140 beats per minute, and BP is 60/0. Breathing is rapid and shallow, aerating both lung elds. His abdomen is moderately distended with no audible peristalsis. Tere are closed ractureso theright orearmandthele lowerleg.
A er rapid IV administration o 2 L o lactated Ringer solution in theupper extremities,hispulseis130and BP 70/0.Tenext immediatestep should beto
A. Obtain cross-tablelateralX-rayso thecervicalspine.
B. Obtain head and abdominalC scans.
C. Obtain supine and lateral decubitus X-rays o the abdomen.
D. Obtain an arch aortogram.
E. Exploretheabdomen.
20. A 36-year-old patient arrives in the trauma bay with a stabwound tothele chest.A er placement o ale thoracostomy tube and uid resuscitation, his breathing is stablewith BP 160/74mm Hgand heart rateo 110beats per minute. C scanning reveals a descending thoracic pseudoaneurysm and nointracranialor intra-abdominal injury.What isthemost appropriatenext step?
A. Open repair with partialle heart bypass
B. Endovascular repair with stent
C. Esmololdrip
D. Admission toSICUwith repeat C in 24hours
21. A patient with penetrating injury to the chest should undergothoracotomyi
A. Tere is more than 500 mL o blood which drains rom thechest tubewhen placed.
B. Tere is more than 200 mL/h o blood or 3 hours rom thechest tube.
C. Tereisan air leakthat persists or >48hours.
D. Tereisdocumented lunginjuryon C scan.
Answer:E
Ideally, a patient seriously injured in an automobile accident should undergoX-rayso thecervicalspine,thechest,and the abdomen. When he has a GCSo 7, C scans o the head are certainlydesirable. I the chest X-rayshows a widened mediastinum,arch aortogramsareindicated.However,thispatient has had no response to a rapid luid challenge, and i he is to survive, bleeding must be controlled immediately. he head injury, although severe, is not responsible or his hypotension and tachycardia. hemost likelyproblem isuncontrolled abdominal hemorrhage. Immediate abdominal exploration o ers the best chance or survival. (See Schwartz 10th ed., p.164.)
Answer:C
Descending thoracic aortic injuries may require urgent i not emergent intervention. However, operative intervention or intracranial or intra-abdominal hemorrhage or unstable pelvic ractures takes precedence. o prevent aortic rupture, pharmacologictherapywith aselectiveβ1 antagonist,esmolol, should be instituted in the trauma bay, with a target SBP o <100 mm Hg and heart rate o <100 beats per minute. Endovascular stenting is now the mainstay o treatment, but open operative reconstruction is warranted, or necessary, in select patients. Endovascular techniques are particularly appropriate in patients who cannot tolerate single lung ventilation, patientsolder than 60yearswhoareat risk or cardiacdecompensation with aorticclamping,or patientswith uncontrolled intracranialhypertension.(SeeSchwartz10th ed.,p.200.)
Answer:B
hemost common injuries rom both blunt and penetrating thoracic trauma are hemothorax and pneumothorax. More than 85%o patients can be de initively treated with a chest tube. he indications or thoracotomy include signi icant initial or ongoing hemorrhage rom the tube thoracostomy and speci ic imaging-identi ied diagnoses. One caveat concerns the patient who presents a ter a delay. Even when the initial chest tube output is 1.5 L, i the output ceases and the lungisre-expanded,thepatient maybemanaged nonoperatively, i hemodynamically stable ( able 7-4). (See Schwartz 10th ed., able7-10,p.200.)
TABLE7-4 Indications oroperative treatment o thoracicinjuries
•Initialtubethoracostomydrainageo>1000mL(penetratinginjury)or>1500mL(bluntinjury)
•Ongoingtubethoracostomydrainageo>200mL/hor3consecutivehoursinnoncoagulopathic patients
•Cakedhemothoraxdespiteplacementotwochesttubes
•Selecteddescendingtornaortas
•Greatvesselinjury(endovasculartechniquesmaybeusedinselectedpatients)
•Pericardialtamponade
•Cardiacherniation
•Massiveairleakromthechesttubewithinadequateventilation
•Trachealormainstembronchialinjurydiagnosedbyendoscopyorimaging
•Openpneumothorax
•Esophagealperoration
•Airembolism
50
22. A er sustainingagunshotwoundtotherightupper quadranto theabdomen,thepatienthasnosignso peritonitis. Her vital signs are stable, and C scan shows a grade III liver injury.Whatisthenextstepin management?
A. Exploratory laparotomy with control o hepatic parenchymalhemorrhage.
B. Admission toSICUwith serialcompleteblood count.
C. Admission toSICUwith repeat C in 24hours.
D. Hepaticangiography.
23. A25-year-old man hasmultipleintra-abdominalinjuries a er a gunshot wound. Celiotomy reveals multiple injuriestosmalland largeboweland major bleeding rom the liver. A er repair o the bowel injuries, the abdomen is closed with towelclips,leavingalargepackin theinjured liver.Within 12hours,there ismassive abdominalswelling with edema uid, and intra-abdominal pressure exceeds35mm Hg.Teimmediatestep in managingthis problem isto
A. Administer albumin intercavernously
B. Givean IVdiuretic
C. Limit IV uid administration
D. Open theincision todecompresstheabdomen
24. Which o the ollowing statements is correct regarding traumaticspleen injury?
A. An elevation in WBC to 20,000/mm3 and platelets to 300,000/mm3 on postoperative day 7 is a common benign ndingin postsplenectomypatients.
B. Delayed rebleeding or rupture will typically occur within 48hourso injury.
C. Common complications a er splenectomy include subdiaphragmatic abscess, pancreatic tail injury, and gastricper oration.
D. Postsplenectomy vaccines against encapsulated bacteria is optimally administered preoperatively or immediatelypostoperative.
Answer:B
he liver’s large size makes it the organ most susceptible to blunt trauma,and it is requentlyinvolved in upper torsopenetrating wounds. Nonoperative management o solid organ injuriesispursuedinhemodynamicallystablepatientswhodo not haveovert peritonitisor other indications or laparotomy. Patients with more than grade II injuries should be admitted to the SICU with requent hemodynamic monitoring, determination o hemoglobin, and abdominal examination. he only absolute contraindication to nonoperative management ishemodynamicinstability.Factorssuch ashigh injurygrade, large hemoperitoneum, contrast extravasation, or pseudoaneurysmsmaypredict complicationsor ailureo nonoperative management. Angioembolization and endoscopic retrograde cholangiopancreatography (ERCP) are use ul adjuncts that can improve the success rate o nonoperative management. he indication or angiography to control hepatic hemorrhage is trans usion o 4 units o RBCs in 6 hours or 6 units o RBCs in 24 hours without hemodynamic instability. (See Schwartz10th ed.,p.203.)
Answer:D
Cardiac, pulmonary, and renal problems develop when invasive ascites compresses the diaphragm and the in erior vena cava. Dialysis, diuresis, and increasing serum oncotic pressure will not correct this problem rapidly enough to save the patient’sli e.Openingtheincisionrelievestheintra-abdominal pressure. here are ew reports o sudden hypotension a ter thismaneuver,but volumeloadinghaslargelyeliminated this problem.(SeeSchwartz10th ed.,p.217.)
Answer:C
Unlikehepaticinjuries,which usuallyrebleed within 48hours, delayed hemorrhage or rupture o the spleen can occur up to weeks a ter injury. Indications or early intervention include initiation o blood trans usion within the irst 12 hours and hemodynamic instability. A ter splenectomy or splenorrhaphy,postoperativehemorrhagemaybeduetolooseningo atie around the splenic vessels, an improperly ligated or unrecognizedshortgastricartery,or recurrentbleeding rom thespleen i splenic repair was used. An immediate postsplenectomy increase in platelets and WBCs is normal; however, beyond postoperative day 5, a WBC count above 15,000/mm3 and a platelet/WBC ratio o <20 are strongly associated with sepsis and should prompt athorough search or underlyingin ection. Acommon in ectiouscomplication a ter splenectomyisasubphrenic abscess, which should be managed with percutaneous drainage.Additionalsourceso morbidityincludeaconcurrent but unrecognized iatrogenic injury to the pancreatic tail duringrapid splenectomyresultingin pancreatic ascites or istula, and a gastric per oration during short gastric ligation. Enthusiasm or splenicsalvagewasdriven bytherare,but o ten atal, complication o overwhelming postsplenectomy sepsis. Overwhelming postsplenectomy sepsis is caused by encapsulated bacteria, Streptococcus pneumoniae, Haemophilus influenzae,
51
25. Te most appropriate treatment or a duodenal hematomathat occurs rom blunt traumais
A. Exploratorylaparotomyand bypasso theduodenum
B. Exploratory laparotomy and evacuation o the hematoma
C. Exploratory laparotomy to rule out associated injuries
D. Observation
26. A19-year-old man ellof hisskateboard,reportingblunt injury to his upper abdomen. Abdominal C and magnetic resonance cholangiopancreatography (MRCP) con rmed hesufered transection o themain pancreatic duct at the middle o the pancreatic body. Which o the ollowing would be the most appropriate next step in management?
A. Nonoperativetreatment
B. Endoscopic retrograde cholangiopancreatography (ERCP)with stentingo pancreaticduct
C. Distalpancreatectomywith splenicpreservation
D. Primaryrepair o pancreaticduct with closed suction drainage
and Neisseria meningitidis,which areresistant to antimicrobial treatment. In patients undergoing splenectomy, prophylaxis against these bacteria is provided via vaccines administered optimallyat14days.(SeeSchwartz10thed.,p.206.)
Answer:D
Duodenalhematomasarecaused byadirect blowtotheabdomenandoccurmoreo teninchildrenthanadults.Bloodaccumulates between the seromuscular and submucosal layers, eventuallycausingobstruction. hediagnosisissuspected by theonset o vomiting ollowingblunt abdominaltrauma;barium examination o the duodenum reveals either the coiled springsign or obstruction.Mostduodenalhematomasin children can be managed nonoperatively with nasogastric suction and parenteral nutrition. Resolution o the obstruction occurs in the majority o patients i this therapy is continued or 7 to 14 days. I surgical intervention becomes necessary, evacuation o the hematoma is associated with equal success but ewer complications than bypass procedures. Despite ew existing data on adults, there is no reason to believe that their hematomas should be treated di erently rom those o children. A new approach is laparoscopic evacuation i the obstruction persistsmorethan 7days.(SeeSchwartz10th ed., p.207.)
Answer:C
Optimal management o pancreatic trauma is determined by where the parenchymal damage is located and whether the intrapancreatic common bile duct and main pancreatic duct remain intact. Patients with pancreatic contusions (de ined as injuries that leave the ductal system intact) can be treated nonoperatively or with closed suction drainage i undergoing laparotomy or other indications. Patients with proximal pancreaticinjuries,de ined asthosethat lieto theright o the superior mesentericvessels,arealsomanagedwithclosedsuction drainage. In contrast, distal pancreatic injuries are managed based upon ductal integrity. Pancreatic duct disruption can be identi ied through direct exploration o the parenchymal laceration, operative pancreatography, endoscopic retrograde pancreatography (ERCP), or magnetic resonance cholangiopancreatography(MRCP).Patientswith distalductaldisruption undergo distalpancreatectomy,pre erablywith splenic preservation. An alternative, which preserves both the spleen and distal transected end o the pancreas, is either a Roux-en-Y pancreaticojejunostomy or pancreaticogastrostomy. I the patient is physiologically compromised, distal pancreatectomy with splenectomy is the pre erred approach. (SeeSchwartz10th ed.,p.207.)
27.
A. End colostomyand mucous stula.
B. Loop colostomy.
C. Exteriorized repair.
D. Resection o theright colon with ileocolostomy.
Answer:D
Numerous large retrospective and several prospective studies have now clearly demonstrated that primary repair is sa e and e ective in the majority o patients with penetrating injuries. Colostomy is still appropriate in a ew patients, but the current dilemma is how to select them. Exteriorized repair isprobablynolonger indicatedsincemost patientswho were once candidates or this treatment are now success ully managed by primary repair. wo methods have been advocated that result in 75 to 90% o penetrating colonic injuries
52
Te most appropriate treatment or a gunshot wound to the hepatic exure o the colon that cannot be repaired primarilyis
28. Which o the ollowing statements is FALSE regarding traumaticgenitourinaryinjury?
A. I exploratorylaparotomyisper ormed or trauma,all blunt and penetrating wounds to the kidneys should beexplored.
B. Renalvascular injuriesarecommon a er penetrating trauma, and can be deceptively tamponaded by surrounding ascia.
C. Success o renal artery repair a er blunt trauma is slim,but can beattempted i injuryoccurred within 5 hoursor patientdoesnothaveanyreserverenal unction (solitarykidneyor bilateralinjury).
D. Suspected ureteral injuries in patients with penetratingtraumaor pelvic racturescan beevaluated intraoperatively with methylene blue or indigo carmine administered intravenously.
E. Bladder injuries with extraperitoneal extravasation can be managed with Foley decompression or 2 weeks.
beingsa elytreated byprimaryrepair. he irst isto repair all per orations not requiring resection. I resection is required due to the local extent o the injury, and it is proximal to the middlecolicartery,theproximalportion o theright colon up to and including the injury is resected and an ileocolostomy per ormed. I resection is required distal to the middle colic artery, an end colostomyis created and the distal colon oversewn and le t within the abdomen. he theory behind this approach is that an ileocolostomy heals more reliably than colocolostomy, because in the trauma patient who has suered shock and maybe hypovolemic, assessingthe adequacy o the blood supply o the colon is much less reliable than in elective procedures. he blood supply o the terminal ileum is never a problem. he other approach is to repair all injuries regardless o the extent and location (including colocolostomy), and reserve colostomy or patients with protracted shock and extensive contamination. he theory used to support this approach is that systemic actors are more important than local actors in determining whether a suture line will heal. Both o these approaches are reasonable and result in the majority o patients being treated by primary repairs. When a colostomy is required, regardless o the theory used to reach that conclusion,per orminga loop colostomyproximal to a distal repair should be avoided because a proximal colostomydoesnot protect adistalsutureline.Allsuturelines and anastomosesareper ormed with therunningsingle-layer technique.(SeeSchwartz10th ed.,p.209.)
Answer:A
When undergoing laparotomy or trauma, the best policy is to explore all penetrating wounds to the kidneys. However, over 90% o blunt injuries are treated nonoperatively; the indications or surgery include parenchymal injuries leading to hypotension and evidence o renovascular injury. I laparotomy is per ormed in the setting o blunt kidney injury or other reasons,expandingor pulsatileperinephrichematomas should beexplored.Injuriesto theuretersareuncommon but may occur in patients with pelvic ractures and penetrating trauma. An injurymaynot be identi ied until a complication (ie, a urinoma) becomes apparent. I an injury is suspected during operative exploration but is not clearly identi ied, methylene blue or indigo carmine is administered IV with observation or extravasation. Bladder injuries are subdivided into those with intraperitoneal extravasation and those with extraperitoneal extravasation. Extraperitoneal ruptures are treated nonoperatively with bladder decompression or 2 weeks,whereasinjurieswithintraperitonealextravasation can beclosedprimarily.Urethralinjuriesaremanagedbybridging the de ect with a Foleycatheter, with or without direct suture repair.Stricturesarenotuncommon butcan bemanagedelectively.(SeeSchwartz10th ed.,p.211.)
53
29. At what pressure is operative decompression o a compartment mandatory?
A. 15mm Hg
B. 25mm Hg
C. 35mm Hg
D. 45mm Hg
30. Which istrueregardingtraumain geriatricpatients?
A. Admission GCS score a er severe head injury is a good predictor o outcome.
B. Rib ractures are associated with pulmonary contusion in 35%o patients,and complicated bypneumoniain 10to30%o patients.
C. Approximately 10% o patients older than 65 years willsustain arib racture rom a all<6 .
D. Chronologicageolder than 65yearsisassociatedwith higher morbidityand mortalitya er trauma.
Answer:D
In comatose or obtunded patients, the diagnosis is more diicult to secure. A compatible history, irmness o the compartment to palpation, and diminished mobility o the joint aresuggestive. hepresenceor absenceo apulsedistalto the a ected compartment isnotoriouslyunreliablein thediagnosis o a compartment syndrome. A rozen joint and myoglobinuria are late signs and suggest a poor prognosis. As in the abdomen,compartmentpressurecan bemeasured. hesmall, hand-heldStryker deviceisaconvenienttool or thispurpose. Pressures greater than 45 mm Hg usually require operative intervention. Patients with pressures between 30 and 45 mm Hg should be care ully evaluated and closely watched. (See Schwartz10th ed.,p.215.)
Answer:B
Mortalityin patients with severe head injurymore than doubles a ter the age o 55 years. Moreover, 25%o patients with a normal GCSscore o 15 had intracranial bleeding, with an associated mortality o 50%. Just as there is no absolute age that predictsoutcome,admission GCSscoreisa poor predictor o individual outcome. here ore, the majority o trauma centers advocate an initial aggressive approach with reevaluation at the72-hour mark to determine subsequent care.Secondly, one o the most common sequelae o blunt thoracic trauma is rib ractures. In act, in one study, 50% o patients older than 65 years sustained rib ractures rom a all o <6 t, compared with only 1% o patients younger than 65 years. Concurrent pulmonary contusion is noted in up to 35% o patients,andpneumoniacomplicatestheinjuriesin 10to30% o patientswith rib ractures.(SeeSchwartz10th ed.,p.221.)
54
1. A22-year- ld man sbr ught t theemergen yr m a er a h use re.Hehasburnsar und h sm uth and h sv e sh arse,butbreath ng sunlab red.Whatm stappr pratenext step n management?
A. Immed ateend tra heal ntubat n.
B. Exam nat n ral av tyand pharynx,w th ber pt laryng s pe ava lable.
C. Pla e n supplemental xygen.
D. Pla ement tw large-b re ntraven us(IV) atheters w th u d resus tat n.
Answer:B
W thd re tthermal njuryt theupper a rway r sm ke nhalat n, rap d and severe a rway edema s a p tent ally lethal threat. Ant pat ng the need r ntubat n and establ sh ng an earlya rway s r t al.Per ralburnsand s nged nasalha r are s gns that the ral av ty and pharynx sh uld be urther evaluated r mu sal njury,butthesephys al nd ngsal ne d n t nd ate an upper a rway njury. Sgns mpend ng resp rat ry mpr m se may n lude a h arse v e, wheezng, r str d r;subje t vedyspnea sapart ularly n ern ng sympt m, and sh uld tr gger pr mpt ele t ve end tra heal ntubat n.In pat entsw th mb ned mult ple trauma,espeally ral trauma, nas tra heal ntubat n may be use ul but sh uld be av ded ral ntubat n s sa e and easy. (See S hwartz10th ed.,p.227.)
2. What per entage burn d es a pat ent have wh has suered burnst neleg( r um erent al), nearm ( r umerent al),and theanter r trunk?
A. 18%
B. 27%
C. 36%
D. 45%
Answer:D
Ageneral dea the burn s ze an be made byus ngthe rule n nes. Ea h upper extrem ty a unts r 9% the t tal b dysur a e area ( BSA), ea h l wer extrem tya unts r 18%, the anter r and p ster r trunk ea h a unts r 18%, theheadandne ka unt r 9%,andtheper neum a unts r 1%. Alth ugh the rule n nes s reas nably a urate r adults, a number m re pre se harts have been devel ped that arepart ularlyhelp ul n assess ngped atr burns.M st emergen y r ms have su h a hart. A d agram the burn an be drawn n the hart, and m re pre se al ulat ns the burn s ze made r m the a mpany ng BSA est mates g ven.
Ch ldren y unger than 4yearshavemu h larger headsand smaller th ghs n pr p rt n t t talb dys zethan d adults. In n ants the head a unts r nearly 20% the BSA; a h ld’s b dy pr p rt ns d n t ully rea h adult per entages unt lad les en e.Even when us ngpre sed agrams, nter bserver var at n mayvarybyas mu h as ±20%. An bserver’s exper en e w th burned pat ents, rather than edu at nal level, appears t be the best pred t r the a ura y burn s ze est mat n. F r smaller burns, an a urate assessment s ze an be made by us ng the pat ent’s palmar hand sur a e, n lud ng the d g ts, wh h am unts r appr x mately 1% BSA.(SeeS hwartz10th ed.,p.229.)
55
8
Burns
CHAPTER
3. A40-year- ld w man sadm tted t theburn un t a er an ndustr al reat aplast smanu a tur ngplant w th burns t the a eand arms.Her ele tr ard gram (ECG) sh ws
S- elevat n, and n t al hem stry panel and arter al bl d gasrevealan an n gap metab l a d s sw th n rmalarter al arb xyhem gl b n.What s the m st apprpr atenext step?
A. C rre t n a d s s by add ng s d um b arb nate t IV u ds.
B. Adm n strat n 100% xygenandhydr x balam n.
C. ransth ra e h ard gram.
D. Bl d ulturew th IVant b t s.
4. Wh h the ll w ng sa mm n sequelae ele tr al njury?
A. Card a arrhythm as
B. Paralys s
C. Bra n damage
D. Catara ts
Answer:B
Hydr gen yan det x tymayals bea mp nent sm ke nhalat n njury.A l tedpat entsmayhaveapers stentla t a d s s r S- elevat n n ECG.Cyan de nh b ts yt hr me x dase, wh h s requ red r x dat ve ph sph rylat n. reatment ns sts s d um th sul ate, hydr x balam n, and 100% xygen. S d um th sul ate w rks by trans rm ng yan de nt a n nt x th yanate der vat ve, but t w rks sl wlyand sn te e t ve ra utetherapy.Hydr x balam n qu kly mplexes w th yan de and s ex reted bythe k dney, and s re mmended r mmed ate therapy. In the maj rty pat ents, the la t a d s s w ll res lve w th vent lat n and s d um th sul ate treatment be mes unne essary. (See S hwartz10th ed.,p.228.)
Answer:D
My gl b nur a requently a mpan es ele tr al burns, but the l n al s gn an e appears t be tr v al. D srupt n mus le ells releases ellular debr s and my gl b n nt the r ulat n t be ltered by the k dney. I th s nd t n s untreated, the nsequen e an be rrevers ble renal a lure. H wever, m dern burn resus tat n pr t ls al ne appear t besu ent treatment r my gl b nur a.
Card a damage, su h as my ard al ntus n r n art n,maybe present.M rel kely,the ndu t n system may bederanged.H useh ld urrent at 110Ve ther d esn damage r ndu es ventr ular br llat n. I there are n eletr ard graph rhythm abn rmal t es present up n n t al emergen y department evaluat n, the l kel h d that they w ll appear later s m nus ule. Even w th h gh-v ltage njur es, a n rmal ard a rhythm n adm ss n generally means that subsequent dysrhythm a sunl kely.Stud es n rm that mm nly measured ard a enzymes bear l ttle rrelat n t ard a dys un t n, and elevated enzymes may be r m skeletalmus ledamage.Mandat ryECGm n t r ngand ard a enzymeanalys s n an ICU sett ng r 24h urs ll w ng njury s unne essary n pat ents w th ele tr al burns, even th se result ng r m h gh-v ltage urrent, n pat ents wh havestable ard a rhythms n adm ss n. he nerv us system s exqu s tely sens t ve t ele trty. he m st devastat ng njury w th requent bra n damage urswhen urrent passesthr ugh thehead,but sp nal rd damage sp ss blewhenever urrenthaspassed r m nes de the b dy t the ther. S hwann ells are qu te sus ept ble, and delayed transverse myel t s an ur days r weeks a ter njury. C ndu t n n t allyrema ns n rmal thr ugh ex st ng myel n, but as myel n wears ut, t s n t repla ed and ndu t n eases. Anter r sp nal artery syndr me r m vas ular dysregulat n an als pre p tatesp nal rd dys un t n. Damage t per pheral nerves s mm n and may ause permanent un t nal mpa rment.Everypat ent w th an ele tral njury must have a th r ugh neur l g exam nat n as part the n t alassessment.Pers stent neur l g sympt ms maylead t hr n pa n syndr mes,and p sttraumat stress d s rders are apparently m re mm n a ter ele tr al burns than thermalburns.
Catara ts are a well-re gn zed sequela h gh-v ltage ele tr alburns. hey ur n 5 t 7% pat ents, requently are b lateral, ur even n the absen e nta t p nts n
56
5. An 8-year- ld b y sbr ught t theemergen yr m a er a dentally t u h ng a h t r n w th h s rearm. On exam nat n, the burned area has weep ng bl sters and s verytender t thet u h.What stheburn depth?
A. F rst degree
B. Se nd degree
C. T rd degree
D. F urth degree
6. Tree h urs a er a burn njury that ns sted r umerent al, th rd-degree burns at the wr st and elb w the r ght arm,apat ent l sessensat n t l ght t u h n h s ngers.M t r un t n h sd g ts,h wever,rema ns nta t. Tem stappr pr atetreatment r th spat entn ww uld ns st
A. Elevat n the extrem ty, D ppler ultras n graphy every 4 h urs, and d stal pulses are absent 8 h urs later, mmed atees har t my.
B. Palpat n r d stalpulsesand mmed atees har t my pulsesareabsent.
C. D ppler ultras n graphy r assessment per pheral wand mmed atees har t my w sde reased.
D. Immed ate es har t my under general anesthes a r m ab ve the elb w t bel w the wr st n b th med aland lateralaspe ts thearm.
7. What s the lu d requ rement a 50-kgman w th rstdegree burns t h s le t arm and leg, r um erent al se nd-degree burn t h s r ght arm, and th rd-degree burns t h s t rs and r ght leg. What s the rate n t al u d resus tat n?
A. 4.5L ver 8h urs, ll wed by4.5L ver 16h urs
B. 4.5L ver 8h urs, ll wed by6L ver 16h urs
C. 6L ver 8h urs, ll wed by6L ver 16h urs
D. 6L ver 8h urs, ll wed by9L ver 16h urs
the head, and typ ally man est w th n 1 t 2 years njury. Ele tr ally njured pat ents sh uld underg a th r ugh phthalm l g exam nat n early dur ng the r a ute are. (See S hwartz10th ed.,p.229.)
Answer:B Burn w unds are mm nly lass ed as super al ( rst degree), part al th kness (se nd degree), ull th kness (th rd degree), and urth degree burns, wh h a e t underlyng s t t ssue.Part alth knessburnsare lass edase thersuper al r deep part alth knessburnsbydepth nv lved derm s.Cl nally, rst-degree burns are pa n ul but d n t blster, se nddegreeburnshavedermal nv lvementandareextremelypa n ul wth weep ng and bl sters, and th rd-degree burns are leathery, pa nless,andn nblan h ng.(SeeS hwartz10thed.,p.229.)
Answer:C
h rd-degree burn njur es are hara ter zed by alm st mplete l ss elast ty the sk n. hus, as s t t ssue swell ng pr gresses, neur vas ular mpr m se may ur. Fa lure t re gn zeth spr blem mayresult n thel ss d stalextremt es. he m st rel able s gns de reased per pheral bl d l w n burned pat entsaresl w ap llaryre llas bserved n the na l beds, the nset neur l g de ts, and de reased r absent D ppler ultras n pulse dete t n. When vas ular mpa rment s d agn sed, mmed ate es har t m es are ndated. Anesthes a s n t requ red r es har t my—the burn area s nsensate be ause sk n nerve end ngs are destr yed by th rd-degreeburns.(SeeS hwartz10th ed.,p.234.)
Answer:A
hem st mm nlyused rmula,theParkland r Baxter rmula, ns sts 3t 4mL/kg/%burn la tated R nger s lut n, wh h hal s g ven dur ng the rst 8 h urs p stburn, andtherema n nghal ver thesubsequent 16h urs. he neptbeh nd nt nu us lu drequ rements ss mple. heburn (and/ r nhalat n njury) dr ves an n lammat ry resp nse that leads t ap llary leak; as plasma leaks nt the extravasular spa e, rystall d adm n strat n ma nta nsthe ntravasular v lume. here re, apat ent re e vesalarge lu d b lus n apreh sp talsett ng remergen ydepartmentthat lu dhas l kelyleaked nt the nterst t um and the pat ent st llrequ res ng ng burn resus tat n a rd ng t the est mates. C nt nuat n lu d v lumes sh uld depend n the t me s n e njury, ur ne utput, and mean arter al pressure. As the leak l ses, the pat ent w ll requ re less v lume t ma nta n these tw resus tat n endp nts.(SeeS hwartz10th ed.,p.230.)
8. A pat ent w th a 90%burn en mpass ng the ent re t rs devel psan n reas ngP 2 and peak nsp rat rypressure. Wh h the ll w ng s m st l kely t res lve th s pr blem?
A. In reasethedel vered t dalv lume.
B. In reasetheresp rat ryrate.
C. In reasetheFio2.
D. Per rm ath ra es har t my.
Answer:D headequa y resp rat n must bem n t red nt nu usly thr ugh ut theresus tat n per d.Earlyresp rat ryd stress maybe due t the mpr m se vent lat n aused by hest wall nelast tyrelated t adeep r um erent alburn w und the th rax. Pressures requ red r vent lat n n rease and arter alPco2 r ses.Inhalat n njury,pneum th rax, r ther
57
9. Wh h the ll w ng s FALSE regard ng s lver sul ad az ne?
A. Used as pr phylax s aga nst burn w und n e t ns w th aw derange ant m r b ala t v ty.
B. Sa e t use n ull and part al th kness burn w unds, aswellassk n gra s.
C. Hasl m ted system abs rpt n.
D. May nh b t ep thel al m grat n n part al th kness w und heal ng.
auses an als result n resp rat ry d stress and sh uld be appr pr atelytreated.
h ra es har t my s seld m requ red, even w th a rum erent al hest wall burn. When requ red, es har t m es areper rmed b laterally n theanter r ax llaryl nes.I there ss gn ant extens n theburn nt theadja ent abd mnalwall,thees har t my n s nssh uld beextended t th s area by a transverse n s n al ng the stal marg ns. (See S hwartz10th ed.,p.230.)
Answer:B
Slver sul ad az ne s ne the m st w dely used n l n al pra t e. Slver sul ad az ne has a w de range ant m r b al a t v ty, pr mar ly as pr phylax s aga nst burn w und n et ns rather than treatment ex st ng n e t ns. It has the added bene ts be ng nexpens ve and eas ly appl ed, and hass th ngqual t es.It sn t s gn antlyabs rbedsystemally and thus has m n mal metab l derangements. Slver sul ad az nehasareputat n r aus ngneutr pen a,but th s ass at n sm rel kelyduet neutr ph lmarg nat n r m the n lammat ryresp nse. rueallerg rea t nst thesul a mp nent s lver sul ad az ne are rare,and at-r sk pat ents an have a small test pat h appl ed t dent y a burn ng sensat n r rash. Slver sul ad az ne destr ys sk n gra ts and s ntra nd ated n burns r d n r s tes n pr x m tyt newly gra ted areas. Als , s lver sul ad az ne may retard ep thel al m grat n n heal ngpart alth knessw unds.(See S hwartz 10th ed.,p.232.)
10. Su ess ul ant b t penetrat n a burn es har an be a h eved w th
A. Ma en dea etate
B. Ne my n
C. Slver n trate
D. Slver sul ad az ne
11. Wh h the ll w ng s true regard ng nutr t nal needs burn pat ents?
A. Te hypermetab l resp nse t burn w unds typallyra sesthebas metab l rateby120%.
B. Oxandr l ne, an anab l ster d, an mpr ve lean b dy mass but an be ass ated w th hypergly em a and l n allys gn antr se n hepat transam n t s.
C. Early enteral eed ng s sa e when burns are less than 20% BSA, therw se enteral eed ng sh uld awa t return b wel un t n t av d eed ng a pat ent w th gastr leus.
D. F r pat ents w th greater than 40% BSA, al r needs are est mated t be 25 k al/kg/day plus 40 k al/% BSA/day.
Answer:A Ma en de a etate s the ant b t agent that penetrates burn es har t rea h the nter a e w th the pat ent’s v able t ssue. h sagent hasthed sadvantagesthat t squ tepa n ul n any part al th kness areas, and t s a arb n anhydrase nh bt r that nter eresw th renalbu er ngme han sms.Chl r de s reta ned, and metab l a d s s results. F r these reas ns, s lver sul ad az ne s m re mm nly used n burn enters unless a maj r pr blem w th burn w und seps s s present. (SeeS hwartz10th ed.,p.232.)
Answer:D
he hypermetab l resp nse n burn njury may ra se basel ne metab l rates by as mu h as 200%. h s an lead t atab l sm mus le pr te ns and de reased lean b dy mass that may delay un t nal re very. Early enteral eedng r pat ents w th burns larger than 20% BSA s sa e, and may redu e l ss lean b dy mass, sl w the hypermetab l resp nse,and result n m ree ent pr te n metab l sm.
Cal ulat ng the appr pr ate al r needs the burn pat ent an be halleng ng. A mm nly used rmula n n nburned pat ents s the Harr s-Bened t equat n, wh h al ulates al r needs us ng a t rs su h as gender, age, he ght, and we ght. h s rmula uses an a t v ty a t r r spe njur es, and r burns, the basal energy expendture s mult pl ed bytw . he Harr s-Bened t equat n may be na urate n burns less than 40% BSA, and n these pat ents the Currer rmula may be m re appr pr ate. h s rmula est mates al r needs t be 25 k al/kg/day plus 40k al/% BSA/day.
58
12. A14-year- ld g rl susta ns a steam burn measur ng6 by 7 n hes ver the ulnar aspe t her r ght rearm. Bl sters devel p ver the ent re area the burn w und, and by the t me the pat ent s seen 6 h urs a er the njury, s me the bl sters have ruptured sp ntane usly. All the ll w ng therapeut reg mens m ght be ns dered appr pr ate r th spat ent EXCEP
A. Appl at n s lver sul ad az ne ream (Slvadene) and da lywashes,but n dress ng.
B. Appl at n ma en de a etate ream (Sul amyl n), but n da lywashes r dress ng.
C. H m gra appl at n w th ut suturest se ure t n pla e,but n da lywashes r dress ng.
D. Heter gra (p gsk n) appl at n w th sutures t se ure t n pla eand da lywashes,but n dress ng.
13. Wh h s FALSE n ern ng surg al treatment burn w unds?
A. angent al ex s n ns sts tangent al sl es burn t ssueunt lbleed ngt ssue sen untered.Tus, ex s n an beass ated w th p tent allys gn ant bl d l ss.
B. Human adaver all gra sa permanent alternat ve t spl t-th kness sk n gra s when there are nsufent d n r s tes.
C. Bleed ng r m tangent alex s n an be helped w th nje t n ep nephr ne tumes en e s lut n, pneumat t urn quets, ep nephr ne s aked mpresses, and br n gen and thr mb n spraysealant.
D. Meshed spl t th kness sk n gra s all w ser sangun us dra nage t prevent gra l ss and pr v de a greater area w und verage.
he anab l ster d xandr l ne has been extens vely stud ed n ped atr pat ents as well, and has dem nstrated mpr vements n lean b dymassand b nedens ty n severely burned h ldren. he we ght ga n and un t nal mpr vements seen w th xandr l ne may pers st even a ter st pp ng adm n strat n the drug. A re ent d uble-bl nded, rand m zed study xandr l ne sh wed de reased length stay, mpr ved hepat pr te n synthes s, and n adverse e e ts n the end r ne un t n, th ugh the auth rs n ted ar se n transam nasesw th un lear l n als gn an e.(See S hwartz10th ed.,p.232.)
Answer:D
A number d erent a eptable reg mens ex st r treat ng small, super al se nd-degree burn njur es. In all ases, the ne r t ep thel um s rst debr ded. p al ant bater al agents then may be appl ed and the w unds treated pen r l sed w th dress ngs hanged da ly r every ther day. B l g dress ngs (h m gra ts r heter gra ts) may be appl ed t super al se nd-degree burns at the t me n t al debr dement. yp ally, these dress ngs qu kly adhere t the w unds, rel eve pa n, and pr m te rap d epthel al zat n. hesedress ngssh uld n t besutured n pla e, h wever, be ause sutur ng reates the p tent al r a l sedspa e n e t n and r nvers n a se nd degree t a ull-th kness njury.I ab l g dress ngd esn t adhere, t sh uld be rem ved mmed ately,and the w und sh uld then be treated w th t p al ant ba ter al agents. (See S hwartz 10th ed.,p.234.)
Answer:B
he strategy earlyex s n and gra t ng n burned pat ents rev lut n zed surv val ut mes n burn are. Ex s n s per rmed w th repeated tangent al sl es us ng a Wats n r G ul an blade unt l v able, d usely bleed ng t ssue rema ns. he d wns de tangent al ex s n s a h gh bl d l ss, th ugh th s may be amel rated us ng te hn ques su h as nst llat n an ep nephr ne tumes en e s lut n underneath theburn.Pneumat t urn quetsarehelp ul n extremty burns, and mpresses s aked n a d lute ep nephr ne s lut n are ne essary adjun ts a ter ex s n. A br n gen and thr mb n spray sealant ( sseel F br n Sealant; Baxter, Deer eld, IL) als has bene al e e ts n b th hem stas s and gra t adheren et thew und bed.
Sn e ull th kness burns are mpra t al r m st burn w unds, spl t-th kness sheet aut gra ts harvested w th a p wer dermat me make the m st durable w und verngs, and have a de ent smet appearan e. In larger burns, meshed aut gra ted sk n pr v desalarger area w und verage. h s als all ws dra nage bl d and ser us lu d t prevent a umulat n under the sk n gra t w th subsequent gra t l ss. Areas smet mp rtan e, su h as the a e, ne k, and hands, sh uld be gra ted w th n nmeshed sheet gra ts t ensure pt mal appearan e and un t n. Opt ns r temp rary w und verage n lude human adaver all gra t,wh h s n rp rated nt thew und but sreje ted by the mmune system and must be eventually repla ed.(See S hwartz10th ed.,p.234.)
59
14.A45-year- ldw man sadm ttedt ah sp talbe ause a th rd-degree burn njury t 40% her BSA, and her w unds are treated w th t p al s lver sul ad az ne ream (Slvadene). Tree days a er adm ss n, a burn w und b psy sem quant tat ve ulture sh ws 104 Pseudomonas rgan sms per gram t ssue. Te pat ent’s nd t n s stable at th s t me. Te m st appr pr ate management r th spat ent w uld bet
A. Repeat theb psyand ulture n 24h urs.
B. Start subes har lys sw th ant b t s.
C. Adm n ster system ant b t s.
D. Surg allyex setheburn w unds.
15.F urteen days a er adm ss n t the h sp tal r a 30% part al th kness burn and hem dynam nstab l ty requ r ng entral ven us a ess, a pat ent devel ps a sp kngtemperature urve.On phys alexam nat n,the entral ven us atheter nsert n s te was red, tender, and warm.Tebest treatment r th s mpl at n st
A. Ex hange entral ven us atheter ver gu dew re, ulturet p prev us atheter.
B. reat pat ent w th IV ant b t s unt l bl d ultures drawn r m atheter arenegat ve.
C. Rem val entral ven us atheter, ulture t p, and pla ement new atheter n ntralaterals te.
D. Rem val atheter and treat pat ent w th ral ant bt sand pa n med at n asneeded.
Answer:B
Ba ter al pr l erat n n a burn w und may ur desp te t p al ant ba ter al agents. When ba ter al pr l erat n has es aped ntr l, as pr ved by quant tat ve burn w und b psy, adm n strat n ant b t s by needle lys s beneath the es har s nd ated. h s therapy s m st e e t ve n tated early, be re nvas ve burn w und seps s has devel ped rw und l n zat n hasrea hedgreaterthan 104 rgan sms per gram t ssue.System ant b t susuallyare ne e t ve atth sp ntbe ausebytheth rddaya teraburn,bl d l wt aburn w und smarkedlyde reased. hus,adequatelevels ant b t aren t a h eved at thees har-v ablet ssue nter a e where the ba ter al pr l erat n s urr ng. Be re the use subes har ant b t s, Pseudomonasseps s burn w unds a mpan edbye thymagangren sumwasun rmly atal n h ldren. On e l n zat n a burn w und has urred, surg al ex s n s extremely danger us, as system seed ng w ll ur.(SeeS hwartz10th ed.,p.232.)
Answer:C Burn pat ents ten requ re entral ven us a ess r lu d resus tat n and hem dynam m n t r ng. Be ause the anat m relat n the r burns t mm nly used a ess s tes, burn pat ents may be at h gher r sk r atheter-related bl dstream n e t ns. he2009CDC NHSN rep rt (http:// www. d .g v/nhsn/dataStat.html) nd ates that Amer an burn enters have h gher n e t us mpl at n rates than any ther ICUs.Be auseburn pat entsmay mm nlyexh b t leuk yt s s w th a d umented bl dstream n e t n, prat ehasbeen t rew rel nes ver agu dew reand t ulturethe atheter t p. H wever, th s may n rease the r sk atheterrelated n e t ns n burned pat entsand a news te sh uld be used at allp ss ble.(SeeS hwartz10th ed.,p.233.)
60
1. Which o the ollowing is FALSE regarding polymorphonuclear neutrophils (PMNs) and their role in wound healing?
A. PMNsreleaseproteasesthatdegradegroundsubstance within thewound site.
B. Neutrophilsuse brin clot generated at thewound site assca olding or migration intothewound.
C. Neutrophil migration is stimulated by local prostaglandins, complement actors, interleukin-1 (IL-1), tumornecrosis actor-α( NF-α),trans orminggrowth actor-β ( GF-β), platelet actor 4, or bacterial products.
D. PMNsarethe rstcellstoin ltratethewound,peaking at 24to48hours.
E. Neutrophilsreleasecytokinesthat later assist with collagen deposition and epithelialclosure.
Wound Healing
Answer:E
Polymorphonuclearneutrophils(PMNs)arethe irstin iltrating cellstoenterthewoundsite,peakingat24to48hours.Increased vascular permeability,localprostaglandin release,and thepresence o chemotactic substances such as complement actors, interleukin-1 (IL-1), tumor necrosis actor-α ( NF-α), transorminggrowth actor-β ( GF-β),platelet actor 4, or bacterial productsallstimulateneutrophilmigration.
he postulated primaryrole o neutrophils is phagocytosis o bacteria and tissue debris. PMNs are also a major source o cytokines early during in lammation, especially NF-α, which mayhave a signi icant in luence on subsequent angiogenesis and collagen synthesis. PMNs also release proteases such as collagenases, which participate in matrix and ground substance degradation in the early phase o wound healing. Other than their role in limitingin ections, these cells do not appear to play a role in collagen deposition or acquisition o mechanicalwound strength.On the contrary,neutrophil actors have been implicated in delayingthe epithelial closure o wounds.(SeeSchwartz10th ed.,p.243.)
2. Teproli erativephaseo wound healingoccurshowlong a er theinjury?
A. 1day
B. 2days
C. 7days
D. 14days
Answer:C
Normal wound healing ollows a predictable pattern that can be divided into overlapping phases de ined by the cellular populations and biochemical activities: (1) hemostasis and in lammation, (2) proli eration, and (3) maturation and remodeling.
heproli erativephaseisthesecond phaseo wound healing and roughly spans days 4 through 12. It is during this phase that tissue continuity is reestablished. Fibroblasts and endothelial cells are the last cell populations to in iltrate the healing wound, and the strongest chemotactic actor or ibroblasts is platelet-derived growth actor (PDGF). Upon entering the wound environment, recruited ibroblasts irst need to proli erate, and then become activated, to carry out their primary unction o matrix synthesis remodeling. his activation is mediated mainly by the cytokines and growth actors released rom wound macrophages. (See Schwartz 10th ed.,p.241.)
61
9
CHAPTER
3. Which o the ollowing is true regarding the broblastic phaseo wound healing?
A. Early during wound healing, the predominant composititon o the matrix is bronectin and type II collagen.
B. A er complete replacement o the scar with type III collagen, the mechanical strength will equal that o uninjured tissue approximately 6 to 12 months postinjury.
C. Even though the tensile strength o a wound reaches a plateau a er several weeks, the tensile strength will increase over another 6 to 12 months due to bril ormation and cross-linking.
D. As the scar matures, matrix metalloproteinases (MMPs)breakdown typeIcollagen and replaceit with typeIIIcollagen.
Answer:C
hematuration and remodelingo thescar beginsduringthe ibroblastic phase, and is characterized bya reorganization o previously synthesized collagen. Collagen is broken down by matrix metalloproteinases (MMPs), and the net wound collagen content is the result o a balance between collagenolysisand collagen synthesis. hereisa net shi t toward collagen synthesis and eventually the reestablishment o extracellular matrixcomposed o arelativelyacellular collagen-rich scar.
Wound strength and mechanical integrity in the resh wound are determined by both the quantity and quality o the newlydeposited collagen. he deposition o matrixat the wound site ollows a characteristic pattern: ibronectin and collagen type III constitute the early matrix sca olding; glycosaminoglycans and proteoglycans represent the next signi icant matrix components; and collagen type I is the inal matrix. By several weeks postinjury the amount o collagen in the wound reaches a plateau, but the tensile strength continues to increase or several more months. Fibril ormation and ibril cross-linking result in decreased collagen solubility, increased strength, and increased resistance to enzymatic degradation o the collagen matrix. Fibrillin, a glycoprotein secreted by ibroblasts,isessential or the ormation o elastic ibers ound in connective tissue. Scar remodeling continues or many (6–12) months postinjury, gradually resulting in a mature,avascular,and acellular scar. hemechanicalstrength o the scar never achieves that o the uninjured tissue. (See Schwartz10th ed.,p.245.)
4. Which o the ollowing is commonly seen in EhlersDanlossyndrome(EDS)?
A. Smallbowelobstructions.
B. Spontaneousthrombosis.
C. Direct or recurrent herniasin children.
D. Abnormalscarringo thehandswith contractures.
Answer:C Ehlers-Danlos syndrome (EDS) is a group o 10 disorders that present as a de ect in collagen ormation. Over hal o the a ectedpatientsmani estgeneticde ectsencodingalphachains o collagen typeV,causingittobeeither quantitativelyor structurallyde ective. hesechangeslead to “classic”EDSwith phenotypic indings that include thin, riable skin with prominent veins,easybruising,poor wound healing,atrophicscar ormation,recurrenthernias,andhyperextensiblejoints.Gastrointestinal (GI) problems include bleeding, hiatal hernia, intestinal diverticula,and rectalprolapse.Smallblood vessels are ragile, making suturing di icult during surgery. Large vessels may develop aneurysms,varicosities,arteriovenous istulas,or may spontaneouslyrupture.(SeeSchwartz10thed.,p.246.)
5. Patients with Mar an syndrome are associated with what geneticdecect?
A. MFN-1genedeletion
B. ypeIcollagen genemutation
C. COL7A1genemutation
D. FBN-1genemutation
Answer:D
Patients with Mar an’s syndrome have tall stature, arachnodactyly, lax ligaments, myopia, scoliosis, pectus excavatum, and aneurysm o the ascending aorta. Patients who su er rom this syndrome are also prone to hernias. Surgical repair o a dissecting aneurysm is di icult, as the so t connective tissue ails to hold sutures. Skin may be hyperextensible, but showsnodelayin wound healing.
he genetic de ect associated with Mar an’s syndrome is a mutation in theFBN-1genewhich encodes or ibrillin.Previously, it was thought that structural alteration o the microibrillar system was responsible or the phenotypic changes seen with the disease. However, recent research indicates an intricaterelationship that FBN-1geneproductsplayin GF-β signaling.(SeeSchwartz10th ed.,p.246.)
62
6. When a longbone racture is repaired byinternal xation with platesand screws
A. Callusat the racturesite ormsmorerapidly.
B. Delayed union isprevented.
C. Direct bone-to-bonehealingoccurswithout so callus ormation.
D. Endochondralossi cation ismorecomplete.
7. Which o the ollowingisFALSEregardinghealingo ullthicknessinjurieso theGItract?
A. Serosalhealingisessentialto orm awater-tightbarrier tothelumen o thebowel.
B. Extraperitoneal segments o bowel that lack serosa havehigher rateso anastomotic ailure.
C. Tere is an early decrease in marginal strength due to an imbalance o greater collagenolysis versus collagen synthesis.
D. Collagen synthesis is done by broblast and smooth musclecells.
E. Tegreatest tensilestrength o theGI tract isprovided bytheserosa.
Answer:C
Precise racture reduction and ixation allows the racture to heal bone-to-bone without the so t callus ormation and endochondral ossi ication, which are characteristic o closed racture management. However, internal reduction does not prevent delayed union, especially when in ection or poor blood supplyarepresent.(SeeSchwartz10th ed.,p.249.)
Answer:E
he submucosa lies radially and circum erentially outside o theselayers,iscomposed o abundant collagenousand elastic ibers, and supports neural and vascular structures. he submucosa is the layer that imparts the greatest tensile strength and greatest suture-holding capacity, a characteristic that should be kept in mind duringsurgical repair o the GI tract. Additionally, serosal healing is essential or quickly achieving a watertight seal rom the luminal side o the bowel. he importance o the serosa is underscored by the signi icantly higher rates o anastomotic ailure observed clinicallyin segmentso bowelthat areextraperitonealandlackserosa(ie,the esophagusand rectum).
he early integrity o the anastomosis is dependent on ormation o a ibrin seal on the serosal side, which achieves watertightness, and on the suture-holding capacity o the intestinal wall, particularly the submucosal layer. here is a signi icant decreasein marginalstrength duringthe irst week due to an early and marked collagenolysis. he lysis o collagen is carried out by collagenase derived rom neutrophils, macrophages, and intraluminal bacteria. Collagenase activity occurs earlyin the healingprocess, and duringthe irst 3 to 5 days collagen breakdown ar exceeds collagen synthesis. he integrity o the anastomosis represents equilibrium between collagen lysis, which occurs early, and collagen synthesis, which takes a ew days to initiate. Collagen synthesis in the GI tract is carried out byboth ibroblasts and smooth muscle cells.(SeeSchwartz10th ed.,p.249.)
8. Steroidsimpair wound healingby
A. Decreasingangiogenesisand macrophagemigration
B. Decreasingplatelet plugintegrity
C. Increasingreleaseo lysosomalenzymes
D. Increasing brinolysis
Answer:A
he major e ect o steroids is to inhibit the in lammatory phaseo woundhealing(angiogenesis,neutrophilandmacrophage migration, and ibroblast proli eration) and the release o lysosomal enzymes. he stronger the anti-in lammatory e ect o the steroid compound used, the greater the inhibitorye ect on wound healing.Steroids used a ter the irst 3to 4 days postinjury do not a ect wound healing as severely as when they are used in the immediate postoperative period. here ore i possible, their use should be delayed or, alternatively, orms with lesser anti-in lammatory e ects should be administered.
In addition to their e ect on collagen synthesis, steroids also inhibit epithelialization and contraction and contribute to increased rates o wound in ection, regardless o the time o administration. Steroid-delayed healing o cutaneous wounds can be stimulated to epithelialize by topical application o vitamin A. Collagen synthesis o steroid-treated wounds also can be stimulated by vitamin A. (See Schwartz 10th ed.,p.253.)
63
9. What type o nerve injury involves disruption o axonal continuitywith preserved Schwann cellbasallamina?
A. Neurapraxia
B. Axonotemesis
C. Neurotmesis
D. Axonolysis
10. Temajor causeo impaired wound healingis
A. Anemia
B. Diabetesmellitus
C. Localtissuein ection
D. Malnutrition
11. Howdoesdiabetesmellitusimpair wound healing?
A. Local hypoxemia, reduced angiogenesis, and infammation duetovascular disease.
B. Glycosylation o proteoglycans and collagen in wound bed duetohyperglycemia.
C. Decreased collagen accretion noted in patients with typeIIdiabetesmellitus.
D. Increased bacterialload toduetohyperglycemia.
Answer:B
here are three types o nerve injuries: neurapraxia ( ocal demyelination), axonotmesis (interruption o axonal continuity but preservation o Schwann cell basal lamina), and neurotmesis (complete transection). Following all types o injury, the nerve ends progress through a predictable pattern o changes involving three crucial steps: (1) survival o axonalcellbodies;(2) regeneration o axons that growacross the transected nerve to reach the distal stump;and (3) migration and connection o the regenerating nerve ends to the appropriatenerveendsor organ targets.
Phagocytes remove the degenerating axons and myelin sheath rom the distal stump (Wallerian degeneration). Regeneratingaxonalsproutsextend rom theproximalstump and probe the distal stump and the surrounding tissues. Schwann cellsensheatheand help in remyelinatingtheregenerating axons. Functional units are ormed when the regenerating axons connect with the appropriate end targets. (See Schwartz10th ed.,p.251.)
Answer:C
Allthe actorslisted impair wound healing,but localin ection is the major problem. he surgeon should make every e ort to remove all devitalized tissue and leave a clean wound or closure.(SeeSchwartz10th ed.,p.252.)
Answer:A
Uncontrolled diabetes results in reduced in lammation, angiogenesis, and collagen synthesis. Additionally, the large and small vessel disease that is the hallmark o advanced diabetes contributes to local hypoxemia. De ects in granulocyte unction, capillary ingrowth, and ibroblast proli eration all have been described in diabetes. Obesity, insulin resistance, hyperglycemia, and diabetic renal ailure contribute signi icantly and independently to the impaired wound healing observed in diabetics.(SeeSchwartz10th ed.,p.253.)
12. Supplementation o which o the ollowing micronutrients improves wound healingin patients without micronutrient de ciency?
A. Vitamin C
B. Vitamin A
C. Selenium
D. Zinc
Answer:B
he vitamins most closely involved with wound healing are vitamin C and vitamin A. here is no evidence that excess vitamin C is toxic; however, there is no evidence that supertherapeuticdoseso vitamin Careo anybene it.
Vitamin A de iciency impairs wound healing, while supplemental vitamin A bene its wound healing in nonde icient humans and animals. Vitamin A increases the in lammatory responsein wound healing,probablybyincreasingthelability o lysosomalmembranes. hereisan increased in luxo macrophages, with an increase in their activation and increased collagen synthesis. Vitamin A directly increases collagen production and epidermal growth actor receptors when it is added in vitro to cultured ibroblasts. As mentioned be ore, supplemental vitamin A can reverse the inhibitory e ects o corticosteroidson wound healing.Vitamin Aalso can restore wound healing that has been impaired by diabetes, tumor ormation, cyclophosphamide, and radiation. Serious injury or stress leads to increased vitamin A requirements. In the severelyinjuredpatient,supplementaldoseso vitaminAhave been recommended. Doses ranging rom 25,000 to 100,000 IU/dayhavebeen advocated.
64
13. Which type o collagen is most important in wound healing?
A. ypeIII
B. ypeV
C. ypeVII
D. ypeXI
14. What isFALSEregardinghealingo cartilage?
A. Cartilage is avascular and depends on di usion o nutrients.
B. Super cial cartilage wounds are not associated with an infammatoryresponse.
C. Cartilage injuries o en heal slowly and result in permanent structuralde ects.
D. Amajorsourceo nutrientstocartilageis romnearby periosteum.
Zincisthemost well-known element in wound healingand hasbeen used empiricallyin dermatologicconditions or centuries. o date, no studyhas shown improved wound healing with zinc supplementation in patients who are not zinc de icient.(SeeSchwartz10th ed.,p.255.)
Answer:A
Although there are at least 18 types o collagen described, the main ones o interest to wound repair are types I and III. ype I collagen is the major component o extracellular matrixin skin. ypeIII,whichisalsonormallypresentin skin, becomes more prominent and important during the repair process.(SeeSchwartz10th ed.,p.244.)
Answer:D
Cartilage consists o cells (chondrocytes) surrounded by an extracellular matrix made up o several proteoglycans, collagen ibers, and water. Unlike bone, cartilage is veryavascular and depends on di usion or transmittal o nutrients across the matrix. Additionally, the hypervascular perichondrium contributes substantially to the nutrition o the cartilage. here ore,injuriestocartilagemaybeassociated with permanent de ectsduetothemeager and tenuousblood supply. he healing response o cartilage depends on the depth o injury. In a super icial injury, there is disruption o the proteoglycan matrix and injuryto the chondrocytes. here is no in lammatory response, but an increase in synthesis o proteoglycan and collagen dependent entirely on the chondrocyte. Un ortunately, the healing power o cartilage is o ten inadequateand overallregeneration isincomplete. here ore, super icialcartilageinjuriesareslowtohealando ten resultin persistent structuralde ects.(SeeSchwartz10th ed.,p.251.)
15. Signs o malignant trans ormation in a chronic wound include
A. Persistent granulation tissuewith bleeding
B. Overturned wound edges
C. Nonhealinga er 2weekso therapy
D. Distaledema
16. What isthedi erencebetween hypertrophicscars(H S) and keloids?
A. Keloidsarean overabundanceo broplasiaasaresult o healing,hypertrophicscarsarea ailureo collagen remodeling.
B. Hypertrophic scars o en regress over time, whereas keloidsrarelyregress.
C. Hypertrophic scars are more common in darkerpigmented ethnicities.
D. Hypertropic scars extend beyond the border o the originalwound.
Answer:B
Malignant trans ormation o chronic ulcers can occur in any long-standing wound (Marjolin ulcer). Anywound that does not heal or a prolonged period o time is prone to malignant trans ormation. Malignant wounds are di erentiated clinically rom nonmalignant wounds by the presence o overturned wound edges. In patients with suspected malignant trans ormations, biopsy o the wound edges must be perormed to rule out malignancy. Cancers arising de novo in chronic wounds include both squamous and basal cell carcinomas.(SeeSchwartz10th ed.,p.259.)
Answer:B
Hypertrophicscars(H S)and keloidsrepresent an overabundance o ibroplasia in the dermal healing process. H S rise above the skin level but stay within the con ines o the originalwound and o ten regressover time.Keloids riseabovethe skin levelaswell,but extend beyond theborder o theoriginal wound and rarelyregressspontaneously(Fig.9-1). Both H S and keloidsoccur a ter traumatotheskin,and maybetender, pruritic, and cause a burning sensation. Keloids are 15 times morecommon in darker-pigmented ethnicities,with individualso A rican,Spanish,andAsian ethnicitiesbeingespecially susceptible.Men and women areequallya ected.Genetically, the predilection to keloid ormation appears to be autosomal dominant with incomplete penetration and variable expression.(SeeSchwartz10th ed.,Figure9-11,p.261.)
65
17. Tetreatment o choice or keloidsis
A. Excision alone
B. Excision with adjuvant therapy(eg,radiation)
C. Pressuretreatment
D. Intralesionalinjection o steroids
FIG.9-1. Recurrent keloid on the neckofa 17-year-old patient that had been revised severaltimes.(Reproduced with permission from MurrayJC,PinnellSR:Keloidsand excessive dermalscarring, in Cohen IK,Diegelmann RF,Lindblad WJ(eds):WoundHealing: BiochemicalandClinicalAspects.Philadelphia:WBSaunders,1993. Copyright Elsevier.)
Answer:B
Excision alone o keloids is subject to a high recurrence rate, ranging rom 45 to 100%. here are ewer recurrences when surgical excision is combined with other modalities such as intralesional corticosteroid injection, topical application o silicone sheets, or the use o radiation or pressure. Surgery is recommended or debulking large lesions or as second-line therapy when other modalities have ailed. Silicone application is relatively painless and should be maintained or 24 hours a day or about 3 months to prevent rebound hypertrophy. It may be secured with tape or worn beneath a pressure garment. he mechanism o action is not understood, but increased hydration o theskin,which decreasescapillary activity, in lammation, hyperemia, and collagen deposition, may be involved. Silicone is more e ective than other occlusivedressingsand isan especiallygood treatment or children and others who cannot tolerate the pain involved in other modalities.(SeeSchwartz10th ed.,p.262.)
66
18. What isFALSEabout peritonealadhesions?
A. Most peritoneal adhesions are a result o intraabdominalsurgery.
B. Intra-abdominal adhesions are the most common causeo smallbowelobstruction.
C. Operations in the upper abdomen have a higher chance o causing adhesions that cause small bowel obstruction,especiallyinvolvingthejejunum.
D. Adhesionsarea leadingcauseo secondaryin ertility in women.
Answer:C
Peritoneal adhesions are ibrous bands o tissues ormed between organs that are normally separated and/or between organs and the internal body wall. Most intra-abdominal adhesions are a result o peritoneal injury, either by a prior surgical procedure or due to intra-abdominal in ection. Postmortem examinations demonstrate adhesions in 67% o patientswith prior surgicalproceduresand in 28%with ahistoryo intra-abdominalin ection.Intra-abdominaladhesions arethemostcommon cause(65–75%)o smallbowelobstruction,especiallyin theileum.Operationsin thelowerabdomen have a higher chance o producing small bowel obstruction. Following rectal surgery, le t colectomy, or total colectomy, there is an 11% chance o developing small bowel obstruction within 1 year, and this rate increases to 30%by10 years. Adhesions also are a leading cause o secondary in ertility in women and can cause substantial abdominaland pelvic pain. Adhesions account or 2% o all surgical admissions and 3% o alllaparotomiesin generalsurgery.(See Schwartz10th ed., p.263.)
19. Which growth actor has been ormulated and approved or treatment o diabetic oot ulcers?
A. PDGF
B. IGF-1
C. IL-8
D. Keritinocytegrowth actor
E. Laminin-5
Answer:A
At present, only platelet-derived growth actor BB (PDGFBB)iscurrentlyapprovedbytheFDA or treatmento diabetic oot ulcers. Application o recombinant human PDGF-BBin a gel suspension to these wounds increases the incidence o totalhealingand decreaseshealingtime.Severalother growth actorshavebeen testedclinicallyandshowsomepromise,but currently none are approved or use (See Schwartz 10th ed., p.267.)
67
1. Te annual age-adjusted cancer incidence rates among men and women are decreasing or all o the ollowing EXCEP
A. Colorectal
B. Oropharynx
C. Lung
D. Tyroid
2. Which o the ollowingisNO ahallmarko cancer?
A. Abilitytoinvadeand metastasize
B. Abilitytoevadeapoptosis
C. Abilitytoevadeautophagy
D. Abilitytoevadeimmunedestruction
3. Characteristics o tumorigenic trans ormation o cells includewhich o the ollowing?
A. Enhanced sur aceadherence
B. Monolayer con uenceinhibition
C. Acquisition o chemoresistance
D. Immortalization
4.
A. Sphase
B. G1phase
C. G2phase
D. G3phase
Answer:D
Incidence rates are declining or most cancer sites, but they are increasingamongboth men and women or melanoma o the skin, cancers o the liver and thyroid. Incidence rates are decreasing or all our major cancer sitesexcept or breastcancer in women.(SeeSchwartz10th ed.,p.274.)
Answer:C
here are six essential alterations in cell physiology that dictate malignant growth: sel -su iciency o growth signals, insensitivity to growth-inhibitory signals, evasion o apoptosis (programmed cell death), potential or limitless replication, angiogenesis, and invasion and metastasis. Recently two additional hallmarks have emerged—reprogramming o energy metabolism and evading immune destruction. (See Schwartz10th ed.,p.277.)
Answer:D
Abnormally proli erating, trans ormed cells outgrow normal cells in the culture dish (ie, in vitro) and commonly display severalabnormalcharacteristics. heseincludelosso contact inhibition (ie, cells continue to proli erate a ter a con luent monolayer is ormed);an altered appearanceand poor adherence to other cells or to the substratum; loss o anchorage dependence or growth;immortalization;and gain o tumorigenicity (ie, the ability to give rise to tumors when injected intoan appropriatehost).(SeeSchwartz10th ed.,p.277.)
Answer:D
hecellcycleisdivided into our phases.Duringthesynthetic or Sphase,thecellgeneratesa singlecopyo itsgeneticmaterial, whereas in the mitotic or M phase, the cellular componentsarepartitioned between two daughter cells. heG1and G2phasesrepresentgapphasesduringwhich thecellsprepare themselves or completion o theSand Mphases,respectively. When cells cease proli eration, they exit the cell cycle and enter thequiescentstatere erredtoasG.(SeeSchwartz10thed., p.279.)
69
10
Oncology
CHAPTER
Tecellcycleincludesallo the ollowingphasesEXCEP
5. Which o the ollowing actors are suggestive o a hereditarycancer?
A. umor development at ayounger than normalage.
B. Presenceo bilateraldisease.
C. Association with paraneoplasticsyndrome.
D. Presenceo multipleprimarymalignancies.
6. Which o the ollowingareassociated with amilialadenomatouspolyposis(FAP)
A. Osteomas
B. Glioblastomamulti orme
C. Meckeldiverticulum
D. Esophagealatresia
7. Which mutated gene malignant disease association is correct
A. PTENand Li-Fraumenisyndrome
B. RETand MEN2syndrome
C. P16and synovialsarcoma
D. BRCA1and adrenocorticalcarcinoma
8. Risk or invasive breast cancer development is increased or each actor EXCEP
A. Ageat menarche<12.
B. Ageat frst livebirth >30.
C. Biopsy-proven atypicalhyperplasia.
D. Nopreviousbreast biopsy.
Answer:C
he ollowing actorsmaysuggestthepresenceo ahereditary cancer:
1. umor development at amuch younger agethan usual.
2. Presenceo bilateraldisease.
3. Presenceo multipleprimarymalignancies.
4. Presentation o acancer in thelessa ected sex(eg,male breast cancer).
5. Clusteringo thesamecancer typein relatives.
6. Occurrence o cancer in association with other conditionssuch asmentalretardation or pathognomonicskin lesions.(SeeSchwartz10th ed.,p.287.)
Answer:A
Familial adenomatous polyposis (FAP) is associated with benignextracolonicmani estationsthatmaybeuse ulinidenti ying new cases, including congenital hypertrophy o the retinal pigment epithelium, epidermoid cysts, and osteomas. In addition to colorectal cancer, patients with FAP are at risk or upper intestinal neoplasms (gastric and duodenal polyps, duodenal and periampullary cancer), hepatobiliary tumors (hepatoblastoma, pancreatic cancer, and cholangiocarcinoma), thyroid carcinomas, desmoid tumors, and medulloblastomas.(SeeSchwartz10th ed.,p.291.)
Answer:B
MEN2 syndrome is caused by gain o unction mutations in theRE gene.Li-Fraumenisyndromeisassociatedwithmutation o P53.Mutationsin p16isassociated with melanomas, as well as cancers o the pancreas, esophagus, head and neck, stomach, breast, and colon. BRCA1 is associated with breast and ovarian carcinoma.(SeeSchwartz10th ed.,pp.291-293.)
Answer:D
Risk actors or the development o breast cancer is summarized in able 10-1. Previous breast biopsies are associated with an increasein risk o invasivebreast cancer.No previous breast biopsycon ersthebaselinerisk.(SeeSchwartz10th ed., able10-8,p.297.)
70
RiskFactor RelativeRisk(%) Ageatmenarche(years) >14 1.00 12–13 1.10 <12 1.21 Ageatfrstlivebirth(years) Patientswithnofrst-degreerelativeswithcancer <20 1.00 20–24 1.24 25–29ornulliparous 1.55 ≥30 1.93 Patientswithonefrst-degree-relativewithcancer <20 1.00 20–24 2.64 25–29ornulliparous 2.76 ≥30 2.83 (Continued)
TABLE10-1 Assessment ofriskforinvasive breast cancer
9. Routine ongoing cancer screening is recommended or which o the ollowingmalignancies?
A. Ovary
B. Leukemia
C. Carcinomao thekidney
D. Sarcoma
10. Depending on the tumor, acceptable approaches to biopsyincludeanyo the ollowingEXCEP
A. Fine-needleaspiration
B. Coreneedlebiopsy
C. Incisionalbiopsy
D. Morcellation
11. Anticancer chemotherapy agents include all o the ollowingEXCEP
A. Alkylatingagents
B. Antitumor antibiotics
C. Prometabolites
D. Plant alkaloids
Answer:A
On the occasion o a periodic health examination, the cancer related checkup should includeexamination or cancerso the thyroid,testicles,ovaries,lymph nodes,oralcavity,andskin,as wellashealthcounselingabouttobacco,sun exposure,dietand nutrition,risk actors,sexualpractices,andenvironmentaland occupationalexposures.(SeeSchwartz10th ed.,p.299.)
Answer:D
Asample o a lesion can be obtained with a needle or with an open incisional or excisional biopsy specimen. Core biopsy specimen,such as ine-needleaspiration,isrelativelysa eand can beper ormed either bydirect palpation (eg,abreast mass or a so t tissue mass) or can be guided by an imaging study (eg, stereotactic core biopsy specimen o the breast). Open biopsy specimens have the advantage o providing more tissue or histologicevaluation and thedisadvantageo beingan operativeprocedure.Incisionalbiopsyspecimensarereserved or very large lesions in which a de initive diagnosis cannot be made byneedle biopsyspecimen. Excisional biopsyspecimens are per ormed or lesions or which either core biopsy specimen isnot possibleor theresultsarenondiagnostic.(See Schwartz10th ed.,p.300.)
Answer:C
Anticancer agents include alkylating agents, antitumor antibiotics, antimetabolites, and plant alkaloids. Antimetabolites are cell-cycle speci ic agents that have their major activity in the S phase o the cell cycle. hese drugs are most e ective in tumorsthat havea high growth raction,and include olate antagonists, purine antagonists, and pyrimidine antagonists. (SeeSchwartz10th ed.,p.307.)
71
RiskFactor(years) RelativeRisk(%) Patientswith ≥2frst-degreerelatives withcancer <20 6.80 20–24 5.78 25–29ornulliparous 4.91 ≥30 4.17 Breastbiopsies(number) Patientsaged<50atcounseling 0 100 1 170 ≥2 2.88 Patientsaged ≥50atcounseling 0 100 1 127 ≥2 1.62
TABLE10-1 Assessment ofriskforinvasive breast cancer (continued)
Source:Modifed rom GailMH,Brinton LA,ByarDP,Corle DK,Green SB,SchairerC,et al. Projecting individualized probabilitieso developing breast cancer orwhite emales who are being examined annually.JournaloftheNationalCancerInstitute. 1989;81:1879-1886.
12. Approved strategies or cancer chemoprevention include allo the ollowingEXCEP
A. Neurontin or malignant peripheral nerve sheath tumor
B. amoxi en or breast cancer
C. Celecoxib or FAPsyndrome
D. 13-cis-retinoicacid or oralleukoplakia
Answer:A he systemic or local administration o therapeutic agents to prevent thedevelopment o cancer,called chemoprevention,is beingactivelyexplored or severalcancer types.In breast cancer, the NSABP Breast Cancer Prevention rial demonstrated thattamoxi en administration reducestherisko breastcancer byone-hal and reduces the risk o estrogen receptor-positive tumors by 69% in high-risk patients. here ore, tamoxi en has been approved by the FDA or breast cancer chemoprevention. he subsequent NSABP P-2 trial demonstrated that raloxi ene is as e ective as tamoxi en in reducing the risk o invasive breast cancer and is associated with a lower risk o thromboembolic events and cataracts but a nonstatistically signi icanthigher risko noninvasivebreastcancer;these indings led the FDA to approve raloxi ene or prevention as well. Several other agents are also under investigation. Celecoxib has been shown to reduce polyp number and polyp burden in patientswith FAP,which led to itsapprovalbytheFDA or these patients. In head and neck cancer, 13-cis-retinoic acid hasbeen shown both toreverseoralleukoplakiaandtoreduce second primary tumor development. (See Schwartz 10th ed., p.315.)
72
1. Hyperacuterejection iscaused by
A. Pre ormed antibodies
B. B-cell–generated antidonor antibodies
C. -cell–mediated allorejection
D. Nonimmunemechanism
2. Temechanism o action o azathioprine(AZA)is
A. Inhibition o calcineurin
B. Inter erencewith DNAsynthesis
C. Bindingo FK-506bindingproteins
D. Inhibition o P7056kinase
Answer:A
Hyperacute rejection, a very rapid type o rejection, results in irreversible damage and gra t loss within minutes to hours a ter organ reper usion. It is triggered bypre ormed antibodies against the donor’s human leukocyte antigen (HLA) or ABO blood group antigens. hese antibodiesactivate a series o eventsthat result in di useintravascular coagulation,causing ischemic necrosis o the gra t. Fortunately, pretransplant blood group typingand cross-matching(in which thedonor’s cells are mixed with the recipient’s serum, and then destruction o thecellsisobserved)havevirtuallyeliminatedtheincidenceo hyperacuterejection.(SeeSchwartz10th ed.,p.324.)
Answer:B
An antimetabolite, azathioprine (AZA) is converted to 6-mercaptopurineandinhibitsboththedenovopurinesynthesisand salvagepurinesynthesis.AZAdecreases -lymphocyte activity and decreases antibody production. It has been used in transplant recipients or more than 40 years, but became an adjunctive agent a ter the introduction o cyclosporine. Withthedevelopmento newer agentssuchasmycophenolate mo etil (MMF), the use o AZA has decreased signi icantly. However,itispre erredin recipientswhoareconsideringconceiving a child, because MMF is teratogenic in emales and can cause birth de ects. AZA might be an option or recipients who cannot tolerate the gastrointestinal (GI) side e ects o MMF.
hemost signi icant sidee ect o AZA,o ten dose-related, is bone marrow suppression. Leukopenia is o ten reversible withdosereduction or temporarycessation o thedrug.Other signi icant side e ects include hepatotoxicity, pancreatitis, neoplasia, anemia, and pulmonary ibrosis. Its most signi icant druginteraction is with allopurinol, which blocks AZA’s metabolism, increasing the risk o pancytopenia. Recommendationsareto not useAZAand allopurinoltogether,or i doingso isunavoidable,to decrease the dose o AZAby75%. (SeeSchwartz10th ed.,p.326.)
73
11
Transplantation
CHAPTER
3. Whicho the ollowingisNO asidee ecto cyclosporine?
A. Interstitialfbrosiso therenalparenchyma
B. Gingivalhyperplasia
C. Hirsutism
D. Pancreatitis
Answer:D (SeeSchwartz10th ed., able11-4,p.327.)
TABLE11–1 Side efectsand drug interactionso the main immunosuppressive drugs
Cyclosporine(CSA) Hypertension, nephrotoxicity,hirsutism, neurotoxicity,gingival hyperplasia, hypomagnesemia, hyperkalemia
Tacrolimus(FK506) Hypertension, nephrotoxicity,alopecia, hyperglycemia, neurotoxicity, hypomagnesemia, hyperkalemia
Sirolimus Thrombocytopeniaand neutropenia,elevated cholesterol,extremity edema,impairedwound healing
Mycophenolatemoetil Leukopenia, thrombocytopenia,GI upset
Corticosteroids Hyperglycemia, osteoporosis,cataracts, myopathy,weightgain
Azathioprine Leukopenia,anemia, thrombocytopenia, neoplasia,hepatitis, cholestasis
Verapamil,diltiazem, clarithromycin, azithromycin, erythromycin,azole antiungals,protease inhibitors,graperuitjuice
Verapamil,diltiazem, clarithromycin, azithromycin, erythromycin,azole antiungals,protease inhibitors,graperuitjuice
Verapamil,diltiazem, clarithromycin, azithromycin, erythromycin,azole antiungals,protease inhibitors,graperuitjuice
Isoniazid,carbamazepine, phenobarbital,phenytoin, riampin,St.John’sWort
Nephrotoxicity:ganciclovir, aminoglycosides,NSAIDs, ACE-Is,andARBs
Isoniazid,carbamazepine, phenobarbital,phenytoin, riampin,St.John’swort
Nephrotoxicity:ganciclovir, aminoglycosides,NSAIDs, ACE-Is,andARBs
Isoniazid,carbamazepine, phenobarbital,phenytoin, riampin,St.John’swort
— Cholestyramine,antacids
Bonemarrowsuppression: valganciclovir,ganciclovir, TMP-SMX
— —
4. Postrenaltransplant gra thrombosisusuallyoccurs
A. Within 2to3days
B. Within 2weeks
C. Within 1month
D. Within 3months
Bonemarrowsuppression: allopurinol,sulonamides
Answer:A
One o the most devastating postoperative complications in kidney recipients is gra t thrombosis. It is rare, occurring in ewer than 1%o recipients. herecipient risk actorsinclude a history o recipient hypercoagulopathy and severe peripheral vascular disease; donor-related risk actors include the use o en bloc or pediatricdonor kidneys,procurement damage, technical actors such as intimal dissection or torsion o vessels, and hyperacute rejection. Gra t thrombosis usually occurs within the irst several days posttransplant. Acute cessation o urine output in recipients with brittle posttransplant diuresis and the sudden onset o hematuria or gra t pain should arouse suspicion o gra t thrombosis. Doppler ultrasound may help con irm the diagnosis. In cases o gra t thrombosis,an urgent thrombectomyisindicated;however,it rarelyresultsin gra t salvage.(SeeSchwartz10th ed.,p.339.)
74
COMMONSIDEEFFECTS OTHERMEDICATIONS THATINCREASEBLOOD LEVELS OTHERMEDICATIONS THATDECREASEBLOOD LEVELS OTHERMEDICATIONS THATPOTENTIATE TOXICITY
— — —
ACE-I= angiotensin-converting enzyme inhibitor;ARB= angiotensin receptorblocker;NSAID= nonsteroidalanti-infammatorydrug;TMP-SMX= trimethoprim-sulamethoxazole
5. Te1-year gra survivala er renaltransplantation is
A. 35–40%
B. 50–55%
C. 75–80%
D. 92–96.5%
6. A er completion o thevascular anastomoses,drainageo atransplanted pancreasisaccomplished byanastomosisto
A. Right colon
B. Le colon
C. Duodenum
D. Bladder or smallbowel
Answer:D
Accordingtothe2010Scienti icRegistryo ransplant Recipients (SR R) annual report, a total o 84,614 adult patients were on the kidney transplant waiting list, including 33,215 added just that year. Yet in 2009, only 15,964 adult kidney transplants were per ormed in the United States (9912 with a deceased donor and 6052 with a living donor). O note, the number o patientsadded tothekidneytransplant waitinglist hasincreasedeveryyear,butthenumber o kidneytransplants per ormed has been declining since 2006. On the positive side, posttransplant outcomes have continued to improve: in 2009,the1-year gra t survivalratewith a livingdonor kidney was96.5%;with a deceased donor kidney,theratewas92.0%. (SeeSchwartz10th ed.,p.334.)
Answer:D
Over the years, di erent surgical techniques have been described or (1) the management o exocrine pancreatic secretions and (2) the type o venous drainage. For the secretions, the two most common techniques are drainage o the duodenal segment to the bladder (bladder drainage) or to the small bowel (enteric drainage) (Figs. 11-1 and 11-2). For venous drainage, systemic venous drainage is pre erred over portalvenousdrainage.(SeeSchwartz10th ed.,Figures11-12 and 11-13,pp.341–343.)
FIG.11-1. Whole-organ transplant with systemicvein and bladder exocrine drainage.(Reproduced rom GruessnerRWG,Sutherland DER,eds.TransplantationofthePancreas.NewYork:Springer,2004; ColorPlate XIV,Figure 8.2.2.2[B].With kind permission o Springer Science + BusinessMedia.)
75
7. Allo the ollowingare absolute contraindications in consideringacandidate or orthotopiccardiactransplantation EXCEP
A. Activein ection
B. Ageover 65years
C. Historyo medicalnoncompliance
D. Severepulmonaryhypertension
Answer:B
In general terms, contraindications to a liver transplant include insu icient cardiopulmonary reserve, uncontrolled malignancy or in ection, and re ractory noncompliance. Older ageisonlyarelativecontraindication:care ullyselected recipients older than 70 years can achieve satis actory outcomes. Patients with reduced cardiopulmonary reserve are unlikelyto survive a liver transplant. Candidates should have anormalejection raction.I coronaryarterialdiseaseispresent, they should undergo revascularization pretransplant. Severe chronic obstructive pulmonary disease (COPD) with oxygen dependence is a contraindication. Severe pulmonary hypertension with a mean pulmonaryarterypressure greater than 35 mm Hg that is re ractory to medical therapy is also a contraindication. Candidates with pulmonary hypertension should be evaluated with a right heart catheterization. (SeeSchwartz10th ed.,p.348.)
8. Heart transplant donors and recipients are matched using the ollowingcriteriaEXCEP
A. Statuson theUNOSwaitinglist
B. Gender
C. Blood type
D. Size
Answer:B
Once a potential deceased donor is identi ied, the surgeon reviews the status report and screening examination results. he donor is initiallymatched to the recipient per the recipient’s status on the UNOSwaiting list, the size match, and the blood type. Results o the donor’s serologic testing, echocardiography, chest X-ray, hemodynamic testing, and possibly coronaryarteryevaluation areassessed,in order todetermine whether or not the donor’s heart can withstand up to 4 hours o cold ischemic time during procurement, transport, and surgery.(SeeSchwartz10th ed.,p.355.)
76
FIG.11-2. Whole-organ transplant with systemicvein and bladder exocrine drainage.(Reproduced rom GruessnerRWG,Sutherland DER,eds.TransplantationofthePancreas.NewYork:Springer,2004; ColorPlate XIV,Figure 8.2.2.2[A].With kind permission o Springer Science + BusinessMedia.)
9. Required laboratory tests in evaluation o a patient under consideration or heart transplantation include all o the ollowingEXCEP
A. Psychosocialevaluation
B. Cardiaccatheterization
C. Dentalexamination
D. Allo theabove
10. Immunologicrejection ismediated bytherecipient’s
A. Eosinophils
B. Lymphocytes
C. Neutrophils
D. Plasmacells
11. In theprevention o gra rejection,cyclosporine
A. Blocks transcription o interleukin-1 (IL-1) and tumor necrosis actor-α( NF-α)
B. Inhibitslymphocytenucleicacid metabolism
C. Resultsin rapid decreasein thenumber o circulatory lymphocytes
D. Selectivelyinhibits -cellactivation
Answer:D
Pretransplant,both candidatesand potentialdonorsareevaluated to ensure their suitability or the procedure. ransplant candidates undergo echocardiography, right and le t heart catheterization,evaluation oranyundiagnosedmalignancies, laboratorytestingto assessthe unction o other organs(such as the liver, kidneys, and endocrine system), a dental examination, psychosocial evaluation, and appropriate screening (such as mammography, colonoscopy, and prostate-speci ic antigen testing). Once the evaluation is complete, the selection committee determines, at a multidisciplinary con erence, whether or not a heart transplant is needed and is likely to be success ul. ransplant candidates who meet all o the center’s criteria are added to the waiting list, according to the UNOS criteria, which are based on health status. (SeeSchwartz10th ed.,p.355.)
Answer:B
ransplants between genetically nonidentical persons lead to recognition and rejection o the organ by the recipient’s immune system, i no intervention is undertaken. he main antigens responsible or this process are part o the major histocompatibility complex (MHC). In humans, these antigens make up the HLA system. he antigen-encoding genes are located on chromosome 6. wo major classes o HLAs are recognized. hey di er in their structure, unction, and tissue distribution. Class I antigens (HLA-A, HLA-B, and HLA-C) are expressed byall nucleated cells. Class II antigens (HLA-DR, HLA-DP, and HLA-DQ) are expressed byantigen presentingcells(APCs)suchasBlymphocytes,dendriticcells, macrophages,and other phagocyticcells.
heprincipal unction o HLAsisto present the ragments o oreign proteins to lymphocytes. his leads to recognition andelimination o the oreign antigen with greatspeci icity. HLAmolecules playa crucial role in transplant recipients as well. hey can trigger rejection o a gra t via two di erent mechanisms. hemost common mechanism iscellular rejection,in which thedamageisdonebyactivated lymphocytes. (SeeSchwartz10th ed.,p.324.)
Answer:D
he introduction o cyclosporine in the early 1980s dramatically altered the ield o transplantation by signi icantly improving outcomes a ter kidney transplantation. Cyclosporine binds with its cytoplasmic receptor protein, cyclophilin, which subsequently inhibits the activity o calcineurin, thereby decreasing the expression o several critical -cell activation genes, the most important being or IL-2. As a result, -cell activation is suppressed. (See Schwartz 10th ed., p.328.)
12. Te most common cause o renal ailure in the United Statesis
A. Chronicglomerulonephritis
B. Chronicpyelonephritis
C. Diabetesmellitus
D. Obstructiveuropathy
Answer:C
Diabetes and hypertension are the leading causes o chronic renal disease. Concomitant cardiovascular disease (CVD) is a common inding in this population. An estimated 30% to 42%o deathswith a unctioningkidneygra tareduetoCVD. here ore, assessment o the potential kidneytransplant candidate’s cardiovascular status is an important part o the pretransplant evaluation.(SeeSchwartz10th ed.,p.335.)
77
13. Tebest method o monitoringthedevelopment o acute rejection in apatient a er cardiactransplantation is
A. Dipyridamolethallium study
B. Electrocardiogram
C. Endomyocardialbiopsy
D. Ultrasound examination o theheart
14. Absolute contraindications to renal transplantation or a patientwithchronicrenal ailureincludeallo the ollowingEXCEP
A. Chronicactivehepatitis
B. Colorectalcancer
C. Psychiatricillness
D. Sicklecelldisease
15. All o the ollowing is true or living renal transplant EXCEP
A. Donor’s kidneys with multiple renal arteries should beavoided.
B. Tedonor’sle kidneyispre erable.
C. Tereisnomedicalbeneft tothedonor.
D. Te intraperitoneal approach is most o en used or harvest.
Answer:C
hegoalo immunosuppression istoprevent rejection,which is assessed by immunosuppressive levels and, early on, by endomyocardial biopsy. Both -cell–mediated (cellular) and B-cell–mediated (antibody-mediated) rejection are monitored.(SeeSchwartz10th ed.,p.356.)
Answer:D
Active in ection or the presence o a malignancy, active substance abuse, and poorlycontrolled psychiatric illness are the ew absolute contraindications to a kidney transplant. Studies have demonstrated the overwhelming bene its o kidney transplants in terms o patient survival, quality o li e, and cost-e ectiveness, so most patients with end stage renal disease (ESRD) are re erred to or consideration o a kidney transplant.However,to achieveoptimaltransplant outcomes, the many risks (such as the surgical stress to the cardiovascular system, the development o in ections or malignancies with long-term immunosuppression, and the psychosocial and inancial impacts on compliance) must be care ully balanced.(SeeSchwartz10th ed.,p.334.)
Answer:B
he kidney, the irst organ to be transplanted rom living donors,isstillthemost common organ donated bytheseindividuals. he donor’s le t kidney is usually pre erable because o the longvascular pedicle. Use o livingdonor kidneys with multiplerenalarteriesshould beavoided,in order to decrease the complexity o the vascular reconstruction and to help avoid gra t thrombosis. Most donor nephrectomies are now per ormed via minimallyinvasive techniques, that is, laparoscopically, whether hand-assisted or not. With laparoscopic techniques, an intraperitoneal approach is most common: it involves mobilizing the colon, isolating the ureter and renal vessels, mobilizing the kidney, dividing the renal vessels and the distal ureter [C6], and removing the kidney (Fig. 11-3). Extensive dissection around the ureter should be avoided, and the surgeon should strive to preserve as much length o the renal artery and vein as possible. (See Schwartz 10th ed., Figure11-4,pp.332–333.)
78
16. Tesinglemostimportant actor in determiningwhether to per orm a transplant between a specifc donor and recipient is
A. Mixed lymphocyte culture assays o the donor and recipient
B. HLAtypeso thedonor and recipient
C. ABO blood typeso thedonor and recipient
D. Peripheral -cellcount o therecipient
17. Temostcommondiagnosisleadingtoahearttransplantis
A. COPD
B. Congenitalheart disease
C. Ischemicdilated cardiomyopathy
D. Idiopathicdilated myopathy
Answer:C
ABO blood typingand HLAtyping(HLA-A,-B,and -DR)are required be ore a kidney transplant. he method o screening or pre ormed antibodies against HLAs (because o prior transplants, blood trans usions, or pregnancies) is evolving. he panel-reactive antibody (PRA) assay is a screening test that examines the ability o serum rom a kidney transplant candidate to lyse lymphocytes rom a panel o HLA-typed donors.Anumericvalue,expressed asa percentage,indicates thelikelihoodo apositivecross-matchwithadonor.Ahigher PRA level identi ies patients at high risk or a positive crossmatch and there ore serves as a surrogate marker to measure the di iculty o inding a suitable donor and the risk o gra t rejection.(SeeSchwartz10th ed.,p.336.)
Answer:C he most common diagnosis leading to a heart transplant is ischemicdilated cardiomyopathy,which stems rom coronary artery disease, ollowed by idiopathic dilated myopathy and congenitalheart disease.About 3000patientsareadded tothe waitinglist each year.(SeeSchwartz10th ed.,p.355.)
79
A B C D E F
FIG.11-3. Laparoscopicle t donornephroureterectomy.A.Take down o splenic lexure o colon to expose the le t renalhilum.B. Dissection o le t uretero the psoasmuscle.C. Dissection o le t renalvein and gonadalvein.Le t ureterwasseen lateralto the dissection.D. Dissection o le t renalartery.Lumbarveinswere clipped and divided.E. Endo-TAstaplerto transect the le t renalartery.F. Placement o portsand Pannenstielincision orthe donorkidneyextraction.
18. Allo the ollowingaresidee ectso cyclosporineadministration or prevention o organ rejection EXCEP
A. Hyperkalemia
B. Hirsutism
C. remor
D. Bonemarrowdepression
Answer:D he metabolism o cyclosporine is via the cytochrome P450 system, resulting in many signi icant drug interactions (see able 11-1). Calcineurin inhibitors are nephrotoxic and constrict the a erent arteriole in a dose-dependent, reversible manner ( able 11-2). hey can also cause hyperkalemia and hypomagnesemia. Several neurologic complications, includingheadaches, tremor, and seizures, also have been reported. Cyclosporinehasseveralundesirablecosmetice ects,including hirsutism and gingival hyperplasia. It is associated with a higher incidence o hypertension and hyperlipidemia than is tacrolimus.(SeeSchwartz10th ed., able11-5,p.328.)
TABLE11–2 Drug interactionsand side efects
associated with calcineurin inhibitors
INTERACTIONS MEDICATIONS
Inhibitionometabolism Clarithromycin,erythromycin, azoleantiungals,diltiazem, verapamil,nicardipine, amiodarone,graperuitjuice, ritonavir,azithromycin
Inductionometabolism Nevirapine,riampin,St.John’s wort,carbamazepine, phenobarbital,phenytoin, caspoungin
Hyperkalemia Potassium-sparingdiuretics, angiotensinconvertingenzyme inhibitors(ACE-Is),angiotensin receptorblockers(ARBs), β-blockers,trimethoprimsulamethoxazole
Nephrotoxicity Nonsteroidalanti-infammatory drugs,aminoglycosides, amphotericin,ACE-Is,ARBs
19. Allo the ollowingaretrueo extracorporealmembrane oxygenation (ECMO)EXCEP
A. Cannulation occursa er withdrawalo li esupport
B. Minimizesischemicinjuryto organso cardiacdeath donors.
C. Organs are per used with warm oxygenated blood a er declaration o cardiacdeath.
D. Cannulation occursbe orewithdrawalo li esupport.
20. Temost signifcant sidee ect o sirolimusis
A. Anemia
B. Leukopenia
C. Impaired wound healing
D. Hypertriglyceridemia
Answer:A
A new development to minimize ischemic injury to organs procured a ter cardiac death has been the application o declaration o cardiac death (DCD) di ers in two key ways:
(1) cannulation occurs be ore withdrawal o li e support and
(2) organs are per used via ECMO with warm oxygenated blooda ter declaration o cardiacdeath. heinitialexperience with organs procured using ECMO has been encouraging. (SeeSchwartz10th ed.,p.331.)
Answer:D
One o the most signi icant side e ects o sirolimus is hypertriglyceridemia, a condition that may be resistant to statins and ibrates. Impaired wound healing (immediately posttransplant in particular),thrombocytopenia,leukopenia,and anemiaalsoareassociated with sirolimus,and theseproblems are exacerbated when it is used in combination with MMF. (SeeSchwartz10th ed.,p.328.)
80
2.
A. Changestostructure
B. Changestoprocess
C. Changestoculture
D. Changestooutcomes
Patient Sa ety
Answer:C
he Donabedian model o measuring quality identi ies three main types o improvements: changes to structure, process, and outcome(Fig.12-1).(SeeSchwartz10th ed.,p.367.)
A. Arteriovenous stulae
B. Myocardialin arction
C. Expandingneckhematoma
D. Localized neurologicde cit
Context: Are we improving the safety culture?
Answer:B
Complications o carotid endarterectomy include central or regional neurologic de icits or bleeding with an expanding neckhematoma.An acutechangein mentalstatusor thepresence o localized neurologic de icit requires an immediate return to theoperatingroom (OR).An expandinghematoma may warrant emergent airway intubation and subsequent trans er to the OR or control o hemorrhage. Intraoperative anticoagulation with heparin during carotid surgery makes bleeding a postoperative risk. Other complications include arteriovenous istulae,pseudoaneurysms,and in ection,allo which aretreated surgically.
Intraoperative hypotension during manipulation o the carotid bi urcation can occur and is related to increased tone rom baroreceptors that re lexly cause bradycardia. Should hypotension occur when manipulating the carotid bi urcation, an injection o 1% lidocaine solution around this structure should attenuate this re lexive response. he most common delayed complication ollowing carotid endarterectomyremainsmyocardialin arction. hepossibilityo apostoperative myocardialin arction should be considered a cause o labileblood pressureand arrhythmiasin high-riskpatients.
(SeeSchwartz10th ed.,p.384.)
81
12
Temost common delayedcomplication ollowingcarotid endarterectomyis
1. Te Donabedian model o measuringqualityidenti es all o the ollowingasmain typeso improvementsEXCEP
CHAPTER
Outcome Process
Structure
FIG.12-1. Donabedian modelformeasuring quality.(From Makary MA,Sexton JB,Freischlag JA,et al,Patientsafetyinsurgery.Ann Surg 243:628,2006.With Permission.)
3. Te most appropriate treatment or a seroma a er a sotissuebiopsyis
A. Multiple attempts o aspiration with application o pressuredressings.
B. Immediatereturn totheOR or drainage.
C. Single attempt at aspiration with return to the ORi it recurs.
D. Observation.
4. Prophylaxisusinglow-doseun ractionatedheparinreduces theincidenceo atalpulmonaryembolisms(PE)by
A. 45%
B. 50%
C. 60%
D. 35%
5. Which o the ollowingisthebest test topredict success ul extubation o apatient?
A. Respiratoryrate
B. Negativeinspiratorypressure
C. obin index
D. Minuteventilation
6. Te root cause o the majority o wrong-site surgeries result rom
A. Communication errors
B. Emergencysurgery
C. Multipleprocedures
D. Multiplesurgeons
Answer:A Lymph node biopsies have direct and indirect complications that include bleeding, in ection, lymph leakage, and seromas. Measures to prevent direct complications include proper surgicalhemostasis,proper skin preparation,and a single preoperative dose o antibiotic to cover skin lora 30 to 60 minutes be ore incision. Bleeding at a biopsy site usually can be controlled with direct pressure. In ection at a biopsy site will appear5to10dayspostoperativelyandmayrequireopeningo the wound to drain the in ection. Seromas or lymphatic leaks resolvewith aspiration o seromasand theapplication o pressure dressings, but may require repeated treatments or even placemento avacuum drain.(SeeSchwartz10th ed.,p.383.)
Answer:B
Deep vein thrombosis(DV ) occursa ter approximately25% o all major surgical procedures per ormed without prophylaxis,and pulmonaryembolism (PE) occursa ter 7%.Despite the well-established e icacy and sa ety o preventive measures, studies show that prophylaxis o ten is underused or used inappropriately. Both low-dose un ractionated heparin and low-molecular-weight heparin have similar e icacy in DV and PEprevention. Prophylaxis usinglow-dose un ractionated heparin has been shown to reduce the incidence o atalPEsby50%.(SeeSchwartz10th ed.,p.374.)
Answer:C
Protocol-driven ventilator weaning strategies are successul and have become part o the standard o care. he use o a weaningprotocol orpatientsonmechanicalventilationgreater than 48 hours reduces the incidence o ventilator-associated pneumonia (VAP) and the overalllength o time on mechanical ventilation. Un ortunately, there is still no reliable way o predicting which patient will be success ully extubated a ter a weaningprogram, and the decision or extubation is based on acombination o clinicalparametersandmeasuredpulmonary mechanics. he obin index( requency[breaths per minute]/ tidal volume [L]), also known as the rapid shallow breathing index, is perhaps the best negative predictive instrument. I the result equals less than 105, then there is nearly a 70% chance the patient will pass extubation. I the score is greater than 105,thepatient hasan approximately80%chanceo ailingextubation.Other parameterssuch asthenegativeinspiratory orce,minuteventilation,andrespiratoryrateareused,but individuallyhavenobetterpredictivevaluethan therapidshallowbreathingindex.(SeeSchwartz10thed.,p.385.)
Answer:A
he risk o per orming wrong-site surgery increases when there are multiple surgeons involved in the same operation or multiple procedures are per ormed on the same patient, especially i the procedures are scheduled or per ormed on di erent areas o the body. ime pressure, emergency surgery, abnormal patient anatomy, and morbid obesity are also thought to berisk actors.Communication errorsaretheroot cause in more than 70%o the wrong-site surgeries reported to heJoint Commission.Other risk actorsincludereceiving an incomplete preoperative assessment; having inadequate
82
7. Which o the ollowing have been shown to decrease the timeo postoperativeileus?
A. Cyclooxygenase-1inhibitors
B. Morphinepatient-controlled analgesia
C. Nasogastric drainage until ull return o bowel unction
D. Erythromycin
8. In order to reducetheoverallrisko stressgastritisin ICU patients mechanically ventilated or >48 hours, their gastricpH levelshould bekept greater than
A. 3
B. 5
C. 2
D. 4
9. Te treatment o choice or a biloma a er laparoscopic cholecystectomyis
A. Reoperation,closureo theleak,and drainage
B. Percutaneousdrainage
C. Biliarystent
D. Observation
10.
Temost requent nosocomialin ection is
A. Urinarytract in ection (U I)
B. Sepsis
C. Pneumonia
D. Fungalin ection
procedures in place to veri y the correct surgical site;or having an organizational culture that lacks teamwork or reveres the surgeon as someone whose judgment should never be questioned.(SeeSchwartz10th ed.,p.378.)
Answer:D
Pharmacologic agents commonly used to stimulate bowel unction include metoclopramide and erythromycin. Metoclopramide’s action is limited to the stomach and duodenum, and it may help primarily with gastroparesis. Erythromycin is a motilin agonist that works throughout the stomach and bowel. Several studies demonstrate signi icant bene it rom the administration o erythromycin in those su ering rom an ileus.Alvimopan,a newer agent and a mu-opioid receptor antagonist, has shown some promise in manystudies or earlier return o gut unction and subsequent reduction in length o stay. Neostigmine has been used in re ractory pan-ileus patients(Ogilviesyndrome) with somedegreeo success.It is recommended or patientsreceivingthistypeo therapyto be in amonitored unit.(SeeSchwartz10th ed.,p.386.)
Answer:D
When patients in the ICU have a major bleed rom stress gastritis, the mortality risk is as high as 50%. It is important to keep the gastric pH greater than 4 to decrease the overall risk or stress gastritis in patients mechanically ventilated or 48 hours or greater and patients who are coagulopathic. Proton pump inhibitors, H2-receptor antagonists, and intragastric antacid installation are all e ective measures. However, patients who are not mechanically ventilated or who do not have a history o gastritis or peptic ulcer disease should not be placed on gastritis prophylaxis postoperativelybecause it carries a higher risk o causing pneumonia. (See Schwartz 10th ed.,p.387.)
Answer:C
A bile leak due to an unrecognized injury to the ducts may present a ter cholecystectomy as a biloma. hese patients may present with abdominal pain and hyperbilirubinemia. hediagnosiso abiliaryleakcan becon irmed bycomputed tomography(C ) scan,endoscopicretrogradecholangiopancreaticogram (ERCP), or radionuclide scan. Once a leak is con irmed,aretrogradebiliarystentandexternaldrainageare thetreatment o choice.(SeeSchwartz10th ed.,p.387.)
Answer:A
he most requent nosocomial in ection is urinary tract in ection (U I). hese in ections are classi ied into complicated and uncomplicated orms. he uncomplicated type is a U I that can be treated with outpatient antibiotic therapy. he complicated U I usually involves a hospitalized patient with an indwelling catheter whose U I is diagnosed as part o a ever workup. he interpretation o urine culture results o lessthan 100,000CFU/mLiscontroversial.Be oretreating suchapatient,oneshouldchangethecatheter andthen repeat the culture to see whether the catheter was simply colonized with organisms. Cultures with more than 100,000 CFU/mL
83
11. Te rst step in treating a 70-kg patient with a platelet count o 12,000 due to heparin-induced thrombocytopenia is
A. Anticoagulation
B. rans usion o our unitso platelets
C. rans usion o eight unitso platelets
D. rans usion o 12unitso platelets
12. VAPinventilatedICUpatientsreachesa70%probabilityat
A. 5days
B. 15days
C. 30days
D. 45days
13. Which o the ollowing is the only thing that has been shown to decrease wound in ections in surgical patients with contaminated wounds?
A. Useo iodophor-impregnated polyvinyldrapes.
B. Salineirrigation o theperitoneum and wound.
C. Antibioticirrigation o theperitoneum and wound.
D. 24hourso appropriateantibioticspostoperatively(in addition topreoperativedose).
should be treated with the appropriate antibiotics and the catheter should be changed or removed as soon as possible. Under-treatmentor misdiagnosiso aU Ican leadtourosepsisand septicshock.(SeeSchwartz10th ed.,p.390.)
Answer:D
hrombocytopenia may require platelet trans usion or a platelet count less than 20,000/mLwhen invasive procedures are per ormed, or when platelet counts are low and ongoing bleeding rom rawsur aceareaspersists.Oneunit o platelets will increase the platelet count by5000 to 7500/mLin adults. It isimportant to delineatethecauseo thelowplatelet count. Usuallythere is a sel -limitingor reversible condition such as sepsis.Rarely,it isdueto heparin-induced thrombocytopenia I and II. Complications o heparin-induced thrombocytopenia II can be serious because o the di use thrombogenic nature o the disorder.Simple precautionsto limit thishypercoagulable state include saline solution lushes instead o heparin solutionsand limitingtheuseo heparin-coatedcatheters. he treatment is anticoagulation with synthetic agents such asargatroban.(SeeSchwartz10th ed.,pp.388–389.)
Answer:C
Pneumoniaisthesecondmostcommon nosocomialin ection and isthemost common in ection in ventilated patients.VAP occurs in 15 to 40% o ventilated ICU patients, with a probability rate o 5% per day, up to 70% at 30 days. he 30-day mortality rate o nosocomial pneumonia can be as high as 40% and depends on the microorganisms involved and the timeliness o initiating appropriate antimicrobials Protocoldriven approaches or prevention and treatment o VAP are recognized asbene icialin managingthesedi icult in ectious complications.(SeeSchwartz10th ed.,pp.384–385.)
Answer:B
No prospective, randomized, double-blind, controlled studies exist that demonstrate antibiotics used beyond 24 hours in the perioperative period prevent in ections. Prophylactic useo antibioticsshould simplynot becontinued beyond this time. Irrigation o the operative ield and the surgical wound with saline solution has shown bene it in controlling wound inoculum.Irrigation withan antibiotic-basedsolution hasnot demonstrated signi icant bene it in controlling postoperative in ection.(SeeSchwartz10th ed.,p.389.)
14. racheostomymaydecreasetheincidenceo VAP,overall length o ventilator time,and the number o ICU patient dayswhen per ormed
A. Be orethe h dayo ventilator support
B. Be orethe10th dayo ventilator support
C. Be orethe15th dayo ventilator support
D. Be orethe20th dayo ventilator support
Answer:B
Although not without risk, tracheostomy decreases the pulmonary dead space and provides or improved pulmonary toilet. When per ormed be ore the 10th day o ventilatory support, tracheostomy may decrease the incidence o VAP, the overall length o ventilator time, and the number o ICU patient days.(SeeSchwartz10th ed.,p.385.)
84
15. Which o the ollowing is a dominant cytokine in the pathogenesis o systemic infammatory response syndrome(SIRS)?
A. Interleukin-2(IL-2)
B. IL-5
C. IL-6
D. IL-7
16. Temost common causeo postrenal ailureis
A. Aclogged urinarycatheter
B. An unintentionalligation o ureters
C. Alargeretroperitonealhematoma
D. Acutekidney ailure
17. Laryngoscopic ndings a er a superior laryngeal nerve injuryinclude
A. Ipsilateralvocalcord in aparamedian position
B. Ipsilateralvocalcord in amiddlingposition
C. Asymmetryo theglotticopening
D. Normalexamination
Answer:C
he systemic in lammatory response syndrome (SIRS) and the multiple organ dys unction syndrome (MODS) carry signi icant mortality risks ( able 12-1). Speci ic criteria have been established or the diagnosis o SIRS ( able 12-2), but twocriteriaarenot required or thediagnosiso SIRS:lowered blood pressureand blood culturespositive or in ection.SIRS is the result o proin lammatory cytokines related to tissue malper usion or injury.
he dominant cytokines implicated in this process include interleukin (IL)-1, IL-6, and tissue necrosis actor ( NF). Other mediators include nitric oxide, inducible macrophagetypenitricoxidesynthase,andprostaglandinI2.(SeeSchwartz 10th ed., ables12-17and 12-18,p.391.)
TABLE12-1 Mortalityassociated with patients exhibiting two ormore criteria orsystemic infammatoryresponse syndrome (SIRS)
PROGNOSIS MORTALITY(%)
TABLE12-2 Inclusion criteria orthe systemic infammatoryresponse syndrome
Temperature>38°Cor<36°C(>100.4°For<96.8°F)
Heartrate>90beats/min
Respiratoryrate>20breaths/minorPaco2 <32mmHg
Whitebloodcellcount<4000or>12,000cells/mm3 or>10% immatureforms
Paco2 = partialpressure ofarterialcarbon dioxide.
Answer:A
Renal ailure can be classi ied as prerenal ailure, intrinsic renal ailure, and postrenal ailure. Postrenal ailure, or obstructive renal ailure, should always be considered when low urine output (oliguria) or anuria occurs. he most common cause is a misplaced or clogged urinary catheter. Other, less common causes to consider are unintentional ligation or transection o uretersduringadi icultsurgicaldissection (eg, colon resection or diverticular disease)or alargeretroperitonealhematoma(eg,rupturedaorticaneurysm).(SeeSchwartz 10th ed.,p.387.)
Answer:C
Superior laryngeal nerve injury is less debilitating, as the common symptom is loss o projection o the voice. he glottis aperture is asymmetrical on direct laryngoscopy, and management islimited to clinicalobservation.(SeeSchwartz 10th ed.,p.384.)
85
2SIRScriteria 5 3SIRScriteria 10 4SIRScriteria 15–20
18. Allo the ollowingaretruestatementsregardingwound in ection EXCEP
A. Irrigation o the operative eld and surgical wound with salinesolution isbene cial.
B. Prophylactic use o antibiotics continued beyond 48hoursisbene cial.
C. Irrigation with an antibiotic-based solution has not been shown tobebene cial.
D. Antibacterial-impregnated polyvinyl placed over the operative wound area or the duration o the surgical procedureisnot bene cial.
Answer:B
No prospective, randomized, double-blind, controlled studies exist that demonstrate antibiotics used beyond 24 hours in the perioperative period prevent in ections. Prophylactic useo antibioticsshould simplynot becontinued beyond this time. Irrigation o the operative ield and the surgical wound with saline solution has shown bene it in controlling wound inoculum.Irrigation withan antibiotic-basedsolution hasnot demonstrated signi icant bene it in controlling postoperative in ection.
Antibacterial-impregnated polyvinyl placed over the operative wound area or the duration o the surgical procedure has not been shown to decrease the rate o wound in ection. Although skin preparation with 70%isopropylalcoholhasthe best bactericidal e ect, it is lammable and could be hazardous when electrocautery is used. he contemporary ormulas o chlorhexidine gluconate with isopropyl alcohol remain moreadvantageous.(SeeSchwartz10th ed.,p.389.)
19. Te most common cause o an empyema in the postoperativepatient is
A. Pneumonia
B. Systemicsepsis
C. Esophagealper oration
D. Retained hemothorax
20. Te primary cause o hyperbilirubinemia in the surgical patient is
A. Sepsis
B. Hematoma rom trauma
C. Cholestasis
D. Increased unconjugated bilirubin duetohemolysis
Answer:A
One o the most debilitating in ections is an empyema, or in ection o the pleural space. Frequently, an overwhelming pneumonia is the source o an empyema, but a retained hemothorax, systemic sepsis, esophageal per oration rom anycause, and in ections with a predilection or the lung(eg, tuberculosis) are potential etiologies as well. (See Schwartz 10th ed.,pp.390–391.)
Answer:C
Hyperbilirubinemia in the surgical patient can be a complex problem. Cholestasis makes up the majority o causes or hyperbilirubinemia, but other mechanisms o hyperbilirubinemia include reabsorption o blood (eg, hematoma rom trauma), decreased bile excretion (eg, sepsis), increased unconjugated bilirubin due to hemolysis, hyperthyroidism, and impaired excretion due to congenital abnormalities or acquired disease. Errors in surgery that cause hyperbilirubinemia largely involve missed or iatrogenic injuries. (See Schwartz10th ed.,p.387.)
86
CHAPTER
PhysiologicMonitoring ofthe SurgicalPatient
1. Tepoint o criticaloxygen delivery(DO2crit)
A. Represents the transition rom supply-independent to supply-dependent oxygen uptake and is decreased in sepsis.
B. Representstheminimalrateo oxygen deliveryneeded or aerobicmetabolism and isdecreased in sepsis.
C. Represents the transition rom supply-independent to supply-dependent oxygen uptake and is increased in sepsis.
D. Representstheminimalrateo oxygen deliveryneeded or aerobicmetabolism and isincreased in sepsis.
Answer:C
Under normalconditionswhen thesupplyo oxygen isplentiul, aerobic metabolism is determined by actors other than the availability o oxygen. However, in pathologic circumstanceswhen oxygen availabilityisinadequate,oxygen utilization (VO2) becomes dependent upon oxygen delivery (DO2). he relationship o VO2 to DO2 over a broad range o DO2 values is commonly represented as two intersecting straight lines. In the region o higher DO2 values, the slope o the line is approximatelyzero, indicatingthat VO2 is largelyindependent o DO2. In contrast, in the region o lowDO2 values, the slope o the line is nonzero and positive, indicating that VO2 is supply-dependent. he region where the two lines intersect is called the point ofcritical oxygen delivery(DO2crit), and representsthetransition rom supply-independent to supplydependent oxygen uptake. Microcirculatory derangements, suchasthoseseeninsepsis,willshi tthispointhigher.Belowa criticalthreshold o oxygen delivery,increased oxygen extraction cannotcompensate or thedeliveryde icit;hence,oxygen consumption begins to decrease. he slope o the supplydependent region o the plot re lects the maximal oxygen extraction capability o the vascular bed being evaluated. (See Schwartz10thed.,p.400.)
2. O the ollowingparameters, which is the least in uenced by an underdamped or overdamped intra-arterial blood pressuremonitoringsystem?
A. Systolicblood pressure
B. Mean arterialblood pressure
C. Diastolicblood pressure
D. Pulsepressure
Answer:B
I the system is underdamped, then the inertia o the system, which is a unction o the mass o the luid in the tubing and the mass o the diaphragm, causes overshoot o the points o maximum positive and negative displacement o the pressure transducer diaphragm during systole and diastole, respectively. hus, in an underdamped system, systolic pressure will be overestimated and diastolic pressure will be underestimated. In an overdamped system, displacement o the diaphragm ailsto tracktherapidlychangingpressurewave orm, andsystolicpressurewillbeunderestimatedanddiastolicpressure willbe overestimated.It isimportant to notethat even in an underdamped or overdamped system, mean pressure will beaccuratelyrecorded,provided thesystem hasbeen properly calibrated.For thesereasons,when usingdirect measurement o intra-arterialpressuretomonitor patients,cliniciansshould makeclinicaldecisionsbasedprimarilyonthemeasuredmean arterialbloodpressure.(SeeSchwartz10th ed.,p.401.)
87 13
3. Regarding electrocardiographic (ECG) monitoring in theICU
A. A standard 3-lead ECG will detect 95% o ischemia, whereasa12-lead ECGwilldetect greater than 98%.
B. Lead V4 is the most sensitive or detecting perioperativeischemia.
C. Astandard3-leadECGwilldetectischemiaatthesame rateasa12-lead ECG;however,it isin erior at identi yingdysrhythmias.
D. Lead V2 is the most sensitive or detecting perioperativeischemia.
4. Regardingpreload,which o the ollowingistrue?
A. It is approximated by the systemic vascular resistance which is calculated bydividing mean arterial pressure (MAP)bycardiacoutput.
B. It is approximated by the right ventricular end-diastolic pressure (EDP) as estimated with pulmonary arteryocclusion pressure.
C. It isapproximated bytheright ventricular EDP asestimated with centralvenouspressure(CVP).
D. It is approximated by the le ventricular EDP as estimated with pulmonaryarteryocclusion pressure.
5. Allo the ollowingaretrueEXCEP
A. Terelationship between EDPand preload islinear.
B. EDP isdetermined byboth volumeand complianceo theventricle.
C. Te relationship between EDP and end-diastolic volume (EDV) can be changed with pharmacologic agents.
D. EDP is o en used as a surrogate or EDV because it is easier toapproximatein theclinicalsetting.
6. Teend-systolicpressure-volumeline
A. Provides a good estimation o le ventricular compliance.
B. Uses small changes in preload and a erload between cardiac cycles to determine contractility, which is represented bythex-intercept o theline.
C. Te slope will become steeper i contractility is increased.
D. Requires preload to be held approximatelyconstant to bemeasured.
7. Te thermodilution technique or determining cardiac output
A. CalculatesQ with theFickequation
B. Underestimatescardiacoutput at lowvalues
C. Should be per ormed with a cold indicator liquid to increasethesignal-to-noiseratio
D. Is in uenced by respiratory cycle due to changes in blood temperatureand Q
Answer:B
Continuous monitoring o the 12-lead ECG is now available in manyICUsand isprovingtobebene icialin certain patient populations.In a studyo 185vascular surgicalpatients,continuous 12-lead ECG monitoring was able to detect transient myocardial ischemic episodes in 20.5% o the patients. his study demonstrated that the precordial lead V4, which is not routinely monitored on a standard 3-lead ECG, is the most sensitive or detecting perioperative ischemia and in arction. o detect 95% o the ischemic episodes, two or more precordial leads were necessary. hus, continuous 12-lead ECG monitoring may provide greater sensitivity than 3-lead ECG or the detection o perioperative myocardial ischemia, and may become standard or monitoring high-risk surgical patients.(SeeSchwartz10th ed.,p.401.)
Answer:D
For therightventricle,centralvenouspressure(CVP)approximatesright ventricular end-diastolicpressure(EDP).For the le t ventricle, pulmonary artery occlusion pressure (PAOP), which is measured by transiently in lating a balloon at the end o a pressure-monitoring catheter positioned in a small branch o thepulmonaryartery,approximatesle t ventricular EDP. he presence o atrioventricular valvular stenosis may alter this relationship. Le t ventricular EDP is the most commonlyused proxy or preload.(SeeSchwartz10th ed.,p.402.)
Answer:A
Clinicians requentlyuseEDP asa surrogate or end-diastolic volume(EDV),butEDPisdeterminednotonlybyvolumebut also by the diastolic compliance o the ventricular chamber. Ventricular compliance is altered by various pathologic conditionsand pharmacologicagents.Furthermore,therelationship between EDP and true preload is not linear,but rather is exponential.(SeeSchwartz10th ed.,p.402.)
Answer:C
I pressure-volume loops are constructed or each cardiac cycle, small changes in preload and/or a terload will result in shi ts o the point de ining the end o systole. hese endsystolicpointsonthepressureversusvolumediagramdescribe a straight line,known as the end-systolicpressure-volumeline. Asteeper slopeo thislineindicatesgreater contractility.(See Schwartz10th ed.,p.402.)
Answer:D
he relationship used by the thermodilution technique or calculatingQ iscalled theStewart-Hamilton equation: Q =[V× ( B – I)×K1 × K2]/∫ B(t)dt
where V is the volume o the indicator injected, B is the temperature o blood, I is the temperature o the indicator, K1 is a constant that is the unction o the speci ic heats o blood and the indicator, K2 is an empirically derived constant, and ∫ B(t)dt is the area under the time-temperature
88
8. All o the ollowing are true regarding the ractional saturation o hemoglobin in mixed venous blood (SVO2) EXCEP
A. It willdecreasewith worseningheart ailure.
B. It willdecreasewith increased sedation.
C. It willdecreasewith worseninganemia.
D. It willdecreasewith ever.
curve.Determination o cardiacoutputbythethermodilution method is generally quite accurate, although it tends to systematically overestimate Q at low values. Changes in blood temperature and Q during the respiratory cycle can in luence the measurement. here ore,resultsgenerallyshould be recorded asthemean o twoor threedeterminationsobtained at random pointsin therespiratorycycle.Usingcold injectate widensthedi erencebetween B and I andtherebyincreases signal-to-noise ratio. Nevertheless, most authorities recommend usingroom temperatureinjectate(normalsalineor 5% dextrosein water)tominimizeerrorsresulting rom warming o the luidasitistrans erred rom itsreservoir toasyringe or injection.(SeeSchwartz10th ed.,p.404.)
Answer:B
he Fick equation or cardiac output can be rearranged as ollows:
CVO2 =Cao2 –VO2/Q
I the small contribution o dissolved oxygen to CVO2 and Cao2 isignored,therearranged equation can berewritten as:
SVO2 = Sao2 – VO2/(Q ×Hgb×1.36)
where SVO2 is the ractional saturation o hemoglobin in mixed venous blood, Sao2 is the ractional saturation o hemoglobin in arterialblood,and Hgb istheconcentration o hemoglobin in blood. husit can beseen that SVO2 isa unction o VO2(ie, metabolic rate), Q , Sao2, and Hgb. Accordingly, subnormal values o SVO2 can be caused by a decrease in Q (eg, due to heart ailure or hypovolemia), a decrease in Sao2 (eg, due to intrinsic pulmonary disease), a decrease in Hgb (ie, anemia), or an increase in metabolic rate (eg, due to seizuresor ever).(SeeSchwartz10th ed.,p.404.)
9. Te Surviving Sepsis Campaign guidelines recommend whicho the ollowingregardingtheinitialresuscitation o sepsis-induced hypoper usion?
A. GoalMAP≥60mm Hg.
B. GoalSVO2 o 80%.
C. Goalurineoutput ≥ 1mL/kg/h.
D. Tat goals o resuscitation be met within the rst 6 hourso management.
10. Noninvasivemethodso measuringcardiacoutput
A. Allow or continuousmeasurement o Q
B. Show excellent correlation with Q as measured by thermodilution
C. Haverarelybeen adopted into clinicalpracticedueto theincreased trainingburden
D. Havesimilar complication ratesastheuseo apulmonaryarterycatheter
Answer:D
he Surviving Sepsis Campaign guidelines or the management o severe sepsis and septic shock recommends that during the irst 6 hours o resuscitation, the goals o initial resuscitation o sepsis-induced hypoper usion should include all o the ollowing: CVP 8 to 12 mm Hg, MAP ≥65 mm Hg, urine output ≥0.5 mL/kg/h. ScVO2 o 70% or SVO2 o 65%.
(SeeSchwartz10th ed.,p.405.)
Answer:A
Noninvasive methods o monitoring cardiac output include impedance cardiography and pulse contour analysis among others. Impedance cardiography is attractive because it is noninvasive, provides a continuous readout o Q , and does not require extensive training. However, measurements o Q obtained by impedance cardiography are not su iciently reliableto beused or clinicaldecision-makingand havepoor correlation with thermodilution. Measurements o Q based on pulse contour monitoring are comparable in accuracy to standard PAC thermodilution methods, but are less invasive since transcardiac catheterization is not needed. he use o pulse contour analysis hasbeen applied usingnoninvasivephotoplethysmographicmeasurementso arterialpressure.However,the accuracyo this technique has been questioned and its clinical utilityremainstobedetermined.(SeeSchwartz10thed.,p.408.)
89
11. Usingpulse pressure variability(PPV) to determine preload responsiveness
A. Is reliable or a patient in rate-controlled atrial brillation,but not or apatient in atrial utter.
B. Is a better predictor o preload responsiveness than CVP.
C. De nes PPV as the diference between the maximal pulsepressureandtheminimalpulsepressureobserved at diferentpointsin therespiratorycycle.
D. Is unreliable in mechanically ventilated patients due todecreased venousreturn duringinspiration.
Answer:B
When intrathoracic pressure increases during the application o positive airway pressure in mechanically ventilated patients, venous return decreases and, as a consequence, le t ventricular stroke volume also decreases. here ore, pulse pressure variation (PPV) during a positive pressure episode can beused to predict theresponsivenesso cardiacoutput to changes in preload. PPV is de ined as the di erence between the maximal pulse pressure and the minimal pulse pressure divided by the average o these two pressures. his approach has been validated by comparing PPV, CVP, PAOP, and systolic pressure variation as predictors o preload responsiveness in a cohort o critically ill patients. Receiver-operating characteristic curves demonstrated that PPV was the best predictor o preload responsiveness. Although atrial arrhythmias can inter ere with the use ulness o this technique, PPV remains a use ul approach or assessing preload responsiveness in most patients because o its simplicity and reliability. (SeeSchwartz10th ed.,p.408.)
12. Strategies or increasingoxygen deliveryin mechanically ventilated,criticallyillpatientsinclude
A. IncreasingSao2 byincreasinginspiratorytime
B. IncreasingSao2 byincreasingrespiratoryrate
C. Increasing SVO2 by switching to a reversed inspiratorytoexpiratoryratioventilation strategy
D. IncreasingSVO2 byincreasingpositiveend-expiratory pressure(PEEP)
Answer:A
Sao2 in mechanicallyventilated patientsdependson themean airway pressure, the raction o inspired oxygen (Fio2), and SVO2. hus,when Sao2 is low,the clinician has onlya limited number o ways to improve this parameter. he clinician can increase mean airway pressure by increasing positive-end expiratory pressure (PEEP) or inspiratory time. Fio2 can be increased to a maximum o 1.0 by decreasing the amount o room air mixed with the oxygen supplied to the ventilator. SVO2 can beincreased byincreasingHgb or Q or decreasing oxygen utilization (eg,byadministeringamusclerelaxantand sedation).(SeeSchwartz10th ed.,p.409.)
13. All o the ollowing are true regarding airway pressures
EXCEP
A. Bronchospasm will cause increased peak pressure with arelativelynormalplateau pressure.
B. Pneumothoraxwillcause increased peak and plateau pressures.
C. Lobar atelectasis will cause increased plateau pressureswith relativelynormalpeakpressures.
D. Plateau pressureisindependent o airwayresistance.
Answer:C
he peak airway pressure measured at the end o inspiration (Ppeak) is a unction o the tidal volume, the resistance o the airways, lung/chest wall compliance, and peak inspiratory low. he airway pressure measured at the end o inspiration when the inhaled volume is held in the lungs by brie ly closingthe expiratoryvalve is termed the plateau airwaypressure (Pplateau).Asastaticparameter,plateau airwaypressureisindependent o the airwayresistance and peak airway low, and is related to the lung/chest wall compliance and delivered tidal volume.I bothPpeak andPplateau areincreased(andtidalvolume isnot excessive),then theunderlyingproblem isadecreasein the compliance in the lung/chest wall unit. Common causes o this problem include pneumothorax, hemothorax, lobar atelectasis, pulmonary edema, pneumonia, acute respiratory distress syndrome (ARDS), active contraction o the chest wall or diaphragmatic muscles, abdominal distention, and intrinsicPEEP,such asoccursin patientswith bronchospasm and insu icient expiratorytimes. When Ppeak is increased but Pplateau isrelativelynormal,theprimaryproblem isan increase in airway resistance, such as occurs with bronchospasm, use o asmall-caliber endotrachealtube,or kinkingor obstruction o theendotrachealtube.(SeeSchwartz10th ed.,pp.409–410.)
90
14. Causeso an increasein end-tidal-CO2 include
A. Massivepulmonaryembolism
B. Reduced cardiacoutput
C. Sustained hyperventilation
D. Reduced minuteventilation
15. Whicho the ollowingisNO an indication or intracranialpressure(ICP)monitoring?
A. Glasgow Coma Scale (GCS) less than or equal to 8 with an abnormalcomputed tomography(C )scan.
B. Severe traumatic brain injury ( BI) in a patient older than 40 years and systolic blood pressure less than 90mm Hg.
C. Intracranial hemorrhage without intraventricular blood.
D. Fulminant hepatic ailure with coma and cerebral edemaon C .
Answer:D
Causes o an increase in Petco2 include reduced minute ventilation or increased metabolic rate. Sudden reduction in end-tidal-CO2 (Petco2)suggestseitherobstruction o thesamplingtubingor a catastrophicevent such aslosso theairway, airway disconnection or obstruction, ventilator mal unction, or a marked decrease in Q . I the airway is connected and patient and the ventilator is unctioning properly, then a sudden decrease in Petco2 should prompt e orts to rule out cardiac arrest, massive pulmonary embolism, or cardiogenic shock. Petco2 can be persistentlylowduring hyperventilation or with an increase in dead space such asoccurs with pulmonary embolization (even in the absence o a change in Q ). (SeeSchwartz10th ed.,p.410.)
Answer:C
Monitoring o intracranial pressure (ICP) currently is recommended in patients with severe BI, de ined as a Glasgow ComaScale(GCS) scorelessthan or equalto 8with an abnormal C scan, and in patients with severe BI and a normal C scan i two or more o the ollowingare present:age older than 40 years, unilateral or bilateral motor posturing, or systolic blood pressure less than 90 mm Hg. ICP monitoring is indicated in patients with acute subarachnoid hemorrhage with coma or neurologic deterioration, intracranial hemorrhage with intraventricular blood, ischemic middle cerebral artery stroke, ulminant hepatic ailure with coma and cerebraledemaon C scan,andglobalcerebralischemiaor anoxia withcerebraledemaonC scan. hegoalo ICPmonitoringis toensurethat cerebralper usion pressure(CPP)isadequateto support per usion o thebrain.(SeeSchwartz10th ed.,p.411.)
16. Currently accepted uses o transcranial Doppler ( CD) includeallo the ollowingEXCEP
A. Diagnosingvasospasma ersubarachnoidhemorrhage
B. Estimatingcerebralper usion pressure
C. Con rming brain death a er clinical examination in patients under the in uence o central nervous system (CNS)depressants.
D. Con rming brain death a er clinical examination in patientswith metabolicencephalopathy.
Answer:B
ranscranial Doppler ( CD) measurements o middle and anterior cerebral artery blood low velocity are use ul or the diagnosis o cerebral vasospasm a ter subarachnoid hemorrhage. Qureshi et al demonstrated that an increase in the middlecerebralarterymean lowvelocityasassessed by CD is an independent predictor o symptomatic vasospasm in a prospective study o patients with aneurysmal subarachnoid hemorrhage. In addition, while some have proposed using CD to estimate ICP, studies have shown that CD is not a reliable method or estimating ICP and CPP, and currently cannot be endorsed or this purpose. CD also is use ul to con irm theclinicalexamination or determiningbrain death in patients with con ounding actors such as the presence o CNSdepressantsor metabolicencephalopathy.(SeeSchwartz 10th ed.,pp.411–412.)
17. Regardingjugular venousoximetryin patientswith BI
A. It requiresplacement o acatheter in thejugular bulb.
B. Low jugular venous oxygen saturation (Sjo2) has not been showtopredict poor clinicaloutcomes.
C. It is less invasive than placing an intraventricular monitor, but does not allow or continuous monitoring.
D. It can replaceICPmonitoringin patientswithout evidenceo regionalvariation in cerebralblood ow.
Answer:A
When the arterial oxygen content, hemoglobin concentration,and the oxyhemoglobin dissociation curve are constant, changes in jugular venous oxygen saturation (Sjo2) re lect changes in the di erence between cerebral oxygen delivery and demand. Generally, a decrease in Sjo2 re lects cerebral hypoper usion,whereasan increasein Sjo2 indicatesthepresence o hyperemia. Sjo2 monitoring cannot detect decreases in regional cerebral blood low i overall per usion is normal or above normal. his technique requires the placement o a
91
18. Monitoring local brain tissue oxygen tension (PbtO2) in patientswith severe BI
A. Has shown that normal ICP and CPP generally precludesthepresenceo brain tissueischemia
B. Has been shown to lower mortality when compared with ICPmonitoringalone
C. Has not been adopted into routine clinical practice due to additional adverse efects rom additional, potentiallyunnecessary,interventions
D. Hasbeen shown toincreasestrokerateasacomplication o catheter placement
catheter in the jugular bulb, usually via the internal jugular vein. Catheters that permit intermittent aspiration o jugular venous blood or analysis or continuous oximetry catheters are available. LowSjo2 is associated with poor outcomes a ter BI. Nevertheless, the value o monitoring Sjo2 remains unproven.I it isemployed,it should not bethesolemonitoring technique, but rather should be used in conjunction with ICP and CPP monitoring. By monitoring ICP, CPP, and Sjo2, earlyintervention withvolume,vasopressors,andhyperventilation has been shown to prevent ischemic events in patients with BI.(SeeSchwartz10th ed.,p.412.)
Answer:B
Whilethestandardo care orpatientswithsevere BIincludes ICP and CPP monitoring, this strategy does not always prevent secondary brain injury. Growing evidence suggests that monitoring local brain tissue oxygen tension (PbtO2) may be a use ul adjunct to ICP monitoring in these patients. Normal values or PbtO2 are 20 to 40 mm Hg, and critical levels are 8 to 10 mm Hg. A recent clinical study sought to determine whether the addition o a PbtO2 monitor to guide therapy in severe BI was associated with improved patient outcomes. Mortalitywassigni icantlylower in thepatientswhohadtherapyguided byPbtO2 monitoring in addition to ICP and CPP (25%;P<0.05). hebene itso PbtO2 monitoringmayinclude the early detection o brain tissue ischemia despite normal ICP and CPP. In addition, PbtO2-guided management may reduce potential adverse e ects associated with therapies to maintain ICPand CPP.(SeeSchwartz10th ed.,p.412.)
92
MinimallyInvasive Surgery
1. Temost common arrhythmiaseen duringlaparoscopyis
A. Atrial brilation
B. Sinustachycardia
C. Prematureventricular contractions
D. Sinusbradycardia
2. Capacitivecoupling
A. Resultswhen energybleeds rom a port sleeveor laparoscopeintoadjacent (but not touching)bowel
B. Isalwaysrecognized at thetimeo surgery
C. Can result in mal unction o the electrocardiogram monitor
D. Can result in inaccurateimagetransmission tothedigitalmonitor
Answer:D
he pressure e ects o the pneumoperitoneum on cardiovascular physiology also have been studied. In the hypovolemic individual, excessive pressure on the in erior vena cava and a reverse rendelenburg position with loss o lower extremity muscle tone may cause decreased venous return and cardiac output. his is not seen in the normovolemic patient. he mostcommon arrhythmiacreatedbylaparoscopyisbradycardia.Arapid stretch o theperitonealmembraneo ten causesa vagovagal response with bradycardia and occasionally hypotension. he appropriate management o this event is desulation o theabdomen,administration o vagolyticagents(eg, atropine), and adequate volume replacement. (See Schwartz 10th ed.,p.418.)
Answer:A
o avoid thermal injury to adjacent structures, the laparoscopic ield o view must include all uninsulated portions o the electrosurgical electrode. In addition, the integrity o the insulation must be maintained and assured. Capacitive coupling occurs when a plastic trocar insulates the abdominal wall rom the current; in turn the current is bled o a metal sleeve or laparoscope into the viscera (Fig. 14-1). his may result in thermalnecrosisand a delayed ecal istula.Another potential mechanism or unrecognized visceral injury may occur with the direct coupling o current to the laparoscope and adjacent bowel. (See Schwartz 10th ed., Figure 14-7, pp.427–428.)
93 14
CHAPTER
3. Which o the ollowing are true regarding sa e laparoscopicsurgeryin pregnancy.
A. Te patient should be position in the lef lateral position.
B. Open abdominal access (Hasson) is recommended versusdirect puncturelaparoscopy(Veressneelde).
C. Te surgery should be per ormed during the second trimester i possible.
D. Allo theabove.
4. Systemic e ects o CO2 rom pneumoperitoneum can causeallo the ollowingEXCEP
A. Hypercarbia
B. Increased myocardialoxygen demand
C. Alterationsin preload
D. Increased afer load
Answer:D
Concerns about the sa ety o laparoscopic cholecystectomy and appendectomy in the pregnant patient have been thoroughlyinvestigated and readilymanaged.Accessto theabdomen in the pregnant patient should take into consideration theheight o the uterine undus,which reachesthe umbilicus at 20 weeks. In order not to damage the uterus or tis blood supply, most surgeons eel that the open (Hasson) approach should be used in avor o direct puncture laparoscopy. he patient should be positioned slightly on the le t side to avoid compression o thevenacavabytheuterus.Becausethepregnancyposesarisk orthromboembolism,sequentialcompression devices are essential or all procedures. Surgery should be per ormed in the second trimester, i possible. Protection o the etus against intraoperative X-rays is imperative. (See Schwartz10th ed.,pp.435–436.)
Answer:C
Alterations in preload are local e ects (pressure speci ic) o CO2 peritoneum.
he physiologic e ects o CO2 pneumoperitoneum can be dividedintotwoareas(1)gas-speci ice ectsand(2)pressurespeci ic e ects (Fig. 14-2). CO2 is rapidly absorbed across the peritoneal membrane into the circulation. In the circulation, CO2 creates a respiratory acidosis by the generation o carbonic acid. Body bu ers, the largest reserve o which lies in bone, absorb CO2 (up to 120 L) and minimize the development o hypercarbia or respiratory acidosis during brie endoscopic procedures. Once the body bu ers are saturated, respiratoryacidosis develops rapidly, and the respiratorysystem assumesthe burden o keepingup with theabsorption o CO2 and itsrelease rom thesebu ers.
In patients with normal respiratory unction, this is not di icult;the anesthesiologist increases the ventilatoryrate or
94
Capacitivecoupled
faultcondition
Capacitively coupledenergy tometal cannula
Plasticcollar overmetal trocar
FIG.14-1. Capacitive coupling occursasa result ofhigh current densitybleeding from a port sleeve orlaparoscope into adjacent bowel.(Reproduced with permission from OdellRC.Laparoscopic electrosurgery,in HunterJG,SackierJM,eds.MinimallyInvasive Surgery.NewYork:McGraw-Hill,1993,p 33.)
5. While per orming a laparoscopic Nissen undoplication duringthe transhiatal dissection the mediastinal pleura is compromised and aCO2 pneumothoraxdevelops.What is theinitialpre erred management o thepneumothorax?
A. Needle thoracostomy over the second intercostal space,mid-clavicular line.
B. Enlargement o the de ect and placement o an 18-French red rubber catheter acrossthede ect.
C. Abort theprocedureand emergent tubethoracostomy with a28-French chest tube.
D. No intervention is needed.Continue with the planned procedure.
6. When compared to traditional laparoscopic surgery, the advantageso computer-enhanced surgeryare
A. Natural wrist movements and improved manual dexterity
B. Ergonomically com ortable workstation with 3-D imaging
C. remor elimination
D. Allo theabove
Local effects
Peritoneal distention
Vagal reaction
Elevated diaphragm
Altered venous return Pain
Systemic effects
Hypercarbia
Acidosis
Increased afterload
Increased catecholamines
Myocardial stress
FIG.14-2. Carbon dioxide gasinsufflated into the peritoneal cavityhasboth localand systemiceffectsthat cause a complex set ofhemodynamicand metabolic alterations.(Reproduced with permission from HunterJG,ed.Baillière’sClinicalGastroenterology: LaparoscopicSurgery.London/Philadelphia:BaillièreTindall,1993, p.758.Copyright Elsevier.)
vitalcapacityon theventilator.I therespiratoryraterequired exceeds 20 breaths per minute, there may be less e icient gas exchange and increasing hypercarbia. Conversely, i vital capacityis increased substantially, there is a greater opportunity or barotrauma and greater respiratory motion-induced disruption o the upper abdominal operative ield. In some situations, it is advisable to evacuate the pneumoperitoneum or reduce the intra-abdominal pressure to allow time or the anesthesiologist to adjust or hypercarbia. Although mild respiratory acidosis probably is an insigni icant problem, more severe respiratory acidosis leading to cardiac arrhythmias has been reported. Hypercarbia also causes tachycardia and increased systemic vascular resistance, which elevates blood pressure and increases myocardial oxygen demand. (SeeSchwartz10th ed.,Figure14-1,pp.417–418.)
Answer:B
When a pneumothoraxoccurs with laparoscopic Nissen undoplication or Heller myotomy, it is pre erable to place an 18-French red rubber catheter with multiple holes cut out o the distal end across the de ect. At the end o the procedure, the distal end o the tube is pulled out a 10-mm port side (as the port is removed), and the pneumothorax is evacuated toaprimitivewater-sealusingabowlo sterilewater or saline. (SeeSchwartz10th ed.,p.419.)
Answer:D
he major revolution in robotic surgery was the development o a master-slave surgical plat orm that returned the wrist to laparoscopic surgeryand improved manual dexterity by developing ergonomically com ortable workstation, with 3-D imaging, tremor elimination, and scaling o movements (eg,large,grosshand movementscan bescaled down to allow suturingwith microsurgicalprecision). hemost recent iteration o the robotic plat orm eatures a second console slave enabling greater assisting and teaching opportunities. (See Schwartz10th ed.,p.429.)
95
CO2
7. A patient undergoing laparoscopic colon resection is noted to have decreased urine output duringthe last hour o thecase.Abolusisgiven attheendo thecase.Onehour later, there is still verypoor urine output. Te appropriate treatment is
A. Repeat bolus
B. Intravenous(IV) urosemide
C. Checkurineelectrolytes
D. Noneo theabove
Answer:D
Low urine output is a normal physiologic response to increased intra-abdominalpressure or up to 1hour a ter surgery. Although the e ect o the pneumoperitoneum on renal blood loware immediatelyreversible, the hormonallymediated changes such as elevated antidiuretic hormone levels decreaseurineoutput or up to 1hour a ter theprocedurehas ended.Intraoperativeoliguriaiscommon duringlaparoscopy, but the urineoutput isnot a re lection o intravascular status; intravenous (IV) luid administration during an uncomplicated laparoscopic procedure should not be linked to urine output.(SeeSchwartz10th ed.,p.418.)
96
Molecularand GenomicSurgery
1. Te process that occurs during translational control o eukaryoticgeneexpression is
A. Protein degradation
B. RNAprocessing
C. Posttranslationalcontrol
D. ranscription
2. All o the ollowing transcription mechanisms occur in eukaryotesEXCEP
A. Chromatin structure changes to allow DNA to be accessibletothepolymerase.
B. TreeseparateRNApolymerasesareinvolved.
C. Proteinsor initiation actorsarenot required.
D. O en packaged with histone and nonhistone proteins intochromatins.
Answer:A
Four major stepsin thecontrolo eukaryoticgeneexpression (Fig. 15-1). ranscriptional and posttranscriptional control determinethelevelo messenger RNA(mRNA) that isavailable to make a protein, while translational and posttranslational control determine the inal outcome o unctional proteins.Notethat posttranscriptionaland posttranslational controls consist o several steps. (See Schwartz 10th ed., Figure16-6,p.446.)
Answer:C
ranscription mechanisms in eukaryotes di er rom those in prokaryotes. he unique eatures o eukaryotic transcription are as ollows: (1) hree separate RNA polymerases are involved in eukaryotes: RNA polymerase I transcribes the precursor o 5.8S, 18S, and 28S rRNAs; RNA polymerase II synthesizestheprecursorso messenger RNA(mRNA)aswell as microRNA;and RNApolymerase III makes tRNAs and 5S
97
CHAPTER15
Nucleus Cytoplasm DNA RNA transcript mRNA mRNA Protein Active protein mRNA turnover Protein turnover Transcription RNA transport Transcriptional control Posttranscriptional control Translational control Posttranslational control Nuclearenvelope RNA degradation Protein degradation Posttranslational modification Translation RNA processing
FIG.15-1. Fourmajorstepsin the controlofeukaryoticgene expression.Transcriptionaland posttranscriptionalcontroldetermine the level ofmessengerRNA(mRNA)that isavailable to make a protein,while translationaland posttranslationalcontroldetermine the finaloutcome of functionalproteins.Note that posttranscriptionaland posttranslationalcontrolsconsist ofseveralsteps.
3. Tehuman genomecontainsapproximately
A. 35,000to40,000genes
B. 20,000to25,000genes
C. 25,000to30,000genes
D. 30,000to35,000genes
4. I cyclin-dependent kinase (CDK) is to a cell as an engine istoacar,then cyclinsand CKIare
A. Tekeyand ignition,respectively.
B. Tegaspedaland brakes,respectively.
C. Tedistributor and thesparkplug,respectively.
D. Tewindowsand thetires,respectively.
rRNAs. (2) In eukaryotes, the initial transcript is o ten the precursor to inal mRNAs, tRNAs, and rRNAs. he precursor isthen modi ied and/or processed into its inal unctional orm. RNA splicing is one type o processing to remove the noncoding introns (the region between coding exons) on an mRNA. (3) In contrast to bacterial DNA, eukaryotic DNA o ten is packaged with histone and nonhistone proteins into chromatins. ranscription will only occur when the chromatin structure changes in such a way that DNA is accessible to the polymerase. (4) RNA is made in the nucleus and transported into cytoplasm, where translation occurs. here ore, unlike bacteria, eukaryotes undergo uncoupled transcription and translation.(SeeSchwartz10th ed.,p.447.)
Answer:C
Genomeisacollectiveterm or allgenespresent in oneorganism. hehuman genomecontainsDNAsequenceso 3billion base pairs, carried by 23 pairs o chromosomes. he human genome has an estimated 25,000 to 30,000 genes, and overall,it is99.9%identicalin allpeople.Approximately3million locations where single-base DNA di erences exist have been identi ied and termed singlenucleotidepolymorphisms.Single nucleotide polymorphisms may be critical determinants o human variation in disease susceptibility and responses to environmental actors.(SeeSchwartz10th ed.,p.449.)
Answer:B
hecellcycleisconnected with signaltransduction pathways aswellasgeneexpression.Although theSandMphasesrarely are subjected to changes imposed byextracellular signals, the G1 and G2 phases are the primary periods when cells decide whether or not to move on to the next phase. During the G1 phase, cells receive green- or red-light signals, Sphase entry or G1arrest,respectively.Growingcellsproli erateonlywhen supplied with appropriate mitogenic growth actors. Cells become committed to entry o the cell cycle only toward the end o G1. Mitogenic signals stimulate the activity o early G1 cyclin-dependent kinases (CDKs) (eg, cyclin D/CDK4) that inhibit the activity o pRb protein and activate the transcription actor called E2Fto induce the expression o batteries o genes essential or G1-S progression. Meanwhile, cells also receiveantiproli erativesignalssuch asthose rom tumor suppressors. hese antiproli erative signals also act in the G1 phase to stop cells’progress into the Sphase byinducingCKI production. For example, when DNA is damaged, cells will repair the damage be ore entering the Sphase. here ore, G1 contains one o the most important checkpoints or cell cycle progression.I the analogyis made that CDKis to a cellas an engine is to a car, then cyclins and CKI are the gas pedal and brakes,respectively.Acceleratedproli eration orimproper cell cycle progression with damaged DNA would be disastrous. Genetic gain-o - unction mutations in oncogenes (that o ten promoteexpression or activityo thecyclin/CDKcomplex)or loss-o - unction mutations in tumor suppressor (that stimulateproduction o CKI)arecausal actors or malignant transormation.(SeeSchwartz10th ed.,p.450.)
98
5. Incellularapoptosis,thereleaseo cytochromecactivatesthe
A. Fasreceptor
B. Death receptor
C. umor necrosis actor receptor
D. Caspasecascade
Answer:D
Asimpli iedviewo theapoptosispathways(Fig.15-2).Extracellular death receptor pathways include the activation o Fas and tumor necrosis actor ( NF) receptors, and consequent activation o the caspase pathway. Intracellular death pathwayindicatesthereleaseo cytochromec rom mitochondria, which alsotriggerstheactivation o thecaspasecascade.Duringapoptosis,cellsundergo DNA ragmentation,nuclear and cell membrane breakdown, and are eventually digested by other cells.(SeeSchwartz10th ed,Figure15-8,p.451.)
FIG.15-2. Asimplified viewofthe apoptosispathways.Extracellulardeath receptorpathwaysinclude the activation ofFasand tumornecrosisfactor(TNF) receptors,and consequent activation ofthe caspase pathway.Intracellular death pathwayindicatesthe release ofcytochrome c from mitochondria, which also triggersthe activation ofthe caspase cascade.During apoptosis,cells undergo DNAfragmentation,nuclear and cellmembrane breakdown,and are eventuallydigested byothercells.
6. Dysregulation o trans orminggrowth actor-beta( GF-β) signalingisassociated with allo the ollowingEXCEP
A. Cancer
B. Inguinalhernias
C. Mar an syndrome
D. Toracicaorticaneurysm
Answer:B
Resistance to trans orming growth actor-beta’s ( GF-β) anticancer action is one hallmark o human cancer cells. GF-β receptorsand SMADsareidenti ied astumor suppressors. he GF-βsignalingcircuitcanbedisruptedinavarietyo waysand in di erent types o human tumors. Some lose GF-β responsiveness through downregulation or mutations o their GF-β receptors. he cytoplasmic SMAD4 protein,which transduces signals rom ligand-activated GF-β receptors to downstream targets, may be eliminated through mutation o its encoding gene. he locus encoding cell cycle inhibitor p15INK4B may be deleted. Alternatively, the immediate downstream target o its actions, CDK4, may become unresponsive to the inhibitory actions o p15INK4B because o mutations that block p15INK4B binding. he resulting cyclin D/CDK4 complexes constitutively inactivate tumor suppressor pRb by hyperphosphorylation.Finally, unctionalpRb,theendtargeto thispathway,maybe lost through mutation o its gene.For example, in pancreatic and colorectal cancers, 100% o cells derived rom these cancers carry genetic de ects in the GF-β signaling pathway. here ore, the antiproli erative pathway converging onto pRb and the cell division cycle is, in one way or another, disrupted in a majority o human cancer cells. Besides cancer, dysregulation o GF-βsignalingalsohasbeen associated with other human diseases such as Mar an syndrome and thoracic aorticaneurysm.(SeeSchwartz10thed.,p.453.)
99
Nucleus Death signal (e.g., TNF or Fas) Death receptor Plasma membrane Activation of caspase cascade Cytochrome c release Death receptor signaling pathway Mitochondrion Normal target cell Apoptotic target cell
7. Te only gene expression detection method that provides in ormation regardingmRNAsizeis
A. Polymerasechain reaction (PCR)
B. Southern blot hybridization
C. Northern blot hybridization
D. Immunoblotting
8. Allo the ollowingarecell-sur acereceptorsEXCEP
A. ransmitter-gated ion channels
B. Seven-transmembrane-G-protein–coupled receptors (GPCRs)
C. Enzyme-linked receptors
D. Adhesivereceptors
Answer:C
Northern blottingre ers to the technique o size ractionation o RNA in a gel and the trans erring o an RNA sample to a solid support (membrane) in such a manner that the relative positions o the RNA molecules are maintained. he resulting membrane then is hybridized with a labeled probe complementary to the mRNA o interest. Signals generated rom detection o the membrane can be used to determine the size and abundanceo thetarget RNA.In principle,Northern blot hybridization is similar to Southern blot hybridization (and henceitsname),with theexception that RNA,not DNA,ison the membrane. Although reverse-transcriptase polymerase chain reaction (PCR) has been used in many applications, Northern analysis is the only method that provides in ormation regarding mRNA size and has remained a standard method or detection and quantitation o mRNA. he process o Northern hybridization involves several steps, as does Southern hybridization, including electrophoresis o RNA samples in an agarose- ormaldehyde gel, trans er to a membrane support, and hybridization to a radioactively labeled DNA probe. Data rom hybridization allow quanti ication o steady-statemRNAlevelsand,at thesametime,providein ormation related to the presence, size, and integrity o discrete mRNAspecies. hus,Northernblotanalysis,alsotermedRNA gelblotanalysis,commonlyisusedin molecular biologystudies relatingtogeneexpression.(SeeSchwartz10th ed.,p.458.)
Answer:D
here are three major classes o cell-sur ace receptors: transmitter-gated ion channels, seven-transmembrane-Gprotein–coupled receptors (GPCRs), and enzyme-linked receptors. he super amily o GPCRs is one o the largest amilieso proteins,representingover 800geneso thehuman genome. Members o this super amily share a characteristic seven-transmembrane con iguration. he ligands or these receptors are diverse and include hormones, chemokines, neurotransmitters, proteinases, in lammatory mediators, and even sensorysignalssuch asodorantsand photons.Most GPCRs signal through heterotrimeric G proteins, which are guanine nucleotide regulatory complexes. hus the receptor serves as the receiver, the G protein serves as the transducer, and the enzyme serves as the e ector arm. Enzyme-linked receptors possess an extracellular ligand-recognition domain and a cytosolic domain that either has intrinsic enzymatic activity or directly links with an enzyme. Structurally, these receptors usually have only one transmembrane-spanning domain. O at least ive orms o enzyme-linked receptors classi ied by the nature o the enzyme activity to which they are coupled, the growth actor receptors, such as tyrosine kinasereceptor or serine/threoninekinasereceptors,mediate diverse cellular events including cell growth, di erentiation, metabolism, and survival/apoptosis. Dysregulation (particularly mutations) o these receptors is thought to underlie conditionso abnormalcellular proli eration in thecontext o cancer. he ollowing sections will urther review two exampleso growth actor signalingpathwaysand their connection with human diseases.(SeeSchwartz10th ed.,p.452.)
100
9. Teprocesso decodingin ormation on mRNAtosynthesizeproteinsiscalled
A. ranscription
B. ranslation
C. Replication
D. Signaling
Answer:B DNA directs the synthesis o RNA; RNA in turn directs the synthesiso proteins.Proteinsarevariable-length polypeptide polymers composed o various combinations o 20 di erent aminoacidsand aretheworkingmoleculeso thecell. he processo decodingin ormation on mRNAto synthesize proteins is called translation (see Fig. 15-3). ranslation takes place in ribosomes composed o rRNA and ribosomal proteins.(SeeSchwartz10th ed.,Figure15-1,pp.445and447.)
FIG.15-3. The flow ofgenetic information from DNAto protein to cellfunctions.The processoftransmission ofgeneticinformation from DNAto RNAiscalled transcription,and the processof transmission from RNAto protein iscalled translation.Proteins are the essentialcontrolling componentsforcellstructure,cell signaling,and metabolism.Genomicsand proteomicsare the study ofthe genetic composition ofa living organism at the DNAand protein level,respectively.The studyofthe relationship between genesand theircellularfunctionsiscalled functionalgenomics.
10.
Tecellcycleperiod in which DNAisduplicated is
Answer:A
Figure 15-4. he cell cycle and its control system. M is the mitosis phase, when the nucleus and the cytoplasm divide; S isthephasewhen DNAisduplicated;G1isthegapbetween M andS;G2isthegapbetween SandM.Acomplexo cyclin and cyclin-dependent kinase (CDK) controls speci ic events o each phase.Without cyclin,CDKisinactive.Di erent cyclin/ CDKcomplexesareshown around thecellcycle.A,B,D,and E stand or cyclin A, cyclin B, cyclin D, and cyclin E, respectively.(SeeSchwartz10th ed.,Figure15-7,p.450.)
101
Genomics Proteomics Functional genomics DNA RNA Proteins Transcription Translation Structure Metabolism Signaling Cellfunctions
S B G1 C M
G2
A.
D.
11. In thetranscription o prokaryotes,bindingo RNApolymerasetothespeci cpromoter region isachieved by
A. Sigma actors
B. Operon
C. Elongation actors
D. Rho actors
12. When per ormingcellculture,cellsshould be
A Maintained in cultureinde nitely
B Fed with resh medium every 2 to 3 days and split when theyreach confuency.
C. Prepared on sur aces wiped with a 50%ethyl alcohol solution.
D. Maintainedin adehumidi edcarbon dioxideincubator at 37°F.
FIG.15-4. The cellcycle and itscontrolsystem.Misthe mitosis phase,when the nucleusand the cytoplasm divide;Sisthe phase when DNAisduplicated;G1isthe gap between Mand S;G2isthe gap between Sand M.Acomplexofcyclin and cyclin-dependent kinase (CDK)controlsspecific eventsofeach phase.Without cyclin, CDKisinactive.Different cyclin/CDKcomplexesare shown around the cellcycle.A,B,D,and Estand forcyclin A,cyclin B,cyclin D,and cyclin E,respectively.
Answer:A Initiation o transcription in prokaryotesbeginswith the recognition o DNA sequences by RNA polymerase. First, the bacterial RNA polymerase catalyzes RNA synthesis through loose bindingto anyregion in the double-stranded DNAand then through speci icbindingtothepromoter region with the assistance o accessory proteins called σ factors (sigma actors). A promoter region is the DNA region upstream o the transcription initiation site. RNA polymerase binds tightly at the promoter sites and causes the double-stranded DNA structure to unwind. Consequently, ew nucleotides can be base-paired with the DNA template to begin transcription. Once transcription begins,the σ actor is released. he growing RNA chain may begin to peel o as the chain elongates. hisoccursin such a waythat therearealwaysabout 10to 12 nucleotides o the growing RNA chains that are base-paired with theDNAtemplate.(SeeSchwartz10th ed.,p.447.)
Answer:B
Cellculturehasbecomeoneo themost power ultoolsin biomedicallaboratories,asculturedcellsarebeingusedinadiversityo biologic ieldsranging rom biochemistryto molecular and cellular biology. hrough their ability to be maintained in vitro,cellscan bemanipulated bytheintroduction o genes o interest (cell trans ection) and be trans erred into in vivo biologic receivers (cell transplantation) to study the biologic e ect o the interested genes (Fig. 15-5). In the common laboratory settings, cells are cultured either as a monolayer (in which cells growas one layer on culture dishes) or in suspension. It is important to know the wealth o in ormation concerning cell culturing be ore attempting the procedure. For example,conditionso culturewilldepend on thecelltypesto
102
B/CDK1 A/CDK1 A/CDK2 E/CDK2 D/CDK4 D/CDK6 G1 G2 S M Mitosis DNAreplication
FIG.15-5. Cellculture and transfection.A.Primarycellscan be isolated from tissuesand cultured in medium fora limited period of time.Aftergeneticmanipulations to overcome the cellaging process, primarycellscan be immortalized into celllinesforlong-term culture.
B.DNAcan be introduced into cellsto produce recombinant gene productsorto analyze the biologic functionsofthe gene.
13. All o the ollowing are involved in gene regulation EXCEP
A Introns
B. Controlo messenger RNA(mRNA)stability
C. Lacko modi cation o mRNA
D. Control o export o mRNA rom the nucleus to the cytoplasm
be cultured (eg, origins o the cells such as epithelial or ibroblasts, or primary versus immortalized/trans ormed cells). It isalso necessaryto usecelltype-speci icculturemedium that varies in combination o growth actors and serum concentrations. I primary cells are derived rom human patients or animals,somecommercialresourceshave a varietyo culture mediaavailable or testing.Generally,cellsaremanipulated in a sterile hood, and the working sur aces are wiped with 70 to 80%ethylalcoholsolution.Culturedcellsareusuallymaintained in ahumidi iedcarbon dioxideincubator at37°C(98.6°F)and need to be examined daily under an inverted microscope to check or possible contamination and con luency (the area cellsoccupyon thedish).Asageneralrule,cellsshould be ed with resh medium every2to3daysand split when theyreach con luency. Depending on the growth rate o cells, the actual time and number o plates required to split cells in two varies rom cell line to cell line. Splitting a monolayer requires the detachment o cells rom plates byusing a trypsin or collagenase treatment, o which concentration and time period vary dependingon celllines.I cultured cellsgrowcontinuouslyin suspension,theyaresplit or subcultured bydilution.
Because cell lines may change their properties when cultured,it isnot possibleto maintain celllinesin cultureinde initely.(SeeSchwartz10th ed.,Figure15-21,p.462.)
Answer:C
Living cells have the necessary machinery to enzymatically transcribe DNA into RNA and translate the mRNA into protein. his machinery accomplishes the two major steps required or gene expression in all organisms: transcription and translation (see Fig. 15-1). However, gene regulation is ar more complex, particularly in eukaryotic organisms. For example, many gene transcripts must be spliced to remove the intervening sequences. he sequences that are spliced o are called introns, which appear to be useless, but in act may carry some regulatory in ormation. he sequences that
103
Cell isolation Primary culture Propagation Tissue sample Productionof recombinantproteins Analysisofgenefunction Transfection withDNA B A
14. Which o the ollowingisaregulator o thecellcycle?
A CDK
B. yrosinekinase
C. PolIIholoenzyme
D Caspase
15. Whicho the ollowingdrugsisa
monoclonalantibodyto an oncogene?
A rastuzumab
B Methotrexate
C. Adriamycin
D Gleevec
arejoined together,and areeventuallytranslated into protein, are called exons. Additional regulation o gene expression includes modi ication o mRNA, control o mRNA stability, and its nuclear export into cytoplasm (where it is assembled intoribosomes or translation).A ter mRNAistranslatedinto protein,thelevelsand unctionso theproteinscan be urther regulated posttranslationally.(SeeSchwartz10th ed.,p.446.)
Answer:A
he machinery that drives cell cycle progression is made up o a group o enzymes called CDKs. Cyclin expression luctuates during the cell cycle, and cyclins are essential or CDK activitiesand orm complexeswith CDK. hecyclin A/CDK1 and cyclin B/CDK1 drive the progression or the M phase, while cyclin A/CDK2 is the primary S phase complex. Early G1 cyclin D/CDK4/6 or late G1 cyclin E/CDK2 controls the G1-S transition. here also are negative regulators or CDK termed CDKinhibitors,which inhibit theassemblyor activity o the cyclin-CDK complex. Expression o cyclins and CDK inhibitors o ten is regulated by developmental and environmental actors.(SeeSchwartz10th ed.,p.450.)
Answer:A
Patients whose tumors overexpress HER-2/neu are candidates or anti–HER-2/neu therapy. rastuzumab (Herceptin) is a recombinant humanized monoclonal antibody directed against HER-2/neu. Randomized clinical trials have demonstrated that single-agent trastuzumab therapyisan activeand well-tolerated option or irst-line treatment o women with HER-2/neu–overexpressing metastatic breast cancer. More recently, adjuvant trials demonstrated that trastuzumab also was highly e ective in the treatment o women with earlystage breast cancer when used in combination with chemotherapy. Patients who received trastuzumab in combination with chemotherapy had between a 40 and 50% reduction in therisko breast cancer recurrenceand approximatelyathird reduction in breast cancer mortality. (See Schwartz 10th ed., p.454.)
104
PARTII SpecifcConsideration
The Skin and SubcutaneousTissue
1. Followingcaustic injuryto the skin with an alkaline agent thee ected areashould initiallybe
A. reated with runningwater or saline or 30minutes
B. reated with runningwater or saline or 2hours
C. reated with aneutralizingagent
D. reated with topicalemollientsand oralanalgesics
Answer:B
hetreatment orbothtypeso injuriesisbasedon neutralization o the inciting solution and starts with running distilled water or saline over the a ected skin or at least 30 minutes or acidicsolutionsand 2hours or alkalineinjuries.It should be noted that neutralizing agents do not o er a signi icant advantage over dilution with water,maydelaytreatment, and may worsen the injury due to the exothermic reaction that may occur. he clinician observes and treats based on the degree o presentation. Many cases are success ully managed conservatively with topical emollients and oral analgesics, and most cases result in edema, erythema, and induration. I signs o deep second-degree burns develop, local wound care may include debridement, Silvadene, and protective petroleum gauze. In severe cases, injury to the underlying vessels, bones,muscle,and tendon mayoccur,and thesecasesmaybe managedwithin 24hoursbyliposuction throughasmallcatheter and then saline injection. Surgery is indicated or tissue necrosis, uncontrolled pain, or deep-tissue damage. Antibiotics should not be administered unless signs o in ection are present.(SeeSchwartz10th ed.,p.479.)
2. Tetreatment o ahydro uoricacid skin burn is
A. Application o calcium carbonategel
B. Irrigation with sodium bicarbonate
C. Injection o sodium bicarbonate
D. Localwound careonly
Answer:A
Injuries that have speci ic additional treatments include hydro luoride burns. Hydro luoride is ound in air conditioning cleaners and petroleum re ineries. reatment o hydro luorideburnsshould includetopicalor locallyinjected calcium gluconatetobind luorineions.Intra-arterialcalcium gluconate can provide pain relie and preserves arteries rom necrosis, whereas intravenous (IV) calcium repletes resorbed calcium stores. opical calcium carbonate gel and quaternary ammonium compoundsdetoxi y luorideions. hismitigates the leaching o calcium and magnesium ions by the hydroluoric acid rom the a ected tissues and prevents potentially severehypocalcemiaand hypomagnesemiathat predisposeto cardiacarrhythmias.(SeeSchwartz10th ed.,pp.479–480.)
107
CHAPTER16
3. Te area most amenable to salvage by resuscitative and wound management techniques ollowing thermal injury iscalled the
A. Zoneo hyperemia
B. Zoneo coagulation
C. Zoneo stasis
D. Zoneo scalding
4. issueischemiaresultingin woundsthat arecharacterized asapartial-thicknessinjurywith ablister isconsidered
A. Stage1
B. Stage2
C. Stage3
D. Stage4
5. Te presence o sul ur granules in a draining wound should lead to the use o which o the ollowing antibiotics?
A. Ri ampin
B. Gentamicin
C. Penicillin
D. Amphotericin
Answer:C
Exposureo theskin to thermalextremesdisruptsitsprimary unction as a barrier to heat loss, evaporation, and microbial invasion. hedepth and extent o injuryaredependent on the duration and temperature o the exposure. he pathophysiologyand management are discussed elsewhere in this book. Brie ly,theepicenter o theinjuryundergoesa varyingextent o necrosis (depending on the exposure), otherwise re erred to as the zone of coagulation, which is surrounded by the zone o stasis, which has marginal per usion and questionable viability. his is the area o tissue that is most amenable to salvage by appropriate resuscitative and wound managementtechniques,which wouldtheoreticallylimit theextento injury. heoutermost area o skin showscharacteristicssimilar toother in lamed tissuesand hasbeen designated thezone o hyperemia. he degree o burn corresponds to histologic layerso thea ected dermisand correlateswith management and prognosis pertaining to timeline o healing and magnitudeo scarring.(SeeSchwartz10th ed.,p.480.)
Answer:B
issuepressuresthat exceed thepressureo themicrocirculation (30 mm Hg) result in tissue ischemia. Frequent or prolongedischemicinsultswillultimatelyresultin tissuedamage. Areas o bony prominence are particularly prone to ischemia, the most common areas beingischial tuberosity(28%), trochanter (19%), sacrum (17%), and heel (9%). issue pressurescan measureup to 300mm Hgin theischialregion duringsittingand150mm Hgoverthesacrum whilelyingsupine. Muscle is more susceptible than skin to ischemic insult due to its relatively high metabolic demand. Wounds are staged as ollows: stage 1, nonblanching erythema over intact skin; stage 2, partial-thickness injury (epidermis or dermis)— blister or crater; stage 3, ull-thickness injury extending down to, but not including, ascia and without undermining o adjacent tissue; and stage 4, ull-thickness skin injury with destruction or necrosis o muscle, bone, tendon, or joint capsule.(SeeSchwartz10th ed.,p.482.)
Answer:C
Actinomycosis should be considered in the di erential diagnosiso anyacute,subacute,or chroniccutaneousswellingo the head and neck. he cervico acial orm o Actinomycetes in ection is the most common presentation, typically as an acute pyogenic in ection in the submandibular or paramandibular area,but in ection could be elsewhere in the mandibular and maxillary regions. he primary skin in ection may spread to adjacent structuressuch asthescalp,orbit,ears,and other areas. Oral in ection may spread to the hypopharynx, larynx, trachea, salivary glands, and sinuses. Actinomycosis can spread beyond boundaries o tissue planes and may also mimic chronic osteomyelitis. reatment consists o a combination o penicillin therapy and surgical debridement. Debulking and debriding in ected tissue arising rom sinus tracts and abscess cavities inhibit actinomycosis growth in most cases.(SeeSchwartz10th ed.,p.484.)
108
6. Initialtreatment o nonpurulent,complicated cellulitisis
A. Vancomycin
B. β-lactam
C. Linezolid
D. Clindamycin
7. A3-mm basalcellcarcinoma (BCC) o the skin should be treated with
A. Biopsyand grosstotalexcision
B. Dermatologiclaser vaporization
C. Excision with 2- to4-mm normalmargin
D. Electrodesiccation
8. richilemmalcysts
A. Arethemost common typeo cutaneouscysts
B. Are ound between the orehead tonosetip
C. Aretypically ound on thescalp o emales
D. Occasionallydevelop bone,tooth,or nervetissue
Answer:B reatment o nonpurulent, complicated cellulitis can begin with a β-lactam, with methicillin-resistant Staphylococcus aureus (MRSA) coverage added i no response is observed. Empiric MRSA coverage is warranted in all other complicated skin and subcutaneous in ections. Vancomycin is the mainstay o therapy, although it is in erior to β-lactams or methicillin-sensitive S. aureus (MSSA) and has a relatively slow onset o e icacyin vitro. Linezolid, daptomycin, tigecycline,and telavancin are other FDA-approved alternatives or MRSAtreatment.Clindamycin isalso approved or S.aureus; however, resistance may develop, and diarrhea can occur in up to 20% (Clostridium difficile related). (See Schwartz 10th ed.,p.483.)
Answer:C
Basalcellcarcinoma(BCC)arises rom thebasallayer o nonkeratinocytes and represents the most common tumor diagnosed in theUnited States.Annuallyit accounts or 25%o all diagnosed cancers and 75%o skin cancers. he primaryrisk actor or disease development is sun exposure (ultraviolet [UV] Braysmorethan UVArays) particularlyduringadolescence;however,other actorsincludeimmunesuppression (ie, organ transplant recipients, human immunode iciency virus [HIV]), chemical exposure, and ionizing radiation exposure. BCCcan alsobea eatureo inheritedconditionssuchasxeroderma pigmentosa, unilateral basal cell nevus syndrome, and nevoid BCCsyndrome. henaturalbehavior o BCCisoneo local invasion rather than distant metastasis. Untreated BCC can result in signi icant morbidity. hirtypercent o casesare ound on the nose, and bleeding, ulceration, and itching are o ten part o theclinicalpresentation.(SeeSchwartz10th ed., p.486.)
Answer:C
herearethreetypeso cutaneouscysts:epidermal,dermoid, and trichilemmal. All o these benign entities comprise epidermis that grows toward the center o the cyst, resulting in central accumulation o keratin to orm a cyst. All clinically appear as a white, creamy substance-containing subcutaneous, thin-walled nodule. Epidermal cysts are the most common cutaneous cyst and histologically characterized by matureepidermiscompletewith granular layer. richilemmal cysts are the second most common lesion; they tend to orm on the scalp o emales, have a distinct odor a ter rupture, histologically lack a granular layer, and have an outer layer resemblingtheroot sheath o ahair ollicle.Dermoid cystsare congenital, ound between the orehead to nose tip, and contain squamous epithelium, eccrine glands, and pilosebaceous units, occasionally developing bone, tooth, or nerve tissue. he eyebrow is the most requent site o presentation. hese cysts are commonly asymptomatic but can become in lamed and in ected, thus necessitating incision and drainage. A ter theacutephasesubsides,theentirecyst should beremoved to prevent recurrence.(SeeSchwartz10th ed.,p.486.)
109
9.
Morethan hal o patientstreated or BCC willexperience arecurrencewithin
A. 6months
B. 1years
C. 2years
D. 3years
10. Teprimaryrisk actor or thedevelopment o squamous cellcarcinoma(SCC)is
A. UVexposure
B. Cigarettesmoking
C. Chemicalagents
D. Chronic,nonhealingwounds
11. In the ABCDE o melanoma, the D stands or diameter greater than
A. 2mm
B. 4mm
C. 6mm
D. 8mm
12. Te most common site o distant metastasis or melanomais
A. Brain
B. Lung
C. Gastrointestinaltract
D. Distant skin
13. Temost common subtypeo melanomais
A. Lentigomaligna
B. Acrallentiginous
C. Super cialspreading
D. Nodular
Answer:D
It is critical or each patient to have routine annual ollow-up that includes ull-body skin examinations. Sixty-six percent o recurrences develop within 3 years, and with a ew exceptionsoccurringdecadesa ter initialtreatment,theremaining recur within 5 years o initial treatment. A second primary BCC maydevelop a ter treatment and, in 40%o cases, presents within the irst 3 years a ter treatment. (See Schwartz 10th ed.,p.487.)
Answer:A
Squamous cell carcinoma (SCC) is the second most common skin cancer, accounting or approximately100,000 cases each year and generally a licting individuals o lighter skin color. heprimaryrisk actor and driving orce or thedevelopment o this common cancer is UV exposure; however, other risks includeenvironmental actorssuch aschemicalagents,physical agents (ionizing radiation), psoralen and UVA (PUVA), HPV-16and -18in ections(immunosuppression),and smoking. Chronic nonhealing wounds, burn scars, and chronic dermatosis are other risk actors, and manydarker skin individuals who develop SCC o ten have a historyo one o these risk actors.Heritableconditionssuch asxeroderma pigmentosum, epidermolysis bullosa, and oculocutaneous albinism arepredisposingrisk actors.(SeeSchwartz10th ed.,p.487.)
Answer:C
Melanoma most commonly mani ests as cutaneous disease, and clinical characteristics include an Asymmetric outline, changing irregular Borders, Color variations, Diameter greater than 6 mm, and Elevation (ABCDE). Other key clinical characteristics include a pigmented lesion that has enlarged, ulcerated, or bled. Amelanotic lesions appear as raised pink, purple, or normal-colored skin papules and are o ten diagnosed late.(SeeSchwartz10th ed.,p.488.)
Answer:B
hemostcommon siteso distantmetastasisarethelungsand liver ollowed bythebrain,gastrointestinaltract,distant skin, and subcutaneous tissue. A limited subset o patients with small-volume,limited distant metastasesto the brain, gastrointestinal tract,or distant skin will be cured with resection or gammakni eradiation.Liver metastasesarebetter dealt without surgical resection unless they arise rom an ocular primary.(SeeSchwartz10th ed.,p.491.)
Answer:C
Melanomagrowth most commonlystartsasalocalized,radial growth phase ollowed by a vertical growth phase that determines metastatic risk. he subtypes o melanoma include lentigo maligna, super icial spreading, acral lentiginous, mucosal, nodular, polypoid, desmoplastic, amelanotic, and so t tissue. he most common subtype is super icial spreading, accounting or 70% o cases. (See Schwartz 10th ed., p.488.)
110
14. Ocular melanoma
A. Exclusivelymetastasizestothelungs
B. Exclusivelymetastasizestothebrain
C. Exclusivelymetastasizestoregionallymph nodes
D. Exclusivelymetastasizestotheliver
15. Te ollowing is NO true in regard to Merkel cell carcinoma
A. It iscommonly oundin whitemen with amedian age o 70years.
B. It ischaracterized bya rapidlygrowing, esh-colored papule.
C. reatment should begin with examination o nodal basins.
D. Recurrenceisuncommon.
Answer:D
Ocular melanomaisthemost common noncutaneousdisease site, and treatment includes photocoagulation, partial resection, radiation, or enucleation. Ocular melanomas exclusively metastasize to the liver and not regional lymph nodes, and some patients bene it rom liver resection. (See Schwartz 10th ed.,p.491.)
Answer:D
his is a rare and aggressive neuroendocrine tumor o the skin most commonly ound in white men and diagnosed at a mean age o 70 years. Risk actors include UV radiation, PUVA,and immunosuppression.Approximatelyonein three cases present on the ace, with the remainder occurring on sun-exposed skin.Arapidlygrowing, lesh-colored papule or plaque characterizes the disease. Regional lymph nodes are involved in 30% o patients, and 50% will develop systemic disease (skin, lymph nodes, liver, lung, bone, brain). here are no standardized diagnostic imaging studies or staging, but computed tomography (C ) o the chest, abdomen, and pelvis and octreotide scans may provide use ul in ormation when clinicallyindicated. A ter examiningthe entire skin or other lesions,treatment should begin byevaluatingthe nodal basins.
Recurrenceiscommon,andonestudyo 95patientsshowed a47%recurrence,with 80%o recurrencesoccurringwithin 2 yearsand 96%occurringwithin 5years.Regionallymph node diseaseiscommon,and70%o patientswillhavenodalspread within 2 years o disease presentation. Five-year overall survival o head and neck disease in surgicallytreated patients is between 40and 68%.(SeeSchwartz10th ed.,p.492.)
16. What is the most common melanoma in patients with darkskin?
A. Nodular
B. Super cialspreading
C. Acrallentiginous
D. Lentigomaligna
17. Kaposisarcoma
A. Excision isthetreatment o choice
B. Ispredominantly ound on theskin
C. Appearsasrubbery,bluenodules
D. Is most ofen seen in patients in their fh decade o li e
Answer:C
Nodular melanomaaccounts or 15to30%o melanomas,and this variant is unique because it begins with a vertical growth phase that partly accounts or its worse prognosis. Lentigo maligna is typically ound in older individuals and primarilylocated in the head and neck region. he acral lentiginous variant accounts or 29to 72%o melanomasin dark-skinned individuals, is occasionally seen in Caucasians, and is ound on palmar, plantar, and subungual sur aces. (See Schwartz 10th ed.,p.488.)
Answer:A
Kaposi sarcoma is diagnosed a ter the i th decade o li e and predominantly ound on the skin but can occur anywhere in thebody.In North America,theKaposisarcoma herpesvirus is transmitted via sexual and nonsexual routes and predominantlya ectsindividualswith compromisedimmunesystems such asthosewith HIVand transplant recipientson immunesuppressing medications. Clinically, Kaposi sarcoma appears as multi ocal, rubbery blue nodules. reatment o acquired immunode iciency syndrome (AIDS)-associated Kaposi sarcoma is with antiviral therapy, and many patients experience adramatictreatment response. hoseindividualswho do not respond and havelimited mucocutaneousdiseasemaybene it rom cryotherapy, photodynamic therapy, radiation therapy, intralesional injections, and topical therapy. Surgical biopsy
111
18. Te ollowingis NO a prognostic indicator or patients with asentinelnodecontainingmetastaticmelanoma
A. Patient age
B. Siteo metastasis
C. Number o positivenodes
D. Tickness, mitotic rate, and ulceration o primary tumor
is important or disease diagnosis, but given the high local recurrence and the act that Kaposi sarcoma represents more o asystemicrather than localdisease,thebene it o surgeryis limited and generallyshould not bepursued except or palliation.(SeeSchwartz10th ed.,p.492.)
Answer:B
Melanoma is characterized according to the American Joint Committeeon Cancer (AJCC) aslocalized disease(stageI and II), regional disease (stage III), or distant metastatic disease (stage IV). Overall tumor thickness, ulceration, and mitotic rate are the most important prognostic indicators o survival. I a sentinel node contains metastatic melanoma, the number o positive nodes;thickness, mitotic rate, and ulceration o the primary tumor; and patient age determine prognosis. With clinicallypositivenodes,thenumbero positivenodes,primary tumorulceration,andpatientagedetermineprognosis. hesite o metastasis is stronglyassociated with prognosis or stage IV disease,andelevatedlactatedehydrogenase(LDH)isassociated withaworseprognosis.(SeeSchwartz10thed.,p.488.)
19. A patient with a 5-mm deep melanoma o the thigh and no clinically positive nodes should undergo which procedure?
A. Resection o theprimaryonly
B. Super cial emoralnoderesection
C. Super cialand deep emoralnoderesection
D. Resection o emoraland inguinalnodalbasins
Answer:A Nonmetastatic in-transit disease should undergo excision to clear margins when easible.However, disease not amenable to complete excision derives bene it rom isolated limb per usion (ILP) and isolated limb in usion (ILI) (Fig. 16-1). hese two modalitiesareusedtotreatregionaldisease,andtheirpurposeis to administer high doseso chemotherapy,commonlymelphalan,toana ectedlimbwhileavoidingsystemicdrugtoxicity.ILI was shown to provide a 31%response rate in one study, while hyperthermicILP provided a63%completeresponseratein an independentstudy.(SeeSchwartz10thed.,Figure16-15,p.491.)
112
Overhead heater Hot air blanket Venous catheter Arterial catheter Pneumatic tourniquet Pump chamber 25cc Syringe Warming coil Esmarch bandage Drug in pre warmed saline
FIG.16-1. Isolated limb infusion.Schematic ofisolated limb infusion oflowerextremity. (FromThompson JF,Kam PC.Isolated limb infusion formelanoma:a simple but effective alternative to isolated limb perfusion.JSurgOnc.2004;88:1-3.Copyright 2004JohnWileyand Sons.Reprinted with permission.)
20. A 65-year-old patient who spends winters in Florida presents with a painless, ulcerated lesion on his right cheek. Te lesion has been present or 1 year. Physical examination o the patient’s neck reveals no lymph node enlargement.Temost likelydiagnosisis
A. Melanoma
B. BCC
C. SCC
D. Sebaceouscyst
21. Te chronic in ammatory disease presenting as pain ul subcutaneousnodulesis
A. Pyodermagangrenosum
B. oxicepidermalnecrolysissyndrome
C. Hidradenitissuppurative
D. Steven-Johnson syndrome
22. Correct statements about toxic epidermal necrolysis ( EN)includeallo the ollowingEXCEP
A. oxicepidermalnecrolysisisbelieved tobean immunologicproblem.
B. Lesions are similar in appearance to partialthickness burns.
C. Teprocessdevelopsatthedermoepidermaljunction.
D. Corticosteroid useisaprimarypart o therapy.
Answer:B he most common orm o BCC (60%) is the nodular variant, characterized by raised, pearly pink papules and occasionally a depressed tumor center with raised borders giving the classic “rodent ulcer”appearance. hisvariant tendsto develop in sunexposedareaso individualsolderthan60years.Super icialBCC accounts or 15%o BCC,isdiagnosedatamean ageo 57years, and typically appears on the trunk as a pink or erythematous plaquewithathinpearlyborder. hein iltrative ormappearson theheadandneckin thelate60swithsimilar clinicalappearance to the nodular variant.An important variant to keep in mind is thepigmented variant o nodular BCC becausethismaybediicult to di erentiate rom nodular melanoma.Other important subtypesincludethemorphea orm variant,accounting or3%o cases and characterized byindistinct borders with a yellowhue, and ibroepithelioma o Pinkus. Histologic subtypes o BCC includenodular andmicronodular (50%),super icial(15%),and in iltrative.(SeeSchwartz10thed.,p.486.)
Answer:C
Hidradenitissuppurativaisachronicin lammatorydiseasepresenting as pain ul subcutaneous nodules. Patients experience appreciable physical, psychological, and economical hardship and decreased qualityo li e when compared with patients who su er rom other chronic dermatologic disease such as psoriasis and alopecia. It is characterized bymultiple abscesses, internetworking sinus tracts, oul-smelling exudate rom draining sinuses,in lammationinthedermis,bothatrophicandhypertrophicscars,ulceration,andin ection,whichmayextenddeepinto the ascia. he diagnosis is made clinicallywithout the need or imagingorlaboratorytests.(SeeSchwartz10thed.,p.467.)
Answer:D
hese in lammatory diseases represent a spectrum o an autoimmune reaction to stimuli such as drugs that result in structural de ects in the epidermal-dermal junction. he cutaneous mani estations o toxic epidermal necrolysis syndrome ( ENS) ollow a prodromal period reminiscent o an upper respiratory tract in ection. A symmetrical macular eruption ollowsstarting rom the aceand trunkand spreading to the extremities. ypically, a Nikolsky sign develops in which lateralpressurecausestheepidermistodetach rom the basal layer. he macular eruption evolves into blisters, causingan extensive super icial partial-thickness skin injurywith exposed dermis.(SeeSchwartz10th ed.,p.477.)
23. Terareadenocarcinomao theapocrineglandthatofen appearsasanonpigmented plaqueis
A. Angiosarcoma
B. ExtramammaryPaget disease
C. Malignant broushistiocytoma
D. Dermato brosarcomaprotuberans
Answer:B
his rare adenocarcinoma o apocrine glands arises in perianal andaxillaryregionsandin genitaliao men andwomen.Clinical presentation is that o erythematous or nonpigmented plaques with an eczema-like appearance that o ten persist a ter ailed treatment rom other therapies.An important characteristicand one that the surgeon must be acutely aware o is the high incidence o concomitant other malignancies with this cutaneous disease.Fortypercento casesareassociatedwithprimarygastrointestinal and genitourinarymalignancies, and a diligent search shouldbemadea teradiagnosiso extramammaryPagetdisease is made. reatment is surgical resection with negative microscopic margins, and adjuvant radiation may provide additional locoregionalcontrol.(SeeSchwartz10thed.,p.493.)
113
1. Which o the ollowing statements about normal breast anatomyistrue?
A. Tebreast typicallycontains10lobes.
B. Cooper ligaments are only ound in the upper quadrantso thebreast.
C. Te upper inner quadrant o the breast contains the most breast tissue.
D. Te tail o Spence extends across the anterior axillary old.
Answer:D
hebreast iscomposed o 15to20lobes,which areeach composed o several lobules. Fibrous bands o connective tissue travel through the breast (Cooper suspensory ligaments), insert perpendicularlyintothedermis,and providestructural support. he mature emale breast extends rom the level o the second or third rib to the in ramammary old at the sixth or seventh rib. It extends transversely rom the lateral border o the sternum to the anterior axillary line. he deep or posterior sur ace o the breast rests on the ascia o the pectoralis major, serratus anterior, and external oblique abdominal muscles, and the upper extent o the rectus sheath. he retromammary bursa may be identi ied on the posterior aspect o the breast between the investing ascia o the breast and the ascia o the pectoralis major muscles. he axillary tail o Spence extends laterallyacross the anterior axillary old. (See Schwartz10th ed.,p.500.)
2. Which o the ollowingchangesin thebreast isNO associated with pregnancy?
A. Accumulation o lymphocytes,plasmacells,andeosinophilswithin thebreast.
B. Enlargement o breast alveoli.
C. Releaseo colostrum.
D. Accumulation o secretory products in minor duct lumina.
Answer:C
With pregnancy,thebreast undergoesproli erativeand developmental maturation. As the breast enlarges in response to hormonal stimulation, lymphocytes, plasma cells, and eosinophils accumulate within the connective tissues. he minor ducts branch and alveoli develop. Development o the alveoli is asymmetric, and variations in the degree o development may occur within a single lobule. With parturition, enlargement o the breastsoccursvia hypertrophyo alveolar epithelium and accumulation o secretory products in the lumina o the minor ducts. Alveolar epithelium contains abundant endoplasmicreticulum,largemitochondria,Golgicomplexes, and dense lysosomes. wo distinct substances are produced bythealveolar epithelium:(1) theprotein component o milk, whichissynthesizedin theendoplasmicreticulum(merocrine secretion); and (2) the lipid component o milk (apocrine secretion),which ormsas reelipid dropletsin thecytoplasm. Milk released in the irst ew days a ter parturition is called colostrum and haslowlipid content but containsconsiderable quantitieso antibodies.(SeeSchwartz10th ed.,p.501.)
115
CHAPTER17 Breast
3. Tebreastreceivesitsbloodsupply romallo the ollowing
EXCEP
A. Brancheso theinternalmammaryartery
B. Brancheso thesuperior epigastricartery
C. Brancheso theposterior intercostalarteries
D. Brancheso thoracoacromialartery
4. Which o the ollowingstatementsisINCORREC ?
A. Level I lymph nodes are those that are lateral to the pectoralisminor muscle.
B. LevelII lymph nodesarelocated deep to thepectoralis minor muscle.
C. LevelIII lymph nodesare located medialto thepectoralisminor muscle.
D. LevelIVlymph nodesaretheipsilateralinternalmammarylymph nodes.
5. Concerning breast development be ore and during pregnancy,which hormonalactivitypairingisINCORREC ?
A. Estrogen:Initiatesductaldevelopment
B. Progesterone:Initiateslobular development
C. Prolactin:Initiateslactogenesis
D. Folliclestimulatinghormone:Cooperligamentrelaxation
Answer:B
he breast receives its principal blood supply rom: (1) perorating branches o the internal mammaryartery;(2) lateral brancheso theposterior intercostalarteries;and (3)branches rom theaxillaryartery,includingthehighest thoracic,lateral thoracic,and pectoralbrancheso thethoracoacromialartery. he second, third, and ourth anterior intercostal per orators and branches o the internal mammaryarteryarborize in the breast as the medial mammary arteries. he lateral thoracic artery gives o branches to the serratus anterior, pectoralis major and pectoralis minor, and subscapularis muscles. It also gives rise to lateral mammary branches. (See Schwartz 10th ed.,p.501.)
Answer:D
Level I includes lymph nodes located lateral to the pectoralis minor muscle; level II includes lymph nodes located deep to the pectoralis minor; and level III includes lymph nodes locatedmedialtothepectoralisminor.(SeeSchwartz10thed., p.502.)
Answer:D
Estrogen initiates ductal development, whereas progesterone is responsible or di erentiation o epithelium and or lobular development. Prolactin is the primaryhormonal stimulus or lactogenesisin latepregnancyand thepostpartum period. he gonadotropins luteinizing hormone (LH) and olliclestimulating hormone (FSH) regulate the release o estrogen and progesterone rom the ovaries. In turn, the release o LH and FSH rom the basophilic cells o the anterior pituitary is regulated by the secretion o gonadotropin-releasing hormone (GnRH) rom the hypothalamus. (See Schwartz 10th ed.,pp.503–504.)
6. Concerninggynecomastia,which o the ollowingistrue?
A. Duringsenescencegynecomastiaisusuallyunilateral.
B. Duringpubertygynecomastiaisusuallybilateral.
C. Is not associated with breast cancer except in EhlersDanlospatients.
D. Isclassi ed asper athree-gradesystem.
Answer:D
In gynecomastia, the ductal structures o the male breast enlarge, elongate, and branch with a concomitant increase in epithelium. During puberty, the condition o ten is unilateral and typicallyoccurs between ages 12 and 15. In contrast, senescent gynecomastia is usually bilateral. Gynecomastia generally does not predispose the male breast to cancer. However, the hypoandrogenic state o Kline elter syndrome (XXY), in which gynecomastia is usually evident, is associated with an increased risk o breast cancer. Gynecomastia is graded based on the degree o breast enlargement, the position o the nipple with re erence to the in ramammary old and the degree o breast ptosis and skin redundancy: Grade I: mild breast enlargement without skin redundancy; Grade IIa: moderate breast enlargement without skin redundancy; Grade IIb: moderate breast enlargement with skin redundancy; and Grade 3: marked breast enlargement with skin redundancyand ptosis.(SeeSchwartz10th ed.,p.505.)
116
7. In ammatory conditions o the breast include all o the ollowingEXCEP
A. Necrotizingviralmastitis
B. Zuskadisease(recurrent preductalmastitis)
C. Mondor disease(super cialbreast thrombophlebitis)
D. Hidradenitissuppurativa
8. Lesionswith malignant potentialincludeallo the ollowingEXCEP
A. Intraductalpapilloma
B. Atypicalductalhyperplasia
C. Sclerosingadenosis
D. Atypicallobular hyperplasia
9. Risk actors or thedevelopment o breast cancer include
A. Earlymenarche
B. Nulliparity
C. Latemenopause
D. Longer lactation period
10. Drugsuse ulin breast cancer prevention include
A. Raloxi ene
B. amoxi en
C. Aspirin
D. Aromataseinhibitors
Answer:A
Zuska disease, also called recurrent periductal mastitis, is a condition o recurrent retroareolar in ections and abscesses. Hidradenitis suppurativa o the nipple-areola complex or axilla is a chronic in lammatory condition that originates within theaccessoryareolar glandso Montgomeryor within the axillary sebaceous glands. Mondor disease is a variant o thrombophlebitis that involves the super icial veins o the anterior chest wall and breast. (See Schwartz 10th ed., pp.506–507.)
Answer:C
Sclerosing adenosis is prevalent during the childbearing and perimenopausal years and has no malignant potential. Multiple intraductal papillomas, which occur in younger women and are less requently associated with nipple discharge, are susceptible to malignant trans ormation. Individuals with a diagnosis o atypical ductal hyperplasia (ADH) are at increased risk or development o breast cancer and should be counseled appropriately regarding risk reduction strategies. Atypicallobular hyperplasia(ALH)resultsin minimaldistention o lobular units with cells that are similar to those seen in lobular carcinoma in situ (LCIS). (See Schwartz 10th ed., pp.509–510.)
Answer:D
Increasedexposuretoestrogen isassociatedwith an increased risk or developing breast cancer, whereas reducing exposure is thought to be protective. Correspondingly, actors that increase the number o menstrual cycles, such as early menarche, nulliparity, and late menopause, are associated with increased risk. Moderate levels o exercise and a longer lactation period, actors that decrease the total number o menstrualcycles,areprotective.(SeeSchwartz10th ed.,p.511.)
Answer:C
he P-2 trial, the Studyo amoxi en and Raloxi ene (known astheSTARtrial),randomlyassigned 19,747postmenopausal women at high risk or breast cancer to receiveeither tamoxien or raloxi ene. he initial report o the P-2 trial showed the two agents were nearly identical in their ability to reduce breast cancer risk, but raloxi ene was associated with a more avorable adverse event pro ile. An updated analysis revealed that raloxi ene maintained 76% o the e icacy o tamoxi en in prevention o invasive breast cancer with a more avorable side-e ect pro ile. Aromatase inhibitors (AIs) have been shown to be more e ective than tamoxi en in reducing the incidence o contralateral breast cancers in postmenopausal women receivingAIs or adjuvant treatment o invasivebreast cancer.(SeeSchwartz10th ed.,p.514.)
117
11. Which o the ollowing is true regarding breast cancer metastasis?
A. Metastases occur afer breast cancers acquire their own blood supply.
B. Batson plexus acilitatesmetastasistothelung.
C. Natural killer cells have no role in breast cancer immunosurveillance.
D. wenty percent o women who develop breast carcinoma metastases will do so within 60 months o treatment.
12. All o the ollowing are true concerning breast LCIS EXCEP
A. Developsonlyin the emalebreast.
B. Cytoplasmic mucoid globules are a distinctive cellular eature.
C. Frequencyo LCIScannot bereliablydetermined.
D. Teaverageageat diagnosisis65to70years.
13. Which o the ollowingconcerningbreast cancer staging iscorrect?
A. Stage I tumors have no metastases to either lymph nodesor distant sites.
B. StageIIItumorsincludesomewith distant metastases (M1disease).
C. In ammatorycarcinomaisconsidered 4disease.
D. N4diseaseincludesmetastasestohighest contralateral axillarynodes.
14. Factors that determine the type o therapy o ered to patients afer diagnosiso breast cancer include allo the ollowingEXCEP
A. Whether or not atherapyhasbeen proven e ectivein clinicaltrials
B. Stageo disease
C. Generalhealth o patient
D. Biologicsubtype
15. Which o the ollowing statements about the management o distalcarcinomain situ (DCIS)istrue?
A. DCIS treated by mastectomy has a local recurrence rateo 2%.
B. ExtensiveDCISshould betreated with tamoxi en ollowed bylumpectomy.
C. Specimen mammography is only use ul or patients with smallamountso DCIS.
D. Postoperative tamoxi en is use ul in DCIS patients whosetumorsareestrogen-receptor (ER)negative.
Answer:A
At approximately the 20th cell doubling, breast cancers acquire their own blood supply (neovascularization). herea ter, cancer cells may be shed directly into the systemic venous blood to seed the pulmonary circulation via the axillary and intercostal veins or the vertebral column via Batson plexus o veins, which courses the length o the vertebral column. hese cells are scavenged bynatural killer lymphocytes and macrophages. Sixty percent o the women who develop distant metastases will do so within 60 months o treatment.
(SeeSchwartz10th ed.,p.518.)
Answer:D
LCIS originates rom the terminal duct lobular units and developsonlyin the emalebreast.Cytoplasmicmucoid globulesareadistinctivecellular eature. he requencyo LCISin thegeneralpopulation cannot bereliablydetermined because it usually presents as an incidental inding. he average age at diagnosisis45years,which isapproximately15to 25years younger than the age at diagnosis or invasive breast cancer. (SeeSchwartz10th ed.,p.519.)
Answer:C (SeeSchwartz10thed., ables17-10and17-11,pp.532and 534.)
Answer:A
Oncea diagnosiso breast cancer ismade,thetypeo therapy o ered to abreast cancer patient isdetermined bythestageo thedisease,thebiologicsubtype,and thegeneralhealth status o theindividual.(SeeSchwartz10th ed.,p.536.)
Answer:A
Women with DCIS and evidence o extensive disease (>4cm o diseaseor disease in more than onequadrant) usually require mastectomy. For women with limited disease, lumpectomy and radiation therapy are generally recommended. For nonpalpable DCIS, needle localization or other image-guided techniquesareused to guidethesurgicalresection.Specimen mammographyisper ormed to ensurethat all visibleevidenceo cancer isexcised.Adjuvanttamoxi en therapy is considered or DCIS patients with estrogen-receptor (ER)-positivedisease. hegold standard against which breast conservation therapy or DCIS is evaluated is mastectomy. Women treated with mastectomy have local recurrence and mortalityrateso <2%.(SeeSchwartz10th ed.,p.537.)
118
16. All o the ollowing are true about accelerated partial breast irradiation (APBI)EXCEP
A. APBI is delivered in an abbreviated ashion and a lower totaldosethan standard courseo wholebreast radiation.
B. Suitable patients or APBI include women older than or equalto60years.
C. Suitable patients or APBI include patients whose tumor marginsaregreater than or equalto2mm.
D. Suitablepatients or APBIincludethosewith multi ocaldisease.
17. Patients not suitable or sentinel lymph node (SLN) biopsyincludeallo the ollowingEXCEP
A. In ammatorycarcinomao thebreast.
B. Prior axillarysurgery.
C. Biopsy-proven distant metastases.
D. Lower inner quadrant o breast primarycarcinoma.
18. Which o the ollowing is true concerning breast cancer duringpregnancy?
A. Metastases to lymph nodes occur in approximately 75%o patients.
B. Approximately 50% o breast nodules developing duringpregnancyaremalignant.
C. Mammographyisespeciallyuse ulin localizingsmall lesions.
D. Tere is risk o chemotherapy teratogenicity i used during the second, but not the third, trimester o pregnancy.
Answer:D
Accelerated partial breast irradiation (APBI) is delivered in an abbreviated ashion (twice daily or 5 days) and at a lower total dose compared with the standard course o 5 to 6 weeks o radiation (50 Gray with or without a boost) in the case o whole breast irradiation. he AS RO guidelines describe patients “suitable” or APBI to include women older than 60 years with a uni ocal, 1, ER-positive tumor with no lymphovascular invasion, and margins o at least 2 mm. Finally, a group elt to be “unsuitable” or APBI includes those with 3or 4disease,ER-negativedisease,multi ocality,multicentricity, extensive lymphovascular invasion (LVI), or positive margins.(SeeSchwartz10th ed.,p.539.)
Answer:D
Clinicalsituationswheresentinellymph node(SLN)dissection isnotrecommendedincludepatientswithin lammatorybreast cancers, those with palpable axillary lymphadenopathy and biopsy-proven metastasis, DCISwithout mastectomy, or prior axillary surgery. Although limited data are available, SLN dissection appears to be sa e in pregnancy when per ormed with radioisotopealone.(SeeSchwartz10thed.,p.545.)
Answer:A
Breast cancer occursin 1o every3000pregnant women,and axillary lymph node metastases are present in up to 75% o these women. Fewer than 25% o the breast nodules developing during pregnancy and lactation will be cancerous. Mammography is rarely indicated because o its decreased sensitivityduringpregnancyand lactation;however,the etus can be shielded i mammography is needed. Chemotherapy administered duringthe irst trimester carries a risk o spontaneous abortion and a 12%risk o birth de ects. here is no evidence o teratogenicity resulting rom administration o chemotherapeutic agents in the second and third trimesters. (SeeSchwartz10th ed.,p.554.)
119
Disordersofthe Head and Neck
1. Which o the ollowing are true about leukoplakia o the vocalcords?
A. Up to40%risko progression toinvasivecarcinoma
B. Ulceration is particularly suggestive o possible malignancy.
C. Initialtherapyincludesantihistamines.
D. Biopsy should be considered only a er 6 months o conservativetherapy.
Answer:B
Leukoplakia o thevocal old representsa whitepatch (which cannot be wiped o ) on the mucosal sur ace, usually on the superior sur ace o the true vocalcord.Rather than a diagnosisper se,theterm leukoplakia describesa indingon laryngoscopic examination. The signi icance o this inding is that it may represent squamous hyperplasia, dysplasia, and/or carcinoma. Lesions exhibiting hyperplasia have a 1 to 3%risk o progression to malignancy. In contrast, that risk is 10 to 30% or those demonstrating dysplasia. Furthermore, leukoplakia may be observed in association with in lammatory and reactive pathologies, including polyps, nodules, cysts, granulomas,and papillomas.Featureso ulceration and erythroplasia areparticularlysuggestiveo possiblemalignancy.Ahistoryo smokingand alcoholabuse should also prompt a malignancy workup.In theabsenceo suspected malignancy,conservative measuresareused or 1month.Anylesionsthatprogress,persist,or recur should beconsidered or excisionalbiopsyspecimen.(SeeSchwartz10th ed.,p.573.)
2. Factors associated with increased incidence o head and neckcancersincludeallo the ollowingEXCEPT
A. Human papillomavirus(HPV)exposure
B. Ultraviolet light exposure
C. Plummer-Vinson syndrome
D. Refuxesophagitis
Answer:D
Human papillomavirus(HPV) isan epitheliotropicvirusthat has been detected to various degrees within samples o oral cavity squamous cell carcinoma. In ection alone is not considered su icient or malignant conversion; however, results o multiple studies suggest a role o HPV in a subset o head and neck squamous cell carcinoma. Multiple reports re lect that up to 40 to 60% o current diagnoses o tonsillar carcinoma demonstrate evidence o HPVtypes 16or 18.Environmentalultraviolet light exposurehasbeen associated with the development o lip cancer. The projection o the lower lip, as it relates to this solar exposure, has been used to explain why themajorityo squamouscellcarcinomasarisealongthe vermilion border o the lower lip.In addition,pipe smokingalso has been associated with the development o lip carcinoma. Factors such as mechanical irritation, thermal injury, and chemical exposure have been described as an explanation or this inding. Other entities associated with oral malignancy include Plummer-Vinson syndrome (achlorhydria, ironde iciency anemia, mucosal atrophy o mouth, pharynx, and esophagus), chronic in ection with syphilis, and immunocompromised status (30- old increase with renal transplant). (SeeSchwartz10th ed.,p.579.)
121
18
CHAPTER
3. Featureso oraltonguecarcinomaincludeallo the ollowingEXCEPT
A. Presentation asulcerated exophyticmass
B. Mayinvolve submandibular and upper cervicallymph nodes
C. Can result in contralateralparesthesias
D. CO2 laser use ul or excision o smallearlytumors
Answer:C
Tumorso thetonguebegin in thestrati ied epithelium o the sur aceand eventuallyinvadeinto thedeeper muscular structures. The tumors may present as ulcerations or as exophytic masses. The regional lymphatics o the oral cavity are to the submandibular space and the upper cervical lymph nodes. The lingual nerve and the hypoglossal nerve may be directly invaded by locally extensive tumors. Involvement can result in ipsilateralparesthesiasand deviation o thetongueon protrusion with asciculations and eventual atrophy. Tumors on thetonguemayoccur on anysur ace,but aremost commonly seen on thelateraland ventralsur aces.Primarytumorso the mesenchymalcomponentso thetongueincludeleiomyomas, leiomyosarcomas, rhabdomyosarcomas, and neuro ibromas. Surgical treatment o small (T1–T2) primary tumors is wide local excision with either primary closure or healing by secondary intention. The CO2 laser may be used or excision. (SeeSchwartz10th ed.,p.583.)
4. Branchial cle cysts, i enlarged, should be removed becauseo which o the ollowing
A. Pronetobecomingsecondarilyin ected
B. Pronetocauseacuteairwayobstructions
C. Possiblepremalignant concerns
D. Association with severehalitosis
5. All o the ollowing are FALSE about salivary gland neoplasmsEXCEPT
A. Account or less than 2% o all head and neck neoplasms
B. I in minor salivary glands, less likely to be malignant than i in theparotid gland
C. Computed tomography (CT) scanning is more accuratethanmagneticresonanceimaging(MRI)indetectinglesions
D. Oncocytomasareusuallymalignant
Answer:A Congenital branchial cle t remnants are derived rom the branchialcle t apparatusthat persistsa ter etaldevelopment. There are several types, numbered according to their corresponding embryologic branchial cle t. First branchial cle t cysts and sinuses are associated intimately with the external auditorycanal(EAC)and theparotid gland.Second and third branchialcle t cystsare ound alongtheanterior border o the sternocleidomastoid (SCM) muscle and can produce drainageviaasinustract totheneckskin.Secondaryin ectionscan occur,producingenlargement,cellulitis,andneckabscessthat requiresoperativedrainage.(SeeSchwartz10th ed.,p.598.)
Answer:A Tumors o the salivary gland are relatively uncommon and represent lessthan 2%o allhead and neck neoplasms.About 85% o salivary gland neoplasms arise within the parotid gland. The majority o these neoplasms are benign, with the most common histology being pleomorphic adenoma (benign mixed tumor). In contrast, approximately 50% o tumors arising in the submandibular and sublingual glands are malignant. Tumors arising rom minor salivary gland tissue carry an even higher risk or malignancy (75%). Diagnosticimagingisstandard or theevaluation o salivarygland tumors.Magneticresonanceimaging(MRI)isthemost sensitivestudyto determineso t-tissueextension and involvement o adjacent structures.Benign epithelialtumorsincludepleomorphic adenoma (80%), monomorphic adenoma, Warthin tumor, oncocytoma, or sebaceous neoplasm. (See Schwartz 10th ed.,p.599.)
122
6. Allo the ollowingaretrueabout tracheostomyEXCEPT
A. Should beper ormedin patientsanticipated tobeintubated morethan 2weeks
B. Improvespatient discom ort ascompared tolongterm oropharyngealintubation
C. Usually spontaneously close within 2 months o removal
D. Doesnot obligatepatient tolosso speech
Answer:C
The avoidance o prolonged orotracheal and nasotracheal intubation decreases the risk o laryngeal and subglottic injury and potential stenosis, acilitates oral and pulmonary suctioning, and decreases patient’s discom ort. When the tracheostomy is no longer needed, the tube is removed and closure o the opening usually occurs spontaneously over a 2-weekperiod.Placement o atracheostomydoesnot obligate a patient to loss o speech. When a large cu ed tracheostomy tube is in place, expecting a patient to be capable o normal speech isimpractical.However,a ter apatient isdownsized to an uncu ed tracheostomytube,intermittent inger occlusion or Passy-Muir valveplacement willallowapatient tocommunicate while still using the tracheostomy to bypass the upper airway.(SeeSchwartz10th ed.,p.602.)
123
Chest Wall,Pleura,and Mediastinum
1. All o the ollowing increase the risk or tracheal stenosis
EXCEP
A. Ageover 70years
B. Radiation
C. Malegender
D. Excessivecorticosteroid therapy
Answer:C
Intubation-related risk actors include prolonged intubation; high tracheostomy through the irst tracheal ring or cricothyroid membrane; transverse rather than vertical incision on the trachea; oversized tracheostomy tube; prior tracheostomyor intubation;and traumaticintubation.Stenosis isalso more common in older patients, in emales, a ter radiation, or a ter excessive corticosteroid therapy, and in the setting o concomitant diseases such as autoimmune disorders, severe re lux disease, or obstructive sleep apnea and the setting o severe respiratory ailure. However, even a properly placed tracheostomycan lead totrachealstenosisbecauseo scarring and local injury. Mild ulceration and stenosis are requently seen a tertracheostomyremoval.Useo thesmallesttracheostomy tube possible, rapid downsizing, and a vertical tracheal incision minimizetherisk or posttracheostomystenosis.(See Schwartz10th ed.,p.607.)
2. Adenoid cysticcarcinomas
A. Spread submucosally
B. Exhibit aggressivegrowth
C. Arenot radiosensitive
D. Havea5-year survivalrateo >50%
Answer:A
Squamous cell carcinomas o ten present with regional lymph node metastases and are requently unresectable at presentation. heir biologic behavior is similar to that o squamous cellcarcinoma o the lung.Adenoid cystic carcinomas,a type o salivary gland tumor, are generally slow-growing, spread submucosally, and tend to in iltrate along nerve sheaths and within thetrachealwall.Although indolent in nature,adenoid cystic carcinomas are malignant and can spread to regional lymph nodes, lung, and bone. Squamous cell carcinoma and adenoid cysticcarcinomasrepresent approximately65%o all tracheal neoplasms. he remaining 35%comprises small cell carcinomas,mucoepidermoid carcinomas,adenocarcinomas, lymphomas,and others.
Postoperative mortality, which occurs in up to 10% o patients,isassociatedwiththelengtho trachealresection,use o laryngeal release, the type o resection, and the histologic type o the cancer. Factors associated with improved longterm survivalincludecompleteresection and useo radiation asadjuvanttherapyin thesettingo incompleteresection.Due totheir radiosensitivity,radiotherapyis requentlygiven postoperatively a ter resection o both adenoid cystic carcinomas and squamous cell carcinomas. A dose o 50 Gray or greater is usual. Nodal positivitydoes not seem to be associated with
125
CHAPTER19
3. Which o the ollowingis NO a non–small-celltumor o thelung?
A. Squamouscellcarcinoma
B. Adenocarcinoma
C. Carcinoid tumor
D. Large-cellcarcinoma
4. Temost common pattern o benign calci cation in hamartomasis
A. Solid
B. Di use
C. Central
D. Popcorn
5. For an adenocarcinoma that has pleural invasion, tumor necrosis, and has lymphovascular invasion the correct subtypeis
A. Minimallyinvasiveadenocarcinoma(MIA)
B. Lepidicpredominant adenocarcinoma(LPA)
C. Invasiveadenocarcinoma
D. Adenocarcinomain situ
6. Te grade o neuroendocrine carcinoma (NEC) that is associated with hemoptysis, pneumonia, and tumor cells arranged in cordsand clustersis
A. GradeVINEC
B. GradeIVNEC
C. GradeIINEC
D. GradeINEC
worse survival. Survival at 5 and 10 years is much better or adenoid cystic (73 and 57%, respectively) than or tracheal cancers (47 and 36%, respectively; P <0.05). For patients with unresectable tumors, radiation may be given as the primary therapy to improve local control, but is rarely curative. For recurrent airway compromise, stenting or laser therapies should be considered part o the treatment algorithm. (See Schwartz10th ed.,pp.610–611.)
Answer:C
he term non–small-cell lung carcinoma (NSCLC) includes manytumor celltypes,includinglargecell,squamouscell,and adenocarcinoma.Historically,thesesubtypeswereconsidered to be a uni orm group based on limited understandingo the distinct clinical behaviors o the subtypes as well as the act that therewere ewtreatment optionsavailable.With increasing understanding o the molecular biologyunderlying these tumor subtypes,however,theapproachtodiagnosisandmanagement and theterminologyused in describingthesetumors isevolvingrapidly.(SeeSchwartz10th ed.,p.614.)
Answer:D Computed tomography (C ) indings characteristic o benign lesions include small size, calci ication within the nodule,and stabilityover time.Four patternso benign calciication arecommon:di use,solid,central,and laminated or “popcorn.” Granulomatous in ections, such as tuberculosis, can demonstratethe irst threepatterns,whereasthepopcorn pattern ismost common in hamartomas.In areaso endemic granulomatous disease, di erentiating benign versus malignant can be challenging. In ectious granulomas arising rom a variety o organisms account or 70 to 80% o this type o benign solitarynodules;hamartomasarethenext most common single cause, accounting or about 10%. (See Schwartz 10th ed.,p.622.)
Answer:B
I lymphovascular invasion, pleural invasion, tumor necrosis, or more than 5 mm o invasion are noted in a lesion that has lepidic growth as its predominant component,minimally invasive adenocarcinoma (MIA) is excluded and the lesion is called lepidic predominant adenocarcinoma (LPA), and the sizeo theinvasivecomponentisrecorded or the stage.(See Schwartz10th ed.,p.615.)
Answer:D
Grade I neuroendocrine carcinoma (NEC) (classic or typical carcinoid) isa low-gradeNEC;80%arisein theepithelium o the central airways. It occurs primarily in younger patients. Because o the central location, it classically presents with hemoptysis,with or without airwayobstruction and pneumonia.Histologically,tumor cellsarearranged in cordsand clusters with a rich vascular stroma. his vascularity can lead to li e-threateninghemorrhage with even simple bronchoscopic biopsy maneuvers. Regional lymph node metastases are seen in 15% o patients, but rarely spread systemically or cause death.(SeeSchwartz10th ed.,p.617.)
126
7. Which o the ollowing is NO a known predictive or prognostictumor marker or adenocarcinoma?
A. EGFR
B. KRAS
C. AFP
D. EML4-ALK
8. Desmoid tumors
A. Arise rom theperiosteum o therib
B. Are treated with wide local excision with a 2- to 4-cm margin
C. Require radical excision (sacri cing neurovascular structures)toobtain 4-cm margins
D. Require chemotherapy to treat or prevent metastatic disease
Answer:C
Establishingaclearhistologicdiagnosisearlyin theevaluation and management o lung cancer is critical to e ective treatment.Molecular signaturesarealsokeydeterminantso treatment algorithms or adenocarcinoma and will likely become important or squamous cell carcinoma as well. Currently, di erentiation between adenocarcinoma and squamous cell carcinoma in cytologic specimens or small biopsy specimens isimperativein patientswith advanced stagedisease,astreatment with pemetrexed or bevacizumab-based chemotherapy is associated with improved progression- ree survival in patients with adenocarcinoma but not squamous cell cancer. Furthermore, li e-threatening hemorrhage has occurred in patients with squamous cell carcinoma who were treated with bevacizumab. Finally, EGFRmutation predicts response to EGFR tumor kinase inhibitors and is now recommended as irst-line therapy in advanced adenocarcinoma. Because adequate tissue is required or histologic assessment and molecular testing, each institution should have a clear, multidisciplinaryapproachtopatientevaluation,tissueacquisition, tissuehandling/processing,andtissueanalysis.In manycases, tumor morphology di erentiates adenocarcinoma rom the other histologic subtypes. I no clear morphology can be identi ied,then additionaltesting or oneimmunohistochemistry marker or adenocarcinoma and one or squamous cell carcinoma will usually enable di erentiation. Immunohistochemistry or neuroendocrine markers is reserved or lesions exhibiting neuroendocrine morphology. Additional molecular testingshould beper ormed on alladenocarcinomaspecimens orknownpredictiveandprognostictumormarkers(eg, EGFR,KRAS,and EML4-ALK usion gene).Ideally,useo tissuesectionsand cellblockmaterialislimited totheminimum necessaryat each decision point. his emphasizes the importance o a multidisciplinary approach; surgeons and radiologistsmustworkin directcooperation with thecytopathologist to ensure that tissue samples are adequate or morphologic diagnosis as well as providing su icient cellular material to enablemolecular testing.(SeeSchwartz10th ed.,p.627.)
Answer:B
Because the lesions have low cellularity and poor yield with ine needle aspiration (FNA), an open incisional biopsy or lesions over 3 to 4 cm is o ten necessary. Surgery consists o wide local excision with a 2- to 4-cm margin and intraoperative rozen section assessment o resection margins. ypically, chest wallresection,includingtheinvolved rib(s) and onerib aboveand belowthetumor with a 4- to 5-cm margin o rib,is required. A margin o less than 1 cm results in much higher local recurrence rates. I a major neurovascular structure would have to be sacri iced,leadingto high morbidity,then a margin o less than 1 cm would have to su ice. Survival a ter wide local excision with negative margins is 90%at 10 years. (SeeSchwartz10th ed.,p.666.)
127
9. A 57-year-old non–small-cell lung cancer patient with a potentiallyresectable tumor ound on computed tomography (C ) scan who can walk on a at sur ace indenitely without oxygen or stopping to rest, secondary to dyspneawillmost likelytolerate
A. Lobectomy
B. Pneumonectomy
C Single-lungventilation
D. Wedgeresection
10. An “onion-peel”
o
A. Chondroma
B Ewingsarcoma
C. Plasmacytoma
D. Osteosarcoma
11. Pancoast tumors are identi ed as involvingall o the ollowingEXCEP
A. Techest wallat or belowthesecond rib.
B. umors o the parietal pleura or deeper structures overlyingthe rst rib.
C. Tesuperior sulcus.
D. Teextremeapexo thechest.
Answer:A
Patients with potentially resectable tumors require care ul assessment o their unctional status and ability to tolerate either lobectomyor pneumonectomy.
he surgeon should irst estimate the likelihood o pneumonectomy, lobectomy, or possibly sleeve resection, based on the C images. A sequential process o evaluation then un olds.
A patient’s history is the most important tool or gauging risk. Speci ic questions regarding per ormance status should be routinely asked. I the patient can walk on a lat sur ace inde initely, without oxygen and without having to stop and rest secondary to dyspnea, he will be very likely to tolerate lobectomy. I the patient can walk up two lights o stairs (up two standard levels), without having to stop and rest secondarytodyspnea,hewilllikelytoleratepneumonectomy.Finally, nearly all patients, except those with carbon dioxide (CO2) retention on arterialblood gasanalysis,willbeableto tolerate periods o single-lung ventilation and wedge resection. (See Schwartz10th ed.,pp.635–636.)
Answer:B
Primitive neuroectodermal tumors (PNE s) (neuroblastomas, ganglioneuroblastomas, and ganglioneuromas) derive rom primordialneuralcrest cellsthat migrate rom themantle layer o the developing spinal cord. Histologically, PNE s and Ewing sarcomas are small, round cell tumors; both possess a translocation between the long arms o chromosomes 11 and 22 within their genetic makeup. hey also share a consistent pattern o proto-oncogene expression and have been ound to express the product o the MIC2 gene. Ewing sarcoma occurs in adolescents and young adults who present with progressive chest wall pain, but without the presence o a mass. Systemic symptoms o malaise and ever are o ten present. Laboratory studies reveal an elevated erythrocyte sedimentation rate and mild white blood cell elevation. Radiographically, the characteristic onion peel appearance is produced by multiple layers o periosteum in the bone ormation. Evidence o bony destruction is also common. he diagnosiscan bemadebya percutaneousneedlebiopsyor an incisionalbiopsy.(SeeSchwartz10th ed.,p.669.)
Answer:A
Carcinomaarisingin theextremeapexo thechest with associated arm and shoulder pain, atrophy o the muscles o the hand, and Horner syndrome presents a unique challenge to the surgeon. Any tumor o the superior sulcus, including tumors without evidence or involvement o the neurovascular bundle, is now commonly known as Pancoast tumors, a ter Henry Pancoast who described the syndrome in 1932. he designation is reserved or tumors involving the parietal pleura or deeper structures overlying the irst rib. Chest wall involvement at or belowthe second rib is not a Pancoast tumor. reatment is multidisciplinary; due to the location o the tumor and involvement o the neurovascular bundle that supplies the ipsilateral extremity, preserving postoperative unction o the extremity is critical. (See Schwartz 10th ed., p.642.)
128
appearance o a rib on C is suggestive
12. Temost likelycauseo aspiration pneumoniais
A. Amixtureo aerobesand anaerobes
B. Aerobesonly
C. Anaerobesonly
D Gram-negativebacteria
Answer:C
Normal oropharyngeal secretions contain many more Streptococcus species and more anaerobes (approximately 1 × 108 organisms/mL) than aerobes (approximately 1 × 107 organisms/mL). Pneumonia that ollows rom aspiration, with or without abscess development, is typically polymicrobial. An average o two to our isolates present in large numbers have been cultured rom lung abscesses sampled percutaneously. Overall, at least 50% o these in ections are caused by purely anaerobic bacteria, 25% are caused by mixed aerobes and anaerobes, and 25% or ewer are caused by aerobes only. In nosocomial pneumonia, 60 to 70%o the organisms are gram-negative bacteria, including Klebsiella pneumoniae, Haemophilus influenzae, Proteus species, Pseudomonas aeruginosa, Escherichia coli, Enterobacter cloacae, and Eikenella corrodens. Immunosuppressed patients may develop abscesses because o the usual pathogens as well as less virulent and opportunistic organisms such as Salmonella species,Legionella species,Pneumocystiscarinii,atypical mycobacteria,and ungi.(SeeSchwartz10th ed.,p.650.)
13. Laboratory evaluation o a chest wall mass showing elevated erythrocytesedimentation ratesindicates
A Osteosarcoma
B. Plasmacytoma
C. Ewingsarcoma
D. Multiplemyeloma
14. Temost common benign chest walltumor is
A. Chondromas
B. Osteochondromas
C. Desmoid tumors
D. Fibrousdysplasia
15. Whicho the ollowingisan indication or surgicaldrainageo alungabscess?
A Abscess>3cm in diameter
B. Hemoptysis.
C. Failure to decrease in size afer 1 week o antibiotic therapy.
D. Persistent ever.
Answer:C
Laboratory evaluations are use ul in assessing chest wall masses or the ollowing:
1. Plasmacytoma: Serum protein electrophoresis demonstrates a single monoclonal spike, which is measuring the overproduction o one immunoglobulin rom the malignant plasmacellclone.
2. Osteosarcoma: Alkaline phosphatase levels may be elevated.
3. Ewing sarcoma: Erythrocyte sedimentation rates may be elevated.
(SeeSchwartz10th ed., able19-18,p.665.)
Answer:A
Chondromas, seen primarily in children and young adults, areoneo themorecommon benign tumorso thechest wall. hey usually occur at the costochondral junction anteriorly and maybecon used with costochondritis,except that apainless mass is present. Radiographically, lesion is lobulated and radiodense; it may have di use or ocal calci ications; and it maydisplace the bonycortex without penetration. Chondromasmaygrowto hugesizesi le t untreated. reatment issurgical resection with a 2-cm margin. Large chondromas may harbor well-di erentiated chondrosarcoma and should be managedwitha4-cm margin topreventlocalrecurrence.(See Schwartz10th ed.,p.666.)
Answer:B
Surgicaldrainageo lungabscessesisuncommon sincedrainage usually occurs spontaneously via the tracheobronchial tree.Indications or intervention arelisted in able19-1.(See Schwartz10th ed., able19-18,p.653.)
129
16. What percentageso chest wallmassesaremalignant?
A 10–20%
B. 20–30%
C 50–80%
D 40–50%
17. Te population most at risk or developing active tuberculosisis
A. Elderly
B Minorities
C Urban residents
D. Human immunode ciencyvirus(HIV)in ected
TABLE19-1 Indications orsurgicaldrainage procedures orlung abscesses
1.Failureomedicaltherapy
2.Abscessundertension
3.Abscessincreasinginsizeduringappropriatetreatment
4.Contralaterallungcontamination
5.Abscess>4–6cmindiameter
6.Necrotizinginectionwithmultipleabscesses,hemoptysis,abscess rupture,orpyopneumothorax
7.Inabilitytoexcludeacavitatingcarcinoma
Answer:C
Patients with chest wall tumors, regardless o etiology, typicallycomplain o a slowlyenlargingpalpable mass(50–70%), chest wall pain (25–50%), or both. Interestingly, growing masses are o ten not noticed by the patient until they su er a trauma to the area. Pain rom a chest wall mass is typically localized to the area o the tumor; it occurs more o ten and moreintenselywith malignant tumors,but it can alsobepresent in up to one-third o patients with benign tumors. With Ewingsarcoma, ever andmalaisemayalsobepresent.Benign chest wall tumors tend to occur in younger patients (average age 26 years), whereas malignant tumors tend to be ound in older patients (average age 40 years). Overall, between 50and 80%o chest walltumorsaremalignant.(SeeSchwartz 10th ed.,p.665.)
Answer:D
uberculosis is a widespread problem that a ects nearlyonethirdo theworld’spopulation.Between 8.3and9million new cases o tuberculosis and 12 million prevalent cases (range 10–13 million) were estimated worldwide in 2011 according to the World Health Organization. Only 10,521 new cases werereportedtotheWorldHealth Organization in theUnited States in 2011. Human immunode iciencyvirus (HIV) in ection is the strongest risk actor or developingactive tuberculosis. he elderly, minorities, and recent immigrants are the most common populations to have clinical mani estations o in ection,yet no age group,sex,or race is exempt rom in ection. In most large urban centers, reported cases o tuberculosis are more numerous amongthe homeless,prisoners, and drug-addicted populations. Immunocompromised patients additionally contribute to an increased incidence o tuberculosis in ection, o ten developing unusual systemic as well as pulmonarymani estations.(SeeSchwartz10th ed.,p.654.)
18. Te ungi associated with the highest mortality rate due toinvasivemycosesin theUnited Statesis
A Aspergillus
B. Cryptococcus
C. Candidia
D Mucor
Answer:A
he genus Aspergillus comprises over 150 species and is the most common cause o mortality due to invasive mycoses in the United States. It is typicallyacute in onset and li e threateningandoccursin thesettingo neutropenia,chronicsteroid therapy, or cytotoxic chemotherapy. It can also occur in the generalintensive care unit population o criticallyillpatients, including patients with underlying chronic obstructive pulmonary disease (COPD), postoperative patients, patients withcirrhosisoralcoholism,andpostin luenzapatients,without anyo these actors present. he species most commonly responsible or clinicaldiseaseincludeA.fumigatus,A.flavus,
130
19. A patient presenting with a history and ndings o dyspnea, wheezing, hemoptysis, and a mediastinal mass in thevisceralcompartment yieldsadiagnosiso
A. Lymphoma
B Tymomawith myastheniagravis
C. Mediastinalgranuloma
D. Germ celltumor
A. niger, and A. terreus Aspergillus is a saprophytic, ilamentous ungus with septate hyphae. Spores (2.5–3 µm in diameter) are released and easily inhaled by susceptible patients; because the spores are microns in size, they are able to reach thedistalbronchiand alveoli.(SeeSchwartz10th ed.,p.655.)
Answer:C able19-2(SeeSchwartz10th ed., able19-28,p.674.)
TABLE19-2 Signsand symptomssuggestive o variousdiagnosesin the setting o a mediastinalmass
Diagnosis
Lymphoma
HistoryandPhysicalFindings
Nightsweats,weightloss,atigue,extrathoracicadenopathy,elevated erythrocytesedimentationrateorC-reactiveproteinlevel,leukocytosis
Thymomawithmyasthenia gravis Fluctuatingweakness,earlyatigue,ptosis,diplopia
Mediastinalgranuloma Dyspnea,wheezing,hemoptysis
Germcelltumor
Malegender,youngage,testicularmass,elevatedlevelsohuman chorionicgonadotropinand/orα-etoprotein
20. Apatient with an anterior mediastinalmassand elevated serum α- etoprotein (AFP)most likelyhas
A. Ateratoma
B. Anonseminomatousgerm-celltumor
C. Aseminomatousgerm-celltumor
D Metastatichepatocellular carcinoma
Answer:B
21. Teprimarysite or malepatientswith malignant pleural e usionsis
A. Gastrointestinaltract
B Lung
C Genitourinarytract
D. Melanoma
CompartmentLocationofMass
Anycompartment
Anterior
Visceral(middle)
he use o serum markers to evaluate a mediastinal mass can be invaluable in some patients. For example, nonseminomatous and seminomatous germ-cell tumors can requently be diagnosed and o ten distinguished rom one another by the levels o α- etoprotein (AFP) and human chorionic gonadotropin (hCG). In over 90% o nonseminomatous germ-cell tumors, either the AFP or the hCG level will be elevated. Results are close to 100%speci ic i the level o either AFP or hCG is greater than 500 ng/mL. Some centers institute chemotherapy based on this result alone, without biopsy con irmation o the diagnosis.In contrast,the AFP levelin patients with mediastinal seminoma is always normal; only 10% will have elevated hCG, which is usually less than 100 ng/mL. Other serum markers, such as intact parathyroid hormone level or ectopic parathyroid adenomas, may be use ul or diagnosing and also or intraoperatively con irming complete resection. A ter success ul resection o a parathyroid adenoma, this hormone level should rapidly normalize. (See Schwartz10th ed.,p.672.)
Answer:B
Malignant pleural e usions may occur in association with a number o di erent malignancies, most commonlylungcancer,breast cancer,and lymphomas,dependingon thepatient’s ageandgender ( ables19-3and19-4).(SeeSchwartz10thed., ables19-35and 19-36,pp.682and 684.)
131
Anterior
TABLE19-3 Primaryorgan site orneoplasm type in male patientswith malignant pleuralefusions
Source:Reproduced with permission rom JohnstonWW.The malignant pleuralefusion: areviewo cytopathologicdiagnoseso 584specimens rom 472consecutive patients. Cancer.1985;56:905.Copyright ©1985American CancerSociety.
22. Eosinophilicgranulomasareassociated with
A. Langerhanscellhistiocytosis(LCH)
B. Parasiticin ections
C. Crohn disease
D. Gardner syndrome
Source:Reproduced with permission rom JohnstonWW.The malignant pleural efusion:a reviewo cytopathologicdiagnoseso 584specimens rom 472consecutive patients.Cancer.1985;56:905.Copyright ©1985American CancerSociety.
Answer:A
Eosinophilicgranulomasarebenign osteolyticlesions.Eosinophilic granulomas o the ribs can occur as solitarylesionsor aspart o amoregeneralized diseaseprocesso thelymphoreticular system termed Langerhanscellhistiocytosis(LCH).In LCH, the involved tissue is in iltrated with large numbers o histiocytes(similar to Langerhanscellsseen in skin and other epithelia), which are o ten organized as granulomas. he cause is unknown. O all LCH bone lesions, 79%are solitary eosinophilic granulomas, 7% involve multiple eosinophilic granulomas,and 14%belongto other ormso moresystemic LCH.Isolatedsingleeosinophilicgranulomascan occur in the ribsor skull,pelvis,mandible,humerus,and other sites. hey arediagnosed primarilyin children between theageso 5and 15 years. Because o the associated pain and tenderness, they may be con used with Ewing sarcoma or with an in lammatoryprocesssuch asosteomyelitis.Healingmayoccur spontaneously,but thetypicaltreatment islimited surgicalresection with a2-cm margin.(SeeSchwartz10th ed.,p.666.)
132
PrimarySiteor TumorType No.ofMale Patients PercentageofMale Patients Lung 140 49.1 Lymphoma/leukemia 60 21.1 Gastrointestinaltract 20 7.0 Genitourinarytract 17 6.0 Melanoma 4 1.4 Miscellaneousless commontumors 10 3.5 Primarysiteunknown 31 10.9 Total 285 100.0
PrimarySiteor TumorType No.ofFemale Patients Percentageof FemalePatients Breast 70 37.4 Femalegenitaltract 38 20.3 Lung 28 15.0 Lymphoma 14 8.0 Gastrointestinaltract 8 4.3 Melanoma 6 3.2 Urinarytract 2 1.1 Miscellaneousless commontumors 3 16 Primarysiteunknown 17 9.1 Total 187 100.0
TABLE19-4 Primaryorgansiteorneoplasmtypein emale patientswithmalignantpleuralefusions
23. A chylothorax is likely to be present in a patient whose pleural uid analysisresultsshowatriglyceridelevelo
A 80mg/100mL
B 100mg/100mL
C. 45mg/100mL
D 130mg/100mL
24. Osteosarcomao therib
A. Is considered nonoperable i pulmonary metastases arepresent
B. Istreated with radiation therapybe oreresection
C. Istreatedwithadjuvantchemotherapybe oreresection
D. Requiresexcision with a6-cm margin
Answer:D
Laboratoryanalysiso thepleural luid showsa high lymphocytecount and high triglyceridelevels.I thetriglyceridelevel is greater than 110 mg/100 mL, a chylothorax is almost certainlypresent (a 99%accuracyrate).I thetriglyceridelevelis lessthan 50mg/mL,thereisonlya 5%chanceo chylothorax. (SeeSchwartz10th ed.,p.686.)
Answer:C
While osteosarcomas are the most common bone malignancy, they represent only 10 to 15% o all malignant chest wall tumors. hey primarily occur in young adults as rapidly enlarging, pain ul masses; however, osteosarcomas can occur in older patients as well,sometimes in association with previous radiation, Paget disease, or chemotherapy. Radiographically,the typicalappearance consistso spiculeso new periosteal bone ormation producing a sunburst appearance. Osteosarcomas have a propensity to spread to the lungs, and up to one-third o patients present with metastatic disease. Osteosarcomas are potentially sensitive to chemotherapy. Currently,preoperativechemotherapyiscommon.A ter chemotherapy,completeresection isper ormedwithwide(4-cm) margins, ollowed by reconstruction. In patients presenting with lungmetastasesthat arepotentiallyamenableto surgical resection,induction chemotherapymaybegiven, ollowed by surgicalresection o theprimarytumor and o thepulmonary metastases. Following surgical treatment o known disease, additional maintenance chemotherapy is usually recommended.(SeeSchwartz10th ed.,p.667.)
25. Excisionalbiopsyo achest wallmassisallowed i
A. Needlebiopsywasnondiagnostic.
B. Imagingrevealsclassicappearanceo achondrosarcoma.
C. It is>3cm.
D. Noneo theabove.
Answer:B
1. Needle biopsy: Pathologists experienced with sarcomas can accurately diagnose approximately 90% o patients usingFNAcytology.Aneedlebiopsy(FNAor core)hasthe advantage o avoidingwound and bodycavitycontamination (apotentialcomplication with an incisionalbiopsy).
2. Incisional biopsy:I a needle biopsy is nondiagnostic, an incisional biopsy may be per ormed, with caveats. First, theskin incision must beplaced directlyover themassand oriented to allow subsequent scar excision; skin laps and drainsshould beavoided.However,i thesurgeon believes a hematoma is likelyto develop, a drain is use ul or limitingso t tissuecontamination bytumor cells.At thetimeo de initivesurgicalresection,theen blocresection includes thebiopsyscar and thedrain tract alongwith thetumor.
3. Excisional biopsy: Any lesion less than 2.0 cm can be excised as long as the resulting wound is small enough to close primarily.Otherwise,excisionalbiopsyis per ormed only when the initial diagnosis (based on radiographic evaluation) indicates that the lesion is benign or when the lesion has the classic appearance o a chondrosarcoma (in which case, de initive surgical resection can be undertaken).(SeeSchwartz10th ed.,p.666.)
133
1. Temost common orm o atrialseptalde ect (ASD)is
A. Sinusvenosusde ect
B. Ostium primum de ect
C. Ostium secundum de ect
D. Combined primum and secundum de ect
CongenitalHeart Disease
Answer:C
Atrialseptalde ects(ASDs) can beclassi ied into threedi erenttypes:(1)sinusvenosusde ects,comprisingapproximately 5 to 10% o all ASDs; (2) ostium primum de ects, which are more correctly described as partial atrioventricular canal de ects;and (3) ostium secundum de ects,which arethemost prevalent subtype,comprising80%o allASDs.(SeeSchwartz 10th ed.,pp.695–696.)
2. Temost common agetocloseasymptomaticASDsis
A. In theimmediatenewborn period
B. A er thechild reaches10kgin weight
C. Age4–5years
D. Duringpuberty
3. Which o the ollowingisNO an acceptabletreatment or aorticvalvestenosiswith ahypoplasticle ventricle(LV)?
A. Balloon valvotomy
B. Intubation and initiation o prostaglandin
C. Surgicalvalvotomy
D. Norwood procedure
Answer:C
ASDs are closed when patients are between 4 and 5 years o age. Children o this size can usually be operated on without the use o blood trans usion and generally have excellent outcomes. Patients who are symptomatic may require repair earlier, even in in ancy. Some surgeons, however, advocate routine repair in in ants and children,as even smaller de ects are associated with the risk o paradoxical embolism, particularlyduringpregnancy. Reddyand colleagues, report 116 neonates weighing less than 2500 g who underwent repair o simpleand complexcardiacde ectswith theuseo cardiopulmonary bypass and ound no intracerebral hemorrhages, no long-term neurologicsequelae,and a lowoperative-mortality rate(10%). heseresultscorrelated with thelength o cardiopulmonarybypassandthecomplexityo repair. heseinvestigatorsalso oundan 80%actuarialsurvivalat1year and,more importantly,thatgrowth ollowingcompleterepair wasequivalent to weight-matched neonates ree rom cardiac de ects. (SeeSchwartz10th ed.,p.697.)
Answer:A
In patients with critical aortic stenosis, the degree o le t ventricular hypoplasia is assessed and based on this the decision or biventricular and univentricular repair is made. Urgent intervention is needed in these critically ill neonates includingintubation,inotropicsupport,and prostaglandin tomaintain ductalpatency or systemicblood low.In thepresenceo hypoplastic LV, isolated aortic valvotomy should not be perormed because studies have demonstrated high mortality in the population ollowing isolated valvotomy. he Norwood procedure is the irst part o the staged single ventricle pathway.(SeeSchwartz10th ed.,p.699.)
135
CHAPTER 20
4.
A. Aorticarch
B. Distaltothele subclavian artery
C. At thediaphragm
D. At thelevelo therenalarteries
5. Te treatment o choice or recurrent COA (a er surgical repair)in apreschool-aged child
A. Resection and primaryanastomosis
B. Resection with interposition gra
C. Balloon dilatation alone
D. Balloon dilatation with stenting
6. Which o the ollowing is a true surgical emergency in a newborn?
A. etralogyo Fallot
B. runcusarteriosus
C. otal anomalous pulmonary venous connection ( APVC)
D. COA
7. TebidirectionalGlenn procedureisused tocorrect
A. ricuspid atresia
B. Patent ductusarteriosus
C. ransposition o thegreat arteries
D. otalanomalouspulmonaryvenousconnection
Answer:B
Coarctation o the aorta (COA) is de ined as a luminal narrowing in the aorta that causes an obstruction to blood low. his narrowing is most commonly located distal to the le t subclavian artery. he embryologic origin o COA is a subject o some controversy. One theoryholds that the obstructing shel , which is largely composed o tissue ound within the ductus, orms as the ductus involutes. he other theory holdsthat a diminished aorticisthmusdevelopssecondaryto decreased aortic lowin in antswith enhanced ductalcirculation.(SeeSchwartz10th ed.,p.705.)
Answer:C
Children younger than 6 months with native COA should be treated with surgical repair, while those requiring intervention atlater agesmaybeidealcandidates or balloon dilatation or primary stent implantation. Additionally, catheter-based therapy should be employed or those cases o restenosis ollowingeither surgicalor primaryendovascular management. (SeeSchwartz10th ed.,p.706.)
Answer:C
otal anomalous pulmonary venous connection ( APVC) occurs in 1 to 2% o all cardiac mal ormations and is characterized by abnormal drainage o the pulmonary veins into the right heart, whether through connections into the right atrium or into its tributaries. Accordingly, the only mechanism by which oxygenated blood can return to the le t heart is through an ASD, which is almost uni ormly present with APVC.
Unique to this lesion is the absence o a de initive orm o palliation. hus, APVCrepresentsoneo theonlytruesurgicalemergenciesacrosstheentirespectrum o congenitalheart surgery.(SeeSchwartz10th ed.,p.707.)
Answer:A
Recognizing the inadequacies o the initial repairs or tricuspid atresia, Glenn described the irst success ul cavopulmonary anastomosis, an end-to-side right pulmonary artery (RPA)-to-superior vena cava (SVC) shunt in 1958, and later modi ied this to allow low to both pulmonary arteries. his end-to-side RPA-to-SVC anastomosis was known as the bidirectional Glenn, and is the irst stage to inal Fontan repair in widespreadusetoday. heFontanrepairwasamajoradvancement in the treatment o congenital heart disease, as it essentially bypassed the right heart, and allowed separation o the pulmonaryand systemiccirculations.(SeeSchwartz10th ed., p.713.)
136
Te most common location or a coarctation o the aorta (COA)is
8. Hypoplasticle heart syndromeissurgicallytreated with
A. Bilateral pulmonary artery banding and stent placement in thepatent ductusarteriosus.
B. Norwood procedure with a Blalock- aussig (B- ) shunt.
C. Norwood procedure with a right ventricle (RV) to pulmonaryarteryconduit (Sanoshunt).
D Allo theabove
9. Te arterial switch operation or transposition o the great vesselsisbest per ormed
A Within 2weekso birth
B. At 1year o age
C. At 10kgo weight
D. In adolescence
10. Which o the ollowingisNO oneo thecomponentso thetetralogyo Fallot ( OF)?
A. ASD
B Ventricular septalde ect
C. Right ventricular hypertrophy
D. Right ventricular out owobstruction
11. Temost commonlyrecommended age or correction o a OFis
A. Neonateyounger than 3months
B. 6monthso age
C 1year o age
D. 4–5yearso age
Answer:D
In 1983, Norwood and colleagues described a two-stage palliative surgical procedure or relie o hypoplastic le t heart syndromethat waslater modi ied to thecurrentlyused threestage method o palliation. Stage 1 palliation, also known as the modified Norwood procedure, bypasses the LV bycreating asingleout lowvessel,theneoaorta,whicharises rom theRV. More recently, Sano introduced a modi ication that includes arch reconstruction and placement o the shunt between the RVand pulmonaryartery(Sanoshunt),which diminishesthe diastolic lowcreated bytheclassicalB- shunt and mayaugmentcoronaryper usion,resultingin improvedpostoperative cardiac unction. A newer approach combines surgical and percutaneous techniques (hybrid procedure). he bilateral pulmonaryarterieso surgicallybanded torestrict excesspulmonaryblood lowa ter pulmonaryvascular resistancedrops and ductal stenting to maintain patency. he hybrid proceduredoesnot requirecardiopulmonarybypass.(SeeSchwartz 10th ed.,pp.717–718.)
Answer:A
he most important consideration is the timing o surgical repair, because arterial switch should be per ormed within 2 weeks a ter birth, be ore the LV loses its ability to pump against systemic a terload. In patients presenting later than 2 weeks, the LV can be retrained with preliminary pulmonary arterybandingand aortopulmonaryshunt ollowed byde initiverepair.Alternatively,theunprepared LVcan besupported ollowing arterial switch with a mechanical assist device or a ew days while it recovers ability to manage systemic pressures.Echocardiographycan beused to assessle t ventricular per ormance and guide operative planning in these circumstances.(SeeSchwartz10th ed.,p.721.)
Answer:A
he our eatureso tetralogyo Fallot ( OF) are(1) malalignment ventricular septalde ect,(2) dextroposition o the aorta, (3) right ventricular out low tract obstruction, and (4) right ventricularhypertrophy. hiscombinationo de ectsarisesasa result o underdevelopment and anterole tward malalignment o thein undibular septum.(SeeSchwartz10th ed.,p.724.)
Answer:A
However, systemic-to-pulmonary shunts, generally a Bshunt,maystillbepre erredwithan unstableneonateyounger than 3 months, when an extracardiac conduit is required because o an anomalous le t anterior descending coronary artery,or when pulmonaryatresia,signi icant branch pulmonaryarteryhypoplasia,or severenoncardiacanomaliescoexist with OF.(SeeSchwartz10th ed.,p.724.)
137
12. Which o the ollowingisthemost common typeo ventricular septal de ect (VSD) to require surgical correction?
A Ostium primum
B Ostium secundum
C. Muscular
D Perimembranous
13. What is the best predictor o spontaneous closure o a VSD?
A Size
B. Ageat diagnosis
C. Gestationalage
D. Lacko electrocardiogram changes
14. Which o the ollowingcardiacabnormalities,allo them well-tolerated during etal li e, becomes a serious problem at birth?
A. Aorticarch
B. Ductusarteriosus
C Foramen ovale
D ricuspid atresia
15. Cor triatriatum is
A Treeatriawheretheright atrium isdivided byadiaphragm with the SVC and in erior vena cava (IVC) drain intoseparateatria.
B Tree atria where the SVC and IVC drain into the superior right atrium and thein erior drainsthrough thetricuspid valvein theRV.
C Tree atria where the le atrium is divided by a diaphragm and the superior and in erior pulmonary veinsdrain intoseparatechambers.
D Tree atria where the le atrium is divided by a diaphragm separatingthechamber receivingpulmonary return rom thechamber drainingthrough themitral valveintotheLV.
Answer:D
Perimembranous ventricular septal de ects (VSDs) are the most common type requiringsurgicalintervention,comprising approximately 80% o cases. hese de ects involve the membranous septum and include the malalignment de ects seen in OF.(SeeSchwartz10th ed.,p.726.)
Answer:B
VSDsmaycloseor narrowspontaneously,and theprobability o closure is inverselyrelated to the age at which the de ect is observed. hus, in ants at 1 month o age have an 80% incidence o spontaneous closure, whereas a child at 12 months o age has only a 25% chance o closure. his has an important impact on operative decision making, because a small or moderate-size VSD may be observed or a period o time in the absence o symptoms. Large de ects and those in severely symptomatic neonates should be repaired during in ancy to relieve symptoms because irreversible changes in pulmonary vascular resistance may develop during the irst year o li e. (SeeSchwartz10th ed.,p.727.)
Answer:D
In etal li e, blood reaching the right atrium has been oxygenated by the placenta. his blood bypasses the highresistance pulmonary circulation to enter the systemic circulation through septal de ects and the ductus arteriosus. When the child is born, the pulmonary circulation becomes important. Septal de ects and the ductus arteriosus can be tolerated by the newborn child, but tricuspid atresia, pulmonic valve stenosis, or other right ventricular out low obstruction orces a right-to-le t shunt, with development o cyanoticheart disease.(SeeSchwartz10th ed.,p.712.)
Answer:D
Cor triatriatum results in obstruction o pulmonary venous return to thele t atrium with adiaphragm dividingthechamber receiving pulmonaryreturn rom the camber in continuity with the mitral valve and LV. he degree o obstruction is variable and depends on the size o enestrations present in thele tatrialmembrane,thesizeo theASD,andtheexistence o other associated anomalies.I thecommunication between thesuperior and in erior chambersislessthan 3mm,patients are symptomatic during the irst year o li e. he a licted in ant willpresent with thestigmatao lowcardiacoutput and pulmonary venous hypertension, as well as congestive heart ailureand poor eeding.(SeeSchwartz10th ed.,p.710.)
138
16. FlowacrossaVSDisdependent upon
A. Sizeo de ect
B. Le and right ventricular pressure and size o the de ect
C. Pulmonary and systemic vascular resistance and de ect size
D. Pulmonaryand systemicvascular resistance
Answer:C hesizeo theVSD determinestheinitialpathophysiologyo thedisease.LargeVSDsareclassi iedasnonrestrictiveand are at least equal in diameter to the aortic annulus. hese de ects allow ree lowo blood rom theLVto theRV,elevatingright ventricular pressures to the same level as systemic pressure. Consequently,thepulmonary-to-systemic lowratio (Qp:Qs) is inversely dependent on the ratio o pulmonary vascular resistance to systemic vascular resistance. Nonrestrictive VSDs produce a large increase in pulmonaryblood low, and the a licted in ant will present with symptoms o congestive heart ailure. However, i untreated, these de ects will cause pulmonary hypertension with a corresponding increase in pulmonary vascular resistance. his will lead to a reversal o low (a right-to-le t shunt), which is known as Eisenmenger syndrome.
Small restrictive VSDs o er signi icant resistance to the passageo blood acrossthede ect,and there oreright ventricular pressure iseither normalor onlyminimallyelevated and Qp:Qs rarely exceeds 1.5. hese de ects are generally asymptomatic because there are ew physiologic consequences. However, there is a long-term risk o endocarditis, because endocardial damage rom the jet o blood through the de ect mayserve as a possible nidus or colonization. (See Schwartz 10th ed.,p.727.)
17. Achild with a largeVSD and no other cardiaclesion can beexpected todevelop allo the ollowingEXCEP
A Cyanosis
B Failuretothrive
C. Le ventricular hypertrophygreater than le ventricular dilation
D. Increased susceptibility to lower respiratory tract in ection
18. Beyond early childhood, high pulmonary blood ow is most apt toproduce
A. Cyanosison exercise
B. Diminished exercisetolerance
C. Periodicepisodeso hemoptysis
D. Right ventricular hypertrophy
19.
Te most important diagnostic assessment modality or evaluatingin antsandchildren with congenitalheart
diseaseis
A. Cardiaccatheterization
B. Chest X-ray
C. ransesophagealechocardiogram
D ransthoracicechocardiogram
Answer:C
A VSD produces a le t-to-right shunt because systemic vascular resistance is greater than pulmonary vascular resistance. heextraworkrequired bytheshunt leadstoincreased basal energy expenditure and a ailure to thrive. he shunt into the pulmonary circulation produces congestion and an associated increased susceptibility to lower respiratory tract in ection. Large ventricular de ects result in dilation but minimal hypertrophy due to decreased a terload. (See Schwartz10th ed.,p.727.)
Answer:B
Highpulmonaryblood lowbeyondin ancymayproducesurprisingly little disability or a period o time, and the diminished exercise tolerancemaybe subtle.Cyanosis,hemoptysis, and pneumonia are not anticipated. With the volume overloadingin theRV,ventricular dilatation ismorecommon than ventricular hypertrophy.(SeeSchwartz10th ed.)
Answer:D
Although chest X-raymayde ineheart sizeand electrocardiograms indicate cardiac rhythm, transthoracic and subcostal echocardiograms provide in ormation on cardiac structure and unction. ransesophageal echocardiogram, o ten very important in adults, is not required in children because children have excellent acoustic windows or the conventional studies. Cardiac catheterization is currently used most requently or therapeutic reasons such as balloon dilatation o an uncomplicated isolated valvular pulmonic stenosis or coil occlusion o apatentductusarteriosus.(SeeSchwartz10th ed., p.697.)
139
20. Te major determinant o operability in patients who haveaVSDis
A. Tesizeo thede ect.
B. Telocation o thede ect.
C Tepulmonaryvascular resistance
D. Teageo thepatient.
21. ransplant- ree survival a er Norwood with B- shunt (system to pulmonary) versusSano shunt (RVto pulmonary shunt) in patients with hypoplastic le heart syndromeis
A. Equalat 12months,though worse or B- shunt with longer term ollow-up
B. Worse or B- shunt at 12monthsand at longer term ollow-up
C. Better at 12months or B- shunt,but equalat longer term ollow-up
D. Worseat 12months or B- shunt,but equalat longer term ollow-up
Answer:C he speci ic anatomy o a ventricular de ect, its size, and the age o the a ected patient are not much hindrance to closure o the de ect, and the major determinant o operability is the degree o pulmonary vascular resistance that is present. It is important todi erentiatebetween pulmonaryarterypressure and vascular resistance. he pressure may be elevated by a largeincreasein blood low,andyettheresistancemaybenormal;conversely,thepressuremaybemarkedlyelevated in the presence o an almost normal blood low i the resistance is increased.When pulmonaryvascular resistanceexceedsonehal the systemic resistance, the de ect generally is considered inoperable. hose patients who have severe pulmonary vascular resistance increase their cardiac output by right-tole t shunting across the de ect because they cannot increase their pulmonary blood low. I the de ect is closed, they have no mechanism to increase cardiac output with exercise. Most cases o VSD are detected todayand the a ected patient success ully operated on within the irst years o li e be ore pulmonaryvascular resistancehasbecomeseverelyelevated.(See Schwartz10th ed.,p.727.)
Answer:D
he postoperative management o in ants ollowing stage 1 palliation is complex because avorable outcomes depend on establishing a delicate balance between pulmonary and systemic per usion. Recent literature suggests that these in ants require adequate postoperative cardiac output in order to supplyboth the pulmonaryand the systemic circulationsand that the use o oximetric catheters to monitor mixed venous oxygen saturation (Svo2) aids clinicians in both the selection o inotropic agents and in ventilatory management. Recent introduction o a modi ication that includes arch reconstruction and placement o the shunt between the RV and the pulmonary artery (Sano shunt) diminishes the diastolic low created bytheclassicalB- shunt and mayaugment coronary per usion, resulting in improved postoperative cardiac unction. A recent prospective, randomized, multi-institutional trial sponsored by the National Institutes o Health, the Systemic Ventricle Reconstruction (SVR) trial, compared the outcomes o neonates having either a modi ied B- shunt versus a Sano shunt. he SVR trial demonstrated that transplantation- ree survival 12 months a ter randomization was higher with theSano shunt than with themodi ied B- shunt (74 vs 64%, P = 0.01). However, the Sano shunt group had moreunintendedinterventions(P=0.003)andcomplications (P = 0.002). Right ventricular size and unction at the age o 14 months and the rate o non atal serious adverse events at theageo 12monthsweresimilar in thetwogroups.Datacollected over amean (± standard deviation) ollow-up period o 32 ± 11 months showed a nonsigni icant di erence in transplantation- ree survival between the two groups (P = 0.06). (SeeSchwartz10th ed.,p.717.)
140
22. Aprematurein ant isdiscovered at birth to haveapatent ductusarteriosuswith moderaterespiratorydistress.Te in ant does not improve a er 48 hours o medical management with uid restriction, diuretics, and respiratory support.Tenext step in management is
A. Acetylsalicylicacid
B Indomethacin
C. Surgicalcorrection o theductus
D. ransvenousocclusion o theductus
23. Which o the ollowingisNO atypeo VSD?
A Perimembranous
B Atrioventricular canal
C. Supracristal
D Sinusvenosal
24. Duringle thoracotomy or repair o patent ductusarteriosus the blood pressure is 70/22. Immediately a er placement o aclip acrosstheduct theblood pressureis
A 70/22
B. 70/40
C 90/22
D 90/40
25. All o the ollowing are true about truncus arteriosus EXCEP
A. runcalvalvesmost commonlyhavethreelea ets.
B Patientsusuallypresentwithmildtomoderatecyanosisandcongestiveheart ailurein theneonatalperiod.
C. Patientsshould undergorepair at 6monthso age
D. Tereisacontinuousle toright shunt.
Answer:B
Prostaglandinsopposecontraction o thesmooth musclethat obliteratestheductus.Indomethacin isaprostaglandin inhibitor and,given intravenously,leadsto closure o the ductus in theprematurein ant.Anationalcooperativestudy ound that indomethacin e ected closure in 79% o 3559 patients studied. Although surgical closure o the ductus is surprisingly well-tolerated in these in ants, operation should not be done unless this therapy does not close the ductus and symptoms arepoorlycontrolled.(SeeSchwartz10th ed.,p.704.)
Answer:D
VSD re ers to a hole between the le t and RVs. hese de ects are common, comprising 20 to 30%o all cases o congenital heart disease, and may occur as an isolated lesion or as part o a morecomplexmal ormation.VSDsvaryin size rom 3to 4 mm to more than 3 cm, and are classi ied into our types based on their location in the ventricular septum: perimembranous, atrioventricular canal, outlet or supracristal, and muscular.(SeeSchwartz10th ed.,p.726.)
Answer:B
Patent ductus arteriosus results in lower aortic diastolic pressure which increases the potential or myocardial ischemia and underper usion o other systemicorgans,whileincreased pulmonary blood low leads to increased work o breathing and decreased gas exchange. Closure o the duct immediately increases diastolic systemic blood pressure, while leaving systolic pressure unchanged. (See Schwartz 10th ed., pp.706–707.)
Answer:C
he truncal valve is most commonly trilea let (60%), but occasionallybicuspid and rarelyquadricuspid (25%).Patients present in the neonatal period with signs and symptoms o congestiveheart ailureandmildtomoderatecyanosis.Apansystolic murmur may be noted at the le t sternal border and occasionallyadiastolicmurmur maybeheard in thepresence o truncal regurgitation. he presence o truncus arteriosus is an indication or surgery. Repair should be undertaken in the neonatal period or as soon as the diagnosis is established to prevent the development o pulmonary hypertension due to pulmonaryover circulation. he presence o Eisenmenger physiology, which is ound primarily in older children, is the only absolute contraindication to correction. (See Schwartz 10th ed.,pp.706–707)
26. Allaretrueregardingclosureo ASDsin adultsEXCEP
A. Atrialarrhythmiasarecommon postoperatively.
B. Postoperative mortality is signi cantly higher with increasingage.
C. Closure o ASDs in patients older than 60 decreases risk o paradoxical embolism, but has little efect on unctionalstatus.
D. Secundum ASD closure is more commonly perormed using transcatheter approach compared with surgicalapproaches.
Answer:C
A ter closure, unctional capacity as measured by standardized survey instruments was signi icantly improved in older patients. Atrial arrhythmias are a common complication o ASDs and are not completely mitigated by closure o the de ect, though atrial and right ventricular size decrease. hough adults are subject to more complications during the postoperative period, mortality is extremely low, as it is in children who undergo closure.Recent advances in transcatheter technology have resulted in the majority o secundum de ects being repaired using these devices. (See Schwartz 10th ed.,p.67.)
141
1. Tebypassconduit with thehighest patencyrateisthe
A. Radialartery
B. Internalthoracicartery
C. Greater saphenousvein
D. Radialartery
2. Which o the ollowingistrueabout anginapectoris?
A. Angina is typically substernal and may radiate to the le upper extremity.
B. “ ypical” angina occurs in approximately 50% o patientswith coronarydisease.
C. “Atypical”anginaoccursmorecommonlyin men.
D. Anginaisatypicalsymptom or mitralstenosis.
Acquired Heart Disease
Answer:B
he most important criterion in conduit selection is gra t patency. he conduit with the highest patency rate (98% at 5yearsand 85–90%at 10years) istheinternalthoracicartery which is most commonlyle t attached proximallyto the subclavian artery(although occasionallyused asa reegra t) and anastomosed distally to the target coronary artery. he use o bothinternalthoracicarterieshasbeenshowntoincreaseeventree survival in a number o studies. (See Schwartz 10th ed., pp.743–744.)
Answer:A
Angina pectoris is the pain or discom ort caused by myocardial ischemia and is typically substernal and may radiate to the le t upper extremity, le t neck, or epigastrium. he varietyo presentationscan makemyocardialischemiadi icultto diagnose. Characteristics o chest pain that make myocardial ischemia less likely include: pleuritic chest pain, pain reproducible by movement or palpation, or brie episodes lasting only seconds. ypical angina is relieved by rest and/or use o sublingual nitroglycerin. Di erential diagnoses to be considered include, but are not limited to, musculoskeletal pain, pulmonary disorders, esophageal spasm, pericarditis, aortic dissection, gastroesophageal re lux, neuropathic pain, and anxiety.(SeeSchwartz10th ed.,p.742.)
3. Aholosystolicmurmer that isaccompanied byaventricular septalde ect isassociated with the ollowingetiology
A. Ventricular llingthat ollowsatrialcontraction
B. Crescendo-decrescendo;occur asblood isejected into thele and right ventricular out owtracts.
C. Flow between chambers that have widely di erent pressuresthroughout systole.
D. Arelativedisproportion between valveori ce size and diastolicblood owvolume.
Answer:C
See able21-1.(SeeSchwartz10th ed., able21-7,p.748.)
143
21
CHAPTER
Murmur Condition
Holosystolic(pansystolic) VSD
Mid-systolic(systolicejection) High owrate,MS,MR,TS,TI
Mechanism/Etiology
SystolicMurmurs
Flowbetweenchambersthathavewidelydierentpressuresthroughoutsystole
Otencrescendo-decrescendoinconguration;occurasbloodisejectedintothelet andrightventricularoutowtracts
Earlysystolic EarlyTI,acuteMR Lesscommon
Midtolatesystolic MR,MVP
Sottomoderatehigh-pitchedmurmursattheLVapex;otenduetoapicaltethering andmalcoaptationoMVleaets;anassociatedclickindicatesprolapseotheMV leaets
DiastolicMurmurs
Earlyhigh-pitched AI,PR
Mid-diastolic MS,TS,PDA*,VSD*,ASD*
Presystolic MS,TS
Systolicanddiastolic PDA
Generallydecrescendoinconguration;occurwhentheassociatedventricular pressuredropssufcientlybelowthatotheoutowtract
Duetoarelativedisproportionbetweenvalveoricesizeanddiastolicblood ow volume;seeninnormalMVandTVwithincreaseddiastolicblood owassociated withtheseconditions*
Occurduringtheperiodoventricular llingthatollowsatrialcontraction(ie,only occurinsinusrhythm)
ContinuousMurmurs
Uncommon,duetoshuntsthatpersistthroughtheendosystoleandthesomeorall odiastole
AI=aorticinsufciency;ASD=atrialseptalde ect;MR=mitralregurgitation;MS=mitralstenosis.MVP=mitralvalve prolapse;PDA= patent ductusarteriosus;PR= pulmonicregurgitation; TI= tricuspid insufciency;TS= tricuspid stenosis;VSD= ventricularseptalde ect.
4. Te ollowingisNO trueo le ventricular aneurysms
A. Ruptureisextremelycommon.
B. Anginaisacommon symptom.
C. Embolicphenomenon israre.
D. Ventricular arrhythmiasarecommon.
5. Temost common arrhythmiaworldwideis
A. Atrial utter
B. Paroxysmalsupraventricular tachycardia(PSV )
C. Wol -Parkinson-White(WPW)syndrome
D. Atrial brillation (AF)
6. Temost common causeo acquired mitralstenosisis
A. Rheumaticdisease
B. Le atrialmyxoma
C. Ballvalvethrombus
D. Previouschest radiation
Answer:A
Symptoms o le t ventricle (LV) aneurysms include angina, congestive heart ailure (CHF), ventricular arrhythmias, and rarely embolic phenomenon. Rupture is extremely uncommon. Patients generallypresent or coronaryarterybypass or during evaluation o CHF or arrhythmias. While transthoracicechocardiographygivespertinentin ormation regarding LV unction, size, mitral valve unction, and the presence o thrombus,it isgenerallyaccepted that cardiacmagneticresonanceimaging(MRI) isthebest diagnosticmodalityto accuratelyidenti yareaso scar andviabletissue,andtobestde ine ventricular geometry.(SeeSchwartz10th ed.,p.767.)
Answer:D
Atrial ibrillation (AF) remains the most common arrhythmia in the world with an overall incidence o 0.4 to 1% that increases to 8% in those older than 80 years. he most serious complication o AF is thromboembolism with resultant stroke, but serious morbidity and mortality may also result rom hemodynamiccompromisedueto losso atrialcontraction, exacerbations o CHF rom atrioventricular asynchrony and tachycardia-induced cardiomyopathy. (See Schwartz 10th ed.,p.771.)
Answer:A
Acquired mitral stenosis (MS) is most o ten caused by rheumatic ever,with approximately60%o patientswith pureMS presenting with a positive clinical history o rheumatic heart disease.Rarely,other conditionscan causeobstruction to illingo the LV,mimickingMS. Acquired causes o mitral valve obstruction include le t atrial myxoma, ball valve thrombus, mucopolysaccharidosis, previous chest radiation, and severe annular calci ication.(SeeSchwartz10th ed.,p.751.)
144
TABLE21-1 Classifcation o cardiacmurmurs
7. What valvular lesion ismost commonly ound in apatient with Ehlers-Danlossyndrome?
A. MS
B. Mitralinsufciency
C. Aorticstenosis(AS)
D. Aorticinsufciency(AI)
Answer:D
hemostcommoncauseo isolatedaorticinsu iciency(AI)in patients undergoing aortic valve replacement (AVR) is aortic root disease,and representsover 50%o such patientsin some studies.Other common causeso AIincludecongenitalabnormalities o the aortic valve, such as bicuspid aortic valve, calci ic degeneration, rheumatic disease, in ective endocarditis, systemic hypertension, myxomatous degeneration, dissection o the ascending aorta, and Mar an syndrome. Less common causes o AI include traumatic injuries to the aortic valve, ankylosingspondylitis,syphiliticaortitis,rheumatoidarthritis, osteogenesisimper ecta,giant cellaortitis,Ehlers-Danlossyndrome, Reiter syndrome, discrete subaortic stenosis, and ventricularseptalde ectswithprolapseo anaorticcusp.Although most o these lesions produce chronic aortic insu iciency, rarely acute severe aortic regurgitation can result, o ten with devastatingconsequences.(SeeSchwartz10th ed.,p.758.)
8. ricuspid stenosisis
A. Causedbysecondarydilation o thetricuspidannulus due to pulmonary hypertension and/or right heart ailure
B. Commonlytheresulto organicdisease,suchasrheumaticheart diseaseand endocarditis
C. Commonlycaused bymitralvalvedisease
D. Commonlyassociated with Mar an syndrome
9. Temost common cardiactumor is
A. Papillary broelastoma
B. Lymphangioma
C Myxoma
D. Metastatictumor
10. Temost common primarycardiactumor in children is
A Fibroelastomas
B. Hemangiomas
C. Lipomas
D Rhabdomyomas
Answer:B
Acquired tricuspid valve ( V) disease can be classi ied as eitherorganicor unctional,anda ectsapproximately0.8%o the general population. ricuspid stenosis is almost always a result o organic disease,namelyrheumatic heart disease and endocarditis.In thecaseo rheumaticdisease,tricuspidstenosiswith or without associated insu iciencyisinvariablyassociated with mitralvalvedisease.Other lesscommon causeso obstruction toright atrialemptyingincludecongenitaltricuspid atresia, right atrial tumors, and endomyocardial ibrosis. (SeeSchwartz10th ed.,pp.762–763.)
Answer:C
Cardiac myxomas are the most common cardiac tumor and are characterized by several distinguishing eatures. About 75% o the time, they arise rom the interatrial septum near the ossa ovalis in the le t atrium. Most others will develop in the right atrium,but,less commonly, theycan arise rom valvular sur aces and the walls o other cardiac chambers. Macroscopically,thesetumorsarepedunculated with a gelatinous consistency,and thesur acemaybesmooth (65%),villous,or riable.Sizevariesgreatlywiththesetumorsandranges rom 1 to 15 cm in diameter. Internally, myxomas are heterogeneous and o ten contain hemorrhage, cysts, necrosis, or calci ication.Histologically,thesetumorscontain cellsthat arise rom amultipotent mesenchymeand arecontained within amucopolysaccharidestroma.(SeeSchwartz10th ed.,p.775.)
Answer:D
In children, rhabdomyomas are the most common primary cardiac tumor, whereas ibromas are the most commonly resected cardiactumor.Rhabdomyomasaremyocardialhamartomas that are o ten multicentric in the ventricles. About 50%o casesareassociated with tuberoussclerosis,and while resection isoccasionallynecessary,most disappear spontaneously. Fibromas are congenital lesions that one-third o the timeare ound in children younger than 1year. hesetumors, conversely, are ordinarily solitary lesions ound in the inner interventricular septum,and theymaypresent with heart ailure, cyanosis, arrhythmias, syncopal episodes, chest pain, or sudden cardiacdeath.(SeeSchwartz10th ed.,p.776.)
145
11. Patientsundergoingmechanicalmitralvalvereplacement
A. Havea target internationalnormalized ratio (INR) o 4to5timesnormal
B Haveincreased le atrialsize
C HaveAF
D. Areat lower risk or thromboembolism
12. An absolutecontraindication to acoronaryarterybypass operation is
A. Acutecoronaryarteryinsufciencywith persistent or progressiveanginadespiteoptimalmedicaltherapy.
B. Acute subendocardial in arction with multivessel coronaryarterydisease.
C Cardiogenicshocka er myocardialin arction
D. Chronic CHF and ischemic cardiomyopathywith no signso angina.
13. Approximately50%o benign cardiactumorsare
A. Figromas
B. eratomas
C. Lipomas
D. Myxomas
14. Temajorityo cardiacmyxomasarise rom
A. Teright atrium
B Valvular sur aces
C Tele atrium
D. Teright ventricle(RV)
Answer:B
Although mechanical valves necessitate systemic anticoagulation, care ul monitoring o the international normalized ratio (INR) reduces the risk o thromboembolic events and hemorrhagic complications, and improves overall survival. Patients undergoing mechanical AVR generally have a target INR o 2 to 3 times normal. Patients undergoing mechanical mitral valve replacement requently have increased le t atrial size, concomitant AF, and are at higher risk or thromboembolism than thoseundergoingmechanicalAOR,and arethus recommended to have a target INR 2.5 to 3.5 times normal. When managed appropriately, the yearly thromboembolic and bleedingrisksin thesepatientsare1to 2%,and 0.5to 2%, respectively.(SeeSchwartz10th ed.,p.749.)
Answer:D
Patients with ischemic cardiomyopathy are a heterogeneous group, and, as with anysurgery, appropriate patient selection is central to success. In one retrospective studyo 96 patients with ischemiccardiomyopathy(ejection raction [EF] ≤25%), age, and poor distal vessel quality were predictors o poor outcomes. Mortality in patients with poor vessel quality was 100%, compared with 90% when vessel quality was air and 10%when it was good. here ore, poor vessel quality should beconsidered acontraindication to surgicalrevascularization even in thepresenceo angina.(SeeSchwartz10th ed.,p.765.)
Answer:D
Cardiac neoplasms are rare, with an incidence ranging rom 0.001 to 0.3% in autopsy studies and a 0.15% incidence in major echocardiographic series. Benign cardiac tumors are most common and account or 75% o primary neoplasms. Approximately 50% o benign cardiac tumors are myxomas, with the remainder being papillary ibroelastomas, lipomas, rhabdomyomas, ibromas, hemangiomas, teratomas, lymphangiomas,and others,in order o decreasing requency.Most malignant primary cardiac tumors are sarcomas (angiosarcoma, rhabdomyosarcoma, ibrosarcoma, leiomyosarcoma, and liposarcoma), with a small incidence o malignant lymphomas. Metastatic cardiac tumors, while still in requent, havebeen reportedtooccur100- oldmoreo ten than primary lesions.(SeeSchwartz10th ed.,p.774.)
Answer:C
Cardiac myxomas are the most common cardiac tumor and are characterized by several distinguishing eatures. About 75% o the time, they arise rom the interatrial septum near the ossa ovalis in the le t atrium. Most others will develop in the right atrium,but,less commonly, theycan arise rom valvular sur aces and the walls o other cardiac chambers. Macroscopically,thesetumorsarepedunculated with a gelatinous consistency,and thesur acemaybesmooth (65%),villous,or riable.(SeeSchwartz10th ed.,p.775.)
146
15. Te ollowingisNO trueo rhabdomyomas.
A. Teyo en requireresection.
B. Teyareo en multicentricin theventricles.
C. Teyarethemost common primarycardiactumor in children.
D. Teyo en disappear spontaneously.
16. A patient presents with a historyo atigue and dyspnea. He is ound to have hepatomegaly, ascites, and an elevated jugular venous pulse.Heart sounds are normal,no murmursarepresent,and theheart iso normalsize.Te pulsepressureisdecreased bypalpation.Electrocardiography (ECG) is normal except or low voltage. Te most likelydiagnosisis
A. Right atrialmyxoma
B Vdisease
C Constrictivepericarditis
D. Primarypulmonaryarteryhypertension
Answer:A
In children, rhabdomyomas are the most common primary cardiac tumor, whereas ibromas are the most commonly resected cardiactumor.Rhabdomyomasaremyocardialhamartomas that are o ten multicentric in the ventricles. About 50%o casesareassociated with tuberoussclerosis,and while resection isoccasionallynecessary,most disappear spontaneously. Fibromas are congenital lesions that one-third o the timeare ound in children younger than 1year. hesetumors, conversely, are ordinarily solitary lesions ound in the inner interventricular septum,and theymaypresent with heart ailure, cyanosis, arrhythmias, syncopal episodes, chest pain, or sudden cardiacdeath.(SeeSchwartz10th ed.,p.776.)
Answer:C
Classic physical examination indings include jugular venous distention with Kussmaul sign, diminished cardiac apical impulses, peripheral edema, ascites, pulsatile liver, a pericardial knock, and, in advanced disease, signs o liver dys unction, such as jaundice or cachexia. he “pericardial knock” is an earlydiastolic sound that re lects a sudden impediment to ventricular illing,similar toan S3 but o higher pitch.
Several indings are characteristic on noninvasive and invasive testing. Central venoud pressure (CVP) is o ten elevated 15 to 20 mm Hg or higher. ECG commonlydemonstrates nonspeci ic low voltage QRS complexes and isolated repolarization abnormalities. Chest X-ray may demonstrate calci ication o the pericardium, which is highly suggestive o constrictive pericarditis in patients with heart ailure, but this is present in only 25% o cases. Cardiac C or MRI (cMRI) typicallydemonstrate increased pericardialthickness (>4 mm) and calci ication, dilation o the in erior vena cava, de ormed ventricular contours, and lattening or le tward shi to theventricular septum.Pericardialadhesionsmayalso beseen on tagged cineMRIstudies.
Asdiscussed,it ismost important todistinguish pericardial constriction rom restrictive cardiomyopathy, which is best done with either echocardiography or right heart catheterization. Findings avoring constriction on echocardiography includerespiratoryvariation o ventricular septalmotion and mitral in low velocity, preserved or increased mitral annulus earlydiastolic illingvelocity,and increased hepatic vein low reversal with expiration. Cardiac catheterization will show increased atrial pressures, equalization o end-diastolic pressure, and early ventricular diastolic illing with a subsequent plateau, called the square-root sign. Additional indings upon catheterization that would avor constriction include respiratory variation in ventricular illing and increased ventricular interdependence, mani est as a discordant change in the total areao theLVand RVsystolicpressurecurvewith respiration. (SeeSchwartz10th ed.,pp.773–774.)
147
17. Each o the ollowinge ectsisanticipated a er insertion o an intra-aorticballoon pump EXCEP
A. Preload decrease
B. Increased totalmyocardialoxygen consumption
C Improvement in cardiacindex
D. Coronaryblood owincrease
Answer:B
heintra-aorticballoon pump (IABP) isthemost commonly used device or mechanicalcirculatorysupport,and it maybe easilydeployed in the catheterization laboratory, in the operating room or at the bedside. he device is inserted percutaneously through the emoral artery into the thoracic aorta. It is synchronized so that the balloon is in lated during diastole and de lated during systole, resulting in augmentation o diastolic per usion o the coronary arteries and decreased a terload. ypically,thisimprovescardiacindexand decreases both preload and myocardial oxygen consumption. (See Schwartz10th ed.,p.768.)
18. Pericarditisisusuallytreated with
A. Ashortcourseo nonsteroidalanti-in ammatorydrugs (NSAIDs).
B. Useo steroidsor intravenousantibiotics.
C. Surgicalexploration and drainage.
D. Observation.
19. Temostcommoncauseo isolatedaorticinAIsufciency in patientsundergoingAVRis
A. Congenitalabnormalitieso theaorticvalve
B. Systemichypertension
C. Aorticroot disease
D. Calci cdegeneration
20. During cardiopulmonary bypass (CPB) anticoagulation, therangeo heparin neededtoincreasetheactivatedclottingtimetogreater than 450secondsis
A. 200to300
B 300to400
C. 400to500
D. 450to550
Answer:A
he pre erred treatment depends on the underlying cause o the pericarditis. he disease usually ollows a sel -limited and benign courseand can besuccess ullytreated with ashort course o nonsteroidal anti-in lammatory drugs (NSAIDs). Some patients may require judicious use o steroids or IV antibiotics. In cases o purulent pyogenic pericarditis, surgical exploration and drainage are occasionally necessary. Rarely, accumulation o luid in the pericardium may lead to tamponade, requiring prompt evacuation o the pericardial space. While pericardiocentesis will typicallysu ice, surgical drainagemayberequired or thick,viscous,or clotted luid or in patients with signi icant scarring rom previous surgeries. More commonly, surgical intervention is required to manage recurrent disease.(SeeSchwartzed.,p.773.)
Answer:C
hemostcommon causeo isolatedAIin patientsundergoing AVR is aortic root disease, and represents over 50% o such patientsin somestudies.Other common causeso AI include congenital abnormalities o the aortic valve, such as bicuspid aorticvalve,calci icdegeneration,rheumaticdisease,in ective endocarditis, systemic hypertension, myxomatous degeneration,dissection o theascendingaorta,and Mar an syndrome. Less common causes o AI include traumatic injuries to the aortic valve, ankylosing spondylitis, syphilitic aortitis, rheumatoid arthritis, osteogenesis imper ecta, giant cell aortitis, Ehlers-Danlos syndrome, Reiter syndrome, discrete subaortic stenosis,and ventricular septalde ectswith prolapseo an aortic cusp. Although most o these lesions produce chronic aortic insu iciency, rarely acute severe aortic regurgitation can result, o ten with devastating consequences. (See Schwartz 10th ed.,p.758.)
Answer:B
he basic cardiopulmonary bypass (CPB) circuit consists o the venous cannulae, a venous reservoir, pump, oxygenator, ilter, and the arterial cannula. Anticoagulation is required during CPB, and 300 to 400 units/kg o heparin is given to increasetheactivated clottingtime(AC ) to greater than 450 seconds. Once adequate anticoagulation is achieved, arterial cannulation is per ormed through a purse-string suture, or through asidegra t which issewn on tothenativeartery.(See Schwartz10th ed.,p.740.)
148
21. Eligibility criteria or mechanical support as destination therapyincludesallo the ollowingEXCEP
A. Medicallyre ractoryNYHAclassIII
B. Peakoxygen consumption <12mL/kg/min
C. Le ventricular ejection raction >25%
D. Age>65
22. Angiosarcomas
A. Are mucous in appearance and are typically ound in theposterior wallo thele atrium
B Areaggressiveand rapidlyinvadeadjacent structures
C. Arebulkytumorsthat usuallyoccur in children
D. Are encapsulated tumors that usually arise rom the epicardium
Answer:C
Current eligibility criteria or mechanical support as destination therapy include: (a) medically re ractory NYHA class III or IV heart ailure or at least 60 days, (b) peak oxygen consumption <12mL/kg/min or ailure to wean rom continuous IV inotropes, (c) le t ventricular ejection raction <25%, and (d) presence o a contraindication or heart transplantation (ie, age >65 years, irreversible pulmonary hypertension, chronic renal ailure, insulin-dependent diabetes with end organ damage, or other clinically signi icant comorbidities). Once a patient has a le t ventricular assist device inserted as destination therapy, close and intensive ollow-up by a multidisciplinary heart ailure team is required in order to optimize medical therapy, reduce device-related morbidity, and improvesurvival.(SeeSchwartz10th ed.,p.770.)
Answer:B
Primary cardiac malignancies are very rare, but when they occur they tend to have a right-sided predominance and requentlydemonstrateextracardiacextension and involvement. Malignant cardiac tumors include angiosarcoma, osteosarcoma, leiomyosarcoma, rhabdomyosarcoma, liposarcoma, and primary cardiac lymphomas. Angiosarcomas are aggressive, rapidly invading adjacent structures, and 47 to 89% o patients present with lung, liver, or brain metastases by the time o diagnosis. Leiomyosarcomas are sessile masses with a mucous appearance that are typically ound in the posterior wall o the le t atrium. Rhabdomyosarcomas are bulky (>10 cm in diameter) tumors that usually occur in children and do not have a predilection or any particular chamber. hey requentlyinvadenearbycardiacstructuresandaremulticentricin 60%o cases.(SeeSchwartz10th ed.,p.776.)
23. Age-related calci c AS causes some degree o AI in approximately A 55% B. 65%
75% D 85%
Answer:C
here are also many primary valvular diseases that cause AI, generally in association with AS. One such disorder is agerelated calci ic AS, which causes some degree o AI in up to 75%o patients. In ective endocarditis mayinvolve the aortic valveapparatusandcauseAIthroughdirectdestruction o the valve lea lets, per oration o a lea let, or ormation o vegetations that inter ere with proper coaptation o the valve cusps. Rheumatic disease causes ibrous in iltration o the valve cusps and subsequent retraction o the valve lea lets, inhibiting apposition o the cusps during diastole and producing a central regurgitant jet. Patients with large ventricular septal de ects or membranous subaortic stenosis may develop progressiveAI,owingtoaVenturie ectthatresultsin prolapseo theaorticvalvelea lets.(SeeSchwartz10th ed.,p.760.)
149
C
ThoracicAneurysmsand AorticDissection
1. Te type o thoracic aortic aneurysm characterized by an outpouchingo theaortais
A. Fusi orm
B. Saccular
C. Pseudoaneurysm
D. Noneo theabove
2. Which o the ollowingis the most common cause o thoracicaorticaneurysms?
A. Atherosclerosis
B. Mar an syndrome
C. akayasu arteritis
D. Nonspeci cmedialdegeneration
3. Te most common complication o extensive repair or distalaorticaneurysmsis
A. Spinalcord ischemia
B. Renal ailure
C. Pulmonarydys unction
D. Le recurrent laryngealnerveinjury
Answer:B
Aortic aneurysms can be either “true” or “ alse.” rue aneurysms can take two orms: usi orm and saccular. Fusi orm aneurysms are more common and can be described as symmetrical dilatations o the aorta. Saccular aneurysms are localized outpouchings o the aorta. False aneurysms, also called pseudoaneurysms, are leaks in the aortic wall that are contained bytheouter layer o theaorta and/or theperiaortic tissue;theyarecausedbydisruption o theaorticwallandlead blood to collect in pouches o ibrotic tissue. (See Schwartz 10th ed.,p.785.)
Answer:D
Nonspeci ic medial degeneration is the most common cause o thoracic aortic disease. Histologic indings o mild medial degeneration, including ragmentation o elastic ibers and loss o smooth muscle cells, are expected in the aging aorta. However, an advanced, accelerated orm o medial degeneration leads to progressive weakening o the aortic wall, aneurysm ormation, and eventual dissection, rupture, or both. he underlyingcauses o medialdegenerative disease remain unknown.(SeeSchwarz10th ed.,p.787.)
Answer:C
Although spinal cord ischemia and renal ailure receive the most attention, several other complications warrant consideration. he most common complication o extensive repairs is pulmonary dys unction. With aneurysms adjacent to the le t subclavian artery, the vagus and le t recurrent laryngeal nerves are o ten adherent to the aortic wall and thus are susceptibletoinjury.(SeeSchwartz10th ed.,p.802.)
151 22
CHAPTER
4. Mar an syndromeiscaused byan abnormalityin which o the ollowingproteins?
A. Elastin
B. Metalloproteinase
C. Collagen
D. Fibrillin
5. Te ollowing is NO true regarding anastomotic pseudoaneurysms
A. Can arise rom deterioration o aortic tissue due to in ection
B. Have increased in incidence with an in ux o cardiovascular surgery
C. Commonlyoccur in patientswith Mar an syndrome
D. Are associated with high incidence o morbidity and rupture
Answer:D
Mar an syndrome is an autosomal-dominant genetic disorder characterized by a speci ic connective tissue de ect that leadsto aneurysm ormation. hephenotypeo patientswith Mar an syndrome typically includes a tall stature, high palate, joint hypermobility, eye lens disorders, mitral valve prolapse, and aortic aneurysms. he aortic wall is weakened by ragmentation o elastic ibers and deposition o extensive amountso mucopolysaccharides(a processpreviouslycalled cystic medial degeneration or cystic medial necrosis). Patients with Mar an syndrome have a mutation in the ibrillin gene located on the long arm o chromosome 15. he traditionally held view is that abnormal ibrillin in the extracellular matrix decreases connective tissue strength in the aortic wall and produces abnormal elasticity, which predisposes the aorta to dilatation rom wall tension caused by le t ventricular ejection impulses. More recent evidence, however, shows that the abnormal ibrillin causes degeneration o the aortic wall matrix by increasing the activity o trans orming growth actor-beta ( GF-β). Between 75 and 85%o patients with Mar an syndrome have dilatation o the ascendingaorta and annuloaortic ectasia (dilatation o the aortic sinuses and annulus). Such aortic abnormalities are the most common causeo deathamongpatientswithMar an syndrome.Mar an syndrome also is requentlyassociated with aortic dissection. (SeeSchwartz10th ed.,p.787.)
Answer:B
Anastomotic pseudoaneurysms can be caused by technical problems or by deterioration o the native aortic tissue, gra t material, or suture. Commonly, they occur in patients with Mar an syndrome. issue deterioration usually is related to either progressivedegenerativediseaseor in ection.Improvementsin sutures,gra t materials,and surgicaltechniqueshave decreased the incidence o thoracic aortic pseudoaneurysms. Should thoracicaorticpseudoaneurysmsoccur,theytypically require expeditious surgical or other intervention because they are associated with a high incidence o morbidity and rupture.(SeeSchwartz10th ed.,p.788.)
6. Temost common causeo death in patientswith typeIV Ehlers-Danlossyndrome
is
A. Myocardialin arction
B. Aorticdissection
C. Ruptured visceralartery
D. Pulmonaryemboli
Answer:C
Ehlers-Danlos syndrome includes a spectrum o inherited connective tissue disorders o collagen synthesis. he subtypes represent di ering de ective steps o collagen production. Vascular type Ehlers-Danlos syndrome is characterized by an autosomal dominant de ect in type III collagen synthesis, which can have li e-threatening cardiovascular maniestations. Spontaneous arterial rupture, usually involving the mesenteric vessels, is the most common cause o death in these patients. horacic aortic aneurysms and dissections are less commonlyassociated with Ehlers-Danlos syndrome, but when theydo occur,theypose a particularlychallengingsurgicalproblem becauseo thereducedintegrityo theaortictissue. An Ehlers-Danlos variant o periventricular heterotopia associated with joint and skin hyperextensibility and aortic dilation has been described as being caused by mutations in the gene encoding ilamin A (FLNA), an actin-binding protein that links the smooth muscle cell contractile unit to the cellsur ace.(SeeSchwartz10th ed.,p.787.)
152
7. Te most common presenting symptom in patients with an ascendingthoracicaneurysm is
A. Anterior chest pain
B Posterior chest pain
C Aorticvalveinsufciency
D. Sudden death
8. Endoleaks
A ypeIandtypeIVgenerallyrequireearlyandaggressive intervention
B. Areuncommon
C Can occur duringtheinitialprocedureor over time
D Arebenign
Answer:A
Initially, aneurysmal expansion and impingement on adjacent structures causes mild,chronic pain. he most common symptom in patientswith ascendingaorticaneurysmsisanterior chestdiscom ort;thepain is requentlyprecordialin location but may radiate to the neck and jaw, mimicking angina. Aneurysms o the ascending aorta and transverse aortic arch can cause symptoms related to compression o the superior vena cava, the pulmonary artery, the airway, or the sternum. Rarely, these aneurysms erode into the superior vena cava or right atrium,causingacutehigh-output ailure.(SeeSchwartz 10th ed.,p.789.)
Answer:A
Another signi icant complication o descending thoracic aortic stent gra ting is endoleak. An endoleak occurs when thereisa persistent lowo blood (visibleon radiologicimaging) into the aneurysm sac, and it may occur during the initial procedure or develop over time. Although endoleaks are a relatively common complication, they are not benign, becausetheyleadtocontinualpressurization o thesac,which can cause expansion or even rupture. hese complications are categorized ( able 22-1) according to the site o the leak. Although all endoleaks may progress such that they can be considered li e-threatening,typeIand typeIIIendoleaksgenerally necessitate early and aggressive intervention. Recently published reporting guidelines aid standardized reporting. (SeeSchwartz10th ed., able22-4,p.806.)
TABLE22-1
TypeI
o and common treatment strategies orendoleak
• Incompletesealbetweenstentgratandaortaattheproximallanding site(TypeIa),thedistallandingsite(TypeIb),orbranchmodule, enestration,orplug(TypeIc)
• Earlyreinterventiontoimprovesealorconversiontoopensurgery TypeII
• Retrogradeperusionosacromexcludedcollateralarteries
• Surveillance;as-neededocclusionwithpercutaneousorother interventions
TypeIII
• Incompletesealbetweenoverlappingstentgratormodule(TypeIIIa), ortearingratabric(TypeIIIb)
• Earlyreinterventiontocovergaportearorconversiontoopensurgery TypeIV
• Perusionosacduetoporosityomaterial
• Surveillance;as-neededreinterventiontorelinestentgrat TypeV
• Expansionosacwithnoidentiablesource
• Surveillance;as-neededreinterventiontorelinestentgrat
153
Classifcation
9. Temost use ulimagingstudy or thoracicaneurysmsis
A Echocardiography
B. Computed tomography(C )scan
C. Magneticresonanceangiography
D. Aortography
10. A patient with Mar an syndrome who has undergone “aorticsurgery”most likelyhad
A. Aorticvalveannuloplasty(annular plication)
B. Aorticroot replacement (valveand ascendingaorta)
C. otal arch replacement with reattachment o the brachiocephalicbranches
D. Elephant trunkrepair o thoracicaneurysm
11. In thecaseo aorticdissection
A Choice o initial treatment is the dependent on dissection local.
B. Diagnosticdelaysarecommon.
C Teyarecategorized aschronica er 14days
D. Contrast-enhanced C isthediagnostico choice.
Answer:B
Computed tomographic (C ) scanning is widely available, provides visualization o the entire thoracic and abdominal aorta, and permits multiplanar and 3-dimensional aortic reconstructions. Consequently, C is the most common— and arguably the most use ul—imaging modality or evaluating thoracic aortic aneurysms. In addition to establishing the diagnosis, C provides in ormation about an aneurysm’s location, extent, anatomic anomalies, and relationship to major branch vessels.C isparticularlyuse ulin determining the absolute diameter o the aorta, especially in the presence o a laminated clot, and also detects aortic calci ication. (See Schwartz10th ed.,pp.790–791.)
Answer:B
Mechanical prostheses necessitate ollowing a li elong anticoagulation regimen. Separate replacement o the aortic valve and ascendingaorta isnot per ormed in patientswith Mar an syndrome, because progressive dilatation o the remaining sinus segment eventually leads to complications that necessitatereoperation. here ore,patientswith Mar an syndrome or thosewith annuloaorticectasiarequiresome orm o aortic root replacement.(SeeSchwartz10th ed.,p.793.)
Answer:B
Because o the variations in severity and the wide variety o potentialclinicalmani estations,the diagnosiso acute aortic dissectioncanbechallenging.141-143 Only3outo every100,000 patientswho present to an emergencydepartment with acute chest, back, or abdominal pain are eventuallydiagnosed with aorticdissection.Not surprisingly,diagnosticdelaysarecommon; delays beyond 24 hours a ter hospitalization occur in up to 39% o cases (Fig. 22-1.) (See Schwartz 10th ed., Figure22-20,p.810.)
154
Managementofacuteaorticdissection
Suspectedacute
Ascendingaortic dissection(StanfordAor DeBakeyIorII)?
Aorticdissection?
Ascendingaortic dissection(StanfordAor DeBakeyIorII)?
Complicateddescending aorticdissection (malperfusion,rupture)?
12. Mortalityrates oroperativerepairo an aorticarchaneurysm havebeen signi cantlyreduced intraoperativelyby
A. Deep hypothermiatoallowcirculatoryarrest
B. Innominateandle carotidarterycannulation topermit oxygenation o thebrain
C Right heart to le subclavian artery bypass to continuebrain per usion
D. Useo an intraaorticballoon pump to maintain distal circulation
Answer:A
Liketheoperationsthemselves,per usion strategiesused duringproximalaorticsurgerydependon theextent o therepair. Aneurysms that are isolated to the ascendingsegment can be replacedbyusingstandardcardiopulmonarybypassanddistal ascending aortic clamping. his provides constant per usion o the brain and other vital organs during the repair. Aneurysmsinvolvingthetransverseaorticarch,however,cannotbe clamped during the repair, which necessitates the temporary withdrawal o cardiopulmonarybypass support;this is called circulatory arrest. o protect the brain and other vital organs during the circulatory arrest period, hypothermia must be initiated be orepump lowisstopped.However,hypothermia is not without risk, and coagulopathy is associated with deep
155 Contrast-enhanced CTscan Transfertooperating room,intubation, diagnosticTEE
dissection
Emergencyoperation Emergencyoperation Transfertointensivecare forfurtherstabilization anddiagnosticwork-up Transfertointensivecareunit forbloodpressurecontrol, anti-impulsetherapy Emergencyendovascular (fenestration,stent)or openintervention Yes Yes Yes Yes Yes Yes No No No No No No Furtherdiagnostic work-up
stable? Anti-impulsetherapy (betablockers), bloodpressurecontrol
Secondarydiagnostic study(MRA,TEE,or aortography)
Hemodynamically
Aorticdissection?
FIG.22-1. Algorithm used to acilitate decisionsregarding treatment o acute aorticdissection.CT= computed tomography;MRA= magnetic resonance angiography;TEE= transesophagealechocardiography.
13. According to the Craw ord classi cation scheme, surgical repair o thoracoabdominal aortic aneurysms with repairs beginning near the le subclavian artery but extending distally into the in rarenal abdominal aorta, o en reachingtheaorticbi urcation isclassi ed as
A Extent I
B. Extent II
C Extent III
D Extent IV
14. reatment o descending aortic dissection by nonoperative,pharmacologicmanagement
A. Has lower morbidity and mortality rates than traditionalsurgicaltreatment.
B. Mostcommoncauseo deathduringnonoperativetreatmentisaorticruptureandend-organmalper usion.
C. AC scan obtained on day2or 3,compared with the initial scan, is sufcient to rule out signi cant aortic expansion.
D. Inadequate blood pressure control has been ound to beassociated with lateaneurysm ormation.
levels o hypothermia (below 20°C), which have been traditionally used in open arch repair. Recently, more moderate levels o hypothermia (o ten between 22°C and 24°C) have been introduced that appear to decrease risksassociated with deep hypothermia while still providing su icient brain protection.(SeeSchwartz10th ed.,p.797.)
Answer:B
Extent I thoracoabdominal aortic aneurysm repairs involve most o the descending thoracic aorta, usually beginning near thele t subclavian artery,and extend down to thesuprarenal abdominal aorta. Extent II repairs also begin near the le t subclavian artery but extend distally into the in rarenal abdominalaorta,and theyo ten reach the aortic bi urcation. Extent III repairs extend rom the lower descending thoracic aorta (below the sixth rib) and into the abdomen. Extent IV repairs begin at the diaphragmatic hiatus and o ten involve theentireabdominalaorta.(SeeSchwartz10th ed.,p.801.)
Answer:C
Nonoperative, pharmacologic management o acute descending aortic dissection results in lower morbidity and mortality rates than traditional surgical treatment does. he most common causes o death duringnonoperative treatment are aortic rupture and end-organ malper usion. here ore, patients are continuallyreassessed ornewcomplications.Atleasttwoserial C scans—usuallyobtained on day2or 3and on day8or 9o treatment—are compared with the initial scan to rule out signi icantaorticexpansion.Oncethepatient’sconditionhasbeen stabilized, pharmacologic management is gradually shi ted rom intravenous to oral medications. Oral therapy, which usually includes a beta antagonist, is initiated when systolic pressure is consistently between 100 and 110 mm Hg and the neurologic,renal,and cardiovascular systemsarestable.Many patients can be discharged a ter their blood pressure is well controlled with oral agents and a ter serial C scans con irm the absence o aortic expansion. Long-term pharmacologic therapyisimportant orpatientswithchronicaorticdissection. Beta blockers remain the drugs o choice. In a 20-year ollowupstudy,DeBakeyand colleagues ound that inadequateblood pressurecontrolwasassociated with lateaneurysm ormation. Aneurysms developed in only 17% o patients with “good” blood pressure control, compared with 45% o patients with “poor”control.(SeeSchwartz10thed.,p.815.)
15. Which o the ollowing is the most typical presenting symptom in apatient with an aorticdissection?
A “ earing”pain
B Paraplegia
C. Abdominalpain
D. Cold le arm
Answer:A
heonset o dissection o ten isassociated with severechest or back pain, classically described as tearing, that migrates distallyasthedissection progressesalongthelength o theaorta. helocation o thepain o ten indicateswhich aorticsegments are involved. Pain in the anterior chest suggests involvement o theascendingaorta,whereaspain in thebackandabdomen generallyindicates involvement o the descendingand thoracoabdominalaorta.(SeeSchwartz10th ed.,p.809.)
156
16. Delayed treatment or ascendingaorticdissection should beconsidered
A. In patients who present with severe acute stroke or mesentericischemia.
B In patients who are elderly and have substantial comorbidity.
C. In patientswhoarein stablecondition and maybenet rom trans er tospecialized centers.
D. In patients who have undergone a cardiac operation in thepast 3weeks.
Answer:D
Because o the risk o aortic rupture, acute ascending aortic dissection is usually considered an absolute indication or emergency surgical repair. However, speci ic patient groups may bene it rom nonoperative management or delayed operation. Delayed repair should be considered or patients who (a) present with severe acute stroke or mesenteric ischemia,(b) areelderlyand havesubstantialcomorbidity,(c) are in stable condition and may bene it rom trans er to specialized centers,or (d) haveundergonea cardiacoperation in the remote past.Regardingthe last group,it is important that the previous operation not be too recent; dissections that occur duringthe irst 3 weeks a ter cardiac surgerypose a high risk o ruptureand tamponade,and such dissectionswarrant early operation.(SeeSchwartz10th ed.,p.813.)
17. Apatient with asubclavian arterymalper usion asa complication o aorticdissection wouldmostlikelyexperience
A Paraplegia
B. Cold,pain ulextremity
C. Incontinence
D Shock
Answer:B
See able22-2.(SeeSchwartz10th ed., able22-5,p.811.)
AnatomicManifestation
SymptomsandSigns
Aorticvalveinsu ciency Dyspnea
Murmur
Pulmonaryrales
Shock
Coronarymalperusion
Chestpainwithcharacteristicso angina
Nausea/vomiting
Shock
Ischemicchangeson
electrocardiogram
Elevatedcardiacenzymes
Pericardialtamponade Dyspnea
Jugularvenousdistension
Pulsusparadoxus
Mufedcardiactones
Shock
Low-voltageelectrocardiogram
Subclavianorilioemoralartery malperusion
Carotidarterymalperusion
Cold,painulextremity
Extremitysensoryandmotordecits
Peripheralpulsedecit
Syncope
Focalneurologicdecit(transientor persistent)
Carotidpulsedecit
Coma
Spinalmalperusion
Mesentericmalperusion
Renalmalperusion
Paraplegia
Incontinence
Nausea/vomiting
Abdominalpain
Oliguriaoranuria
Hematuria
157
TABLE22-2 Anatomiccomplicationso aorticdissection and theirassociated symptomsand signs
A. <0.9
B. <0.6
C. >0.9
D. >0.6
2. Allo the ollowingarecorrect regardingabdominalaortic aneurysm (AAA)ruptureEXCEP
A. It is the 10th leading cause o death or men in the United States.
B. Risko ruptureislowwhen below5.5cm.
C. Overall mortality rate o AAA rupture is higher than 70%.
D. Men haveahigher risko rupturethan women.
CHAPTER
ArterialDisease
Answer:A
here is increasing interest in the use o the ankle-brachial index (ABI) to evaluate patients at risk or cardiovascular events. An ABI less than 0.9 correlates with increased risk o myocardialin arction and indicatessigni icant,although perhaps asymptomatic, underlying peripheral vascular disease. (SeeSchwartz10th ed.,p.829.)
Answer:D
Despite more than 50,000 patients undergoingelective repair o abdominalaorticaneurysm (AAA) each year in theUnited States, approximately 15,000 patients die annually as a result o ruptured aneurysm, making it the 10th leading cause o death or men. heruptureriskisquitelowbelow5.5cm and begins to rise exponentially therea ter. his size can serve as an appropriate threshold or recommending elective repair provided one’s surgical mortality is below 5%. For each size strata,however,women appear tobeat higher risk or rupture than men, and a lower threshold o 4.5 to 5.0 cm maybe reasonablein good-riskpatients.
Overall mortality o AAA rupture is 71 to 77%, which includes all out-o -hospital and in-hospital deaths, as compared with 2 to 6% or elective open surgical repair. Nearly hal o allpatientswith ruptured AAAwilldiebe orereaching thehospital.For theremainder,surgicalmortalityis45to50% and has not substantially changed in the past 30 years. (See Schwartz10th ed.,pp.850–851.)
Answer:A
A. Anterior compartment
B. Lateralcompartment
C. Deep posterior compartment
D. Super cialposterior compartment
he most commonly a ected compartment is the anterior compartment in theleg.Numbnessin the web space between the irst and second toes is diagnostic due to compression o the deep peroneal nerve. Compartment pressure is measured by inserting an arterial line into the compartment and recording the pressure. Although controversial, pressures greater than 20 mm Hgare an indication or asciotomy. (See Schwartz10th ed.,p.888.)
159 23
1. An ankle-brachialindex(ABI)that suggestsincreased risk o myocardialin arction would be
3. Te compartment most commonly a ected in a lower leg compartment syndromeisthe
4. Magnetic resonance angiography (MRA) is contraindicated in the ollowingpatient groupsEXCEP thosewith
A. Pacemakers
B. Intracerebralshunts
C. Cochlear implants
D. Metallicstents
Answer:D
Magnetic resonance angiography (MRA) has the advantage o not requiring iodinated contrast agents to provide vessel opaci ication (Fig. 23-1). Gadolinium is used as a contrast agent or MRA studies, and because it is generally not nephrotoxic, it can be used in patients with elevated creatinine. MRAiscontraindicatedin patientswith pacemakers,de ibrillators, spinal cord stimulators, intracerebral shunts, cochlear implants, and cranial clips. Patients with claustrophobia may require sedation to be able to complete the test. he presence o metallic stents causes arti acts and signal drop-out; however, these can be dealt with using alternations in image acquisition and processing. Nitinol stents produce minimal arti act.Comparedtoother modalities,MRAisrelativelyslow and expensive. However, due to its noninvasive nature and decreased nephrotoxicity,MRAisbeingused more requently or imaging vasculature in various anatomic distributions. (SeeSchwartz10th ed.,Figure23-5,pp.832–833.)
160
FIG.23-1. Magneticresonance angiogram (MRA)o aorticarch and carotid arteries.Thisstudycan provide a three-dimensional analysiso vascularstructure such asaorticarch branches,carotid and vertebralarteries.
5. Tepre erred procedure or treatment o typicalocclusive diseaseo theaortaand both iliacarteriesis
A. Endovascular stenting
B. Extra-anatomicbypass
C. Aortoiliacendarterectomy
D. Aortobi emoralbypass
6. Temost common causeo ischemicstrokeis
A. Carotid arterydisease
B. Idiopathic
C. Cardiogenicemboli
D. Lacunar
7. Tetreatment o acuteembolicmesentericischemiais
A. Observation
B. Anticoagulation
C. Trombolysis
D. Operativeembolectomy
Answer:D
In most cases, aortobi emoral bypass is per ormed because patients usually have disease in both iliac systems. Although one side may be more severely a ected than the other, progression does occur, and bilateral bypass does not complicate theprocedureor addtothephysiologicstresso theoperation. Aortobi emoral bypass reliably relieves symptoms, has excellent long-term patency (approximately 70–80% at 10 years), and can becompleted with atolerableperioperativemortality (2–3%).(SeeSchwartz10th ed.,p.876.)
Answer:C
Ischemic strokes are due to hypoper usion rom arterial occlusion or, less commonly, to decreased low resulting rom proximal arterial stenosis and poor collateral network. Common causes o ischemic strokes are cardiogenic emboli in 35%, carotid artery disease in 30%, lacunar in 10%, miscellaneous in 10%, and idiopathic in 15%. he term cerebrovascular accident is o ten used interchangeably to re er to an ischemicstroke.(SeeSchwartz10th ed.,p.838.)
Answer:D
heprimarygoalo surgicaltreatment in embolicmesenteric ischemia is to restore arterial per usion with removal o the embolus rom the vessel. he abdomen is explored through a midline incision, which o ten reveals variable degrees o intestinalischemia rom themid-jejunum totheascendingor transversecolon. hetransversecolon isli ted superiorly,and the small intestine is re lected toward the right upper quadrant. he superior mesenteric artery (SMA) is approached at the root o the small bowel mesentery, usually as it emerges rom beneath the pancreas to cross over the junction o the third and ourth portionso theduodenum.Alternatively,the SMA can be approached byincising the retroperitoneum lateral to the ourth portion o the duodenum, which is rotated medially to expose the SMA. Once the proximal SMA is identi ied and controlled with vascular clamps, a transverse arteriotomy is made to extract the embolus, using standard balloon embolectomy catheters. In the event the embolus has lodged more distally, exposure o the distal SMA may be obtained in theroot o thesmallbowelmesenterybyisolating individualjejunaland ilealbranchesto allowa morecomprehensive thromboembolectomy. Following the restoration o SMA low,an assessment o intestinalviabilitymust bemade, and nonviable bowel must be resected. Several methods have been described to evaluatetheviabilityo theintestine,which include intraoperative intravenous luorescein injection and inspection with a Wood’s lamp, and Doppler assessment o antimesenteric intestinal arterial pulsations. A second-look procedure should be considered in many patients and is perormed 24 to 48 hours ollowing embolectomy. he goal o the procedure is reassessment o the extent o bowel viability, which may not be obvious immediately ollowing the initial embolectomy. I nonviable intestine is evident in the secondlook procedure, additional bowel resections should be perormed at that time.(SeeSchwartz10th ed.,pp.863–864.)
161
8. Tecorrect classi cation or thedegree o stenosisin the internalcarotid arteryo a patient with a luminaldiameter o 69%is
A Mild
B Moderate
C. Severe
D Nostenosis
9. Tetreatment o nonocclusivemesentericischemiais
A. Observation
B. Catheter in usion o papaverine
C Stentingtoprevent urther spasm
D. Operativebypasso theSMA
Answer:B
Atherosclerotic plaque ormation is complex, beginning with intimal injury, platelet deposition, smooth muscle cell prolieration, and ibroplasia, and leading to subsequent luminal narrowing. With increasing degree o stenosis in the internal carotid artery, low becomes more turbulent, and the risk o atheroembolization escalates. he severityo stenosisiscommonly divided into three categories according to the luminal diameter reduction: mild (<50%), moderate (50–69%), and severe (70–99%). Severe carotid stenosis is a strong predictor or stroke.(SeeSchwartz10th ed.,p.838.)
Answer:B
he treatment o nonocclusive mesenteric ischemia is primarily pharmacologic with selective mesenteric arterial catheterization ollowed by in usion o vasodilatory agents, such as tolazoline or papaverine. Once the diagnosis is made on the mesenteric arteriography (Fig. 23-2), intra-arterial papaverine is given at a dose o 30 to 60 mg/h. his must be coupled with the cessation o other vasoconstricting agents. (SeeSchwartz10th ed.,Figure23-42,pp.863and 865.)
10. Hollenhorst plaqueis ound within the
A Internalcarotid artery
B Retinalvessels
C. Peripheralarteries
D Renalarteries
Answer:B
Patients who su er cerebrovascular accidents typically present with three categories o symptoms including ocular symptoms, sensory/motor de icit, and/or higher cortical dys unction. he common ocular symptoms associated with extracranial carotid artery occlusive disease include amaurosis ugax and presence o Hollenhorst plaques. Amaurosis ugax, commonly re erred to as transient monocular blindness, is a temporary loss o vision in one eye that patients typically describe as a window shutter coming down or grey sheddingo the vision. his partial blindness usuallylasts or
162
FIG.23-2. Mesentericarteriogram showing non-occlusive mesenteric ischemia asevidenced bydi use spasm o intestinal arcadeswith poor illing o intramuralvessels.
11. Te most accurate diagnostic test with the lowest morbidityin thediagnosiso renalarterystenosisis
A Angiography
B. Computed tomography(C )scan
C MRA
D Renalsystemicrenin index
12. Te risk o a recurrent ipsilateral stroke in patients with severecarotid stenosisisapproximately
A 20%
B 30%
C. 40%
D 50%
13. Which o the ollowing statements concerning carotid bodytumorsistrue?
A Over 50%arehereditary
B. Require resection o the underlying carotid artery with reconstruction or cure.
C Areassociated with catecholaminerelease
D Areusuallybenign
a ew minutes and then resolves. Most o these phenomena (>90%) are due to embolic occlusion o the main artery or the upper or lower divisions. Monocular blindness progressing over a 20-minute period suggests a migrainous etiology. Occasionally,thepatient willrecallno visualsymptomswhile the optician notes a yellowish plaque within the retinal vessels,which isalsoknown asHollenhorst plaque. heseplaques are requentlyderived rom cholesterolembolization rom the carotid bi urcation and warrant urther investigation. (See Schwartz10th ed.,p.839.)
Answer:C
MRA has the advantage o not requiring iodinated contrast agentstoprovidevesselopaci ication (seeFig.23-1).Gadolinium is used as a contrast agent or MRA studies, and because it is generallynot nephrotoxic, it can be used in patients with elevated creatinine.(SeeSchwartz10th ed.,p.832.)
Answer:C
Currently,moststrokeneurologistsprescribebothaspirin and clopidogrel or secondary stroke prevention in patients who have experienced a transient ischemic attack ( IA) or stroke. In patients with symptomatic carotid stenosis, the degree o stenosis appears to be the most important predictor in determining risk or an ipsilateral stroke. he risk o a recurrent ipsilateral stroke in patients with severe carotid stenosis approaches40%.(SeeSchwartz10th ed.,p.841.)
Answer:D he carotid body originates rom the third branchial arch and rom neuroectodermal-derived neural crest lineage. he normalcarotid bodyislocated in the adventitia or periadventitial tissue at the bi urcation o the common carotid artery (Fig. 23-3). he gland is innervated by the glossopharyngealnerve.Itsblood supplyisderivedpredominantly rom the external carotid artery but can also come rom the vertebral artery. Carotid body tumor is a rare lesion o the neuroendocrinesystem.Other glandso neuralcrest origin areseen in the neck, parapharyngeal spaces, mediastinum, retroperitoneum, and adrenal medulla. umors involving these structures have been re erred to as paraganglioma, glomus tumor, or chemodectoma. Approximately 5 to 7% o carotid body tumorsaremalignant.Although chronichypoxemiahasbeen invokedasastimulus or hyperplasiao carotidbody,approximately35%o carotid bodytumorsarehereditary. herisko malignancyisgreatest in youngpatientswith amilialtumors. (SeeSchwartz10th ed.,Figure23-27,p.849.)
163
A. Diet
B. Cigarettesmoking
C Diabetesmellitus
D. Age
A. Tebuttock
B Tequadriceps
C Tecal muscles
D. Temetatarsophalangealjoint
Answer:A
he presence o or progression to a greater than 80%stenosis correlated highlywith either thedevelopment o atotalocclusiono theinternalcarotidarteryornewsymptoms. hemajor risk actorsassociated with diseaseprogression werecigarette smoking,diabetesmellitus,and age. hisstudysupported the contention that it isprudent to ollowaconservativecoursein the management o asymptomatic patients presenting with a cervicalbruit.(SeeSchwartz10th ed.,p.842.)
Answer:D
Progression o the underlyingatherosclerotic process is more likely to occur in patients with diabetes, those who continue to smoke, and those who ail to modi y their atherosclerotic risk actors. In comparison, rest pain is constant, and usually occurs in the ore oot across the metatarsophalangeal joint. It is worse at night and requires placing the oot in a dependent position to improve symptoms. (See Schwartz 10th ed., p.890.)
16. Fibromuscular dysplasia(FMD)is
A. More common in men than women in the ourth or h decadeo li e.
B. Commonlypresent bilaterallyin thecarotid artery
C. Surgicalcorrection iso en indicated.
D. Usuallyinvolvesshort vesselswith manybranches.
Answer:B
Fibromuscular dysplasia (FMD) usually involves mediumsized arteries that are longand have ewbranches (Fig. 23-4). Women in the ourth or i th decade o li e are more commonlya ected than men.Hormonale ectson thevesselwall are thought to play a role in the pathogenesis o FMD. FMD o the carotid arteryis commonlybilateral, and in about 20% o patients, the vertebral artery is also involved. An intracranial saccular aneurysm o the carotid siphon or middle cerebral arterycan be identi ied in up to 50%o the patients with FMD. Four histologic types o FMD have been described in the literature. he most common type is medial ibroplasia, which maypresent as a ocalstenosisor multiple lesions with intervening aneurysmal outpouchings. he disease involves the media with the smooth muscle being replaced by ibrous connective tissue. Commonly, mural dilations and microaneurysms can be seen with this type o FMD. FMD should be suspected when an increased velocity is detected across a
164
15. Rest pain seen with occlusiveperipheralvascular disease in thelower extremitymost commonlyoccursin
14. Te ollowing are major risk actors associated with carotid stenosisdiseaseprogression EXCEP
A B
FIG.23-3A&B. A.Carotid body tumor(arrow)located adjacent to the carotid bulb.B.Following periadventitialdissection,the carotid bodytumorisremoved.
17. Te best initial treatment or a groin pseudoaneurysm
a er angiographyis
A Surgicalrepair
B. Ultrasound-guided compression
C. Ultrasound-guided injection o thrombin
D. Observation
stenotic segment without associated atherosclerotic changes on carotid duplex ultrasound. Antiplatelet medication is the generally accepted therapy or asymptomatic lesions. Endovascular treatment is recommended or patients with documented lateralizing symptoms. Surgical correction is rarely indicated.(SeeSchwartz10th ed.,Figure23-24,pp.847–848.)
Answer:C
Percutaneous catheter aspiration should be the initial treatment or cal vessel embolization, but, or larger emboli, such asthosethat lodgein thepro unda emorisor common emoral arteries, surgical embolectomy may be required because theembolicmaterialcontainsatheroscleroticplaque,which is not amenable to transcatheter aspiration or catheter-directed thrombolysis. he incidence o pseudoaneurysm ormation at the puncture site is 0.5%. he treatment o choice or pseudoaneurysms larger than 2 cm in diameter is percutaneous thrombin injection under ultrasound guidance. Arterial rupture may complicate the procedure in 0.3% o cases.
165
FIG.23-4. Acarotid ibromusculardysplasia with typical characteristicso multiple stenoseswith intervening aneurysmal outpouching dilatations.The disease involvesthe media with the smooth muscle being replaced by ibrousconnective tissue.
18. Teprimarycauseo renalarteryocclusivelesionsis
A. Fibromuscular dysplasia
B. Atherosclerosis
C Renalarteryaneurysm
D. akayasu arteritis
amponade o the ruptured artery with an occlusion balloon should beper ormed,and acovered stent should beplaced.In case o ailure, surgical treatment is required. (See Schwartz 10th ed.,p.880.)
Answer:B
Approximately 80% o all renal artery occlusive lesions are causedbyatherosclerosis,whichtypicallyinvolvesashortsegment o therenalarteryostia and representsspillover disease rom a severely atheromatous aorta (Fig. 23-5). Atherosclerotic lesions are bilateral in two-thirds o patients. Individuals with this disease commonly present during the sixth decadeo li e.Men area ected twiceas requentlyaswomen. Atherosclerotic lesions in other territories such as the coronary, mesenteric, cerebrovascular, and peripheral arterial circulation arecommon.When a unilaterallesion ispresent,the disease process equally a ects the right and le t renal arteries. he second most common cause o renal artery stenosis is FMD,which accounts or 20%o casesand ismost requently encountered in young,o ten multiparouswomen.FMDo the renal artery represents a heterogeneous group o lesions that can produce histopathologic changes in the intima, media, or adventitia. he most common variety consists o medial ibroplasia,in which thickened ibromuscular ridgesalternate with attenuated media producing the classic angiographic “string o beads” appearance. he cause o medial ibroplasia remains unclear. Most common theories involve a modi ication o arterial smooth muscle cells in response to estrogenic stimuli duringthereproductiveyears,unusualtraction orces on a ected vessels, and mural ischemia rom impairment o vasa vasorum blood low. Fibromuscular hyperplasia usually a ects the distal two thirds o the main renal artery, and theright renalarteryisa ected more requentlythan thele t. Other less common causes o renal artery stenosis include renal artery aneurysm (compressing the adjacent normal renal artery), arteriovenous mal ormations, neuro ibromatosis, renal artery dissections, renal artery trauma, akayasu arteritis, and renal arteriovenous istula. (See Schwartz 10th ed.,Figure23-43,pp.866–867.)
166
FIG.23-5. Occlusive disease o the renalarterytypicallyinvolves the renalostium (arrow),asa spilloverplaque extension rom aortic atherosclerosis.
19. Carotid coiling
A Isassociated with losso elasticity
B. Ismorecommon in men than women
C. Aremost o en acquired in both children and adults
D Aremost likelyduetoembolicepisodes
Answer:A
A carotid coil consists o an excessive elongation o the internalcarotid arteryproducingtortuosityo thevessel(Fig.23-6). Embryologically, the carotid artery is derived rom the third aortic arch and dorsal aortic root and is uncoiled as the heart and great vessels descend into the mediastinum. In children, carotid coils appear to be congenital in origin. In contrast, elongation and kinking o the carotid artery in adults are associated with the loss o elasticity and an abrupt angulation o the vessel. Kinking is more common in women than men. Cerebral ischemic symptoms caused by kinks o the carotid arteryaresimilar tothose rom atheroscleroticcarotid lesionsbut aremorelikelyduetocerebralhypoper usion than embolic episodes. Classically, sudden head rotation, lexion, or extension can accentuate the kink and provoke ischemic symptoms.Mostcarotidkinksandcoilsare oundincidentally on carotid duplex scan. However, interpretation o the Doppler requencyshi ts and spectral analysis in tortuous carotid arteries can be di icult because o the uncertain angle o insonation. Cerebral angiography, with multiple views taken in neck lexion,extension,and rotation,isuse ulin thedetermination o the clinical signi icance o kinks and coils. (See Schwartz10th ed.,Figure23-23,p.847.)
167
FIG.23-6. Excessive elongation o the carotid arterycan result in carotid kinking (arrow),which can compromise cerebralblood low and lead to cerebralischemia.
20. Complicationso endovascular treatment or mesenteric ischemia
A. O en becomeli ethreatening.
B Can include distal embolization resulting in acute intestinalischemia.
C. Can includeaccess-sitethrombosis.
D. Dissectionsnever occur.
21. Te most common location or the development o atheroscleroticdiseaseis
A Terenalartery
B. Tecoronaryarteries.
C. Teabdominalaorta.
D. Tearteriesin thecircleo Willis.
22. Angiographindications or renalarteryrevascularization includeallo the ollowingEXCEP
A. Documented renalarterystenosis
B FMDlesion
C. A ected/una ected kidneyrenin ration >1.5to1
D. Pressuregradient >10mm Hg
Answer:C
Complicationsarenot common and rarelybecomeli ethreatening. heseincludeaccess-sitethrombosis,hematomas,and in ection. Dissection can occur duringpercutaneous transluminal angioplasty (P A) and is managed with placement o a stent. Balloon-mounted stents are pre erred over the selexpanding ones because o the higher radial orce and the more precise placement. Distal embolization has also been reported, but it never resulted in acute intestinal ischemia, likelyduetotherich networko collateralsalreadydeveloped. (SeeSchwartz10th ed.,p.865.)
Answer:A
Obstructive lesions o the renalarterycan produce hypertension, resulting in a condition known as renovascular hypertension, which is the most common orm o hypertension amenable to therapeutic intervention, and a ects 5 to 10% o allhypertensive patients in the United States.Patients with renovascular hypertension are at an increased risk or irreversible end-organ dys unction, including permanent kidney damage, i inadequate pharmacologic therapies are used to control the blood pressure. he majority o patients with renalarteryobstructivediseasehavevascular lesionso either atherosclerotic disease or ibrodysplasia involving the renal arteries. he proximal portion o the renal artery represents the most common location or the development o atherosclerotic disease. It is well established that renal artery intervention, either bysurgical or endovascular revascularization, provides an e ective treatment or controlling renovascular hypertension as well as preserving renal unction. he decision or intervention iscomplexand needsto consider a varietyo anatomic, physiologic, and clinical eatures, unique or theindividualpatient.(SeeSchwartz10th ed.,p.866.)
Answer:D
See able23-1.(SeeSchwartz10th ed., able23-12,p.869.)
TABLE23-1 Indications orrenalarteryrevascularization
AngiographyCriteria
• Documentedrenalarterystenosis(>70%diameterreduction)
• Fibromusculardysplasialesion
• Pressuregradient>20mmHg
• Aected/unaectedkidneyreninratio>1.5to1
ClinicalCriteria
• Reractoryorrapidlyprogressivehypertension
• Hypertensionassociatedwith ashpulmonaryedemawithout coronaryarterydisease
• Rapidlyprogressivedeteriorationinrenalunction
• Intolerancetoantihypertensivemedications
• Chronicrenalinsu ciencyrelatedtobilateralrenalarteryocclusive diseaseorstenosistoasolitaryunctioningkidney
• Dialysis-dependentrenalailureinapatientwithrenalarterystenosis butwithoutanotherdefnitecauseoendstagerenaldisease
• Recurrentcongestiveheartailureor ashpulmonaryedemanot attributabletoactivecoronaryischemia
168
23. Aortoiliac disease represented by di use aortoiliac diseaseabovetheiliacarteryisclassi ed as
A. ype1
B ypeII
C ypeIII
D. ypeIV
24. Tetreatment o choice or typeBiliaclesionsis
A. Observation
B Endovascular therapy
C. Surgery
D. Intravenous(IV)antibiotics
Answer:B
SeeFigure23-7.(SeeSchwartz10th ed.,Figure23-50,p.873.)
FIG.23-7. Aortoiliac disease can be classi ied into three types. Type Irepresents ocaldisease a ecting the distalaorta and proximalcommon iliacartery.Type IIrepresentsdi use aortoiliac disease above the inguinalligament.Type IIIrepresentsmultisegment occlusive diseasesinvolving aortoiliac and in ra-inguinal arterialvessels.
Answer:B
hemost commonlyused classi ication system o iliaclesions hasbeen set orth bythe ransAtlanticInter-SocietyConsensus( ASC)groupwithrecommendedtreatmentoptions. his lesion classi ication categorizes the extent o atherosclerosis and hassuggested atherapeuticapproach based on thisclassiication ( able 23-2 and Fig.23-8).Accordingto this consensusdocument,endovasculartherapyisthetreatmento choice or type A lesions, and surgery is the treatment o choice or type D lesions.Endovascular treatment is the pre erred treatment or typeBlesions,and surgeryisthepre erred treatment or good-risk patients with type C lesions. In comparison to the 2000 ASC document, the commission has not only madeallowances or treatment o moreextensivelesions,but also takesinto account thecontinuingevolution o endovascular technology and the skills o individual interventionalists when stating the patient’s comorbidities, ully in ormed patient pre erence, and the local operator’s long-term success rates must be considered when making treatment decisions or type Band type C lesions. (See Schwartz 10th ed., Figure23-55and able23-14,p.875.)
169
TypeI TypeII TypeIII
25. Carotid bi urcation occlusive disease resulting in stroke isusuallycaused by
A Atheroemboli
B. Trombosis
C Rupture
D Dissection
TABLE23-2
TypeAlesions
TASCclassi cation o aortoiliacocclusive lesions
• UnilateralorbilateralstenosesoCIA
• Unilateralorbilateralsingleshort(≤3cm)stenosisoEIA
TypeBlesions
• Short(≤3cm)stenosisoinrarenalaorta
• UnilateralCIAocclusion
• Singleormultiplestenosistotaling3–10cminvolvingtheEIAnot extendingintotheCFA
• UnilateralEIAocclusionnotinvolvingtheoriginsointernaliliacartery orCFA
TypeClesions
• BilateralCIAocclusions
• BilateralEIAstenoses3–10cmlongnotextendingintotheCFA
• UnilateralEIAstenosisextendingintotheCFA
• UnilateralEIAocclusionthatinvolvestheoriginsointernaliliacartery and/orCFA
• HeavilycalcifedunilateralEIAocclusionwithorwithoutinvolvement ooriginsointernaliliacarteryand/orCFA
TypeDlesions
• Inrarenalaortoiliacocclusion
• Diusediseaseinvolvingtheaortaandbothiliacarteriesrequiring treatment
• DiusemultiplestenosesinvolvingtheunilateralCIA,EIA,andCFA
• UnilateralocclusionsobothCIAandEIA
• BilateralocclusionsoEIA
• IliacstenosesinpatientswithAAArequiringtreatmentandnot amenabletoendogratplacementorotherlesionsrequiringopen aorticoriliacsurgery
AAA= abdominalaorticaneurysm;CFA= common emoralartery;CIA= common iliac artery;EIA= externaliliacartery.
TypeAlesions
TypeClesions
TypeBlesions
Answer:A
TypeDlesions
Stroke due to carotid bi urcation occlusive disease is usually caused byatheroemboli(Fig.23-9). hecarotid bi urcation is an area o low lowvelocityand lowshear stress.Astheblood circulates through the carotid bi urcation, there is separation o lowinto the low-resistance internal carotid arteryand the high-resistanceexternalcarotid artery.(SeeSchwartz10th ed., Figure23-13,p.838.)
170
FIG.23-8. Schematicdepiction o TASCclassiication o aortoiliac occlusive lesions.
26. Crescendo IAs re ers to a syndrome comprising repeated IAswithin
A. A short period o time that is characterized by completeneurologicrecoveryin between.
B. A lengthy time period that is characterized by completeneurologicrecoveryin between.
C. A short period o time that is characterized bya partialneurologicrecoveryin between.
D. A lengthy time period that is characterized by a partialneurologicrecoveryin between.
Answer:A IAis a ocalloss o neurologic unction,lasting or less than 24 hours. Crescendo IAs re er to a syndrome comprising repeated IAs within a short period o time that is characterized by complete neurologic recovery in between. At a minimum, the term should probably be reserved or those with either daily events or multiple resolving attacks within 24hours.Hemodynamic IAsrepresent ocalcerebralevents that are aggravated by exercise or hemodynamic stress and typically occur a ter short bursts o physical activity, postprandially, or a ter getting out o a hot bath. It is implied that thesearedueto severeextracranialdiseaseand poor intracranial collateral recruitment. Reversible ischemic neurologic de icits re er to ischemic ocal neurologic symptoms lasting longer than 24 hours but resolving within 3 weeks. When a neurologic de icit lasts longer than 3 weeks, it is considered a completed stroke. Stroke in evolution re ers to progressive worsening o the neurologic de icit, either linearly over a 24-hour period or interspersed with transient periods o stabilization and/or partial clinical improvement. (See Schwartz 10th ed.,pp.838–839.)
171
Internal carotid artery Emboli
Ulcer Plaque Common carotid artery
External carotid artery
Superior thyroid artery
FIG.23-9. Stroke due to carotid biurcation occlusive disease is usuallycaused byatheroemboliarising rom the internalcarotid arterywhich providesthe majorityo blood lowto the cerebral hemisphere.With increasing degree o stenosisin the carotid artery, lowbecomesmore turbulent,and the risko atheroembolization escalates.
27. Late postoperative complications o aortobi emoral bypassgra ingincludeallo the ollowingEXCEP
A. Gra in ection
B. Anastomotic stula
C Paraplegia
D. Aortourinary stula
28. Te best diagnostic imaging modality or identi ying lower extremityocclusivediseaseis
A. MRA
B C angiography
C. Ultrasound
D. Contrast angiography
29.
A. Patientswith tissuelossareclassi ed asstageII.
B. Patientswith rest pain areclassi ed asstageIII.
C Asymptomaticpatientsareclassi ed asstageI
D. Patientswith claudication areclassi ed asstageIV.
Answer:C
See able23-3.(SeeSchwartz10th ed., able23-15,p.878.)
TABLE23-3 Perioperative complicationso aortobi emoralbypassgra ting
MedicalComplications
• Perioperativemyocardialinarction
• Respiratoryailure
• Ischemia-inducedrenalailure
• Bleedingromintravenousheparinization
• Stroke
Procedure-RelatedComplications
Early
• Declampingshock
• Gratthrombosis
• Retroperitonealbleeding
• Groinhematoma
• Bowelischemia/inarction
• Peripheralembolization
• Erectiledysunction
• Lymphaticleak
• Chylousascites
• Paraplegia
Late
• Gratinection
• Anastomoticpseudoaneurysm
• Aortoentericfstula
• Aortourinaryfstula
• Gratthrombosis
Answer:D
Contrast angiography remains the gold standard imaging study.Usingcontrast angiography,interventionistscan locate and size the anatomic signi icant lesions and measure the pressure gradient across the lesion, as well as plan or potentialintervention.Angiographyis,however,semi-invasiveand should be con ined to patients or whom surgical or percutaneous intervention is contemplated. Patients with borderline renal unction may need to have alternate contrast agents, such as gadolinium or carbon dioxide, to avoid contrastinduced nephrotoxicity.(SeeSchwartz10th ed.,p.882.)
Answer:B
he Fontaine classi ication uses our stages: Fontaine I is the stagewhen patientsareasymptomatic;FontaineII iswhen they havemild(IIa)or severe(IIb)claudication;FontaineIIIiswhen theyhaveischemicrest pain;andFontaineIViswhen patients su er tissueloss,such asulceration or gangrene( able23-4). (SeeSchwartz10th ed., able23-17,pp.883–884.)
172
Accordingto theFontaineclassi cation system or lower extremityocclusivedisease
173
30. Temost common sourceo distalemboliis
A Teheart
B. Atheroscleroticlesions
C. Dilated cardiomyopathy
D Diseased valves
31. An absolutecontraindication tothrombolytictherapyis
A. Pregnancy
B Intracranialtumor
C. Intracranialtraumawithin thepast 3months
D. Cardiopulmonary resuscitation within the past 10 days
Answer:A heheart isthemost common sourceo distalemboli,which accounts or more than 90% o peripheral arterial embolic events. Atrial ibrillation is the most common source. Sudden cardioversion results in the dilated noncontractile atrial appendage regaining contractile activity, which can dislodge thecontained thrombus.Other cardiacsourcesincludemural thrombus overlying a myocardial in arction or thrombus orming within a dilated le t ventricular aneurysm. Mural thrombi can also develop within a ventricle dilated by cardiomyopathy. Emboli that arise rom a ventricular aneurysm or rom a dilated cardiomyopathy can be very large and can lodge at the aortic bi urcation (saddle embolus), thus rendering both legs ischemic. Diseased valves are another source o distal embolization. Historically, this occurred as a result o rheumatic heart disease. Currently, subacute endocarditis andacutebacterialendocarditisarethemorecommon causes. In ected emboli can seed the recipient vessel wall, creating mycoticaneurysms.(SeeSchwartz10th ed.,p.885.)
Answer:C
Patients with small-vessel occlusion are poor candidates or surgerybecausetheylackdistaltargetvesselstouse orbypass. hesepatientsshouldbeo eredatrialo thrombolysis,unless they have contraindications to thrombolysis or their ischemia is so severe that the time needed to achieve adequate lysis is considered too long. he major contraindications o thrombolysis are recent stroke, intracranial primary malignancy,brain metastases,or intracranialsurgicalintervention. Relative contraindications or per ormance o thrombolysis include renal insu iciency, allergy to contrast material, cardiacthrombus,diabeticretinopathy,coagulopathy,and recent arterial puncture or surgery ( able 23-5). (See Schwartz 10th ed., able23-21,p.887.)
TABLE23-5
Contraindicationsto thrombolytictherapy
AbsoluteContraindications
Establishedcerebrovascularevents(includingtransientischemicattack) withinlast2months
Activebleedingdiathesis
Recent(<10days)gastrointestinalbleeding
Neurosurgery(intracranialorspinal)withinlast3months
Intracranialtraumawithinlast3months
Intracranialmalignancyormetastasis
RelativeMajorContraindications
Cardiopulmonaryresuscitationwithinlast10days
Majornonvascularsurgeryortraumawithinlast10days
Uncontrolledhypertension(>180mmHgsystolicor>110mmHg diastolic)
Punctureononcompressiblevessel
Intracranialtumor
Recenteyesurgery
MinorContraindications
Hepaticailure,particularlywithcoagulopathy
Bacterialendocarditis
Pregnancy
Diabetichemorrhagicretinopathy
174
32. Te term chronic limb ischemia (CLI) is reserved or patientswith objectivelyproven arterialocclusivedisease and symptomslasting or morethan
A 1week
B 2weeks
C. 3weeks
D. 4weeks
Grade Category
ClinicalDescription
Answer:B
he term chronic limb ischemia (CLI) is reserved or patients with objectively proven arterial occlusive disease and symptomslasting ormorethan2weeks.Symptomsincluderestpain andtissueloss,suchasulceration organgrene( able23-6). he diagnosis should be corroborated with noninvasive diagnostic tests, such as the ABI, toe pressures, and transcutaneous oxygen measurements. Ischemic rest pain most commonly occursbelowan anklepressureo 50mm Hgor atoepressure lessthan 30mm Hg.2Ulcersarenot alwayso an ischemicetiology ( able 23-7). (See Schwartz 10th ed., ables 23-24 and 23-25,pp.889–890.)
0 0 Asymptomatic—nohemodynamicallysignifcant occlusivedisease
ObjectiveCriteria
Normaltreadmillorreactivehyperemiatest
Abletocompletetreadmillexercisea;APaterexercise >50mmHgbutatleast20mmHglowerthanrestingvalue I 2 Moderateclaudication Betweencategories1and3
1 Mildclaudication
3 Severeclaudication
Cannotcompletestandardtreadmillexercisea andAPater exercise<50mmHg
RestingAP<40mmHg, atorbarelypulsatileankleor metatarsalPVR;TP<30mmHg IIIb 5
IIb 4 Ischemicrestpain
Minortissueloss—nonhealingulcer,ocal gangrenewithdiusepedalischemia
6 Majortissueloss extendingaboveTMlevel, unctionalootnolongersalvageable
aFive minutesat 2milesperhouron a 12%incline o treadmillexercise.
bGradesIIand III,categories4,5,and 6,are encompassed bythe term chroniccriticalischemia. AP= ankle pressure;PVR= pulse volume recording;TM= transmetatarsal;TP= toe pressure.
RestingAP<60mmHg,ankleormetatarsalPVRatorbarely pulsatile;TP<40mmHg
Sameascategory5
NeuropathicUlcer
IschemicUlcer
Painless Painul
Normalpulses
Regularmargins,typically punched-outappearance
Otenlocatedonplantarsuraceo oot
Presenceocalluses
Lossosensation,reexes,and vibration
Increasedinblood ow (arteriovenousshunting)
Dilatedveins
Absentpulses
Irregularmargin
Commonlylocatedontoes, glabrousmargins
Callusesabsentorinrequent
Variablesensoryfndings
Decreasedinblood ow
Collapsedveins
Dry,warmoot Coldoot
Bonydeormities
Nobonydeormities
Redorhyperemicinappearance Paleandcyanoticinappearance
175
TABLE23-7 Symptomsand signso neuropathiculcer versusischemiculcer
TABLE23-6 Clinicalcategorieso chroniclimb ischemia
33. Te percentage o patients with vein gra s that will develop intrinsic stenosis within the rst 18 months ollowingimplantation is
A 5%
B 10%
C. 15%
D. 20%
34. Te
ollowingaretrueo cryopreserved gra sEXCEP
A. Pronetoearlythrombosis.
B Lessexpensivethan prostheticgra s
C. Moreproneto ailurethan prostheticgra s.
D. Pronetoaneurysmaldegeneration.
35. When lower extremity occlusive disease extends to involvethepoplitealarteryor tibialvessels,theappropriate out ow vessels or per orming bypass in order o descendingpre erenceare
A. Below-knee popliteal artery, posterior tibial artery, anterior tibialartery,above-kneepoplitealartery
B. Above-knee popliteal artery, anterior tibial artery, posterior tibialartery,below-kneepoplitealartery
C. Anterior tibialartery,posterior tibialartery,peroneal artery,below-kneepoplitealartery,above-kneepoplitealartery
D. Above-knee popliteal artery, below-knee popliteal artery, posterior tibial artery, anterior tibial artery, and peronealartery
36. Giant cellarteritis
A. endstooccur in whitemen older than 50years
B. Haslowremission rates
C. Is associated with ischemic optic neuritis in the majorityo patients
D. Isdiagnosed bytemporalarterybiopsy
Answer:C
Fi teen percent o vein gra ts will develop intrinsic stenoses within the irst 18 months ollowing implantation. Consequently, patients with vein gra ts were entered into duplex surveillance protocols (scans every 3 months) to detect elevated (>300cm/s) or abnormallylow(<45cm/s) gra t velocities early. Stenoses greater than 50%, especially i associated with changes in ABI, should be repaired to prevent gra t thrombosis.Repair usuallyentailspatch angioplastyor shortsegment venous interposition, but P A/stenting is an option or short, ocal lesions. Gra ts with stenoses that are identiied and repaired prior to thrombosis have assisted primary patency identical to primary patency, whereas a thrombosed autogenous bypass has limited longevity resulting rom ischemicinjurytothevein wall.(SeeSchwartz10th ed.,p.898.)
Answer:B
Cryopreserved gra ts are usually cadaveric arteries or veins that have been subjected to rate controlled reezing with dimethyl sul oxide (DMSO) and other cryopreservants. Cryopreserved vein gra tsaremoreexpensivethan prosthetic gra ts and are more prone to ailure. he endothelial liningis lost as part o the reezingprocess, makingthese gra ts prone to early thrombosis. Cryopreserved gra ts are also prone to aneurysmal degeneration. Despite the act that these gra ts have not per ormed as well as prosthetic bypasses and autogenous vein bypasses in clinical practice, they can still play a role when revascularization is required ollowing removal o in ected prosthetic bypass gra ts, especially when the autogenousvein isunavailabletocreateanewbypassthrough clean tissueplanes.(SeeSchwartz10th ed.,p.899.)
Answer:D
When thediseaseextendstoinvolvethepoplitealarteryor the tibial vessels, the surgeon must select an appropriate out low vesseltoper ormabypass.Suitableout lowvesselsarede ined as uninterrupted low channels beyond the anastomosis into the oot. Listed in order o descending pre erence, they are as ollows: above-knee popliteal artery, below-knee popliteal artery, posterior tibial artery, anterior tibial artery, and peroneal artery. In patients with diabetes, it is requently the peroneal artery that is spared. Although it has no direct low into the oot,collateralization to theposterior tibialand anterior tibialarteriesmakesit an appropriateout lowvessel.(See Schwartz10th ed.,p.898.)
Answer:D
Giant cellarteritisisalsoknown astemporalarteritis,which is a systemic chronic in lammatory vascular disease with many characteristics similar to those o akayasu disease. he histologic and pathologic changes and laboratory indings are similar. Patients tend to be white women older than 50 years, with ahigh incidencein Scandinaviaand women o Northern European descent. Genetic actors may play a role in disease pathogenesis,with a human leukocyte antigen (HLA) variant having been identi ied. Di erences exist between akayasu and giant cell arteritis in terms o presentation, disease location, and therapeutic e icacy. he in lammatory process
176
A. Kawasakidisease
B. Giant cellarteritis
C Hypersensitivityangitis
D Buerger disease
38. Te ollowing is true regarding polyarteritis nodosa (PAN)EXCEP
A. Is predominantly treated with steroid and cytotoxic agent therapy.
B. Predominantlya ectswomen over men bya2:1ratio.
C Presenting symptoms include low-grade ever, malaise,and myalgias.
D. Maybesufcientlydiagnosed byskin biopsy.
typically involves the aorta and its extracranial branches, o which the super icial temporal artery is speci ically a ected. heclinicalsyndromebeginswith a prodromalphaseo constitutionalsymptoms,includingheadache, ever,malaise,and myalgias. hepatientsmaybeinitiallydiagnosedwithcoexistingpolymyalgia rheumatica;an HLA-related association may exist between thetwodiseases.Asaresult o vascular narrowing and end-organ ischemia, complications may occur such asvisualalterations,includingblindnessand muralweakness, resultingin acuteaorticdissection that maybedevastating.
Ischemic optic neuritis resulting in partial or complete blindness occurs in up to 40%o patients and is considered a medical emergency. Cerebral symptoms occur when the disease process extends to the carotid arteries. Jaw claudication and temporal artery tenderness may be experienced. Aortic lesionsareusuallyasymptomaticuntillater stagesand consist o thoracicaneurysmsand aorticdissections.
he diagnostic gold standard is a temporal artery biopsy, which will showthe classic histologic indings o multinucleated giant cells with a dense perivascular in lammatory in iltrate. reatmentregimensarecenteredon corticosteroids,and giant cell arteritis tends to rapidly respond. Remission rates arehigh,and treatment tendsto haveabene icialand preventative e ect on the development o subsequent vascular complications.(SeeSchwartz10th ed.,p.901.)
Answer:C
See able23-8.(SeeSchwartz10th ed., able23-28,p.903.)
Large-VesselVasculitis
Takayasu’sarteritis
Giantcellarteritis
Behçet’sdisease
Medium-VesselVasculitis
Polyarteritisnodosa
Kawasaki’sdisease
Buerger’sdisease
Small-VesselVasculitis
Hypersensitivityangiitis
Answer:B
Polyarteritis nodosa (PAN) is another systemic in lammatory disease process, which is characterized by a necrotizing in lammation o medium-sized or small arteries that spares the smallest blood vessels (ie, arterioles and capillaries). his diseasepredominantlya ectsmen over women bya2:1ratio. PAN develops subacutely, with constitutional symptoms that last or weeksto months.Intermittent,low-grade evers,malaise,weight loss,and myalgiasarecommon presentingsymptoms. As medium-sized vessels lie within the deep dermis, cutaneous mani estations occur in the orm o livedo reticularis,nodules,ulcerations,and digitalischemia.Skin biopsies o theselesionsmaybesu icient or diagnosis.In lammation may be seen histologically, with pleomorphic cellular in iltrates and segmental transmural necrosis leading to aneurysm ormation. Neuritis rom nerve in arction occurs in 60%o patients, and gastrointestinal complications occur in
177
37. Tedisorder most likelyinvolved in systemicsmallvessel vasculitiswould be
TABLE23-8 Classi cation o vasculitisbased on vessel involvement
up to 50%. Additionally, renal involvement is ound in 40% and mani ests as microaneurysms within the kidney or segmental in arctions. Cardiac disease is a rare inding except at autopsy, where thickened, diseased coronary arteries may be seen,aswellaspatchymyocardialnecrosis.Patientsmaysuccumb to renal ailure, intestinal hemorrhage, or per oration. End-organ ischemia rom vascular occlusion or aneurysm rupture can be disastrous complications with high mortality rates. hemainstayo treatment issteroid and cytotoxicagent therapy.Upto50%o patientswithactivePANwillexperience remission with high dosing.(SeeSchwartz10th ed.,p.903.)
178
Venousand LymphaticDisease
1. All o the ollowing regarding venous anatomy is true EXCEP
A. Veins are thin-walled, collapsible, and highly distensible to a diameter several times greater than that in the supineposition.
B. Tevenousintimaiscomposed o anonthrombogenic endothelium that produces endothelium-derived relaxing actorssuch asnitricoxideand prostacyclin
C. Venousvalvesclose in response to caudal-to-cephalad blood owat avelocityo at least 30cm/s2 .
D. Tein erior venacava(IVC),common iliacveins,portalvenoussystem,and cranialsinusesarevalveless
Answer:C
Veins are thin-walled, highly distensible, and collapsible. heir structurespeci icallysupportstheprimary unctionso veinsto transport blood toward theheart and serveasareservoir toprevent intravascular volumeoverload.
he venous intima is composed o a nonthrombogenic endothelium with an underlying basement membrane and an elastic lamina. he endothelium produces endotheliumderived relaxing actors such as nitric oxide and prostacyclin, which help maintain a nonthrombogenic sur ace through inhibition o platelet aggregation and promotion o platelet disaggregation.
Circum erential rings o elastic tissue and smooth muscle located in themediao thevein allow or changesin vein caliber with minimal changes in venous pressure. he adventitia is most prominent in large veins and consists o collagen, elastic ibers, and ibroblasts. When a vein is maximally distended, its diameter maybe several times greater than that in thesupineposition.
In the axial veins, unidirectional blood low is achieved with multiple venous valves. he in erior vena cava (IVC), common iliacveins,portalvenoussystem,and cranialsinuses arevalveless.In theaxialveins,valvesaremorenumerousdistallyin theextremitiesthan proximally.Each valveconsistso two thin cusps o a ine connective tissue skeleton covered by endothelium. Venous valves close in response to cephaladto-caudal blood low at a velocity o at least 30 cm/s2. (See Schwartz10th ed.,p.915.)
2. Chronicvenousinsu ciency(CVI) ischaracterized byall o the ollowingEXCEP
A. Incompetenceo venousvalves,venousobstruction
B. Preserved microcirculatory and cutaneous lymphatic anatomy
C. Eczemaand dermatitis
D. Lipodermatosclerosis
Answer:B
Chronicvenousinsu iciency(CVI)mayleadtocharacteristic changes in the skin and subcutaneous tissues in the a ected limb.CVIresults rom incompetenceo venousvalves,venous obstruction, or both. Most CVI involves venous re lux, and severe CVI o ten re lects a combination o re lux and venous obstruction. It is important to remember that although CVI originates with abnormalities o the veins, the target organ o CVI is the skin, and the underlying physiologic and biochemical mechanisms leading to the cutaneous abnormalities associated with CVI are poorlyunderstood. A typical leg a ected by CVI will be edematous, with edema increasing
179 24
CHAPTER
over the course o the day. he legmayalso be indurated and pigmented with eczema and dermatitis. hese changes are associated with excessiveproteinaceouscapillaryexudateand deposition o a pericapillary ibrin cu that may limit nutritional exchange. In addition, an increase in white blood cell (WBC) trapping within the skin microcirculation in CVI patients may lead to microvascular congestion and thrombosis. Subsequently, WBCs maymigrate into the interstitium and release necrotizing lysosomal enzymes, potentially leadingtotissuedestruction and eventualulceration.
Fibrosis can eventually develop rom impaired nutrition, chronic in lammation, and at necrosis (lipodermatosclerosis). Hemosiderin deposition due to the extravasation o red cells and subsequent lysis in the skin contributes to the characteristic pigmentation o chronic venous disease (Fig. 24-1). Ulceration can develop with longstanding venous hypertension and is associated with alterations in microcirculatory and cutaneous lymphatic anatomy and unction. he most common location o venousulceration isapproximately3cm proximal to the medial malleolus (Fig. 24-2). (See Schwartz 10th ed.,Figures24-2and 24-3,p.917.)
180
FIG.24-1. Characteristichyperpigmentation o chronicvenous insu iciency.
3. Venous thromboembolism (V E) is associated with all o the ollowingEXCEP
A. Increased morbidityand mortality
B. Pulmonaryhypertension
C. Postthromboticsyndrome
D. Nochangein uturerisko venousthromboembolism
Answer:D
heincidenceo venousthromboembolism (V E)isapproximately 100 per 100,000 people per year in the general population, with 20% o the diagnoses made within 3 months o a surgical procedure. O the symptomatic patients, one-third will present with pulmonary embolism (PE) and two-thirds with deep vein thrombosis (DV ). he estimated number o caseso V Emaywellbe over 600,000per year in the United States, making it a major U.S. health problem. Furthermore, death occurs in 6% o DV and 12% o PE cases within 1 month o diagnosis.NotonlydoesV Eposeaveritablethreat to li e, but it also places patients at higher risk or recurrence and post-V Esequelae such as pulmonaryhypertension and postthromboticsyndrome,with 4%and up to 30%incidence, respectively.(SeeSchwartz10th ed.,p.918.)
4. Virchowtriad ischaracterized by:
A. Stasis
B. Endothelialdamage
C. Hypercoagulability
D. Allo theabove
Answer:D
hree conditions, irst described by Rudol Virchow in 1862, contributeto V E ormation:stasiso blood low,endothelial damage,and hypercoagulability.O theserisk actors,relative hypercoagulability appears most important in most cases o spontaneousV E, or so-called idiopathicVTE, whereas stasis and endothelial damage likely play a greater role in secondaryV E,or so-called provoked VTE,occurringin association with transient risk actors such as immobilization, surgical procedures,and trauma.(SeeSchwartz10th ed.,p.918.)
181
FIG.24-2. Venousulceration located proximalto the medial malleolus.
5. Risk actors or inherited V Einclude all o the ollowing EXCEP
A. Dysfbrinogenemia
B. Factor VLeiden
C. Protein Cdefciency
D. von Willebrand disease
Answer:D
Von Willebrand Disease is the most common genetic bleeding disorder and is characterized by a propensity to bleed. All other disorders listed are associated with increased risk o V E ( able 24-1). (See Schwartz 10th ed., able 24-2, pp. 918–919)
TABLE24-1
Acquired
Advancedage
Hospitalization/immobilization
Hormonereplacementtherapyandoralcontraceptiveuse
Pregnancyandpuerperium
Priorvenousthromboembolism
Malignancy
Majorsurgery
Obesity
Nephroticsyndrome
Traumaorspinalcordinjury
Long-haultravel(>6hours)
Varicoseveins
Antiphospholipidantibodysyndrome
Myeloprolierativedisease
Polycythemia
Inherited
FactorVLeiden
Prothrombin20210A
Antithrombindefciency
ProteinCdefciency
ProteinSdefciency
FactorXIelevation
Dysfbrinogenemia
MixedEtiology
Homocysteinemia
FactorVII,VIII,IX,XIelevation
Hyperfbrinogenemia
ActivatedproteinCresistancewithoutactorVLeiden
6. May-Turner syndromeisan anatomical actor associated with increased DV ormation, and is characterized by which o the ollowing?
A. Narrowing o the le iliac vein at the site where the right iliacarterycrossesover it.
B. Narrowingo the le renal vein as it traverses beneath thesuperior mesentericartery.
C. Subclavian vein narrowing due to repetitive upper extremitye ort.
D. Arapidlyexpandinghemangioma.
A. 1%
B. 3%
Answer:A
Anatomic actorsmayalsocontributetodevelopmento DV . At thesitewheretheright iliacarterycrossesover thele t iliac vein,thele t iliacvein maybecomechronicallynarrowed predisposing to ilio emoral venous thrombosis, so-called MayThurner syndrome. External compression o major veins by masses o various types can also lead to venous thrombosis. (SeeSchwartz10th ed.,p.919.)
Answer:C
Scoring systems have been developed that take into account thenumber o V Erisk actorsin an individualpatient. hese riskstrati ication scores,such astheRogersscoreand Caprini score, provide individual patient risk strati ication and recommendations or prophylactic anticoagulation. he ninth edition o the American College o Chest Physicians (ACCP) Guidelines or Prevention o V E in Non-Orthopedic SurgicalPatientsacknowledgesboth theRogersand Capriniscores and providesrecommendations or V E.Orthopedicsurgical
182
Riskfactorsforvenousthromboembolism
7. ACaprini score o ≥5in a generalsurgerypatient without thromboprophylaxis is associated with what percentage risko developingaDV ?
C. 6%
D. 10%
LevelofRisk
Verylowrisk
Generalorabdominopelvicsurgery
Lowrisk
Generalorabdominopelvicsurgery
Moderaterisk
Generalorabdominopelvicsurgery
patients are generally excluded rom risk assessment scores because o the disproportionately increased risk o V E in orthopedic surgery compared with the general and abdominopelvic surgery population ( able 24-2). (See Schwartz 10th ed., able24-3,p.919.)
ApproximateDvtRiskwithout Thromboprophylaxis(%)
SuggestedThromboprophylaxisOptions
<0.5%(Rogersscore<7;Capriniscore0) Nospecifcthromboprophylaxis
Earlyambulation
~1.5%(Rogersscore7–10;Capriniscore1–2) Mechanicalprophylaxis
~3.0%(Rogersscore>10;Capriniscore3–4)
LMWH(atrecommendeddoses),LDUH,or mechanicalprophylaxis
Highbleedingrisk Mechanicalprophylaxis
Highrisk
Generalorabdominopelvicsurgery
Highbleedingrisk Generalorabdominopelvicsurgeryorcancer
~6%(Capriniscore ≥5)
LMWH(atrecommendeddoses),ondaparinux andmechanicalprophylaxis
Mechanicalthromboprophylaxis
Extended-durationLMWH(4weeks)
DVT= deep vein thrombosis;INR= internationalnormalized ratio;LDUH= low-dose un ractionated heparin;LMWH= lowmolecularweight heparin;VTE= venous thromboembolism.
Source:Summaryo recommendations rom Gould MK,Garcia DA,Wren SM,et al.Prevention o VTEin nonorthopedicsurgicalpatients:antithrombotictherapyand prevention o thrombosis:American College o Chest PhysiciansEvidence-Based ClinicalPractice Guidelines,9th ed.Chest.2012;141:227S.
8. Phlegmasia cerulea dolensisbest described as
A. Asymptomatic,but extensiveDV .
B. Isolated poplitealvein thrombosis.
C. Extensive DV o the major axial deep venous channels o the lower extremity potentially complicated by venousgangreneand/or theneed or amputation.
D. Painlesslower extremityswelling.
Answer:C
Clinical symptoms may worsen as DV propagates and involvesthemajor proximaldeep veins.ExtensiveDV o the major axialdeep venouschannelso thelower extremitywith relative sparing o collateral veins causes a condition called phlegmasia cerulea dolens (Fig. 24-3). his condition is characterized bypain and pittingedema with associated cyanosis. When the thrombosis extends to the collateral veins, massive luid sequestration and more signi icant edema ensue, resultingin acondition known asphlegmasia alba dolens. he a ectedextremityin phlegmasia alba dolensisextremelypainulandedematousandpalesecondarytoarterialinsu iciency rom dramatically elevated below lower knee compartment pressures. Both phlegmasia cerulean dolens and phlegmasia alba dolens can be complicated by venous gangrene and the need or amputation. (See Schwartz 10th ed., Figure 24-4, pp.919–920.)
183
FIG.24-3. Phlegmasia cerulea dolenso the le t leg.Note the bluish discoloration.
TABLE24-2 Thromboembolism riskand recommended thromboprophylaxisin surgicalpatients
9. According to the American College o Chest Physicians, therecommended duration o long-term antithrombotic therapya er provoked DV is
A 2weeks
B. 1month
C. 3months
D. 6months
Answer:C he recommended duration o antithrombotic therapy is strati ied based on whether theDV wasprovoked or unprovoked, whether it was the irst or a recurrent episode, where the DV is located, and whether malignancy or thrombophilia is present. In patients with proximal DV , several randomized clinical trials have demonstrated that shorter-term antithrombotic therapy (4 to 6 weeks) is associated with a higher rateo V Erecurrencethan 3to6monthso anticoagulation.46,47,48In thesetrials,most o thepatientswith transient risk actors had a low rate o recurrent V E, and most recurrenceswerein patientswith continuingrisk actors. he ACCPrecommendation,there ore,isthat 3monthso anticoagulation are su icient to prevent recurrent V E in patients with DV occurring around the time o a transient risk actor (e.g.,hospitalization,orthopedicor major generalsurgery) ( able24-3).(SeeSchwartz10th ed., able24-4,p.923.)
TABLE24-3 SummaryofAmerican College ofChest Physicians recommendationsregarding duration oflong-term antithrombotic therapyfordeep vein thrombosis(DVT)
ClinicalSubgroup
FirstepisodeDVT/transientrisk/ surgery
FirstepisodeDVT/unprovoked
Antithrombotic
TreatmentDuration
VKAorLMWHor3months
VKAorLMWHor3months
Considerorlong-termtherapyi:
•ProximalDVT
•Minimalbleedingrisk
•Stablecoagulation monitoring
DistalDVT/unprovoked
•Symptomatic
•Asymptomaticandnorisk actorsorprogression
SecondepisodeDVT/unprovoked
DVTandcancer
10. All o the ollowingare indications or placement o IVC fltersEXCEP
A. Septicthromboembolism.
B. Bleeding complication rom anticoagulation therapy
o acuteV E.
C. Recurrent DV or PE despite adequate anticoagulation therapy.
D. Severepulmonaryhypertension.
11. All o the ollowing are appropriate therapies or suppurativethrombophlebitisEXCEP
A. Nonsteroidalanti-in ammatorydrugs
B. Antibiotics
C. Systemicsteroid therapy
D. Removalo existingindwellingvenouscatheters
VKAor3months
Serialimagingin2weeks,i progressionVKAor3months
VKAorextendedtherapy
LMWHorextendedtherapyover
VKA
LMWH= lowmolecularweight heparin;VKA= vitamin Kantagonist.
Source:Summaryo recommendations rom Kearon C,AklEA,Comerota AJ,et al.Antithrombotictherapy orVTEdisease:antithrombotictherapyand prevention o thrombosis:American College o Chest PhysiciansEvidence-Based ClinicalPractice Guidelines, 9th ed.Chest.2012;141:e419.
Answer:A
Placement o an IVC ilter is indicated or patients who have mani estationso lower extremityV Eandabsolutecontraindicationsto anticoagulation,those who have a bleedingcomplication rom anticoagulation therapyo acuteV E,or those who develop recurrent DV or PE despite adequate anticoagulation therapy and or patients with severe pulmonary hypertension.(SeeSchwartz10th ed.,p.924.)
Answer:C
reatment o super icial vein thrombophlebitis (SV ) is quite variable. A Cochrane Review reported that lowmolecular-weight heparins (LMWHs) and nonsteroidal anti-in lammatory drugs both reduce the rate o SV extension or recurrence. opical medications appear to improve
184
12. Which o ollowing statements regarding injection sclerotherapy or varicoseveinsistrue?
A Sclerotherapy can be success ul in veins <3 mm in diameter and in telangiectatic vessels. Sclerosing agents include hypertonic saline, sodium tetradecyl sul ate,and polidocanol.
B. Compressive elastic bandages should be wrapped around the leg a er sclerotherapy, and need to be worn or 24hours.
C. Elasticstockingsshould beworn or only1weeka er sclerotherapy.
D. Tere are minimal complications associated with sclerotherapy.
localsymptoms.Surgicaltreatment,combined with theuseo graduated compression stockings, is associated with a lower rate o V Eand SV progression. he treatment is individualized and depends on the location o the thrombus and the severity o symptoms. In patients with SV not within 1 cm o the sapheno emoral junction, treatment consists o compression and administration o an anti-in lammatorymedication such as indomethacin.In patients with suppurative SV , antibiotics and removal o any existing indwelling catheters are mandatory. Excision o the vein may be necessary but is usually reserved or patients with systemic symptoms or when excision o the involved vein is straight orward. I the SV extends proximally to within 1 cm o the sapheno emoraljunction,extension into thecommon emoralvein ismore likelyto occur.In thesepatients,anticoagulation therapy or 6 weeksandgreatsaphenousvein (GSV)ligation appear equally e ective in preventing thrombus extension into the deep venoussystem.(SeeSchwartz10th ed.,p.928.)
Answer:A Interventional management includes injection sclerotherapy, surgical therapy, or a combination o both techniques. Injection sclerotherapy alone can be success ul in varicose veins <3mm in diameter and in telangiectaticvessels.Sclerotherapy acts bydestroyingthe venousendothelium.Sclerosingagents includehypertonicsaline,sodium tetradecylsul ate,and polidocanol. Concentrations o 11.7 to 23.4% hypertonic saline, 0.125to0.250%sodium tetradecylsul ate,and0.5%polidocanol are used or telangiectasias. Larger varicose veins require higher concentrations: 23.4% hypertonic saline, 0.50 to 1% sodium tetradecylsul ate,and 0.75to 1.0%polidocanol.Elastic bandages are wrapped around the leg a ter injection and worn continuously or 3to5daystoproduceapposition o the in lamed vein walls and prevent thrombus ormation. A ter the bandages are removed, elastic compression stockings should be worn or a minimum o 2 weeks. Complications rom sclerotherapy include allergic reaction, local hyperpigmentation, thrombophlebitis, DV , and possible skin necrosis.(SeeSchwartz10th ed.,p.929.)
13. Heparin-induced thrombocytopenia (HI ) is characterized bywhich o the ollowing?
A. Diagnosis based on prior exposure to heparin with platelet count <120,000and/or platelet declineo 40% ollowingheparin exposure.
B. Results rom heparin-associated antiplatelet antibodies (HAAbs) directed against platelet actor 4 complexed with heparin.
C Low incidence in patients with repeat exposure to heparin.
D. Minimalassociation with thromboticcomplications.
Answer:B
Heparin-induced thrombocytopenia (HI ) results rom heparin-associated antiplatelet antibodies (HAAbs) directed against platelet actor 4 complexed with heparin. HI occurs in 1 to 5%o patients being treated with heparin. In patients with repeat heparin exposure (such as vascular surgery patients),theincidenceo HAAbsmaybeashigh as21%.HI occursmost requentlyin thesecondweeko therapyandmay lead to disastrous venous or arterial thrombotic complications. here ore,platelet countsshould bemonitored periodicallyin patientsreceivingcontinuousheparin therapy.
HI is diagnosed based on previous exposure to heparin, platelet count lessthan 100,000,and/or platelet count declineo 50% ollowingexposure.Allheparin must bestopped and alternativeanticoagulationinitiatedimmediatelytoavoidthrombotic complications,whichmayapproach50%over thesubsequent30 daysina ectedindividuals.(SeeSchwartz10thed.,p.922.)
185
14. Direct thrombin inhibitingmedicationsincludewhich o the ollowing
A. War arin
B Enoxaparin
C Argatroban
D. Fondaparinux
15. Which o the ollowing is a true statement about lymphedema?
A. Secondary lymphedema is less common than primarylymphedema.
B. Axillary node dissection leading to lymphedema o the arm is the most common cause o secondary lymphedemain theUnited States.
C. Filariasis and other environmental exposures are uncommon causeso lymphedemaglobally.
D. Surgeryisthemainstayo therapy or lymphedema.
Answer:C
Direct thrombin inhibitors (D Is) include recombinant hirudin,argatroban,and bivalirudin. heseantithromboticagents bind to thrombin, inhibiting the conversion o ibrinogen to ibrin as well as thrombin-induced platelet activation. hese actionsareindependent o antithrombin. heD Isshould be reserved or (a) patients in whom there is a high clinical suspicion or con irmation o HI , and (b) patients who have a historyo HI or test positive or heparin-associated antibodies.In patientswith established HI ,D Isshould beadministered or at least 7days,or untiltheplatelet count normalizes. War arin maythen beintroduced slowly,overlappingtherapy with aD I or at least 5days.(SeeSchwartz10th ed.,p.922.)
Answer:B
heoriginalclassi ication system,describedbyAllen,isbased on thecauseo thelymphedema.Primarylymphedemais urther subdivided into congenital lymphedema, lymphedema praecox,andlymphedematarda.Congenitallymphedema may involve a single lower extremity,multiple limbs,the genitalia, or the ace. heedematypicallydevelopsbe ore2yearso age and may be associated with speci ic hereditary syndromes ( urner syndrome, Milroy syndrome, Klippel- renaunayWeber syndrome). Lymphedema praecox is the most common orm o primary lymphedema, accounting or 94% o cases. Lymphedema praecox is ar more common in women, with the gender ratio avoringwomen 10:1. he onset is during childhood or the teenage years, and the swelling involves the oot and cal . Lymphedema tarda is uncommon, accounting or <10% o cases o primary lymphedema. he onset o edemaisa ter 35yearso age.
Secondarylymphedemais ar morecommon than primary lymphedema. Secondary lymphedema develops as a result o lymphaticobstruction or disruption.Axillarynodedissection leadingto lymphedemao thearm isthemost common cause o secondary lymphedema in the United States. Other causes o secondarylymphedema include radiation therapy,trauma, in ection, and malignancy. Globally, ilariasis (an in ection caused by Wuchereria bancrofti, Brugia malayi, and Brugia timori) and environmental exposure to minerals in volcanic soil resultingin podoconiosis in bare oot populations are the most common causeso secondarylymphedema.
Control o chronic limb swelling through compression is the mainstay o therapy or lymphedema. (See Schwartz 10th ed.,p.934.)
186
16. Mesentericvein thrombosis(MV ) isassociated with all o the ollowingEXCEP
A. MV is less common in patients with hypercoagulablestates,malignancy,or cirrhosis.
B. 5 to 15%o cases o acute mesenteric ischemia occur asaresult o MV .
C Patients with MV are treated with uid resuscitation,heparin anticoagulation,and bowelrest.
D. Computed tomography (C ) scan and magnetic resonance imaging (MRI) are 100% sensitive and 98% specifc or MV .
Answer:A
Five to 15%o cases o acute mesenteric ischemia occur as a result o mesenteric vein thrombosis (MV ). Mortality rates in patientswith MV mayapproach 50%. he usualpresenting symptom is nonspeci ic abdominal pain and distention, o ten accompanied by nausea, vomiting, and diarrhea. Peritoneal signs, suggesting intestinal in arction, are present in ewer than hal o MV patients. MV is more common in patients with a hypercoagulable states, malignancy, and cirrhosis. MV occurs as a rare complication o laparoscopic surgery.
Most cases o MV are diagnosed with contrast-enhanced computed tomography(C ) scanningor magneticresonance imaging (MRI) in the course o an evaluation or abdominal pain. hesensitivityand speci icity or C and MRIapproach 100% and 98%, respectively. Ultrasound can also be used and has reported sensitivity and speci icity o 93%and 99%, respectively.
PatientswithMV aretreatedwith luidresuscitation,heparin anticoagulation,andbowelrest.(SeeSchwartz10thed.,p.929.)
187
CHAPTER
The Esophagusand DiaphragmaticHernia
1. Locationso anatomicnarrowingo theesophagusseen on an esophagram inc udea o the o owingEXCEP
A. Lower esophagea sphincter
B. Crossingo the e mainstem bronchusandaorticarch
C. Toracicout et
D. Cricopharyngea musc e
Answer:C
hree norma areas o esophagea narrowing are evident on the barium esophagogram or during esophagoscopy. he uppermost narrowing is ocated at the entrance into the esophagus and is caused by the cricopharyngea musc e. Its umina diameter is 1.5 cm, and it is the narrowest point o the esophagus. he midd e narrowing is due to an indentation o the anterior and e t atera esophagea wa caused bythecrossingo the e t main stem bronchusand aorticarch. he umina diameter at this point is 1.6 cm. he owermost narrowingisatthehiatuso thediaphragmandiscausedbythe gastroesophagea sphincter mechanism. he umina diameter at this point varies somewhat, depending on the distention o the esophagus by the passage o ood, but has been measured at 1.6 to 1.9 cm. hese norma constrictions tend to ho d up swa owed oreign objects,and theover yingmucosaissubject to injuryby swa owed corrosive iquids because o their s ow passagethrough theseareas.(SeeSchwartz10th ed.,p.942.)
2. Tecervica esophagusreceivesitsb ood supp yprimari y rom the
A. Interna carotid artery
B. In erior thyroid artery
C. Superior thyroid artery
D. In erior cervica artery
E. Facia artery
Answer:B
hecervica portion o theesophagusreceivesitsmain b ood supp y rom the in erior thyroid artery. he thoracic portion receives its b ood supp y rom the bronchia arteries, with 75%o individua s having one right-sided and two e t-sided branches. wo esophagea branches arise direct y rom the aorta. he abdomina portion o the esophagus receives its b ood supp y rom the ascending branch o the e t gastric arteryand rom in erior phrenicarteries.On enteringthewa o the esophagus, the arteries assume a -shaped division to orm a ongitudina p exus,givingrisetoan intramura vascuar network in themuscu ar and submucosa ayers.Asa consequence, the esophagus can be mobi ized rom the stomach tothe eve o theaorticarch without ear o devascu arization and ischemic necrosis. Caution shou d be exercised as to the extent o esophagea mobi ization in patients who have had a previous thyroidectomy with igation o the in erior thyroid arteries proxima to the origin o the esophagea branches. (SeeSchwartz10th ed.,pp.945–946.)
189 25
3. A o the o owingcrania nervesareinvo ved in theswaowingmechanism EXCEP
Answer:C Swa owing can be started at wi , or it can be re exive ye icited by the stimu ation o areas in the mouth and pharynx, amongthem the anterior and posterior tonsi ar pi arsor the posterior atera wa s o the hypopharynx. he a erent sensory nerves o the pharynx are the g ossopharyngea nerves and thesuperior aryngea brancheso thevagusnerves.Once aroused by stimu i entering via these nerves, the swa owing centerin themedu acoordinatesthecomp eteacto swa owingbydischargingimpu ses through crania nerves V,VII,X, XI, and XII, as we as the motor neurons o C1 to C3. Dischargesthroughthesenervesoccur in arather speci icpattern and ast or approximate y 0.5 second. Litt e is known about the organization o the swa owing center, except that it can trigger swa owing a ter a variety o di erent inputs, but the response is a ways a rigid y ordered pattern o out ow. Foowing a cerebrovascu ar accident, this coordinated outow may be a tered, causing mi d to severe abnorma ities o swa owing. In more severe injury, swa owing can be gross y disrupted, eading to repetitive aspiration. (See Schwartz 10th ed.,p.948.)
4. A o these are parts o the human antire ux mechanism
EXCEP
A. Adequategastricreservoir
B. Mechanica y unctioning ower esophagea sphincter (LES)
C. Mucussecretingce so thedista esophagus
D. E cient esophagea c earance
Answer:C
I the pharyngea swa ow does not initiate a perista tic contraction, then the coincident re axation o the ower esophagea sphincter (LES) is unguarded and re ux o gastric juice can occur. his may be an exp anation or the observation o spontaneous ower esophagea re axation, thought by some to be a causative actor in gastroesophagea re ux disease (GERD). he power o the worm-drive pump o the esophagea body is insu icient to orce open a va ve that does not re ax. In dogs, a bi atera cervica parasympathetic b ockade abo ishes the re axation o the LESthat occurs with pharyngea swa owingor distention o theesophagus.Consequent y, vaga unction appears to be important in coordinating the re axation o theLESwith esophagea contraction.
he antire ux mechanism in human beings is composed o three components: a mechanica y e ective LES, e icient esophagea c earance, and an adequate y unctioning gastric reservoir. A de ect o any one o these three components can ead to increased esophagea exposure to gastric juice and the deve opmento mucosa injury.(SeeSchwartz10thed.,p.949.)
5. Physio ogic re ux happens most common y when a person is
A. Awakeand supine
B. Awakeand upright
C. As eep and supine
D. As eep and semi-erect
Answer:B
On 24-hour esophagea pH monitoring, hea thy individua s have occasiona episodes o gastroesophagea re ux. his physio ogic re ux is more common when awake and in the upright position than during s eep in the supine position. When re ux o gastric juice occurs, norma subjects rapid y c ear the acid gastric juice rom the esophagus regard ess o their position.
herearesevera exp anations ortheobservationthatphysio ogic re uxin norma subjects is more common when they areawakeand in theupright position than durings eep in the supineposition.First,re uxepisodesoccur in hea thyvo unteersprimari yduringtransient osseso thegastroesophagea barrier, which maybe due to a re axation o the LESor intragastric pressure overcoming sphincter pressure. Gastric juice can a so re uxwhen a swa ow-induced re axation o theLES
190
A. V B. VII
VIII D. X E. XI
XII
C.
F.
A. Gastrin
B. Estrogen
C. Somatostatin
D. CCK
E. G ucagon
isnot protected byan oncomingperista ticwave. heaverage requencyo these“unguardedmoments”oro transient osses o the gastroesophagea barrier is ar ess whi e as eep and in thesupineposition than whi eawakeand in theupright position.Consequent y,thereare ewer opportunities or re uxto occur in the supine position. Second, in the upright position, there is a 12-mm Hg pressure gradient between the resting, positive intra-abdomina pressure measured in the stomach and the most negative intrathoracic pressure measured in the esophagus at midthoracic eve . his gradient avors the ow o gastric juice up into the thoracic esophagus when upright he gradient diminishes in the supine position. hird, the LESpressure in norma subjects is signi icant y higher in the supine position than in the upright position. his is due to the apposition o the hydrostatic pressure o the abdomen to the abdomina portion o the sphincter when supine. In the upright position, the abdomina pressure surrounding the sphincter is negative compared with atmospheric pressure, and, as expected, the abdomina pressure gradua y increases themorecauda yit ismeasured. hispressuregradient tends tomovethegastriccontentstowardthecardiaandencourages theoccurrenceo re uxinto theesophaguswhen theindividua is upright. In contrast, in the supine position, the gastroesophagea pressure gradient diminishes, and the abdomina hydrostatic pressure under the diaphragm increases, causing an increase in sphincter pressure and a more competent cardia.(SeeSchwartz10th ed.,p.949.)
Answer:A
he LEShas intrinsic myogenic tone, which is modu ated by neura and hormona mechanisms. A pha-adrenergic neurotransmitters or beta b ockers stimu ate the LES, and a pha b ockers and beta stimu ants decrease its pressure. It is not c ear to what extent cho inergic nerve activity contro s LES pressure. he vagus nerve carries both excitatory and inhibitory ibers to the esophagus and sphincter. he hormones gastrin and moti in have been shown to increase LES pressure; and cho ecystokinin, estrogen, g ucagon, progesterone, somatostatin, and secretin decrease LES pressure. he peptides bombesin, l-enkepha in, and substance P increase LES pressure; and ca citonin gene-re ated peptide, gastric inhibitory peptide, neuropeptide Y, and vasoactive intestina po ypeptide decrease LES pressure. Some pharmaco ogic agents such as antacids, cho inergics, agonists, domperidone, metoc opramide, and prostag andin F2 are known to increase LES pressure; and anticho inergics, barbiturates, ca cium channe b ockers, ca eine, diazepam, dopamine, meperidine, prostag andin E1 and E2, and theophy ine decrease LES pressure. Peppermint, choco ate, co ee, ethano , and at are a associated with decreased LESpressure and maybe responsib e or esophagea symptoms a ter a sumptuous mea . (See Schwartz 10th ed.,pp.949–950.)
191
6. A o the o owinghormonesdecreaseLEStoneEXCEP
7. Temost common causeo ade cient LESis
A. Inadequateovera ength
B. Mean restingpressure>6mm Hg
C. Inadequateintra-abdomina ength
D. Fai ureo receptivere axation
Answer:C
It is important that a portion o the tota ength o the LESbe exposed to thee ectso an intra-abdomina pressure. hat is, duringperiodso e evatedintra-abdomina pressure,theresistance o the barrier wou d be overcome i pressure were not app ied equa y to both the LESand stomach simu taneous y. hus, in the presence o a hiata hernia, the sphincter resides entire y within the chest cavity and cannot respond to an increase in intra-abdomina pressure because the pinch va ve mechanismis ostandgastroesophagea re uxismore iab eto occur. here ore,apermanent yde ectivesphincter isde ined byoneor moreo the o owingcharacteristics:An LESwith a mean restingpressureo essthan 6mm Hg,an overa sphincter ength o <2 cm, and intra-abdomina sphincter ength o <1 cm. Compared to norma subjects without GERD these va ues are be ow the 2.5 percenti e or each parameter. he most common cause o a de ective sphincter is an inadequate abdomina ength.(SeeSchwartz10th ed.,p.966.)
8. Maxima esophagea mucosa damage is caused by exposureto
A. Acidic uid a one
B. Acidic uid, ood contents,and pepsin
C. Acidic uid,trypsin,and ood contents
D. Acidic uid,pepsin,and bi esa ts
E. Neutra uid,pepsin,and trypsin
9. Te incidence o metap astic Barrett esophagus (BE) progressingtoadenocarcinomais
A. Lessthan 0.1%per year
B. 0.2to0.5%per year
C. 1to3%per year
D. 3to5%per year
E. Greater than 5%per year
Answer:D
hepotentia injuriouscomponentsthatre uxintotheesophagusinc udegastricsecretionssuch asacid and pepsin,aswe as bi iary and pancreatic secretions that regurgitate rom the duodenum into the stomach. here is a considerab e body o experimenta evidence to indicate that maxima epithe ia injuryoccursduringexposuretobi esa tscombined with acid and pepsin. hese studies have shown that acid a one does minima damage to the esophagea mucosa, but the combination o acid and pepsin is high y de eterious. Simi ar y, the re uxo duodena juicea onedoes itt edamagetothemucosa, a though thecombination o duodena juiceand gastricacid is particu ar ynoxious.(SeeSchwartz10th ed.,p.967.)
Answer:B
I re ux o gastric juice is a owed to persist and sustained or repetitive esophagea injury occurs, two seque ae can resu t. First, a umina stricture can deve op rom submucosa and eventua y intramura ibrosis. Second, the tubu ar esophagus may become rep aced with co umnar epithe ium. he co umnar epithe ium isresistant toacid and isassociated with the a eviation o the comp aint o heartburn. his co umnar epithe ium o ten becomes intestina ized, identi ied histo ogica y by the presence o gob et ce s. his specia ized intestina metap asia (IM) is current yrequired or the diagnosis o Barrett esophagus (BE). Endoscopica y, BE can be quiescent or associatedwith comp icationso esophagitis,stricture,Barrett u ceration, and dysp asia. he comp ications associated with BEmaybeduetothecontinuousirritation rom re uxed duodenogastric juice. his continued injury is pH-dependent and may be modi ied by medica therapy. he incidence o metap astic Barrett epithe ium becoming dysp astic and progressing to adenocarcinoma is approximate y 0.2 to 0.5%per year.(SeeSchwartz10th ed.,pp.968–969.)
192
10.
Te
histo ogicha marko BEis
A. Co umnar epithe ium
B. Gob et ce s
C. Parieta ce s
D. Cuboida epithe ium
11. Re ie rom respiratory symptoms can be expected in approximate y what percent o patients with re ux associated asthmawith medica therapy
A <10%
B. 25%
C 50%
D 75%
12. A o the o owingpatientsaregood candidates or antire uxsurgeryEXCEP
A. A 31-year-o d man with typica GERD with disease becomingresistant tomedica therapy.
B A55-year-o dwomanwithdiseasewe contro edwith PPIswhowishestodiscontinuemedica therapy.
C. A 75-year-o d man with new onset heartburn which isnot re ieved byPPIs.
D. A 52-year-o d man with vo ume re ux and a arge paraesophagea hernia.
Answer:B
he de inition o BE has evo ved considerab y over the past decade. raditiona y, BE was identi ied by the presence o co umnar mucosa extendingat east 3cm into theesophagus. It is now recognized that the specia ized, intestina -type epithe ium ound in theBarrett mucosa istheon ytissuepredisposedtoma ignantdegeneration.Consequent y,thediagnosis o BE is present y made given any ength o endoscopica y identi iab e co umnar mucosa that proves,on biopsy,to show IM. A though ong segments o co umnar mucosa without IM do occur,theyare uncommon and might becongenita in origin.
heha marko IMisthepresenceo intestina gob et ce s. here is a high preva ence o biopsy-demonstrated IM at the cardia, on the gastric side o the squamoco umnar junction, in the absence o endoscopic evidence o a co umnar- ined esophagus(CLE).Evidenceisaccumu atingthatthesepatches o whatappearstobeBarrettinthecardiahaveasimi arma ignant potentia as in the onger segments, and are precursors or carcinomao thecardia.(SeeSchwartz10th ed.,p.969.)
Answer:C
Once the diagnosis is estab ished, treatment may be initiated with either proton pump inhibitor (PPI) therapyor antire ux surgery. A tria o high-dose PPI therapy may he p estab ish the actsthat re uxispart yor comp ete yresponsib e or the respiratory symptoms. It is important to note that the persistence o symptoms in the ace o aggressive PPI treatment does not necessari y ru e out re ux as a possib e co actor or so eetio ogy.
A though there is probab y some e ements o a p acebo e ect,re ie o respiratorysymptomscan be anticipated in up to 50% o patients with re ux-induced asthma treated with antisecretory medications. However, when examined objective y,<15%o patientscan beexpected to haveimprovement in their pu monary unction with medica therapy. In proper yse ected patients,antire uxsurgeryimprovesrespiratory symptoms in near y 90% o chi dren and 70% o adu ts with asthmaand re uxdisease.Improvementsin pu monary unction can be demonstrated in around 30%o patients. Uncontro ed studies o the two orms o therapy (PPI and surgery) and the evidence rom the two randomized contro ed tria s o medica versus surgica therapyindicate that surgica va ve reconstruction isthemoste ectivetherapy orre ux-induced asthma. hesuperiorityo thesurgeryoverPPIismostnoticeab ein thesupineposition,which correspondswith thenadir o PPI b ood eve sand resu tant acid breakthrough and isthe timein thecircadian cyc ewhen asthmasymptomsareattheir worst.(SeeSchwartz10th ed.,p.971.)
Answer:C
Studies o the natura history o GERD indicate that most patients have a re ative y benign orm o the disease that is responsive to i esty e changes and dietary and medica therapy,and do not need surgica treatment.Approximate y25to 50%o thepatientswith GERD havepersistent or progressive disease, and it is this patient popu ation that is best suited to surgica therapy. In the past, the presence o esophagitis and a structura y de ective LES were the primary indications or surgica treatment, and many internists and surgeons
193
13. Preoperative testing or antire ux surgery typica y inc udesa o the o owingEXCEP
A. Computed tomography (C ) scan o the chest and abdomen
B Contrast esophagram
C. 24hour pH probe
D. Esophagea manometry
E. Esophagogastroduodenostomy
were re uctant to recommend operative procedures in their absence. However, one shou d not be deterred rom considering antire ux surgery in a symptomatic patient with or without esophagitis or a de ective sphincter, provided the disease process has been objective y documented by 24-hour pH monitoring. his is particu ar y true in patients who have become dependent upon therapy with PPIs, or require increasingdosesto contro their symptoms.It isimportant to note that a good response to medica therapyin this group o patients predicts an exce ent outcome o owing antire ux surgery.
In genera , the key indications or antire ux surgery are (a) objective y proven gastroesophagea re ux disease, and (b) typica symptoms o gastroesophagea re ux disease (heartburn and/or regurgitation) despite adequate medica management, or (c) a younger patient unwi ing to take i eongmedication.In addition,a structura yde ective LEScan a so predict which patients are more ike y to ai with medica therapy. Patients with norma sphincter pressures tend to remain we contro ed with medica therapy,whereaspatients with a structura y de ective LES may not respond as we to medica therapy, and o ten deve op recurrent symptoms within 1to 2yearso beginningtherapy.Such patientsshou d be considered or an antire ux operation, regard ess o the presenceor absenceo endoscopicesophagitis.(See Schwartz 10th ed.,p.972.)
Answer:A
Be ore proceeding with an antire ux operation, severa actors shou d be eva uated. he c inica symptoms shou d be consistent with the diagnosis o gastroesophagea re ux. Patients presenting with the typica symptoms o heartburn and/or regurgitation who have responded, at east part y, to PPI therapy, wi genera y do we o owing surgery, whereas patients with atypica symptoms have a ess predictab eresponse.Re uxshou d a so beobjective ycon irmed by either the presence o u cerative esophagitis or an abnorma 24-hour pH study.
he propu sive orce o the body o the esophagus shou d be eva uated byesophagea manometryto determine i it has su icient power to prope a bo us o ood through a new y reconstructed va ve. Patients with norma perista tic contractions can be considered or a 360° Nissen undop ication or a partia undop ication, depending on patient and surgeon pre erences. When perista sis is absent a partia undop ication is probab ythe procedure o choice, but on yi acha asia hasbeen ru ed out.
Hiata anatomy shou d a so be assessed. In patients with sma er hiata hernias endoscopy eva uation usua y provides su icient in ormation. However, when patients present with a very arge hiatus hernia or or revision surgery a ter previous antire ux surgery, contrast radio ogy provides better anatomica in ormation. he concept o anatomic shorteningo the esophagusiscontroversia ,with divergent opinions he d about how common this prob em is. Be ievers c aim that anatomic shortening o the esophagus compromises the abi ity o the surgeon to per orm an adequate repair without tension, and that this can ead to an increased incidence o breakdown or thoracic disp acement o the repair. Some o
194
14.
Te va ve created during an antire ux procedure shou d beat east
those who ho d this view c aim that esophagea shortening is present when a barium swa ow X-ray identi ies a s iding hiata hernia that wi not reduce in the upright position, or that measures more than 5 cm in ength at endoscopy. When identi iedthesesurgeonsusua yundertakeaddagastrop asty to the antire ux procedure. Others c aim that esophagea shorteningisoverdiagnosed and rare yseen,and that themorbidity o adding a gastrop asty outweighs any bene its. hese surgeons wou d recommend a standard antire ux procedure in a patients undergoing primary surgery. (See Schwartz 10th ed.,pp.972–973.)
Answer:C
he primary goa o antire ux surgery is to sa e y create a new antire ux va ve at the gastroesophagea junction (GEJ), whi e preserving the patient’s abi ity to swa ow norma y and to be ch to re ieve gaseous distention. Regard ess o the choice o the procedure, this goa can be achieved i attention is paid to some basic princip es when reconstructing the antire ux mechanism. First, the operation shou d create a ap va ve which prevents regurgitation o gastric contents into the esophagus. his wi resu t in an increase in the pressure o the dista esophagea sphincter region. Fo owing a Nissen undop ication,theexpected increaseistoa eve twice the resting gastric pressure (ie, 12 mm Hg or a gastric pressure o 6 mm Hg). he extent o the pressure rise is o ten ess o owing a partia undop ication, a though with a types o undop ication the ength o the reconstructed va ve shou d be at east 3 cm. his not on y augments sphincter characteristics in patients in whom theyare reduced be ore surgery, but prevents un o ding o a norma sphincter in response to gastric distention. Preoperative and postoperative esophagea manometry measurements have shown that the resting sphincter pressure and the overa sphincter ength can be surgica y augmented over preoperative va ues, and that the changein the ormer isa unction o thedegreeo gastricwrap around the esophagus. However, the aim o any undop ication is to create a oose wrap, and to maintain the position o thegastric undusc oseto thedista intra-abdomina esophagus, in a ap va ve arrangement. he e icacyo this re ies on the c ose re ationship between the undus and the esophagus, notthe“tightness”o thewrap.(SeeSchwartz10thed.,p.973.)
15. A oupet undop ication invo ves
A. A180°anterior wrap
B. A90°posterior wrap
C A180°posterior wrap
D. A270°posterior wrap
Answer:D
Partia undop ications were deve oped as an a ternative to the Nissen procedure in an attempt to minimize the risk o post undop ication side e ects, such as dysphagia, inabi ity to be ch, and atu ence. he commonest approach has been a posterior partia or oupet undop ication. Some surgeons use this type o procedure or a patients presenting or antire ux surgery, whereas others app y a tai ored approach in which a partia undop ication isconstructed in patientswith impaired esophagea moti ity,in which thepropu sive orceo the esophagus is thought to be insu icient to overcome the out owobstruction o acomp ete undop ication. he oupet posterior partia undop ication consistso a270°gastric undop ication around the dista 4 cm o esophagus. It is usua y stabi ized byanchoringthewrap posterior yto thehiata rim. (SeeSchwartz10th ed.,pp.975–976.)
195
2cm C. 3cm D 4cm E. 5cm
A 1cm B.
16. What percentage o patients shou d be expected to have re ie o symptomsat 5yearsout rom antire uxsurgery?
A. <50%
B 50–60%
C 60–80%
D. 80–90%
E >90%
17. An upwarddis ocation o both thecardiaandgastric undusiswhich typeo hiata hernia?
A. I
B II
C III
D. IV
Answer:D Studies o ong-term outcome o owing both open and aparoscopic undop ication document the abi ity o aparoscopic undop ication to re ieve typica re ux symptoms (heartburn,regurgitation, and dysphagia) in more than 90%o patients at o ow-up interva s averaging 2 to 3 years and 80 to 90%o patients5yearsor more o owingsurgery. hisincudes evidence-based reviews o antire ux surgery, prospective randomized tria scomparingantire uxsurgerytoPPItherapyand opento aparoscopic undop icationandana ysiso U.S.nationa trendsinuseandoutcomes.(SeeSchwartz10thed.,p.977.)
Answer:C
With the advent o c inica radio ogy, it became evident that a diaphragmatichernia wasa re ative ycommon abnorma ity andwasnota waysaccompaniedbysymptoms. hreetypeso esophagea hiata herniawereidenti ied:(a)thes idinghernia, typeI,characterized byan upward dis ocation o thecardiain the posterior mediastinum;(b) the ro ing or paraesophagea hernia(PEH),typeII,characterized byan upward dis ocation o the gastric undus a ongside a norma y positioned cardia; and (c) the combined s iding-ro ing or mixed hernia, type III, characterized by an upward dis ocation o both the cardia and the gastric undus. he end stage o type I and type II hernias occurs when the who e stomach migrates up into the chest by rotating 180° around its ongitudina axis, with the cardia and py orus as ixed points. In this situation the abnorma ityis usua yre erred to as an intrathoracicstomach (Fig. 25-1). In some taxonomies, a type IV hiata hernia is dec ared when an additiona organ, usua y the co on, herniates as we . ype II-IV hiata hernias are a so re erred to as paraesophageal hernia (PEH), as a portion o the stomach is situated adjacent to the esophagus, above the GEJ. (See Schwartz10th ed.,Figure25-39D,pp.980–981.)
196
18. Temost common orm o esophagea cancer diagnosed in theUnited Statesis
A. Adenocarcinoma
B. Squamouscarcinoma
C. Anap asticcarcinoma
D. Leiomyosarcoma
19. Squamous ce carcinomas o the esophagus most common yoccur
A. At theGEJ
B. In thecervica and upper thoracicesophagus
C. In the ower thoracicesophagus
D. Even ydistributed throughout theesophagus
25-1. Radiogram ofan intrathoracicstomach.Thisisthe end stage ofa large hiatalhernia regardlessofitsinitialclassification. Note that the stomach hasrotated 180°around itslongitudinal axis,with the cardia and pylorusasfixed points.(Reproduced with permission from DeMeesterTR,Bonavina L.Paraesophagealhiatal hernia,in:NyhusLM,Condon REeds.Hernia,3rd ed.Philadelphia: Lippincott;1989,p 684.)
Answer:A
Adenocarcinoma o the esophagus, once an unusua ma ignancy, is diagnosed with increasing requency and now accounts or more than 50% o esophagea cancer in most Western countries. he shi t in the epidemio ogy o esophagea cancer rom predominant ysquamouscarcinomaseen in association with smoking and a coho , to adenocarcinoma in thesettingo BE,isoneo themostdramaticchangesthathave occurredinthehistoryo humanneop asia.A thoughesophagea carcinomaisare ative yuncommon ma ignancy,itspreva ence is exp oding, arge y secondary to the we -estab ished association between gastroesophagea re ux,BE,and esophagea adenocarcinoma.Oncea near yuni orm y etha disease, surviva has improved s ight y because o advances in the understandingo itsmo ecu ar bio ogy,screeningand surveiance practices, improved staging, minima y invasive surgica techniques, and neoadjuvant therapy. (See Schwartz 10th ed.,p.1003.)
Answer:B
It is estimated that 8% o the primary ma ignant tumors o the esophagus occur in the cervica portion. hey are a most a wayssquamousce cancer,witharareadenocarcinomaarising rom a congenita in et patch o co umnar ining. hese tumors, particu ar y those in the postcricoid area, represent a separate patho ogic entity or two reasons: (a) hey are more common in women and appear to be a unique entityin this regard; and (b) the e erent ymphatics rom the cervica
197
FIG.
20. Te preoperative test most heavi y corre ated with the abi itytoto eratean esophagectomyis
A DLCO
B. FEV1
C. Abi itytoc imba ight o stairs
D FVC
esophagusdrain comp ete ydi erent y rom thoseo thethoracicesophagus. he atter drain direct yintotheparatrachea and deep cervica or interna jugu ar ymph nodes(LNs) with minima owin a ongitudina direction. Except in advanced disease,it isunusua or intrathoracicLNstobeinvo ved.(See Schwartz10th ed.,p.1005.)
Answer:B
Patients undergoing esophagea resection shou d have suicient cardiopu monary reserve to to erate the proposed procedure. he respiratory unction is best assessed with the orced expiratoryvo umein 1second,which idea yshou d be 2Lor more.Anypatient with a orced expiratoryvo umein 1 second o <1.25 L is a poor candidate or thoracotomy, because he or she has a 40% risk o dying rom respiratory insu iciencywithin 4years.In patientswith poor pu monary reserve,the transhiata esophagectomyshou d beconsidered, as the pu monary morbidity o this operation is ess than is seen o owing thoracotomy. C inica eva uation and e ectrocardiogram are not su icient indicators o cardiac reserve. Echocardiography and dipyridamo e-tha ium imaging provide accurate in ormation on wa motion, ejection raction, and myocardia b ood ow.Ade ect on tha ium imagingmay require urther eva uation withpreoperativecoronaryangiography.Arestingejection raction o <40%,particu ar yi there isnoincreasewith exercise,isan ominoussign.In theabsence o invasive testing, observed stair-c imbing is an economica (a beit not quantitative) method o assessing cardiopu monary reserve. Most individua s who can c imb three ights o stairs without stopping wi do we with two- ie d open esophagectomy, especia y i an epidura catheter is used or postoperativepain re ie .(SeeSchwartz10th ed.,p.1007.)
21. Which test most accurate yassess the stage o esophagea cancer?
A. High-reso ution C scan
B Magneticresonanceimaging(MRI)
C Echocardiography
D. Endoscopicu trasonography(EUS)
E. Esophagogastroduodenoscopy(EGD)
22. Which o the o owingpatientswou d not beconsidered acandidate or esophagectomy?
A. A 55-year-o d man with GEJ adenocarcinoma conned tothemuscu arismucosa.
B. A 47-year-o d woman with mid-esophagea cancer and an invo ved cervica LN.
C. A 60-year-o d man with a arge GEJ carcinoma with invasion intothep eurawithoutama ignantefusion
D. A 70-year-o d woman with a sma GEJ cancer and threepatho ogicLNsnearbyon EUS.
Answer:D
For years, c inica staging, contrast radiography, endoscopy, and C scanning ormed the backbone o esophagea cancer staging. More recent y, preoperative decision making is guided by endoscopic u trasonography (EUS) and positron emission tomography(PE )scanning.
EUS provides the most re iab e method o determining depth o cancer invasion. In the absence o en arged LNs, the degreeo wa invasiondictatessurgica therapy.(SeeSchwartz 10th ed.,p.1008.)
Answer:B
I the tumor invades into the submucosa, without visib e LN invo vement, most individua s wou d suggest esophagectomy withLNdissection,aspositivenodescanbe oundin20to25% o thosewith cancer imited to themucosa and submucosa.I EUSdemonstrates spread through the wa o the esophagus, especia y i LNs are en arged, then induction chemoradiation therapy (neoadjuvant therapy) shou d be strong y considered. Last y, when the EUS demonstrates invasion o the trachea, bronchus, aorta, or spine, then surgica resection is
198
23.Te technique o resecting an esophagea cancer which remains symptomatic a er de nitive chemoradiotherapy isre erred toas
A. Pa iativeesophagectomy
B. Sa vageesophagectomy
C. Rescueesophagectomy
D. Noneo theabove,theprocedureisnot per ormed
rare yindicated.I thereisinvasion intothep eura( 4a),then surgica resection can beconsidered in theabsenceo ama ignant e usion. hus,it can beseen that thetherapyo esophagea cancer is arge y driven by the indings o an endoscopic u trasonography. It is di icu t to provide modern treatment o esophagea cancer without access to this moda ity. (See Schwartz10th ed.,p.1008.)
Answer:B
Salvageesophagectomyisthenomenc atureapp ied toesophagectomy per ormed a ter ai ure o de initive radiation and chemotherapy. he most requent scenario is one in which distant disease (bone, ung, brain, or wide LN metastases) renders the patient nonoperab e at initia presentation. hen, systemicchemotherapy,usua ywith radiation o theprimary tumor, destroys a oci o metastasis, as demonstrated by C and C -PE , but the primary remains present and symptomatic. Fo owing a period o observation, to make sure no new disease wi become evident, sa vage esophagectomy is per ormed, usua y with an open two- ie d approach. Surprising y,thecurerateo sa vageesophagectomyisnot inconsequentia .Onein ourpatientsundergoingthisoperationwi be disease ree 5 years ater, despite the presence o residua cancer in the operative specimen. Because o the dense scarringcreated byradiation treatment,thisprocedureisthemost technica ycha engingo a esophagectomytechniques.(See Schwartz10th ed.,p.1011.)
24. Patients with dysphagia secondary to esophagea cancer treated with radiation can expect thebene t to ast
A. <1month
B 2–3months
C. 6–12months
D. >12months
25.
A. 2weeks
B. 4–6weeks
C. 6–8weeks
D. 8–10weeks
E >10weeks
Answer:B
Primary treatment with radiation therapy does not produce resu ts comparab e with those obtained with surgery. Current y,theuseo radiotherapyisrestricted to patientswho are not candidates or surgery,and isusua ycombined with chemotherapy.Radiation a oneisused or pa iation o dysphagia but the bene it is short- ived, astingon y2 to 3 months. Furthermore, the ength and course o treatment are di icu t to justi y in patients with a imited i e expectancy. Radiation is e ective in patients who have hemorrhage rom the primary tumor.(SeeSchwartz10th ed.,p.1012.)
Answer:C
he timingo surgerya ter chemoradiation induction is genera y e t to be optima between 6 and 8 weeks o owing the comp etion o induction therapy.Ear ier than thistime,active in ammation may make the resection hazardous, and the patients have not had time to recover u y rom the chemoradiation. A ter 8 weeks, edema in the periesophagea tissue startstoturn toscar tissue,makingdissection moredi icu t. With chemoradiation,thecomp eteresponserates or adenocarcinoma range rom 17 to 24%. No tumor is detected in the specimen a ter esophagectomy. Patients demonstrating a comp ete response to chemoradiation have a better surviva rate than thosewithout comp eteresponse,but distant ai ure remainscommon.(SeeSchwartz10th ed.,pp.1012–1013.)
199
How ong a er comp etion o neoadjuvant chemoradiotherapyshou d esophagectomybeper ormed?
26. Teoptima treatment o an incidenta ydiscovered 3cm eiomyoma o the upper esophagus in a 45-year-o d otherwisehea thyman is?
A. Observation
B. Esophagectomy
C Enuc eation
D. Endoscopicresection
27. Fo owinganight o heavydrinking,a43-year-o d otherwise hea thy man has sudden onset o severe chest pain a er vomiting.Esophagram con rmsesophagea rupture just proxima to the GEJ.What is the pre erred operative exposure?
A. Right thoracotomy
B. Right thoracotomywith aparotomy
C. Le thoracotomy
D. Le thoracotomywith aparotomy
E. Mid ine aparotomy
28. A 34-year-o d man presents to the emergency department (ED) a er an episode o hematemesis. EGD conrms a Ma ory-Weiss tear with no residua b eeding. reatment shou d consist o
A. Esophagectomy
B. Observation
C. Proxima gastrectomywith esophago-jejunostomy
D. Injection o botu inum toxin
Answer:C
Despitetheir s owgrowth and imited potentia or ma ignant degeneration, eiomyomas shou d be removed un ess there are speci ic contraindications. he majority can be removed by simp e enuc eation. I , during remova , the mucosa is inadvertent y entered, the de ect can be repaired primari y. A ter tumor remova , the outer esophagea wa shou d be reconstructed by c osure o the musc e ayer. he ocation o the esion and the extent o surgery required wi dictate the approach. Lesions o the proxima and midd e esophagus requirearight thoracotomy,whereasdista esophagea esions require a e t thoracotomy. Videothoracoscopic and aparoscopicapproachesarenow requent yused. hemorta ityrate associated with enuc eation is ow,and successin re ievingthe dysphagia is near 100%. Large esions or those invo ving the GEJmayrequireesophagea resection.(SeeSchwartz10th ed., pp.1017–1018.)
Answer:C
hekeytooptimum managementisear ydiagnosis. hemost avorab e outcome is obtained o owing primary c osure o the per oration within 24 hours, resu ting in 80 to 90% surviva . he most common ocation or the injuryis the e t atera wa o theesophagus,just abovetheGEJ. o get adequate exposure o the injury, a dissection simi ar to that described or esophagea myotomy is per ormed. A ap o stomach is pu ed up and the soi ed at pad at the GEJ is removed. he edges o the injury are trimmed and c osed primari y. he c osure is rein orced with the use o a p eura patch or construction o a Nissen undop ication. (See Schwartz 10th ed.,pp.1018–1019.)
Answer:B
Ma ory-Weiss tears are characterized by arteria b eeding, which may be massive. Vomiting is not an ob igatory actor, as there may be other causes o an acute increase in intraabdomina pressure, such as paroxysma coughing, seizures, andretching. hediagnosisrequiresahighindexo suspicion, particu ar y in the patient who deve ops upper gastrointestina (GI) b eeding o owing pro onged vomiting or retching. Upper endoscopy con irms the suspicion by identi ying one or more ongitudina issures in the mucosa o the herniated stomach asthesourceo b eeding.
In the majority o patients, the b eeding wi stop spontaneous y with nonoperative management. In addition to b ood rep acement,thestomach shou d bedecompressed and antiemetics administered, as a distended stomach and continued vomiting aggravate urther b eeding. A SengstakenBakemore tube wi not stop the b eeding, as the pressure in the ba oon is not su icient to overcome arteria pressure. Endoscopic injection o epinephrine may be therapeutic i b eeding does not stop spontaneous y. On y occasiona y wi surgery be required to stop b ood oss. he procedure consists o aparotomy and high gastrotomy with oversewing o the inear tear.Morta ityisuncommon,andrecurrenceisrare. (SeeSchwartz10th ed.,p.1020.)
200
29. Success u treatment o aZenker diverticu um invo ves
A. Diverticu opexy
B. Resection o thediverticu um
C. Observation
D. Either diverticu opexyor resection withcricopharyngea myotomy
Answer:D
When a pharyngoesophagea diverticu um is present, ocaization o thepharyngoesophagea segment iseasy. hediverticu um is care u y reed rom the over ying areo ar tissue to exposeitsneck,just be owthein erior pharyngea constrictor and above the cricopharyngeus musc e. It can be di icu t to identi ythecricopharyngeusmusc ein theabsenceo adiverticu um. A bene it o oca anesthesia is that the patient can swa ow and demonstrate an area o persistent narrowing at the pharyngoesophagea junction. Furthermore, be ore c osing the incision, ge atin can be ed to the patient to ascertain whether the symptoms have been re ieved, and to inspect the openingo theprevious ynarrowed pharyngoesophagea segment.Under genera anesthesia,and in theabsenceo adiverticu um,thep acement o anasogastrictubetothe eve o the manometrica y determined cricopharyngea sphincter he ps in oca ization o the structures. he myotomy is extended cepha ad by dividing 1 to 2 cm o in erior constrictor musc e o the pharynx, and caudad by dividing the cricopharyngea musc eand thecervica esophagus or a ength o 4to 5cm.I a diverticu um is present and is arge enough to persist a ter a myotomy, it may be sutured in the inverted position to the prevertebra ascia using a permanent suture (ie, diverticu opexy). I the diverticu um is excessive y arge so that it wou d beredundant i suspended,or i itswa sarethickened,then a diverticu ectomyshou dbeper ormed. hisisbestper ormed undergenera anesthesiabyp acingaMa oneydi ator(48F)in the esophagus, a ter contro ing the neck o the diverticu um and a ter myotomy. A inear stap er is p aced across the neck o the diverticu um and the diverticu um is excised dista to thestap e ine. hesecurityo thisstap e ineande ectiveness o the myotomy may be tested be ore hospita discharge with awater so ub econtrastesophagogram.Postoperativecomp ications inc ude istu a ormation, abscess, hematoma, recurrent nerve para ysis, di icu ties in phonation, and Horner syndrome. he incidence o the irst two can be reduced by per orming a diverticu opexy rather than diverticu ectomy. (SeeSchwartz10th ed.,p.989.)
30. Which o the o owing disorders invo ves simu taneous nonperista ticcontractionso theesophagus?
A. Acha asia
B. Difuseesophagea spasm (DES)
C. Hypertensive ower esophagea sphincter
D. Nutcracker esophagus
Answer:B
he c assic manometric indings in these patients are characterized by the requent occurrence o simu taneous waveorms and mu tipeaked esophagea contractions, which may be o abnorma y high amp itude or ong duration ( ab e25-1).Keytothediagnosiso di useesophagea spasm (DES) is that there remain some perista tic wave orms in excess o those seen in acha asia. A criterion o 30%or more perista ticwave ormsout o 10wet swa owshasbeen used to di erentiate DES rom vigorous acha asia. However, this igureisarbitraryand o ten debated.
he LESin patients with DESusua y shows a norma resting pressure and re axation on swa owing. A hypertensive sphincter with poor re axation maya so be present. In patients with advanced disease, the radiographic appearance o tertiary contractions appears he ica , and has been termed corkscrew esophagus or pseudodiverticu osis. Patients with segmenta or DEScan compartmenta ize the esophagus and deve op an epiphrenic or midesophagea diverticu um between two areas o highpressureoccurringsimu taneous y.(SeeSchwartz10th ed., ab e25-9,pp.991–992.)
201
TABLE25-1
Achalasia
Manometriccharacteristicsofthe primary esophagealmotilitydisorders
Incompleteloweresophagealsphincter(LES)relaxation(<75% relaxation)
Aperistalsisintheesophagealbody
ElevatedLESpressure ≤26mmHg
Increasedintraesophagealbaselinepressuresrelativetogastricbaseline
DifuseEsophagealSpasm(DES)
Simultaneous(nonperistalticcontractions)(>20%ofwetswallows)
Repetitiveandmultipeakedcontractions
Spontaneouscontractions
Intermittentnormalperistalsis
Contractionsmaybeofincreasedamplitudeandduration
NutcrackerEsophagus
Meanperistalticamplitude(10wetswallows)indistalesophagus ≥180mmHg
Increasedmeandurationofcontractions(>7.0s)
Normalperistalticsequence
HypertensiveLowerEsophagealSphincter
ElevatedLESpressure(≥26mmHg)
NormalLESrelaxation
Normalperistalsisintheesophagealbody
InefectiveEsophagealMotilityDisorders
Decreasedorabsentamplitudeofesophagealperistalsis(<30mmHg)
Increasednumberofnontransmittedcontractions
Source:Reproduced with permission from DeMeesterTR,et al.:Physiologicdiagnostic studies,in Zuidema GD,OrringerMB,(eds):Shackelford’sSurgeryoftheAlimentaryTract, 3rd ed,Vol.I.Philadelphia:W.B.Saunders,1991,p.115.Copyright Elsevier.
202
1. Teconsistentlylargest arterytothestomach isthe
A. Right gastric
B. Le gastric
C. Right gastroepiploic
D. Le gastroepiploic
2. Which o the ollowinginhibitsgastrin secretion?
A. Histamine
B. Acetylcholine
C. Aminoacids
D. Acid
3. Helicobacter pylori in ection primarilymediates duodenal ulcer pathogenesisvia
A. Antral alkalinization leading to inhibition o somatostatin release
B. Direct stimulation o gastrin release
C. Localin ammation with autoimmuneresponse
D. Upregulation o parietalcellacid production
CHAPTER
Answer:B
he consistently largest artery to the stomach is the le t gastric artery, which usually arises directly rom the celiac trunk and divides into an ascending and descending branch along the lesser gastric curvature. Approximately 20% o the time, the le t gastric artery supplies an aberrant vessel that travels in thegastrohepaticligament (lesser omentum) to thele t side o theliver.Rarely,thisistheonlyarterialblood supplyto this part o theliver,and inadvertent ligation maylead to clinically signi icanthepaticischemiain thisunusualcircumstance.(See Schwartz10th ed.,p.1037.)
Answer:D
Luminal peptides and amino acids are the most potent stimulants o gastrin release, and luminal acid is the most potent inhibitor o gastrin secretion. he latter e ect is predominantly mediated in a paracrine ashion by somatostatin released rom antralDcells.Gastrin-stimulatedacidsecretion is signi icantlyblocked byH2 antagonists, suggestingthat the principal mediator o gastrin-stimulated acid production is histamine rom mucosal enterochroma in-like (ECL) cells. Acetylcholinereleased bythevagusnerveleadstostimulation o ECLcells,which in turn produce histamine.(See Schwartz 10th ed.,p.1045.)
Answer:A
Helicobacter pyloripossesstheenzymeurease,which converts ureaintoammoniaandbicarbonate,thuscreatingan environment around thebacteria that bu erstheacid secreted bythe stomach. H. pylori in ection is associated with decreased levels o somatostatin, decreased somatostatin messenger RNA production,and ewer somatostatin-producingD cells. hese e ects are probably mediated by H. pylori-induced local alkalinization o the antrum (antral acidi ication is the most potent antagonist to antral gastrin secretion), and H. pylorimediated increases in other local mediators and cytokines. he result is hypergastrinemia and acid hypersecretion, presumablyleading to the parietal cell hyperplasia seen in many patients with duodenal ulcer. Other mechanisms whereby H. pylori can induce gastroduodenal mucosal injury include the production o toxins (vacA and cagA), local elaboration o cytokines (particularly interleukin-8) by in ected mucosa,
203 26
Stomach
4. Tee ect o erythromycin on gastricemptyingisthrough its unction asa
A. Dopamineantagonist
B. Cholinergicagonist
C. Motilin agonist
D. Cholinergicantagonist
5. Which o the ollowingissecreted bygastricparietalcells?
A. Pepsinogen
B. Intrinsic actor
C. Gastrin-releasingpeptide
D. Ghrelin
E. Histamine
6. Te most accurate diagnostic test or Zollinger-Ellison syndrome(ZES)is
A. Fastingserum gastrin
B. Computed tomography(C )scan
C. Endoscopy
D. Secretin stimulation test
recruitment o in lammatory cells and release o in lammatory mediators, recruitment and activation o local immune actors, and increased apoptosis. (See Schwartz 10th ed., pp.1054–1055.)
Answer:C
Erythromycin is a common prokinetic agent used to treat delayedgastricempting,andworksasamotilin agonist.Domperidone and metoclopramide, two other commonly used medications, unction as dopamine antagonists ( able 26-1). (SeeSchwartz10th ed., able26-4,p.1050.)
Metoclopramide
Erythromycin
Domperidone 10mgPOqid Dopamineantagonist
Answer:B
Activated parietal cells secrete intrinsic actor in addition to hydrochloric acid. Presumablythe stimulants are similar, but acidsecretion andintrinsic actorsecretion maynotbelinked. Intrinsic actor binds to luminal vitamin B12, and the complex is absorbed in the terminal ileum via mucosal receptors. Vitamin B12 de iciency can be li e-threatening, and patients with total gastrectomy or pernicious anemia require B12 supplementationbyanonentericroute.(SeeSchwartz10thed., p.1044.)
Answer:D
All patients with gastrinoma have an elevated gastrin level, and hypergastrinemia in the presence o elevated basal acid output(BAO)stronglysuggestsgastrinoma.Patientswithgastrinoma usually have a BAO >15 mEq/h or >5 mEq/h i they havehad apreviousprocedure or pepticulcer.Acid secretory medications should be held or several days be ore gastrin measurement, because acid suppression may alsely elevate gastrin levels. Causes o hypergastrinemia can be divided into those associated with hyperacidity and those associated with hypoacidity (Fig. 26-1). he diagnosis o ZollingerEllison syndrome (ZES) is con irmed by the secretin stimulation test. An intravenous (IV) bolus o secretin (2 U/kg) is given and gastrin levelsarechecked be oreand a ter injection. An increase in serum gastrin o 200 pg/mL or greater suggests the presence o gastrinoma. Patients with gastrinoma should have serum calcium and parathyroid hormone levels determined to rule out multiple endocrine neoplasia type 1 (MEN1)and,i present,parathyroidectomyshould beconsidered be ore resection o gastrinoma. (See Schwartz 10th ed., Figure26-46,p.1072.)
204
Agent TypicalAdultDose MechanismofAction
TABLE26-1 Drugsthat accelerate gastricemptying
10mgPOqid
Dopamineantagonist
250mgPOqid
Motilinagonist
FIG.26-1. Algorithm ordiagnosisand management o hypergastrinemia.B1 = Billroth I;B2 = Billroth II;BAO= basalacid output;Bx= biopsy; ECL= enterochroma in-like;EGD= esophagogastroduodenoscopy;GJ= gastrojejunostomy;H2RA= histamine 2receptorantagonist;insu = insu iciency;MEN1= multiple endocrine neoplasia type 1;PPI= proton pump inhibitor;R/O= rule out;SB= smallbowel;S/P= statuspost; TV= truncalvagotomy;TVand A= truncalvagotomyand antrectomy.
7. Which o the ollowing is the preoperative imaging study o choice or gastrinoma?
A. C scan
B. Magneticresonanceimaging(MRI)
C. Endoscopicultrasound (EUS)
D. Angiographiclocalization
E. Somatostatin receptor scintigraphy
Answer:E
C will detect most lesions >1cm in size and magnetic resonance imaging(MRI) is comparable. Endoscopic ultrasound (EUS)ismoresensitivethan theseother noninvasiveimaging tests, but it still misses many o the smaller lesions, and may con use normal lymph nodes or gastrinomas. Currently, the preoperative imaging study o choice or gastrinoma is somatostatin-receptor scintigraphy (the octreotide scan). When the pretest probability o gastrinoma is high, the sensitivity and speci icity o this modality approach 100%. Gastrinomacellscontain typeIIsomatostatin receptorsthat bind the indium-labeled somatostatin analogue (octreotide) with high a inity, making imaging with a gamma camera possible. Currently, angiographic localization studies are in requently per ormed or gastrinoma. (See Schwartz 10th ed., pp.1072–1073.)
205 Elevated serum gastrin (off PPI+ H2RA) Measure BAO and gastric pH Secretin stimulation test BAO >10 mEq/h pH <2 BAO low pH >2 Previous GI surgery • Vagotomy? • Massive SB resection Atrophic gastritis Pernicious anemia Renal insuff No Yes Confirm with EGD/Bx test/treat H. pylori give B12 Significant elevation in serum gastrin in response to IV secretin? G-cell +/or ECL cell hyperplasia Loop GJ with antral alkalinization Takedown GJ Or TV and A Consider TV + antrectomy Retained antrum on “duodenal stump” Resect retained antrum Or Convert B2 to B1 Antral stasis S/P gastrectomy B2 Confirm with EGD + Bx Consider resection Zollinger-Ellison syndrome Octreotide scan R/O MEN 1
8. Patients taking nonsteroidal anti-in ammatory drugs (NSAIDs) or aspirin need concomitant acid suppressing medication i which o the ollowingispresent?
A. Ageover 50
B. Heavysmokinghistory
C. Concurrent steroid intake
D. Heavyalcoholconsumption
Answer:C
he overallrisk o signi icant serious adverse gastrointestinal (GI)eventsin patientstakingnonsteroidalanti-in lammatory drugs (NSAIDs) is more than three times that o controls ( able26-2). hisrisk increasesto ivetimesin patientsolder than 60 years. Factors that clearly put patients at increased risk or NSAID-induced GI complications include age >60, prior GI event, high NSAID dose, concurrent steroid intake, and concurrent anticoagulant intake. Alcohol is commonly mentioned as a risk actor or peptic ulcer disease (PUD), but con irmatory data are lacking. High doses o H2 blockers havebeen shown tobelesse ectivethan proton pump inhibitors (PPIs) in preventing GI complications in these high risk patientson antiplatelettherapy,butclearlytheyarebetter than noacidsuppression.(SeeSchwartz10thed., able26-6,p.1058.)
aMUCOSAand VIGORtrialsincluded onlyrheumatoid arthritispatients;CLASStrialincluded osteoarthritis(73%)and rheumatoid arthritis(27%).
bIncidence orMUCOSAtrialrepresentsdoubling o resultsprovided at 6months(although median ollow-up was<6months).Incidences orVIGORand CLASStrialsrepresent rates per100patient-years,althoughVIGORmedian ollow-up was9monthsand CLASSdata include onlythe rst 6monthso the study.
cIncludesper orations,obstructions,bleeding,and uncomplicated ulcersdiscovered on clinicallyindicated work-up.
dIncludesper oration,obstruction,bleeding (documented due to ulcerorerosionsin MUCOSAand CLASS;majorbleeding inVIGOR).
e21%o patientsin CLASSstudywere taking low-dose aspirin.
Note:Alldi erencesbetween controlsand studydrugswere signi cant except clinicalupperGIeventsin overallCLASSstudy(P= .09).
Source:Reproduced with permission rom Laine L.Approachesto nonsteroidalanti-infammatorydrug use in the high-riskpatient.Gastroenterology120:594,2001.Copyright Elsevier.
9. Te optimal initial management o a patient hospitalized or ableedingpepticulcer is
A. Ulcer oversew
B. Vagotomyand pyloroplasty
C. Distalgastrectomy
D. IntravenousPPIs
Answer:D
he management o bleeding peptic ulcer is summarized in the algorithm in Fig. 26-2. All patients admitted to hospital with bleeding peptic ulcer should be adequately resuscitated and started on continuous IV PPI. Seventy- ive percent o patientswillstopbleedingwith thesemeasuresalone,but 25% will continue to bleed or will rebleed in hospital. Among the high risk group, endoscopic hemostatic therapy is indicated and usually success ul. Only then should surgical intervention be considered, with indications including massive hemorrhage unresponsive to endoscopic control and trans usion requirement o more than our to six units o blood, despite attempts at endoscopic control. Long-term maintenance PPI therapy should be considered in all patients admitted tohospitalwith ulcer complications.(SeeSchwartz10th ed., Figure26-42,pp.1061,1064–1065,and 1069.)
206
AnnualizedIncidenceb TherapiesUsed ClinicalUpperGIEventsc ComplicatedUpperGIEventsd Studya NSAIDControl StudyDrugs Control StudyDrug Control StudyDrug MUCOSA NSAIDs(n=4439) Misoprostol200µg qid+NSAID(n=4404) 3.1% 1.6% 1.5% 0.7% CLASS Ibuproen800mgtid, dicloenac75mgbid (n=3987) Celecoxib400mgbid (n=3995) 3.5% 2.1% 1.5% 0.8% (Noaspirine:2.9%) 1.4% 1.3% 0.4% VIGOR Naproxen500mgbid (n=4047) Roecoxib50mgqd (n=4029) 4.5% 2.1% 1.4% 0.6%
TABLE26-2 Hospitalization ratesforGIeventswith and without NSAIDuse in selected large populations
FIG.26-2. Algorithm orthe treatment o bleeding pepticulcer.ASA= acetylsalicylicacid;EGD= esophagogastroduodenoscopy;IV= intravenous;OR= operating room;PPI= proton pump inhibitor;PRBC= unit o packed red blood cells;PT= prothrombin time;PTT= partial thromboplastin time;Rx= treatment.
10. Which o the ollowing options is the least pre erable reconstruction or patients undergoing antrectomy or PUD?
A. Billroth I.
B. Billroth II.
C. Roux-en-Ygastrojejunostomy.
D. Allareequallypre erable.
Answer:C
Following antrectomy, GI continuity may be reestablished with a Billroth I gastroduodenostomy or a Billroth II loop gastrojejunostomy. Since antrectomy routinely leaves a 60 to 70%gastric remnant, routine reconstruction as a Roux-en-Y gastrojejunostomyshould beavoided.Although theRoux-en-Y operation is an excellent procedure or keeping duodenal contentsout o thestomach and esophagus,in thepresenceo a largegastricremnant,thisreconstruction willpredisposeto marginalulceration and/or gastricstasis.(SeeSchwartz10th ed., p.1063.)
207 Hospitaladmission Bleedingpepticulcer Bleedingrecurs inhospital OR 20%highrisk 80%Lowrisk Bleedingstops Discharge Bleedingrecurs Bleedingpersists >4PRBCtransfused/24h Deepulcererodingbigvessel Hemodynamicinstability HemostaticRxunavailable EndoscopichemostaticRx Consultsurgeon Resuscitate ContinuousIVPPIdrip EGD LifelongPPI Test+RxH.pylori AvoidNSAIDs/ASAifpossible Yes Shock? No Yes Transfusion? No Yes ActivebleedingonEGD? No Yes VisiblevesselonEGD? No Yes AbnormalPT,PTT,orplatelets? No
11. A 55-year-old executive who is seen because of severe epigastric pain is found on esophagogastroduodenoscopyto have a large ulcer in the duodenal bulb and tests positive for H. pylori. He is treated for H. pylori and instructed to quit smoking,but hissymptomspersist and he is referred to you for further management. At this time,it would bemost appropriatetorecommend
A. NSAIDcessation and ureabreath test
B. Highlyselectivevagotomy
C. Truncalvagotomyand antrectomy
D. Truncalvagotomyand pyloroplasty
Answer:A
Theindicationsfor surgeryin PUD arebleeding,perforation, obstruction, and intractability or nonhealing. Intractability should be an unusual indication for peptic ulcer operation nowadays.Thepatientreferredforsurgicalevaluationbecause of intractable PUD should raise red flags for the surgeon: maybe the patient has a missed cancer, is noncompliant, or hasHelicobacter despitethepresenceofa negativetest or previous treatment (differential for intractability, Table 26-3). In this setting, the patient with persistent symptoms despite appropriate treatment requires further evaluation before any consideration of operative treatment. If surgery is necessary, alesser operation maybe preferable. (See Schwartz10th ed., Table26-13,pp.1059and 1069–1071.)
Cancer
Gastric
Pancreatic Duodenal
PersistentH.pyloriinfection
Testsmaybefalse-negative
Considerempirictreatment
Noncompliantpatient
Failuretotakeprescribedmedication
SurreptitioususeofNSAIDs
Motilitydisorder
Zollinger-Ellisonsyndrome
12. Which bloodgroupisassociatedwith an increasedriskof gastriccancer?
Answer:A
Gastric cancer is more common in patients with pernicious anemia, blood group A, or a family history of gastric cancer. When patients migrate from a high-incidence region to a low-incidence region, the risk of gastric cancer decreases in the subsequent generations born in the new region. This strongly suggests an environmental influence on the development of gastric cancer. Environmental factors appear to be more related etiologically to the intestinal form of gastric cancer than themoreaggressivediffuseform.Thecommonly accepted riskfactorsfor gastriccancer arelisted in Table26-4. (SeeSchwartz10th ed.,Table26-15,pp.1074–1075.)
Increaserisk
Familyhistory
Diet(highinnitrates,salt,fat)
Familialpolyposis
Gastricadenomas
Hereditarynonpolyposiscolorectalcancer
H.pyloriinfection
Atrophicgastritis,intestinalmetaplasia,dysplasia
Previousgastrectomyorgastrojejunostomy(>10yearsago)
Tobaccouse
Ménétrier'sdisease
Decreaserisk
Aspirin
Diet(highfreshfruitandvegetableintake)
VitaminC
208
A. A B. B
C. AB D. O
TABLE26-3 Diferentialdiagnosiso intractabilityor nonhealing pepticulcerdisease
TABLE26-4 Factorsincreasing ordecreasing the risko gastriccancer
13. Asubtotalgastrectomywith D2dissection per ormed or Stage3gastricadenocarcinomain theantrum includes
A. Grosslynegativemarginso 2cm
B. Morethan 15lymph nodesremoved
C. Billroth IIreconstruction
D. Splenectomy
14. Testandard treatment or an isolated3cm gastrointestinalstromaltumor (GIS )in thebodyo thestomach is
A. Imatinib
B. Endoscopicablation
C. Wedgeresection
D. Subtotalgastrectomy
Answer:B
Surgical resection is the only curative treatment or gastric cancer and most patients with clinically resectable locoregional disease should have gastric resection. he standard operation or gastric cancer is radical subtotal gastrectomy, which entails ligation o the le t and right gastric and gastroepiploicarteriesat theorigin,aswellastheen blocremovalo thedistal75%o thestomach,includingthepylorusand 2cm o duodenum, the greater and lesser omentum, and all associated lymphatic tissue. Generally, the surgeon strives or a grosslynegativemargin o atleast5cm.Morethan 15resected lymph nodes are required or adequate staging, even in the low-risk patient. he operation is deemed an adequate cancer operation provided that tumor- ree margins are obtained, >15lymph nodesareremoved,andallgrosstumor isresected. In the absence o involvement bydirect extension, the spleen and pancreatictailarenot removed.Reconstruction isusually byBillroth Igastrojejunostomyor Roux-en-Yreconstruction. (SeeSchwartz10th ed.,p.1081.)
Answer:C
Gastrointestinal stromal tumors (GIS s) are submucosal tumors that are slow growing, and arise rom interstitial cells o Cajal (ICC). Prognosis in patients with GIS s depends mostlyon tumor sizeand mitoticcount,and metastasis,when it occurs, is typically by the hematogenous route. Any lesion >1cm can behavein amalignant ashion and mayrecur. hus, all GIS s are best resected along with a margin o normal tissue—wedge resection with clear margins is adequate surgical treatment. rue invasion o adjacent structures by the primary tumor is evidence o malignancy. I sa e, en bloc resection o involved surrounding organs is appropriate to remove all tumor when the primary is large and invasive. Five-year survival ollowingresection or GIS is about 50%. Most patients with low-grade lesions are cured (80% 5-year survival), but most patients with high-grade lesions are not (30% 5-year survival). Imatinib, a chemotherapeutic agent that blockstheactivityo thetyrosinekinaseproduct o c-kit, yields excellent results in many patients with metastatic or unresectable GIS , and is also recommended in high risk groups as an adjuvant therapy. Fig. 26-3 shows an algorithm or treatment o patients with GIS (See Schwartz 10th ed., Figure26-59,pp.1085–1086.)
209
Primarylocalizeddisease Recurrentormetastaticdisease Imatinib
Surgery* Embolization* RFA* Sunitinib Othernewagents Diffuse progression Focal progression Response/ stabledisease Unresectableorresection requiringextensivesurgery orriskoforgandysfunction
Imatinib Continueimatinib Stillunresectable
15. Whicho the ollowingoptionsisthebestmanagemento alow-gradegastriclymphomao thegastricantrum?
A. H.pylorieradication
B. Chemotherapy± radiation therapy
C. Wedgeresection
D. Antrectomy
imatinib-resistantdisease istreatable
Answer:A
210
Low-grade mucosa-associated lymphoid tissue (MAL ) lymphoma, essentially a monoclonal proli eration o B cells, presumably arises rom a background o chronic gastritis associated with H. pylori. hese relatively innocuous tumors then undergo degeneration to high-grade lymphoma, which istheusualvarietyseen bythesurgeon.Remarkably,when the H. pylori are eradicated and the gastritis improves, the lowgrade MAL lymphoma o ten disappears. hus, low-grade MAL lymphoma is not a surgical lesion. An algorithm or gastric lymphoma treatment is ound in Fig. 26-4. (See Schwartz10th ed.,Figure26-58,pp.1084–1085.) Resectable Surgery Postopimatinib
Considerresectionofprimary withminimalmetastaticdisease especiallywhensymptomatic
*Ifallgrossdiseaseorall
RFA=radiofrequencyablation
FIG.26-3. Algorithm orthe treatment o gastrointestinalstromaltumor.(Reproduced with permission rom Gold JS,DeMatteo RP, Combined surgicaland moleculartherapy:Thegastrointestinalstromaltumormodel.Ann Surg 244:176,2006.)
Confinedtogastricwall Not(11:18)translocation
H.pylorieradicationtherapy Re-evaluateat12mo
Lymphomaregression
Low-grade(indolent)MALT
Lymphnodeinvolvement t(11:18)translocation
H.pylorieradicationtherapy Re-evaluateat3–6mo
Lymphomapersists
StageIIIorIV
H.pylorieradicationtherapy andchemo**+/–XRT*
Closefollow-up
StageII Chemo**+XRT* StageI XRT*
*XRT:externalbeamradiationtherapy,approximately30Gywith10Gyboost **Chemo:chemotherapyregimensincludechlorambucil,fludarabinel, andcyclosphosphamide,vincristine,prednisone(COP)+/–rituximab
High-grade(aggressive)
StageI,II,III Chemo*+XRT**
StageIV Chemo*+/–XRT**
Follow-up
Furtherchemotherapy
*Chemo:chemotherapyregimenusuallycyclophosphamide, doxorobicin,vincristine,prednisone(CHOP)+/–rituximab **XRT:externalbeamradiation,approximately30Gywith10Gyboost
16. ypeIIIgastriccarcinoid tumors
A. O en donot requireresection
B. Areassociated with hypergastrinemia
C. Aresporadiclesions
D. Havebetter outcomesthan typeIand IItumors
Answer:C
Surgery
ypeIIIgastriccarcinoidsaresporadictumors,mosto ten solitary(usually>2cm),occurmorecommonlyinmen,andbehave more aggressively than types I and II. Unlike types 1 and II, they are not associated with hypergastrinemia. ype I gastric carcinoidsarethemost common typeo gastriccarcinoid,and occur in patients with chronic hypergastrinemia secondary to pernicious anemia or chronic atrophic gastritis. ype II is rare,and isassociated with MEN1and ZES.Gastriccarcinoids shouldallberesected,andsmalllesions(<2cm)con inedtothe mucosamaybetreated endoscopicallywith endoscopicmucosal resection (EMR) i there are only a ew lesions (<5) and i margins are histologicallynegative. Locallyinvasive lesions, or those >2 cm, should be removed by radical gastric resection and lymphadenectomy. Survival is excellent or node-negative patients (>90% 5-year survival); node-positive patients have
211
Noresidualdisease Residualdisease
FIG.26-4. Algorithm orthe treatment o gastriclymphoma.MALT= mucosa-associated lymphoid tissue.(Reproduced with permission rom Yoon SS,Coit DG,PortlockCS,et al,Thediminishingroleofsurgeryinthetreatmentofgastriclymphoma.Ann Surg 240:28,2004.)
17. Watermelon stomach isbest treated by
A. Acid-reducingagents
B. Betablockers
C. Antrectomy
D. otalgastrectomy
a 50% 5-year survival. he 5-year survival or patients with type I gastric carcinoid is close to 100%; or patients with type III lesions, the 5-year survival is less than 50%. Most type III patients have nodal or distant metastases at the time o diagnosis, and some present with symptoms o carcinoid syndrome. (See Schwartz 10th ed., p. 1086.)
Answer:C
he parallel red stripes atop the mucosal olds o the distal stomach give this rare entity its name. Histologically, gastric antral vascular ectasia (GAVE) is characterized by dilated mucosal blood vessels that o ten contain thrombi, in the lamina propria. Mucosal ibromuscular hyperplasia and hyalinization o ten arepresent (Fig.26-5). hehistologicappearance can resemble portal hypertensive gastropathy, but the latter usually a ects the proximal stomach, whereas watermelon stomach predominantlya ectsthedistalstomach.
Beta blockers and nitrates, use ul in the treatment o portal hypertensive gastropathy, are ine ective in patients with gastricantralvascular ectasia.Patientswith GAVEareusually elderly women with chronic GI blood loss requiring trans usion. Most have an associated autoimmune connective tissue disorder, and at least 25%have chronic liver disease.Nonsurgical treatment options include estrogen and progesterone, and endoscopic treatment with the neodymium yttriumaluminum garnet (Nd:YAG)laser or argon plasmacoagulator. Antrectomy may be required to control blood loss, and this operation is quite e ective but carries increased morbidityin this elderly patient group. Patients with portal hypertension and antralvascular ectasia should beconsidered or transjugular intrahepaticportosystemicshunt ( IPSS).(SeeSchwartz 10th ed.,Figure26-61,pp.1088–1089.)
212
FIG.26-5. Gastricantralvascularectasia (watermelon stomach). (Reproduced with permission rom Goldman H.Mucosal hypertrophyand hyperplasia o the stomach,in Ming S-C,Goldman H,eds.PathologyoftheGastrointestinalTract,2nd ed.Baltimore: Williams&Wilkins;1998,p.548.)
18. reatment or severe early dumping a er gastrectomy that ispersistent despitean antidumpingdiet and fber is
A. Expectant management
B. Oralglucose or symptoms
C. Octreotide
D. Surgicalconversion toaRoux-en-Ydrainage
Answer:C
Dumping is a phenomenon consisting o a constellation o postprandial symptoms thought to be the result o the abrupt delivery o a hyperosmolar load into the small bowel due to ablation o thepylorusor decreased gastriccompliance.Early dumping occurs 15 to 30 minutes a ter a meal, with patients becoming diaphoretic, weak, light-headed, and tachycardic. Late dumping occurs hours later, and is due to a reactive hypoglycemia. Late dumping is relieved by the administration o sugar. hemedicaltherapy or thedumpingsyndrome consists o dietary management and somatostatin analogue (octreotide). O ten, symptoms improve i the patient avoids liquids duringmeals. Hyperosmolar liquids (eg, milk shakes) maybeparticularlytroublesome. hereissomeevidencethat adding dietary iber compounds at mealtime may improve the syndrome. I dietary manipulation ails, the patient is started on octreotide,100µgsubcutaneouslytwicedaily. his can be increased up to 500 µg twice daily i necessary. he long-actingdepotoctreotidepreparation isuse ul.Octreotide not only ameliorates the abnormal hormonal pattern seen in patients with dumping symptoms, but also promotes restoration o a asting motility pattern in the small intestine (ie, restoration o the migrating motor complex [MMC]). Only a very small percentage o patients with dumping symptoms ultimatelyrequire surgery. here ore, the surgeon should not rush to re-operate on the patient with dumping syndromes. (SeeSchwartz10th ed.,p.1091.)
19. Ménétrier diseaseischaracterized by
A. Hypertrophicgastric oldsand hypoproteinemia
B. A tortuous submucosal congenital arteriovenous mal ormation
C. Gastricantralvascular ectasia
D. Epithelialhyperplasiaand hypergastrinemia
Answer:A here are two clinical syndromes characterized by epithelial hyperplasia and giant gastric olds: ZES and Ménétrier disease. he latter is characteristically associated with proteinlosing gastropathy and hypochlorhydria. A ew patients with theseunusualdiseaseshavebeen success ullytreated with the epidermal growth actor receptor blocking monoclonal antibody cetuximab. here may be an increased risk o gastric cancer with this disease, and gastric resection may be indicated or bleeding, severe hypoproteinemia, or cancer. he other options describe Dieula oy lesions, watermelon stomach,and ZES,respectively.(SeeSchwartz10th ed.,p.1088.)
213
The SurgicalManagement ofObesity
1. What body mass index (BMI or weight(kg)/height [m2]) de nition o obesity serves as the standard indication or bariatric surgery when medical therapy has ailed and comorbid conditionsexist?
A. Overweight (BMI25.0–29.9)
B. ClassIobesity(BMI30.0–34.9)
C. ClassIIobesity(BMI35.0–39.9)
D. ClassIIIobesity(BMI40.0and greater)
2. Which o the ollowing bariatric procedures is primarily intended to induce weight loss through malabsorption o ingested nutrients?
A. Adjustablegastricband
B. Sleevegastrectomy
C. Roux-en-Ygastricbypass(RYGB)
D. Duodenalswitch
3. Complications o adjustable gastric banding which have diminished its popularityas a bariatric procedure include allo the ollowingEXCEP
A. Mortalityrisk
B. Slippageo theband
C. Failuretoloseweight
D. Port and tubingcomplications
4. Earlypostoperativecomplicationsa er aRYGBprocedure includeallo the ollowingEXCEP
A. Anastomoticleak rom astapleline
B. Dilation o thedistalgastricremnant
C. Pulmonaryembolus
D. Hyperglycemia
Answer:C
A 1991 NIH Consensus Con erence recommended that bariatric surgery was indicated or a body mass index (BMI) o 35.0–39.9when medicaltherapyhas ailedandcomorbidconditionsexist.When nocomorbid conditionsexist,aBMIo 40 or greater is required. his standard continues to be used by insurers,although recent studies have shown bene it in lower BMIgroups.(SeeSchwartz10th ed.,p.1100.)
Answer:D
Bariatric procedures are classi ied as restrictive, malabsorptive, or a combination o restrictive and malabsorptive in the mechanism o weight loss that they induce. Restrictive operations include adjustable gastric band and gastric sleeve, malabsorptive operations include duodenal switch and biliopancreatic diversion, and combined restrictive and malabsorptive procedures include the Roux-en-Y gastric bypass (RYGB).(SeeSchwartz10th ed.,p.1103.)
Answer:A
he adjustable gastric band procedure, usually per ormed laparoscopically, has the lowest cost and mortality risk o all the bariatric procedures, but is the least e ective or weight loss. In addition, slippage and erosion o the band and complications related to the maintenance and use o the port or adjustingthesizeo thebandcontributetoitslosso popularity. (SeeSchwartz10th ed.,p.1112.)
Answer:D
Early postoperative complications a ter RYGB include anastomotic leak rom a staple line, gastric remnant dilation due to downstream obstruction, intra-abdominal bleeding, and pulmonary complications such as atelectasis and pulmonary embolus.Diabetes,i present,usuallyimprovespromptlya ter RYGBand hyperglycemia is unlikely to be problematic. (See Schwartz10th ed.,pp.1116–1117.)
215 27
CHAPTER
5. Small bowel obstruction a er RYGBshould be treated as an urgent surgicalemergencybecause
A. It is requently due to an incarcerated internal hernia which can progresstobowelnecrosisand per oration
B. Abdominaldistension risksdisruption o suturelines.
C. Signs and symptoms o peritonitis, such as pain, ever, and leukocytosis,areusuallymasked in theobese.
D. Nasogastric intubation will not decompress the distal gastricremnant.
Answer:A
Small bowel obstruction a ter RYGB is requently due to an incarcerated internal hernia at the location o the closure, or lack thereo , o the mesenteric de ect. his can progress rapidly to strangulation and necrosis o the bowel with subsequent per oration. Adverse outcomes with this complication have resulted in the uni orm recommendation that small bowel obstruction in this setting should be regarded as a surgical emergency. Abdominal distention and di iculties with nasogastric intubation are not relevant concerns. (See Schwartz10th ed.,p.1117.)
6. In addition to weight loss,thebene tso RYGBincludeall o the ollowingEXCEP
A. Reduced long-term mortality.
B. Resolution o type2diabetesmellitus.
C. Resolution o sleep apnea.
D. Resolution o craving or sweets.
7. Te gastric sleeve procedure originated as part o what operation?
A. Esophagealresection
B. Billroth Igastrectomy
C. Duodenalswitch
D. Resection o thegastriccardia
8. In addition to the e ects o weight loss, the resolution o type 2 diabetes mellitus a er the gastric sleeve procedure and RYGBisthought tobecontributed toby
A. Reduced ghrelin production
B. Increasedsecretion o glucagon-likepeptide-1(GLP-1)
C. Appetitesuppression
D. Allo theabove
9. Adolescent patients with morbid obesity are increasingly being re erred or consideration o bariatric procedures dueto ailureo medicalmanagement and therisksassociated with a li etime o obesity. What nutritional de ciencies require li e-long treatment a er RYGB, the most common procedureper ormed in thisagegroup?
A. Perniciousanemiaduetovitamin B12 de ciency
B. Iron de ciencyanemia
C. De ciencieso vitaminsA,E,D,and K
D. Allo theabove
Answer:D
he long-term bene its o RYGB include a high incidence o resolution o the comorbidities o obesity, such as type 2 diabetes and sleep apnea, and a reduced mortality risk due to these and other illnesses. he success o the operation is dependent upon the patient’s success in adopting a healthy eatinghabit,despitecontinuedcravings or snacksandsweets. (SeeSchwartz10th ed.,p.1116.)
Answer:C
he irst stage o the duodenal switch procedure is creation o a gastric sleeve. his is intended to promote weight loss in morbidly obese candidates whose massive obesity creates an unacceptable risk o perioperative complications o a prolonged procedure. hegastricsleeveportion o theprocedure was seen to be so e ective that the second stage o the duodenal switch was sometimes postponed inde initely, and the gastricsleevebecameatreatment option alone.(SeeSchwartz 10th ed.,p.1121.)
Answer:D
Gastrectomy removes much o the ghrelin-producing portion o thestomach,andisthoughttocontributetoweightloss a ter both gastric sleeve and RYGB procedures. GLP-1, the enteric hormone which augments insulin release, is dramaticallyincreaseda terRYGB,andisincreaseda tergastricsleeve as well. A pro ound suppression o appetite and ood craving hasbeen ound to ollowtheseprocedures,presumablydueto thealteredhormonalstatuso peptideswhicha ectthesatiety centerso thecentralnervoussystem.(SeeSchwartz10th ed., p.1125.)
Answer:D
Loss o intrinsic actor produced in the gastric undus, impaired iron absorption, and a de iciency o the at-soluble vitamins present li e-long risks a ter RYGB and other malabsorptive bariatric procedures. Vitamin replacement and nutritional monitoring are there ore mandatory in bariatric patients.(SeeSchwartz10th ed.,p.1120.)
216
10.Te gastric sleeve procedure and RYGB result in similar degreeso resolution o allo the ollowingEXCEP
A. Gastroesophagealrefuxdisease(GERD)
B. Hypertension
C. ype2diabetesmellitus
D. Obstructivesleep apnea
Answer:A he resolution o obesity-related comorbidities a ter gastric sleeve procedures is nearly equivalent to that seen a ter RYGB, with the exception o the resolution o gastroesophagealre luxdisease(GERD).Whereasmorethan 90%o RYGB patientsreport relie o GERD,somestudiesshowan increase in GERDsymptomsa ter gastricsleevesurgery.(SeeSchwartz 10th ed.,p.1128.)
217
1. Where is the largest number o hormone-producing cells ound in thebody?
A. Tepituitary
B. Tesmallintestine
C. Tepancreas
D. Teliver
2. Which o the ollowing eatures is characteristic o the ileum,asopposed tothejejunum?
A. Tepresenceo valvulaeconniventes
B. Tepresenceo Peyer patches
C. Larger vasarecta
D. Less attymesentery
3. Within the intestine, epithelial cells originate rom stem cells, proli erate in the crypts,and migrate up the villus in 2to5days.Tisprocessreplacescellsthatareremoveddue to apoptosis or ex oliation. Tis rapid turnover makes the smallintestinesusceptibleto
A. Radiation damage
B. Starvation
C. Exogenoussteroids
D. Hypothermia
4. Apocket-orsock-likeoutpouchingon theanti-mesenteric side o the distal ileum, called a Meckel diverticulum, is caused by
A. Excessivetraction on theintestineduringchildbirth.
B. Increased intraluminalpressure.
C. Apersistent vitellineduct.
D. Amutation o thec-Mecgene.
5. How much uid normally enters the adult small intestine each day?
SmallIntestine
Answer:B
hesmallintestineisthebody’slargest reservoir o hormoneproducingcells.Multiplespecializedcellswithin theintestinal mucosa respond to luminal stimuli and secrete over 30 peptide hormones which regulate the unctions o the intestine, other organs in the gastro-entero-pancreato-biliary system, theheart,and thebrain (SeeSchwartz10th ed.,p.1145.)
Answer:B
he entire small intestine contains valvulae conniventes, also known as plicae circularis. he jejunum has larger vasa recta, a larger diameter, and a less atty mesentery. he ileum contains prominent lymphoid ollicles called Peyer patches. (See Schwartz10th ed.,p.1138.)
Answer:A
hehigh cellular turnover rateo enterocytesmakesthesmall intestine susceptible to damage by inhibitors o proli eration such as radiation and cytotoxic chemotherapy. (See Schwartz 10th ed.,p.1138.)
Answer:C
he embryonic gut communicates with the yolk sac bymean o the vitelline duct. Failure o this structure to obliterate by the end o gestation can result in a Meckel diverticulum. (See Schwartz10th ed.,p.1139.)
Answer:D
Eight to nine liters o luid enters the small intestine daily, o which over 80% is absorbed. his includes 2 L rom oral intake, 1.5 Lo saliva, 2.5 Lo gastric juice, 1.5 Lo biliopancreaticsecretions,and 1Lo luid secreted bythesmallintestine.(SeeSchwartz10th ed.,p.1140.)
219 28
CHAPTER
2L B. 4L C. 6L D. 8L
A.
6. How are the digestion products o carbohydrates, such as glucose, galactose, and ructose, absorbed through the intestine?
A. Bypassivedi usionacrossenterocyteplasmamembranes.
B. By acilitated di usion via speci c transporters such as sodium-glucosecotransporter 1(SGL 1),glucosetransporter2(GLU 2),andglucosetransporter5(GLU 5).
C. Byendocytosiso enterocyteson thevillus.
D. By acilitated di usion through tight junctionsbetween enterocytes.
7. What doesthe“enterohepaticcirculation”re er to?
A. Tesuperior mesenteric-portalvenouscircuit.
B. Tesecretion o cholesterolin thebileanditsreabsorption in thedistalileum.
C. Te secretion o bile acids by the liver and their reabsorption in thedistalileum.
D. Tesecretion o cholecystokinin bythejejunum andits stimulation o bile ow.
8. Tesecretin-glucagon amilyo gut hormonesincludesall o the ollowingstructurallyrelated peptidesEXCEP
A. Somatostatin (SS )
B. Glucose-dependent insulinotropicpolypeptide(GIP)
C. Glucagon-likepeptide-1(GLP-1)
D. Vasoactiveintestinalpolypeptide(VIP)
9. Temost common causeo smallbowelobstruction is
A. Incarcerated hernia
B. Crohn’sdisease
C. Malignancy
D. Postoperativeadhesions
10. A closed-loop obstruction is particularly dangerous because
A. Intraluminal pressure rises high enough to cause ischemiaand necrosis.
B. Teobstruction ispainless.
C. Bacterialovergrowth resultsin sepsis.
D. Te obstructive segment is not apparent on imaging studies.
Answer:B
he three terminal products o carbohydrate digestion are transported through the enterocyte brush border membrane via acilitativetransporterproteinssuchasthesodium-glucose cotransporter 1(SGL 1),glucosetransporter 2(GLU 2),and glucose transporter 5 (GLU 5). here is evidence o overexpression o thesetransporters,particularlySGL 1,in diabetes and obesity,and newtherapeutic approaches or these conditionsaredesigned to inhibit thesetransporters.(SeeSchwartz 10th ed.,p.1141.)
Answer:C
Bileacidsactasdetergentswhichincreasethesolubilityo lipid micelleswhich aretaken up bythebrush border membraneo thejejunum,whereover 90%o at isabsorbed. hebileacids themselvesremain in theintestinallumen and arereabsorbed in thedistalileum wheretheyenter theportalvenouscirculation and are re-secreted in the bile. (See Schwartz 10th ed., p.1143.)
Answer:A
Peptide hormones produced by enteroendocrine cells o the intestine are grouped into amilies based on their amino acid structural similarity. he secretin-glucagon amily o hormones includes glucose-dependent insulinotropic polypeptide (GIP), glucagon-like peptide-1 (GLP-1), vasoactive intestinal polypeptide (VIP), peptide histidine isoleucine (PHI), growth hormone-releasing hormone (GHRH), and pituitary adenylyl cyclase-activating peptide (PACAP). (See Schwartz10th ed.,p.1145.)
Answer:D
Intra-abdominal adhesions related to prior abdominal surgery accounts or 75% o cases o small bowel obstruction. Cancer-related small bowel obstruction is almost always due to extrinsiccompression or entrapment o thebowelbya primaryor metastatictumor;primarysmallbowelmalignancies arerare.(SeeSchwartz10th ed.,p.1146.)
Answer:A
A closed-loop obstruction, in which an intestinal segment is obstructed both proximally and distally, as in a volvulus, is particularly dangerous because intraluminal pressure rises quicklyand can causevenouscongestion and arterialobstruction which leads to necrosis o the intestinal wall and per oration. It classically presents with “pain out o proportion to thephysicalexam,”and isusuallyapparent on C scan which requentlyshows a U-shaped or C-shaped dilated bowel loop associated with a radial distribution o mesenteric vessels converging toward a torsion point. (See Schwartz 10th ed., p.1147.)
220
11. Terapy o a small bowel obstruction usually consists o prompt surgical correction. In patients with no evidence o closed-loopobstruction,andin whom thereisno ever or leukocytosis or tachycardia, a period o care ul observation with nasogastricdecompression maybesuccess ul in allo the ollowingconditionsEXCEP
A. Partialsmallbowelobstruction.
B. Obstruction in theearlypostoperativeperiod.
C. Obstruction duetoCrohn disease.
D. Obstruction duetoan internalhernia.
12. Interventionswhich mayreducetheincidenceand duration o postoperative ileus include all o the ollowing EXCEP
A. Epiduralanalgesia
B. Am-opioid receptor antagonist
C. Intravenouserythromycin
D. Avoiding excess intra- and postoperative uid administration
13. Risk actors or the development o Crohn’s disease includeallo the ollowingEXCEP
A. Havinga amilymember with Crohn’sdisease
B. Smoking
C. HavingChineseancestry
D. HavingAshkenaziJewish ancestry
14. Te primary genetic de ect associated with Crohn’s diseaseisa mutation o theNOD2geneon chromosome16. Tisgeneencodes or aprotein product which
A. Mediates the innate immune response to microbial pathogens
B. Activatesstellatecellstoproducecollagen
C. Regulatestherateo crypt-to-villusenterocytemigration
D. Mediates the production o enterocyte alkaline phosphatase
15. In theresection o a stenoticarea o intestinein a patient with Crohn’sdisease,thebest approach is
A. Aresection margin o 2cm rom grossdisease.
B. Aresection margin o 12cm rom grossdisease.
C. Aresection margin 2cm rom microscopicdiseaseon rozen section.
D. A resection margin 12 cm rom microscopic disease on rozen section.
Answer:D
Partial small bowel obstruction and early postoperative obstruction can mimic ileus, and may respond to nonoperative therapy. Crohn disease usually responds to medical therapy, although recurrent obstruction is an indication or surgical correction. Obstruction due to an internal hernia requires prompt surgical intervention to avoid strangulation and necrosis.(SeeSchwartz10th ed.,p.1149.)
Answer:C
Epidural analgesia (with reduced systemic narcotic administration), avoiding excess intra- and postoperative luid administration, and administration o alvimopan, a muopioid receptor antagonist, have all been associated with reduced incidence and/or duration o postoperative ileus. Prokinetic agents such as metoclopramide and erythromycin arerarelyuse ul.(SeeSchwartz10th ed.,p.1153.)
Answer:C
herisko havingCrohn’sdiseaseistwo-to our oldhigher in AshkenaziJewish amilies,15timeshigher in amilymembers o a patient with Crohn’s disease, and is increased in higher socioeconomicgroups,and amongsmokers. heincidencein China is 1% o the incidence in the United States, although thisnumber isincreasing.(SeeSchwartz10th ed.,p.1153.)
Answer:A he protein product o the NOD2 gene mediates the innate immuneresponsetomicrobialpathogens.Avarietyo de ects in immune regulatory mechanisms such as overresponsiveness o mucosal cells to enteric lora-derived antigens can lead to de ective immune tolerance and sustained in lammation.(SeeSchwartz10th ed.,p.1153.)
Answer:A
here are no di erences in the recurrence rates or resection with a 2-cm margin or a 12-cm margin rom gross disease. he additional bowel lost may contribute to eventual short gut syndrome in a patient who requires multiple resections, so minimizing bowel loss is a priority. here is no bene it to achieving rozen section negative margins in the resection o Crohn’s strictures; positive margin resections have the same recurrence rate as negative margin resections. he e ort to obtain a rozen section negative margin carries the risk o removing more intestine than is necessary. (See Schwartz 10th ed.,p.1157.)
16. Te ailureo an enterocutaneous stulatohealon aregimen o total parenteral nutrition and antisecretory therapymaybeduetowhich o the ollowing?
A. A oreign bodyin the stulatract.
B. Epithelialization o the stulatract.
C. Downstream obstruction o the stulized segment o intestine.
D. Allo theabove.
Answer:D
Factors which prevent healing o an enterocutaneous istula include oreign body, epithelialization o the istula tract, downstream obstruction,radiation enteritis,associated in ection (abscess or sepsis), malignancy, and a short (<2 cm) istulatract.(SeeSchwartz10th ed.,p.1158.)
221
17. Which primarymalignancyo thesmallintestineismost common?
A. Adenocarcinomao theduodenum
B. Carcinoid tumor o theileum
C. Lymphomao thejejunum
D. Gastrointestinal stromal tumor (GIS ) o the duodenum
18. Adenocarcinoma o the duodenum is associated with what hereditaryoncologicsyndrome?
A. Hereditarynonpolyposiscolorectalcancer (HNPCC)
B. Familialadenomatouspolyposis(FAP)
C. Peutz-Jegherssyndrome
D. Von Hippel-Lindau (VHL)syndrome
19. Which o the ollowing statements is true regarding GIS s?
A. Most occur in thesmallintestine.
B. GIS sareusuallymetastaticwhen rst diagnosed.
C. GIS stypicallypresent with GIhemorrhage.
D. GIS sareusuallyresponsivetocytotoxicchemotherapy.
Answer:A
Adenocarcinomas o the duodenum are the most common primary small bowel malignancy and account or 35 to 50% o the total. Lymphoma and gastrointestinal stromal tumors (GIS s) o the small bowel are the least common and each accounts or 10 to 15% o the total. (See Schwartz 10th ed., p.1159.)
Answer:B
Duodenal carcinoma is a late mani estation o the amilial adenomatous polyposis (FAP) syndrome. A ter resolution o thecolonicdiseasebytotalcolectomy,patientswith FAPmust be ollowed with periodic upper gastrointestinal (GI) endoscopytomaintain surveillance or duodenaltumors.Duodenal cancer isthe leadingcause o death amongpatientswith FAP. (SeeSchwartz10th ed.,p.1158.)
Answer:C
GIS s are a orm o sarcoma which occur most commonly (70%) in the stomach. hey more requently present with GI hemorrhage than other small bowel malignancies. hey are usually re ractory to conventional cytotoxic chemotherapy, but are not usually metastatic on initial diagnosis. A radical lymphadenectomyis not usuallyrequired;a segmental resection o the involved portion o the small intestine is usually su icient surgicaltreatment.(SeeSchwartz10th ed.,p.1162.)
20. Methodsto prevent radiation enteritiso thesmallbowel duringpelvicirradiation or gynecologicor rectalmalignancyincludewhich o the ollowing?
A. ilt table positioning in rendelenburgposition duringradiation therapytreatments.
B. Closure (reapproximation) o the pelvic peritoneum afer primaryresection.
C. Placement o an absorbable mesh sling to suspend small intestine out o the pelvis duringpostoperative radiation therapy.
D. Allo theabove.
21. What ectopic tissue is commonly ound in a Meckel diverticulum?
A. Gastricmucosa
B. Ectopicpancreas
C. Splenic ollicles
D. Ovarian ollicles
22. A patient with recent onset o ascites afer an episode o acute pancreatitis undergoes paracentesis, which reveals cloudywhite uid.What therapyisindicated?
A. Surgicalexploration
B. Low- at diet
C. otalparenteralnutrition ( PN)and octreotide
D. Octreotideand weeklyparacentesis
Answer:D
Inadditiontolimitingradiationexposuretolessthan5000cGy, avoiding radiation to the small intestine a ter pelvic surgery can involve steep rendelenburg positioning during radiation therapy sessions, closure o the pelvic peritoneum at the level o the sacral promontory to prevent small bowel illing thepelvis,and creatingo an absorbablemesh slingtoprevent thesmallintestine rom illingthepelviccavity.(SeeSchwartz 10th ed.,p.1163.)
Answer:A
Approximately60%o Meckeldiverticula contain ectopictissue, o which over 60%consists o gastric mucosa. Pancreatic acini are next most common, ollowed by pancreatic islets, endometriosis, and hepatobiliarytissues.Gastric mucosa can ulcerateand bleed,theetiologyo which can behard to determineunlesstheMeckeldiverticulum isknown.(SeeSchwartz 10th ed.,pp.1163–1164).
Answer:C
Chylous ascites can develop as a complication o operative procedures or in lammatory conditions such as acute pancreatitis. Lymphatic drainage rom damaged lymphatics can heal when the patient is made NPO, and maintained on PN and octreotide. Medium-chain triglycerides have been advocated as an oral diet, but temporary cessation o oral eeding and octreotide comprise the most success ul therapy. (See Schwartz10th ed.,pp.1169–1170.)
222
23. Short bowel syndrome has been arbitrarily de ned in adultsashavingasmallintestineo lessthan whatlength?
A. 300cm
B. 200cm
C. 100cm
D. 50cm
24. Common causes o short bowel syndrome include all o the ollowingEXCEP
A. Mesentericischemia
B. Malignancy
C. Crohn’sdisease
D. Radiation enteritis
25. Afer an emergency operation or bowel in arction in which morethan hal o the smallintestinewasremoved and a jejunostomy created, high volume ostomy losses cause recurrent dehydration.Management o thiscondition includeswhich o the ollowing?
A. Proton pump inhibitors or histamine-2 receptor antagonists
B. Octreotide
C. Loperamide
D. Allo theabove
Answer:B
A unctional de inition, in which insu icient absorptive capacity results in diarrhea, dehydration, and malnutrition is more appropriate, but a standard de inition o short bowel syndrome o 200 cm has been used widely. (See Schwartz 10th ed.,p.1171.)
Answer:D
In adults, the common etiologies o short bowel syndrome include mesenteric ischemia, malignancy, and Crohn’s disease. In pediatric patients, common causes include intestinal atresias, volvulus, and necrotizing enterocolitis. Radiation enteritis usually involves isolated segments o small bowel o lessthan 50%o total small intestinal length. (See Schwartz 10th ed.,p.1171.)
Answer:D
Reducing gastric secretion with proton pump inhibitors or histamine-2 receptor antagonist, reducing gastroenteropancreatic secretions with octreotide, and inhibiting motility with agents such as loperamide or diphenoxylate, are use ul approaches to prevent dehydration as the short gut adapts to itsnewlength. otalparenteralnutritionisalsoo tenrequired. (SeeSchwartz10th ed.,p.1171.)
223
1. A74-year-old man with biopsy-proven rectal adenocarcinomaisundergoingalowanterior resection.Which layers must bestapled through when resectingthedistalportion o resection specimen?
A. Mucosa, submucosa, circular muscle layer, longitudinalmusclelayer,and serosa
B. Mucosa, submucosa, longitudinal muscle layer, circular musclelayer,and serosa
C. Mucosa, submucosa, longitudinal muscle layer, and circular musclelayer
D. Mucosa,submucosa,circular musclelayer,and serosa
2. Which layer o muscle joins together to orm the internal analsphincter?
A. Circum erentialmusclelayer
B. Longitudinalmusclelayer
C. Puborectalismuscle
D. Circular musclelayer
3. A24-year-old woman with medicallyre ractoryulcerative colitis decides to undergo a total colectomy. During this procedure,wherewould it bemost appropriateto look or thein erior mesentericvein in order toligateit?
A. Look or thein erior mesentericartery;theveinso the colon usuallyparallelwith thecorrespondingarteries.
B. Tein erior mesentericarterycan beligatedwithin the peritoneum, where it joins with the superior mesentericartery.
C. Tein erior mesentericvein iso en ligated at thein erior edgeo thepancreas,just belowwhereit joinswith thesplenicvein.
D. Tein erior mesentericvein willnot beligated or this procedure.
Colon,Rectum,and Anus
Answer:C
he wall o the colon and rectum are made o ive separate layers: mucosa, submucosa, circular muscle layer, longitudinal muscle layer, and serosa. he mid and lower rectum lack serosa so this layer would not be stapled through i the surgeon were stapling through the mid or lower rectum. (See Schwartz10th ed.,p.1176.)
Answer:A
he inner circular muscle joins to orm the internal anal sphincter. he subcutaneous, super icial, and deep external sphincter surroundsit. hedeep externalanalsphincter isan extension o the puborectalis muscle. (See Schwartz 10th ed., p.1176.)
Answer:C
he in erior mesenteric vein does not run with the in erior mesentericartery.Instead,it travelscraniallyin theretroperitoneum over the psoas and then posterior to the pancreas to join the splenic vein. he vein is o ten ligated at the in erior edge o the pancreas during a colectomy. (See Schwartz 10th ed.,p.1177.)
225 29
CHAPTER
4. An anatomy class is dissecting out the rectum. What are the correct ascial arrangements that they will encounter duringthisdissection?
A. Tepresacral asciaseparatestherectum rom thepresacral venous plexus and the pelvic nerves; Waldeyer ascia extends orward and downward and attaches to the ascia propria at the anorectal junction. Denonvilliers ascia separates the rectum rom the prostate and seminalvesiclesin men and romthevaginain women.
B. Te presacral ascia extends orward and downward and attaches to the ascia propria at the anorectal junction; Waldeyer ascia separates the rectum rom theprostateand seminalvesiclesin men and rom the vagina in women; Denonvilliers ascia separates the rectum rom the presacralvenousplexusand thepelvicnerves.
C. Te presacral ascia separates the rectum rom the prostate and seminal vesicles in men and rom the vaginain women;Waldeyer asciaextends orwardand downwardandattachestothe asciapropriaattheanorectal junction; Denonvilliers ascia separates the rectum rom the presacral venous plexus and the pelvic nerves.
D. Te presacral ascia separates the rectum rom the prostate and seminal vesicles in men and rom the vagina in women; Waldeyer ascia extends backward and downward and attachesto the asciapropriaat the anorectal junction; Denonvilliers ascia separates the rectum rom thepresacralvenousplexusandthepelvic nerves.
5. Choosethecorrect de nition o intestinalmalrotation
A. At the 4th week o gestation the midgut herniates through the abdominal cavity, rotates 270° clockwise around the superior mesenteric artery and then travels toitsrestingplaceintheabdomenduringthe10thweek.
B. At the 4th week o gestation the midgut herniates through the abdominal cavity, rotates 270° counterclockwise around the superior mesenteric artery and then travels to its restingplace in the abdomen during the12th week.
C. At the 6th week o gestation the midgut herniates through theabdominalcavity,rotates270°clockwise around thesuperior mesentericarteryand then travels to its resting place in the abdomen during the 12th week.
D. At the 6th week o gestation the midgut herniates through the abdominal cavity, rotates 270° counterclockwise around the superior mesenteric artery and then travels to its restingplace in the abdomen during the10th week.
Answer:A
he presacral fascia separates the rectum rom the presacral venous plexus and the pelvic nerves. he rectosacral ascia (Waldeyer fascia) extends orward and downward and attaches to the ascia propria at the anorectal junction. Anteriorly, Denonvilliers fascia separates the rectum rom the prostate and seminal vesicles in men and rom the vagina in women.(SeeSchwartz10th ed.,p.1177.)
Answer:D
During the 6th week o gestation, the midgut herniates out o the abdominal cavity and rotates 270° counterclockwise around the superior mesenteric artery and travels to its restingposition in the 10th week o gestation.Failure o the midgut to rotate and return to the abdominal cavity during the 10th week o gestation results in intestinal malrotation. (See Schwartz10th ed.,pp.1175,1179.)
226
6. A62-year-oldmanhasper orateddiverticulitisandundergoesan emergent le hemicolectomywith adivertingloop ileostomy.I hehasahigh output ileostomyandisat risko diversion colitis,which attyacidsarenot beingabsorbed?
A. Butyricacid and propionicacid
B. Propionicacid and palmiticacid
C. ricosicacid and butyricacid
D. Lauricacid and palmiticacid
7. A 58-year-old mother o 10 su ers rom ecal incontinence. Usually, de ecation occurs by increased intraabdominal pressure via the Valsalva maneuver, increased rectalcontraction,and relaxation o thepuborectalismuscle, which orms a “sling”around the distal rectum, ormingarelativelyacuteanglethatdistributesintra-abdominal orces onto the pelvic foor. With de ecation, this angle straightens, allowing downward orce to be applied along the axis o the rectum and anal, and opening o the anal canal. A dys unction at which point o this pathway can lead to ecalincontinence?
A. Injurytothepuborectalis.
B. Decreased rectalcontraction.
C. Repair o the internal or external sphincter during delivery.
D. Hypertrophicinternaland externalanalsphincters.
8. A healthy48-year-old physician with no amilyhistoryo cancer and who strictly adheres to a high protein, high ber diet,exercises vetimesper week or 50minutes,and takes vitamin C supplements dailyper orms a ecaloccult blood test (FOB )on hersel and testspositive.Should she haveany urther colon screening?
A. No,vitamin Ccan producea alse-positiveresult
B. Yes, all positive FOB requires urther investigation with acolonoscopy.
C. Yes, all positive FOB requires urther investigation with FOB in 1year.
D. No,shehasno risk actors or colon cancer and should ollowthe USPS F screening guidelines or colorectal cancer.
9. A22-year-old collegestudent presentsto clinicwith a history o intermittent diarrhea or the past 5 days a er returning rom Mexico. On urther questioning, she has had previous episodes o diarrhea or the past 2 years, unrelated to travel.A er a physicalexamination,what are appropriate tests that should be ordered to appropriately workup thispatient?
A. Stoolwet-mount and stoolculture.
B. Sigmoidoscopyand colonoscopy,but onlyi noperitonealsignson physicalexamination.
C. Add Sudan red tostoolsample.
D. Allo theabove.
Answer:A
Short-chain atty acids (acetic acid, butyric acid, and propionic acid) are produced by bacterial ermentation o dietary carbohydrates and are an important source o energy or the colonic mucosa, and metabolism by colonocytes provides energy or processes such as active transport o sodium. Diversion o eces by an ileostomyor colostomy can result in “diversion colitis” which is associated with mucosal atrophy and in lammation.(SeeSchwartz10th ed.,p.1179.)
Answer:A
De ecation proceeds by coordination o increasing intraabdominal pressure via the Valsalva maneuver, increased rectal contraction, relaxation o the puborectalis muscle, and opening o the anal canal. Impaired continence may result rom poor rectal compliance, injury to the internal and/or external sphincter or puborectalis, or nerve damage or neuropathy.(SeeSchwartz10th ed.,p.1180.)
Answer:B
Fecaloccult blood test (FOB ) hasbeen a nonspeci ictest or peroxidase contained in hemoglobin; consequently, occult bleeding rom anygastrointestinalsourcewillproduceapositive result. Similarly, many oods (red meat, some ruits and vegetables,and vitamin C)willproducea alse-positiveresult. AnypositiveFOB mandates urther investigation,usuallyby colonoscopy.(SeeSchwartz10th ed.,p.1182.)
Answer:D
Stool wet-mount and culture can o ten diagnose in ection. Sigmoidoscopy or colonoscopy can be help ul in diagnosing in lammatory bowel disease or ischemia. However, i the patient has abdominal tenderness, particularly with peritoneal signs, or any other evidence o per oration, endoscopy is contraindicated. For chronic diarrhea tests or malabsorption and metabolic investigations should be conducted along with colonoscopy.Biopsiesshould betaken even i thecolonic mucosa appears grossly normal. (See Schwartz 10th ed., p.1184.)
227
10. A 76-year-old man undergoes an emergent sigmoidectomy or a per orated colon mass. Te surgeon per orms a Hartmann procedure and brings up a colostomy. In an emergency setting, where is the most appropriate location toseat acolostomy?
A. Abovethebeltline,within therectusabdominusmuscle,away rom thecostalmargin
B. Belowthebeltline,within therectusabdominusmuscle,near theiliaccrest
C. Abovethebeltline,within therectusabdominusmuscle,near thecostalmargin
D. Belowthebeltline,within therectusabdominusmuscle,away rom theiliaccrest
11. Apreviouslyhealthy46-year-oldwoman withahistoryo rectal adenocarcinoma, rst discovered on colonoscopy
1year agowhoisstatuspost lowanterior resection with a diverting loop ileostomy returns to clinic 3 months a er herlowanteriorresection orapreoperativeappointment or her ileostomyreversal.Over thepast3monthsshehas had good ileostomy output as well as occasional loose stools per rectum. What workup does she need to have prior toileostomyreversal?
A. Adigitalrectalexamination to palpatetheanastomosisand check or patency.
B. No examination is needed as this was a simple diversion and shehascontinued topassstoolper rectum.
C. A fexible sigmoidoscopyor contrast enema to check or patency.
D. A colonoscopy to evaluate or polyps not previously seen on previouscolonoscopy.
12. A 75-year-old woman undergoes a right hemicolectomy and end ileostomy or right-sided per orated diverticulitis. What is the most concerningadverse outcome in the short term o thisprocedureand willrequirerevision?
A. Skin breakdown caused bysuccusentericus.
B. Stomanecrosisabovethelevelo the ascia.
C. Stomanecrosisbelowthelevelo the ascia.
D. Stomalretraction belowthelevelo the ascia.
13. A 19-year-old man with medically re ractor ulcerative colitisundergoesatotalcolectomywithJ-pouchcreation. What are some o the late complications o ileal pouchanalreconstruction?
A. Morethan eight bowelmovementsper day
B. Nocturnalincontinence
C. Pouchitis
D. Smallbowelobstruction
E. Allo theabove
Answer:A
In an emergencyoperation,likethisone,wherethestomasite has not been marked, an attempt should be made to place a stomawithin therectusmuscleand away rom both thecostal margin and iliaccrest.In emergencies,placement high on the abdominal wall is pre erred to a low-lyingsite. (See Schwartz 10th ed.,p.1192.)
Answer:C
A lexibleendoscopicexaminationandacontrastenema(Gastrogra in) are recommended prior to closure to ensure that the anastomosis has not leaked and is patent. (See Schwartz 10th ed.,p.1193.)
Answer:C
Stoma necrosis may occur in the early postoperative period and is usually caused by skeletonizing the distal small bowel and/or creating an overly tight ascial de ect. Limited mucosal necrosis above the ascia may be treated expectantly, but necrosis below the level o the ascia requires surgical revision. Stoma retraction may occur early or late and may be exacerbated byobesity. Local revision maybe necessary. (See Schwartz10th ed.,p.1193.)
Answer:E
he unctional outcome o ileal pouch-anal reconstruction is not always per ect. Patients should be counseled to expect 8 to 10 bowel movements per day. Up to 50% have some degree o nocturnal incontinence. Pouchitis occurs in nearly 50%o patientswho undergo theoperation or chroniculcerativecolitis,and smallbowelobstruction iscommon.Pouches ailin 5to10%o patients.(SeeSchwartz10th ed.,p.1194.)
228
14. A 50-year-old woman who underwent a total colectomy with ilealpouch-analreconstruction 5yearsago presents to the emergency room with diarrhea, ever, 2 weeks o malaise, and severe abdominal pain. What is the most appropriatedi erentialdiagnosis?
A. Parasitic in ection, ulcerative colitis o the remaining rectalcu ,undiagnosed Crohn disease.
B. Bacterial or viral in ection, undiagnosed Crohn disease,and pouchitis.
C. Rectal cancer o remaining rectal cu , bacterial or viralin ection,and undiagnosed Crohn disease.
D. Parasitic in ection, bacterial or viral in ection, and pouchitis.
15. A68-year-old man is undergoinga right hemicolectomy or a cecal mass. He asks what the current research has shown about decreasingpostoperativein ection a er this procedure. When should antibiotics always be used or thisprocedure?
A. Oral antibiotics should be used in combination with bowelpreparation.
B. Parenteral antibiotic prophylaxis at the time o surgery a er the skin incision is made and redosed as needed duringtheprocedure.
C. Parenteral antibiotic prophylaxis at the time o surgerybe oretheskin incision ismade.
D. Oral antibiotics should be used postoperatively to decreaserisko anastomosisleak.
16. A22-year-old woman presentsto theclinicwith a 3-year historyo bloodydiarrhea, abdominal pain, and anorectal stulas. Her ather had similar symptoms during his 20’s and has had multiple abdominal surgeries. What is the percentage o patients with this disease who have amilymemberswith thesamedisease?
A. 5–10%
B. 10–20%
C. 10–30%
D. 20–40%
17. A25-year-old man isundergoingworkup to determinei he hasulcerativecolitis,Crohn disease,or indeterminate colitis. What diagnostic ndings would indicate that he hasCrohn disease?
A. Atrophic mucosa, crypt abscesses, infammatory pseudopolyps,scarred and shortened colon,continuousinvolvement o rectum and colon.
B. Mucosalulcerations,noncaseatinggranulomas, brosis, strictures, and stulas in the colon with deep serpiginousulcers.
C. Atrophic mucosa, noncaseating granulomas, strictures,“cobblestone”appearanceon endoscopy.
D. Mucosal ulcerations, crypt abscesses, infammatory pseudopolyps, continuous involvement o colon and rectum.
Answer:B
his patient is likely presenting with pouchitis. Pouchitis is an in lammatorycondition that a ectsboth ileoanalpouches and continent ileostomy reservoirs. he incidence o pouchitis ranges rom 30 to 55%. Symptoms include increased diarrhea, hematochezia, abdominal pain, ever, and malaise. Diagnosis is made endoscopically with biopsies. Di erential diagnosisincludesin ection and undiagnosed Crohn disease. (SeeSchwartz10th ed.,p.1194.)
Answer:C
Prospective randomized trials are needed to better understand the role o oral antibiotic prophylaxis in colorectal surgery. In contrast, long-standing, convincing data support the e icacyo parenteral antibiotic prophylaxis at the time o surgery. Broad-spectrum parenteral antibiotic(s) with activity against aerobic and anaerobic enteric pathogens should be administered just prior to the skin incision and redosed as needed depending on the length o the operation. here is no proven bene it to usingantibioticspostoperativelya ter an uncomplicated colectomy.(SeeSchwartz10th ed.,p.1194.)
Answer:C
Family history may play a role in in lammatory bowel diseaseas10to30%o patientswith in lammatoryboweldisease report a amilymember with the same disease.(See Schwartz 10th ed.,p.1195.)
Answer:B
Ulcerative colitis is a mucosal process in which the colonic mucosa and submucosa are in iltrated with in lammatory cells. he mucosa may be atrophic, and crypt abscesses are common.Endoscopically,themucosais requently riableand may possess multiple in lammatory pseudopolyps. In longstanding ulcerative colitis, the colon may be oreshortened and the mucosa replaced by scar. A key eature o ulcerative colitisisthecontinuousinvolvement o therectum and colon; rectalsparingor skip lesionssuggest adiagnosiso Crohn disease.Crohn diseaseisatransmuralin lammatoryprocessthat can a ect anypart o thegastrointestinaltract rom mouth to anus.Mucosalulcerations,an in lammatorycellin iltrate,and noncaseating granulomas are characteristic pathologic indings. Chronic in lammation may ultimately result in ibrosis, strictures, and istulas in either the colon or small intestine. he endoscopic appearance o Crohn colitis is characterized by deep serpiginous ulcers and a “cobblestone” appearance. (SeeSchwartz10th ed.,p.1195.)
229
18. What structures are most likelyto be site o extracolonic diseasein infammatoryboweldisease?
A. Liver,biliarytree,joints,skin,eyes
B. Biliarytree,lungs,heart,spleen
C. Joints,skin,biliarytree,bladder
D. Skin,liver,pancreas,joints,eyes
Answer:A he liver is a common site o extracolonic disease in in lammatoryboweldisease.Fattyin iltration o theliver ispresent in 40to50%o patients,andcirrhosisis oundin 2to5%.Primary sclerosing cholangitis is a progressive disease characterized by intra- and extrahepatic bile duct strictures. Forty to 60% o patients with primary sclerosing cholangitis have ulcerative colitis.Pericholangitis is also associated with in lammatory bowel disease and may be diagnosed with a liver biopsy. Bile duct carcinoma is a rare complication o long-standing in lammatory bowel disease. Arthritis also is a common extracolonic mani estation o in lammatory bowel disease, and the incidence is20timesgreater than in the generalpopulation. Erythema nodosum is seen in 5 to 15% o patients with in lammatory bowel disease and usually coincides with clinicaldiseaseactivity.Pyodermagangrenosum isan uncommon but serious condition that occurs almost exclusively in patients with in lammatory bowel disease. Up to 10% o patientswith in lammatoryboweldiseasewilldevelop ocular lesions. hese include uveitis, iritis, episcleritis, and conjunctivitis.(SeeSchwartz10th ed.,p.1196.)
19. An 18-year-old woman is undergoing workup to determine i she has ulcerative colitis, Crohn disease, or indeterminate colitis. What diagnostic ndings would indicatethat shehasulcerativecolitis?
A. Atrophic mucosa, crypt abscesses, infammatory pseudopolyps,scarred and shortened colon,continuousinvolvement o rectum and colon.
B. Mucosal ulcerations, noncaseating granulomas, brosis, strictures, and stulas in the colon with deep serpiginousulcers.
C. Atrophic mucosa, noncaseating granulomas, strictures,“cobblestone”appearanceon endoscopy.
D. Mucosal ulcerations, crypt abscesses, infammatory pseudopolyps, continuous involvement o colon and rectum.
Answer:A
Ulcerative colitis is a mucosal process in which the colonic mucosa and submucosa are in iltrated with in lammatory cells. he mucosa may be atrophic, and crypt abscesses are common.Endoscopically,themucosais requently riableand may possess multiple in lammatory pseudopolyps. In longstanding ulcerative colitis, the colon may be oreshortened and the mucosa replaced by scar. A key eature o ulcerative colitisisthecontinuousinvolvement o therectum and colon; rectalsparingor skip lesionssuggest adiagnosiso Crohn disease.Crohn diseaseisatransmuralin lammatoryprocessthat can a ect anypart o thegastrointestinaltract rom mouth to anus.Mucosalulcerations,an in lammatorycellin iltrate,and noncaseating granulomas are characteristic pathologic indings. Chronic in lammation may ultimately result in ibrosis, strictures, and istulas in either the colon or small intestine. he endoscopic appearance o Crohn colitis is characterized by deep serpiginous ulcers and a “cobblestone” appearance. (SeeSchwartz10th ed.,p.1195.)
20. Tegoalso medicaltherapy or infammatoryboweldisease are to decrease infammation and alleviate symptoms.Mild tomoderatefaresaretreated in theclinicand more severe symptoms may require hospitalization. What isthe rst-linetherapy or infammatoryboweldiseasein theoutpatient setting?
A. Salicylates,such assul asalazineand 5-acetylsalicylic acid
B. Antibiotics, such as metronidazole and fuoroquinolones
C. Corticosteroids
D. Azathioprineand 6-mercaptopurine
Answer:A
Sul asalazine (Azul idine), 5-acetyl salicylic acid (5-ASA), and related compounds are irst-line agents in the medical treatment o mild to moderate in lammatory bowel disease. hese compounds decrease in lammation by inhibition o cyclooxygenase and 5-lipoxygenase in the gut mucosa. hey require direct contact with a ected mucosa or e icacy. Multiple preparations are available or administration to di erent sitesin thesmallintestineand colon.Antibioticsareo ten used to decrease the intraluminal bacterial load in Crohn disease. Metronidazole has been reported to improve Crohn colitis and perianal disease, but the evidence is weak. Fluoroquinolonesmayalsobee ectivein somecases.In theabsence o ulminant colitis or toxic megacolon, antibiotics are not used to treat ulcerative colitis. Corticosteroids (either oral or parenteral) are a key component o treatment or an acute exacerbation o either ulcerative colitis or Crohn disease.
230
21. Athin and ill appearing26-year-old man presents to the emergency department (ED) with evers, chills, severe abdominalpain,and arigid abdomen.Whiledoingahistoryandphysical,it isnoted that hehasahistoryo ulcerative colitis. What would be indications that stoma creation would bemoreappropriatethan aprimaryanastomosisin thispatient?
A. Long-standing history o ulcerative colitis with multiplecolon polyps.
B. A prealbumin o 6.0 in a patient who has been on corticosteroids.
C. Ablood glucoselevelo 300in a patient who nished acorticosteroid course3weeksago.
D. Diarrheamorethan 10timesper day or monthswith albumin o 3.6.
22. A 24-year-old woman presents to the ED with ever, severeabdominalpain with guardingon palpation,and a history o 5 days o bloody stools. She has a history o ulcerativecolitis.What aretheindications or emergency surgery or ulcerativecolitis?
A. Hemorrhage with continued decrease in hematocrit levelsin spiteo blood trans usion.
B. Hemodynamic instability requiring trans er to the ICUwith declinein statusover a48-hour period a er admission.
C. Severe abdominal pain and diarrhea that does not respond to bowel rest, hydration, and parenteral corticosteroids.
D. Cecum measured at 9 cm in diameter on computed tomography(C )scan.
E. Allo theabove.
23. A 55-year-old woman with a history o long-standing Crohn disease presents to the clinic with a 1-month history o abdominal pain and a new area o induration, fuctuance, and oul-smelling drainage rom a ormer midlineincision.What arethemostcommon indications or surgery or Crohn disease?
A. Internal stulaor abscess
B. Obstruction
C. oxicmegacolon
D. Strictures
Corticosteroidsarenonspeci icinhibitorso theimmunesystem, and 75 to 90%o patients will improve with the administration o these drugs. Azathioprine and 6-mercatopurine (6-MP) are antimetabolite drugs that inter ere with nucleic acid synthesis and thus decrease proli eration o in lammatorycells. heseagentsareuse ul or treatingulcerativecolitis and Crohn disease in patients who have ailed salicylate therapyor whoaredependent on,or re ractoryto,corticosteroids. (SeeSchwartz10th ed.,p.1196.)
Answer:B
Patients with in lammatory bowel disease are o ten malnourished. Abdominal pain and obstructive symptoms may decrease oral intake. Diarrhea can cause signi icant protein loss. Ongoingin lammation produces a catabolic physiologic state.Parenteralnutrition should bestronglyconsidered early in thecourseo therapy or either Crohn diseaseor ulcerative colitis. he nutritional status o the patient also should be considered when planning operative intervention, and nutritional parameters such as serum albumin, prealbumin, and trans errin should be assessed. In extremely malnourished patients,especiallythose who are also beingtreated with corticosteroids, creation o a stoma is o ten sa er than a primary anastomosis.(SeeSchwartz10th ed.,p.1197.)
Answer:E
Emergency surgery is required or patients with massive li ethreatening hemorrhage, toxic megacolon, or fulminant colitis who ail to respond rapidly to medical therapy. Patients with signs and symptoms o ulminant colitis should be treated aggressively with bowel rest, hydration, broad-spectrum antibiotics, and parenteral corticosteroids. Colonoscopy and barium enema are contraindicated, and antidiarrheal agents should be avoided. Deterioration in clinical condition or ailure to improve within 24 to 48 hours mandates surgery. (See Schwartz10th ed.,p.1197.)
Answer:A
he most common indications or surgery are internal fistula or abscess(30–38%o patients)and obstruction (35–37% o patients). Crohn disease o the large intestine may present as fulminant colitis or toxic megacolon. In this setting, treatment is identical to treatment o ulminant colitis and toxic megacolon secondary to ulcerative colitis. Resuscitation and medical therapy with bowel rest, broad-spectrum antibiotics, and parenteral corticosteroids should be instituted. I the patient’s condition worsens or ails to rapidly improve, total abdominal colectomy with end ileostomy is recommended. (SeeSchwartz10th ed.,p.1199.)
231
24. A23-year-old man presentsto theclinicwith severepain on de ecation that began 2monthsago.Hehastried conservative management at home with sitz baths but his pain has become so severe that he has started to restrict how much he eats to prevent having bowel movements. On rectal examination, a ssure is ound. What would indicatethat this ssure rom Crohn disease?
A. Deep and broad ulcer located in thelateralposition.
B. Shallow and broad ulcer located in the anterior position.
C. Deep and narrow ulcer located in the posterior midlineposition.
D. Shallow and narrow ulcer located in the lateral position.
25. A 65-year-old man presents to the ED with evers, abdominalpain,andbloodystools or thepast 2days.On C scan he is ound to have diverticulitis with scant ree air and a small fuid collection associated with the sigmoid colon.What istheetiologyo diverticulosis?
A. Lack o dietary ber causes smaller stools volume requiringhigher intraluminalpressure.
B. Chronic contraction causes muscular hypertrophy andcausescolon toactassegmentsrather than acontinuoustube.
C. Temucosaandmuscularismucosaherniatethrough thecolon wall.
D. Allo theabove.
26. A 72-year-old woman presents to the clinic to discuss surgicalmanagement o her long-standingdiverticulosis. What would be an indication or a colectomy in this patient?
A. Tree episodeso diverticulitis requiringhospitalization in an otherwiseasymptomaticpatient.
B. A single episode o diverticulitis in an immunosuppressed patient.
C. Acurrent episodeo complicated diverticulitisresultingin eculent peritonitis.
D. Inabilityto exclude malignancyin a patient who was recently hospitalized or her rst episode o complicated diverticulitis.
Answer:A
he most common perianal lesions in Crohn disease are skin tags that are minimally symptomatic. Fissures are also common. ypically, a issure rom Crohn disease is particularly deep or broad and perhaps better described as an anal ulcer. hese issuresare o ten multiple and located in a lateralposition rather than anterior or posterior midline as seen in an idiopathic issure in ano. A classic-appearing issure in ano located laterally should raise the suspicion o Crohn disease. (SeeSchwartz10th ed.,p.1200.)
Answer:D
he majority o colonic diverticula are false diverticula in which the mucosa and muscularis mucosa have herniated through the colonic wall. hese diverticula occur between the teniae coli, at points where the main blood vessels penetrate the colonic wall (presumably creating an area o relative weakness in the colonic muscle). hey are thought to be pulsion diverticula resulting rom high intraluminalpressure. hemost accepted theoryisthat a lack o dietary iber results in smaller stoolvolume,requiringhigh intraluminalpressure and high colonic wall tension or propulsion. Chronic contraction then results in muscular hypertrophy and development o the process o segmentation in which the colon acts like separate segments instead o unctioning as a continuous tube. As segmentation progresses, the high pressures are directed radiallytowardthecolon wallrather than todevelopment o propulsive waves that move stool distally. he high radialpressuresdirected against thebowelwallcreatepulsion diverticula.(SeeSchwartz10th ed.,p.1201.)
Answer:B
Manysurgeons now will not advise colectomyeven a ter two documented episodes o diverticulitis assuming the patient is completely asymptomatic and that carcinoma has been excluded by colonoscopy. Immunosuppressed patients are generally still advised to undergo colectomy a ter a single episode o documented diverticulitis. Medical comorbidities should be considered when evaluating a patient or elective resection,andtheriskso recurrentdiseaseshouldbeweighed against the risks o the operation. Because colon carcinoma may present in an identical ashion to diverticulitis (either complicated or uncomplicated),allpatientsmust beevaluated or malignancya ter resolution o theacuteepisode.Colonoscopyis recommended 4to 6 weeks a ter recovery. Inabilityto exclude malignancy is another indication or resection. (See Schwartz10th ed.,p.1202.)
232
27. A 63-year-old woman presents to the ED with a 2-day history o le lower quadrant abdominal pain and is oundtobe ebrileto38.6°C.Her whiteblood cell(WBC) count is 15,000.On C scan she is ound to have colonic infammation with an associated pericolic abscess. What isher Hincheystage?
A. StageI
B. StageII
C. StageIII
D. StageIV
28. A 68-year-old woman presents to the ED with a 2-day history o le lower quadrant abdominal pain and is ound tobe ebrileto39°C.Her WBCcount is12,000.On C scan she is ound to have colonic infammation with an associated retroperitoneal abscess. What is her Hincheystage?
A. StageI
B. StageII
C. StageIII
D. StageIV
29. A 62-year-old woman presents to the ED with a 2-day historyo severele lower quadrant abdominalpain and is ound to be ebrile to 39°C. On physical examination her abdomen is rigid. Her WBC count is 21,000. On C scan she is ound to have diverticula and gross intraabdominal ree air and ree fuid. She is taken to the operating room (OR) or an emergent exploratorylaparotomyand she is ound to have eculent material intraabdominally.What isher Hincheystage?
A. StageI
B. StageII
C. StageIII
D. StageIV
30. A58-year-old man presents to the clinic with a 2-month historyo the sensation o urinatingair. He has a history o diverticulitis, with his last episode requiring hospitalization being6 months ago. What are the most common stulasthat develop in complicated diverticulitis?
A. Colovaginal stulas
B. Coloenteric stulas
C. Colocutaneous stulas
D. Colovesical stulas
31. A32-year-oldman presentstotheEDwith a2-month historyo alternatingdiarrheaandconstipation,rectalbleeding, a 20-lb weight loss, and worsening atigue. What are the most common genetic mutations that could have led tothedevelopment o thispatient’scolon cancer?
A. APC,deleted in colorectalcarcinoma(DCC),p53
B. APC,BRCA1,K-ras
C. DCC,p53,and MYH geneon chromosome6p
D. MYH geneon chromosome1p,APC,K-ras
Answer:A
heHincheystagingsystemiso ten used todescribetheseverity o complicated diverticulitis: Stage I includes colonic in lammation with an associated pericolic abscess; stage II includes colonic in lammation with a retroperitoneal or pelvic abscess; stage III is associated with purulent peritonitis; and stageIVisassociated with ecalperitonitis.(SeeSchwartz 10th ed.,p.1202.)
Answer:B
heHincheystagingsystemiso ten used todescribetheseverity o complicated diverticulitis: Stage I includes colonic in lammation with an associated pericolic abscess; stage II includes colonic in lammation with a retroperitoneal or pelvic abscess; stage III is associated with purulent peritonitis; and stageIVisassociated with ecalperitonitis.(SeeSchwartz 10th ed.,p.1202.)
Answer:D
heHincheystagingsystemiso ten used todescribetheseverity o complicated diverticulitis: Stage I includes colonic in lammation with an associated pericolic abscess; stage II includes colonic in lammation with a retroperitoneal or pelvic abscess; stage III is associated with purulent peritonitis; and stageIVisassociated with ecalperitonitis.(SeeSchwartz 10th ed.,p.1202.)
Answer:D
Approximately 5% o patients with complicated diverticulitis develop istulas between the colon and an adjacent organ. Colovesical istulasaremost common, ollowed bycolovaginal and coloenteric istulas.Colocutaneous istulasarea rarecomplication o diverticulitis.(SeeSchwartz10th ed.,p.1203.)
Answer:A
Mutations may cause activation of oncogenes (K-ras) and/or inactivationoftumorsuppressorgenes(APC,deletedin colorectalcarcinoma[DCC],p53).Colorectalcarcinomaisthought to develop rom adenomatous polyps by accumulation o these mutations in what has come to be known as the adenomacarcinoma sequence. he APCgene is a tumor suppressor gene Mutations in both alleles are necessary to initiate polyp ormation. Mutation o K-ras results in an inability to hydrolyze guanosinetriphosphate(G P),thusleavingtheG-protein permanentlyin theactive orm.It isthought that thisthen leadsto uncontrolled celldivision.MYH isa baseexcision repair gene, and biallelic deletion results in changes in other downstream
233
molecules. Since its discovery, MYH mutations have been associated with an AFAP phenotype in addition to sporadic cancers. hetumor suppressor genep53hasbeen wellcharacterized in anumber o malignancies. hep53protein appears to be crucial or initiating apoptosis in cells with irreparable genetic damage. Mutations in p53 are present in 75% o colorectalcancers.(SeeSchwartz10th ed.,pp.1204–1205.)
32. A64-year-old man presents to the clinic with a 1-month history o 10-lb weight loss and rectal bleeding. His hematocrit is ound tobe27.On colonoscopyheis ound to have a large cecal mass. What risk actors should he havemodi ed toprevent development o colon cancer?
A. Eatingahigh- at,high-protein,andlow-carbohydrate diet and drinkingabottleo wineaday.
B. Eating a high- ber, low- at, and low-protein diet, drinkingasixpackso beer aday.
C. Eatingahigh- ber,low- at,andhigh-protein diet and drinkingonlyon holidays.
D. Eatinga high- at,low- ber diet and drinkingonlyon holidays.
33. A 63-year-old man undergoes a screening colonoscopy and is ound to haveapolyp in hissigmoid colon.Which typeo polyp ismost associated with malignancy?
A. ubular adenoma
B. Villousadenoma
C. ubulovillousadenoma
D. 1cm polyp
34. A50-year-old woman presentstotheED a er havingher rst colonoscopyon her birthday.Sheendorsesle lower quadrant abdominal pain and is ebrile to 38.5°C. What complications a er a colonoscopy require emergent laparotomy?
A. Smallper oration in astablepatient.
B. Hypotension and temperature o 38°C in an otherwisestablepatient.
C. Rigid abdomen and severe abdominal pain in a patient with poor bowelpreparation.
D. Findingso alarge,fat sessilepolyp.
Answer:C
Adiet high in saturated or polyunsaturated atsincreasesrisk o colorectal cancer, while a diet high in oleic acid (olive oil, coconut oil, ish oil) does not increase risk. Animal studies suggest that ats may be directly toxic to the colonic mucosa and thus may induce early malignant changes. In contrast, a diet high in vegetable fiber appears to be protective. A correlation between alcohol intake and incidence o colorectal carcinomahasalsobeen suggested.Ingestion o calcium,selenium, vitamins A, C, and E, carotenoids, and plant phenols maydecreasetherisko developingcolorectalcancer.Obesity and sedentary li estyle dramatically increase cancer-related mortality in a number o malignancies, including colorectal carcinoma. his knowledge is the basis or primary prevention strategies to eliminate colorectal cancer by altering diet and li estyle.(SeeSchwartz10th ed.,p.1204.)
Answer:B
Adenomatous polyps are common, occurring in up to 25% o the population older than 50 years o age in the United States. Byde inition, these lesions are dysplastic. he risk o malignant degeneration isrelated toboth thesizeand typeo polyp. ubular adenomas are associated with malignancy in only5%o cases,whereasvillousadenomasmayharbor cancer in up to 40%. ubulovillous adenomas are at intermediate risk (22%).Invasive carcinomas are rare in polyps smaller than 1cm;theincidenceincreaseswith size. herisko carcinomain apolyp larger than 2cm is35to50%.Although most neoplasticpolypsdonotevolvetocancer,mostcolorectalcancersoriginateasapolyp.(SeeSchwartz10th ed.,p.1205.)
Answer:C Complications o polypectomy include perforation and bleeding. A small per oration (microperforation) in a ully prepared, stable patient may be managed with bowel rest, broad-spectrum antibiotics, and close observation. Signs o sepsis, peritonitis, or deterioration in clinical condition are indications or laparotomy. Bleeding may occur immediately a ter polypectomy or may be delayed. he bleeding will usually stop spontaneously, but colonoscopy may be required to resnare a bleeding stalk or cauterize the lesion. Occasionally angiography and in usion o vasopressin may be necessary. Rarely,colectomyisrequired.(SeeSchwartz10thed.,p.1206.)
234
35. A 36-year-old woman with a complicated amily history includingher grandmother, mother,and sister who were diagnosed with colon cancer and a younger sister who was recently diagnosed with pancreatic cancer presents to the clinic or genetic counseling. What is a correct statement about thissyndrome?
A. It is the result o mismatch repair genes such as FAP and screeningcriteria look or three amilymembers who have related cancers or two amily members diagnosed be oreage50.
B. It is the result o an autosomal dominant condition and screeningcriteria look or three amilymembers with the diagnosis o adenocarcinoma or two relativesdiagnosed be oreage50.
C. It istheresult o mismatch repair genessuch asPMS2 or MSH6 and three a ected relatives with histologically veri ed adenocarcinoma o the large bowel in two successive generations o a amily with one patient diagnosed be oreage50.
D. It is the result o an autosomal dominant condition and three a ected relatives with histologically veried adenocarcinoma o the large bowel in two successive generations o a amily with one patient diagnosed be oreage50.
Answer:C
Hereditary nonpolyposis colorectal cancer (HNPCC; Lynch syndrome) is extremelyrare (1–3%o all colon cancers). he genetic de ects associated with HNPCC arise rom errors in mismatch repair,the phenotypic result beingMSI.HNPCC is inherited in an autosomaldominant pattern and ischaracterized by the development o colorectal carcinoma at an early age(averageage,40–45years).Approximately70%o a ected individuals will develop colorectal cancer. Cancers appear in the proximal colon more o ten than in sporadic colorectal cancer and have a better prognosis regardless o stage. he risk o synchronous or metachronous colorectal carcinoma is 40%. HNPCC may also be associated with extracolonic malignancies, including endometrial carcinoma, which is most common, and ovarian, pancreas, stomach, small bowel,biliary,and urinarytract carcinomas. hediagnosiso HNPCCismadebased on amilyhistory. heAmsterdam criteria or clinical diagnosis o HNPCC are three a ected relatives with histologicallyveri ied adenocarcinoma o the large bowel (one must be a irst-degree relative o one o the others) in two successivegenerationso a amilywith onepatient diagnosed be ore age 50 years. HNPCC results rom mutations in mismatch repair genes, and like FAP, speci ic mutations are associated with di erent phenotypes. For example, mutationsin PMS2or MSH6result in amoreattenuated orm o HNPCC when compared to mutations in other genes. MSH6inactivation alsoappearstobeassociated with ahigher risk or endometrialcancer.Further signi icanceo thesespeci icmutationsremainsto bedetermined.(SeeSchwartz10th ed.,p.1207.)
36. A 67-year-old man presents to the ED with a 2-month historyo nausea,emesis,20-lb weight loss,and worsening diarrhea until 4 days ago, when he stopped passing fatus and having bowel movements. A C scan shows a large obstructingright colon mass that maybe involving the omentum with two liver lesions. What should be resected in thiscase?
A. Right hemicolectomy, involved omentum, and as manyo theperitonealmassesyou can.
B. Right hemicolectomy with arterial supply and as manynodesasyou can.
C. Right hemicolectomy with arterial supply, at least 12nodes,theinvolved omentum,and resectableliver masses.
D. Don’t do a colon resection. Tis patient has distant metastasesand should bediverted or palliation.
Answer:C
he objective in treatment o carcinoma o the colon is to removetheprimarytumoralongwithitslymphovascularsupply.Becausethelymphaticso the colon accompanythe main arterialsupply,thelength o bowelresected dependson which vessels are supplying the segment involved with the cancer. Any adjacent organ or tissue, such as the omentum, that has been invaded should beresected en blocwith thetumor.I all o thetumor cannotberemoved,apalliativeprocedureshould beconsidered,although it important to notethat “debulking” is rarely e ective in colorectal adenocarcinoma. I the metastatic disease is low volume (isolated or potentially resectable liver lesions) and the resection o the primary tumor is straight orward (segmentalabdominalcolectomy),it isprobablyreasonableto proceed with resection.On theother hand, i the metastatic disease is high volume (carcinomatosis), especially i the primary tumor is minimally symptomatic, theoperation should beaborted in order to acilitateearlysystemicchemotherapy.(SeeSchwartz10th ed.,p.1212.)
235
37. A 30-year-old man presents to the ED a er a witnessed syncopal episode. He has been having bloody diarrhea and intermittent crampy abdominal pain or the past 3months.Aweeklater hehasacolonoscopyand is ound to have ulcerative colitis based on colonoscopy ndings and mucosal biopsies. Which eature o listed below is NO seen in ulcerativecolitis?
A. Teterminalileum showsinfammatorychanges.
B. Te colon is shortened and mucosa is replaced by scars.
C. Rectal sparing with infammation seen in the transverseand descendingcolon.
D. Atrophicmucosawith crypt abscesses.
Answer:C
Ulcerative colitis is a mucosal process in which the colonic mucosa and submucosa are in iltrated with in lammatory cells. he mucosa may be atrophic, and crypt abscesses are common. In long-standing ulcerative colitis, the colon may be oreshortened and the mucosa replaced byscar.Ulcerative colitis does not involve the small intestine, but the terminal ileum may demonstrate in lammatory changes (“backwash ileitis”). A key eature o ulcerative colitis is the continuous involvement o the rectum and colon; rectal sparing or skip lesions suggest a diagnosis o Crohn disease. (See Schwartz 10th ed.,p.1195.)
236
1. Teincidenceo appendectomy or acuteappendicitiswas decreasing in the United Status until the 1990s, at which point the requency o appendectomy or nonper orated appendicitis began to rise. What is one potential explanation or thisobservation?
A. Increased use o diagnostic imaging and detection o appendicitisthat otherwisewould haveresolved.
B. Increased incidence o obesityand the impact o periappendicular at on luminalobstruction.
C. Increasing incidence o in ammatory bowel disease and thepotentialmitigation o ulcerativecolitissymptomsseen with appendectomy.
D. Reimbursement patterns have changed in the United States, avoringaggressivesurgicaldecision making.
2. What imaging ndingwould excludeappendicitis?
A. Acomputed tomographic(C ) scan with a nonvisualized appendix.
B. A barium enema where a short (2 cm) appendix was clearlyidenti ed.
C. An ultrasound study with a compressible appendix that is<5mm in diameter.
D. A C scan showing an edematous but retrocecal appendix.
Answer:A
While the true reason is unknown, some have suggested that the quality and usage o diagnostic imaging in the past 20 to 30yearshasresultedin thedetection o acuteappendicitisthat would have otherwise spontaneously resolved. While appendectomy may mitigate the clinical symptoms o ulcerative colitis, this is likely not responsible or the broad reduction in observed appendectomy. Obesity is not known to impact appendicitis incidence. Reimbursement patterns should hope ully not impact surgical decision making so directly. (SeeSchwartz10th ed.,p.1243.)
Answer:C
Graded compression ultrasonography is inexpensive and rapid. he appendix is identi ied as a nonperistaltic, blind ending loop o bowel. he compressibility and anteroposterior dimensions are measured. hickening o the wall as well as peri-appendiceal luid with a noncompressible appendix are suggestive o appendicitis while an easily compressible, narrow appendix excludes the diagnosis. Failure to identi y theappendixon imagingdoesnot de initelyruleout appendicitis.A ecalithin themidappendixmayallowproximal illing o the appendix with barium in the presence o appendicitis. Sonographic sensitivity or appendicitis is 55 to 96% while speci icityis85to98%.(SeeSchwartz10th ed.,p.1245.)
3. A 25-year-old man presents with migratory right lower quadrant (RLQ) pain, leukocytosis, and a C scan consistent with acute, uncomplicated appendicitis. He is physiologically normal and it is 2 AM. You are planning an appendectomy, what di erence might be expected in his outcome i his operation is delayed until the next morning?
A. Increased risko an intra-abdominalabscess.
B. Increased risko surgical-sitein ection.
C. Decreased operativetime.
D. Increased risko per oration.
E. No di erence in per oration rates, surgical-site in ection,abscess,conversion rateor operativetime.
Answer:E
herehavebeen threeretrospectivestudiescomparingurgent versus emergent appendectomy. No di erence was ound in the incidence o complicated appendicitis,surgical-site in ections,abscess ormation,or conversion to an open procedure. While hospital length o stay was longer in the urgent group (as might be anticipated given the delay in de initive surgical care) this was not statistically or clinically di erent rom the emergent group. It may be sa e in physiologically normal patients with uncomplicated appendicitis to wait 12 to 24 hours and book them as an “urgent” case. (See Schwartz 10th ed.,p.1250.)
237 30
The Appendix
CHAPTER
4. A55-year-oldman hasC evidenceo complicatedappendicitis with a contained abscess in the RLQ. He is mildly tachycardic, a ebrile, and normotensive with ocal RLQ tenderness but no peritonitis. What is the optimal approach tothispatient?
A. Immediatelaparotomy.
B. Laparoscopicexploration and abscessdrainage.
C. Percutaneous drainage, intravenous (IV) uids, bowel rest,and broad spectrum antibiotics.
D. IV uids,bowelrest,and broad spectrum antibiotics.
Answer:C
Conservative management o the physiologically stable patient with complicated appendicitis has been shown to be associated with ewer overall complications, ewer bowel obstructions, ewer intra-abdominal abscesses, and ewer reoperations.Whilepatientswithperitonitisorhemodynamic instability should proceed to the operating room, conservative management o more stable patients with complicated appendicitisis avored. hismaynot necessarilybetruein the pediatric population, however, as two prospective randomized trials in children demonstrate equivalent or superior outcomes with early operative intervention. (See Schwartz 10th ed.,p.1251.)
5. A 23-year-old woman who is 28 weeks pregnant presents with right-sided abdominal pain, leukocytosis, and an abdominal ultrasound that does not visualize the appendix.What intervention would you recommend?
A. Exploratorylaparoscopy.
B. AbdominalC scan.
C. Abdominalmagneticresonanceimaging(MRI)scan.
D. Serialclinicalobservations.
6. A34-year-old man presentsto your clinicaskingabout an elective appendectomy. He has no history o appendicitis. What are possible indications or appendectomy in this patient?
A. Planned traveltoaremoteplacewith nosurgicalcare.
B. Patientswith Crohn diseasewherethececum is reeo grossdisease.
C. Aspart o Ladd procedure.
D. Allo theabove.
7. Whilereviewingpathologyo arecent laparoscopicappendectomy, you note that in addition to acute appendicitis, thepatienthada1.5-cmcarcinoidtumorlocatedatthebase o theappendix.Tepatientisotherwisehealthyandrecoveringwell rom surgery.What wouldyourecommend?
A. Noadditionaltherapynecessary.
B. Right hemicolectomy.
C. Radicalappendectomy.
D. Adjuvant chemotherapy.
Answer:C
Appendicitis complicates 1/766 births and is rare in the third trimester. he rate o negative appendectomyin the pregnant patient appears to be about 25%higher than in nonpregnant patients. his is not, however, a benign procedure as a negativeappendectomyisassociatedwitha4%risko etallossand a 10%risk o earlydelivery. he American College o Radiologyrecommendstheuseo nonionizingradiation techniques as ront-lineimagingin pregnantwomen.Serialexaminations would be inappropriate as rates o etal loss are considerably higher in patients with complicated appendicitis and the greatest opportunityto improve etal outcomes is to improve diagnosticaccuracy.(SeeSchwartz10th ed.,p.1256.)
Answer:D
Incidental appendectomy is generally not indicted. A ew select indications could be considered and they include children about to undergo chemotherapy, the disabled who cannot describe pain or react normally to pain, patients with Crohn diseasewhen thececum is reeo macroscopicdisease, and those patients planning to travel to remote areas with limited surgical care. While part o the traditional teaching, the ubiquity o antibiotics and the evolving understanding o our ability to treat at least some appendicitis nonoperativelymay urther limit theindications or elective,incidental appendectomy.(SeeSchwartz10th ed.,p.1257.)
Answer:B
Appendicealcarcinoid isoneo themost common neoplasms to identi y in an appendectomy specimen. Lesions that are <1 cm generally do not require additional therapy. Lesions larger than 1 or 2 cm, involving the appendiceal base or with lymph node metastasis o mesenteric invasion warrant right hemicolectomy. A radical appendectomy is not a described operation and adjuvant chemotherapy could be considered but onlya ter de initive surgical care. (See Schwartz 10th ed., p.1258.)
238
8. An 8-year-old boypresentstotheemergencydepartment complaining o generalized abdominal pain or the past 24hours.Laboratorytestsrevealaleukocytosiso 13,000 and heistender in theRLQ on physicalexamination.He is taken to the operating room or laparoscopic appendectomy. Removal o the appendix has been associated with aprotectivee ect o which o the ollowing?
A. Crohn colitis
B. Ulcerativecolitis
C. Clostridiumdifcile
D. Carcinoid
9. Which o the ollowing physical signs is associated with thecorrect de nition suggestiveo acuteappendicitis?
A. Rovsingsign:pain in theRLQ on palpation o thelef lower quadrant
B. Dunphy sign:pain in the RLQ with palpation on the lef
C. Obturator sign:pain with extension o theleg
D. Iliopsoas sign: pain on internal rotation o the right hip
Answer:B
he appendix is an immunologic organ involved in secretion o immunoglobulins. An inverse association between appendectomy and development o ulcerative colitis has been reported. Routine resection o the normal appendix to improvetheclinicalcourseo ulcerativecolitisisnot generally indicated.(SeeSchwartz10th ed.,p.1243.)
10. A29-year-old woman presentswith RLQ pain, ever,and leukocytosis.Prior to imagingstudiestheAlvarado score is used to determine the patient’s likelihood o having appendicitis. All o the ollowing variables make up the AlvaradoscoreEXCEP
A. Anorexia
B. Lef shif o neutrophils
C. Iliopsoassign
D. RLQ pain
E. Fever
Answer:A Appendicitis usually starts with periumbilical pain that migrates to the RLQ. Patients o ten have associated gastrointestinal symptoms such as anorexia, nausea, and vomiting. On physicalexamination,patientso ten pre er toremain lying supine and o ten guard due to peritoneal irritation. Rebound tenderness is when the examiner presses on the RLQ and the patient experiences a sudden pain upon removal o the hand. Rovsing sign is RLQ pain that is induced by palpation o the le t lower quadrant and is highlysuggestive o a RLQ in lammatory process. Dunphy sign elicits pain with coughing and is related to in lammation o the peritoneum. he obturator sign occurs with internal rotation o the right hip. Lastly, the iliopsoassign ispain withextension o therighthip,attributed toaretrocecalappendix.(SeeSchwartz10th ed.,p.1244.)
Answer:C he Alvarado score is the most widespread scoring system use ul or ruling out appendicitis and selecting patients or urther imaging or intervention ( able 30-1). he Alvarado score is calculated using RLQ tenderness, elevated temperature, rebound tenderness, migration o pain, anorexia, nausea/vomiting, leukocytosis, and a le t shi t on leukocyte di erentialaspredictive actors.Severalonlinecalculatorsare reelyavailable.(SeeSchwartz10th ed., able30-2,p.1245.)
239
TABLE30-1 Scoring systems
Score:<3:Lowlikelihood ofappendicitis.
Score:0–4:Lowprobability.Outpatient follow-up. 4–6:Considerfurtherimaging. 5–8:Indeterminate group.Active observation ordiagnosticlaparoscopy. ≥7:High likelihood ofappendicitis. 9–12:High probability.Surgicalexploration.
11.A 34-year-old man undergoes an unevent ul appendectomy or acute,nonper orated appendicitis.Tepathology reportnotesreads:acutein ammation witha1-cm adenocarcinomao themid appendix.Tispatient should have
A. No urther treatment
B. Chemotherapy
C. Regionalradiation
D. Right hemicolectomy
E. Ileocecectomy
12.A45-year-old woman presentswith RLQ pain.AC scan isper ormed.What ndingon C scan ismost suggestive o appendiceallymphoma?
A. Appendicealdiameter >2.5cm or surroundingsof tissuethickening
B. Lacko contrast llingtheappendix
C. 1-cm massat thebaseo theappendix
D. Prominent aorticlymph nodes
Answer:D
Primaryadenocarcinoma o theappendixisrare. hreetypes o adenocarcinoma exist; mucinous, colonic, and adenocarcinoid. he most common presentation o adenocarcinoma is acute appendicitis. he recommended treatment or all patients diagnosed with adenocarcinoma is a ormal right hemicolectomy.Patients are at risk or both synchronous and metachronousneoplasms—hal o which willoriginate in the GItract.(SeeSchwartz10th ed.,p.1258.)
Answer:A Lymphoma o the appendix is uncommon. he most common typeso appendiceallymphomain decreasingorder are non-Hodgkin, Burkitt, and leukemia. Findings on C scan includeappendicealdiameter o >2.5cm or surroundingso t tissuethickening. hemanagement iscon inedtoappendectomy.Right hemicolectomyisindicated i thetumor extends beyond the appendix onto the cecum or into the mesentery. I requiring a right hemicolectomy, postoperatively the patient will require a staging workup and possible adjuvant chemotherapy.(SeeSchwartz10th ed.,p.1259.)
240
AlvaradoScore37 AppendicitisInfammatoryResponseScore38,39 Findings Points Findings Points Migratoryrightiliacfossapain 1 Vomiting 1 Anorexia 1 Painintherightinferiorfossa 1 Nauseaorvomiting 1 Reboundtendernessormusculardefense Tenderness:rightiliacfossa 2 Light 1 Reboundtendernessrightiliacfossa 1 Medium 2 Fever≥36.3°C 1 Strong 3 Leukocytosis≥10× 109 cells/L 2 Bodytemperature ≥38.5°C 1 Shifttotheleftofneutrophils 1 Polymorphonuclearleukocytes 70%–84% 1 ≥85% 2 Whitebloodcellcount 10.0–14.9× 109 cells/L 1 ≥150× 109 cells/L 2 C-reactiveproteinconcentration 10–49g/L 1 ≥50g/L 2
1. With regard to hepatic anatomy, the alci orm ligament dividesthe_______ rom the_______.
A. Caudatelobe,quadratelobe
B. Right lobe,le lobe
C. Le medialsection,le lateralsection
D. Le medialsection,right lobe
Right triangular ligament
Answer:C
he alci orm ligament dividesthele t lateralsection rom the le t medial section. he plane between the gallbladder ossa and the in erior vena cava (IVC—re erred to as Cantlie’sline) divides the right and le t lobes. he alci orm ligament,along with the round, triangular, and coronary ligaments may be divided in a bloodless plane duringliver resection (Figs. 31-1 to31-3).(SeeSchwartz10th ed.,p.1265.)
Diaphragm
Left triangular ligament
241 31
Liver
CHAPTER
Falciform ligament
Round ligament
FIG.31-1. Hepaticligamentssuspending the liverto the diaphragm and anteriorabdominalwall.
2. Te most common variant o normal hepatic artery anatomyis
A. Replaced le hepaticartery rom thele gastricartery
B. Completelyreplaced common hepatic artery rom the superior mesentericartery(SMA)
C. Replaced right and le hepaticarteries
D. Replaced right hepaticartery rom theSMA
FIG.31-3. Couinaud liversegments(IthroughVIII)numbered in a clockwise manner.The left lobe includessegmentsIIto IV,the right lobe includessegmentsVtoVIII,and the caudate lobe issegment I. IVC= inferiorvena cava.
Answer:D
Understanding the anatomic variants o the hepatic arterial supply is important to avoid complications during liver surgery. hestandard arterialanatomyisas ollows:thecommon hepatic artery arises rom the celiac trunk, and then divides into the gastroduodenal and proper hepatic artery. In a standard con iguration, the proper hepatic artery gives rise to the right gastric artery, but this is variable. he proper hepaticarterythendividesintotherightandle thepaticartery (Fig. 31-4). However, this standard arterial con iguration only occurs in 76% o patients. he most common variants include replaced right hepaticartery rom the SMA(10–15%), replaced le t hepaticartery rom thele t gastricartery(3–10%), replaced right and le t hepatic arteries (1–2%), and the completelyreplaced common hepaticartery rom theSMA(1–2%) (Fig.31-5).(SeeSchwartz10thed.,p.1266.)
242
Rightlobe Leftlobe Rightlobe Leftlobe VIII IVa IVb II III V VII VI IVC Caudatelobe I III V VII VI II IVb
Liverinsitu
Foramenof winslow
Gastrohepatic ligament
Openhepatoduodenalligament
FIG.31-2. In situ liverhilaranatomywith hepatoduodenaland gastrohepatic ligaments.Foramen ofWinslow isdepicted.
RHA
Hepaticartery proper
Rightgastric artery
Commonhepaticartery
Gastroduodenalartery
Leftgastric artery
Celiactrunk
Splenicartery
243
LHA
FIG.31-4. Arterialanatomyofthe upperabdomen and liver,including the celiac trunkand hepaticarterybranches.a.= artery;LHA= left hepaticartery;RHA= right hepatic artery.
Replacedrighthepatic arteryfromSMA(10–15%)
Replacedlefthepaticartery fromleftgastricartery(3–10%)
Replacedrightandreplacedleft hepaticarteries(1–2%)
Completelyreplacedcommon hepaticarteryfromSMA(1–2%)
FIG.31-5. Common hepaticarteryanatomic variants.SMA= superiormesentericartery.
3. Which o the ollowingcorrectlypairsthesegmentso the liver and their associated systemicvenousdrainage?
A. SegmentsI,II,III:Right hepaticvein
B. Segment IV:Right hepaticvein
C. Segment I:IVC
D. Segment V,VI,VII,VIII:Lef hepaticvein
IVC and 3 HVs
Answer:C
There are three hepatic veins (right, middle, and le t) that serve as the out low or the hepatic circulation and drain into the suprahepatic IVC. The right hepatic vein drains segments V to VIII; the middle hepatic vein drains segment IV, as well as segments V and VIII;and the le t hepatic vein drains segments II and III (Fig. 31-6). The caudate lobe (segmentI)drainsdirectlyin totheIVC.(SeeSchwartz10thed., Figure31–8,p.1267.)
4. Withrespecttotheenterohepaticcirculation o bile,where arethemajorityo bilesaltsreabsorbed?
A. Duodenum
B. Proximaljejunum
C. Terminalileum
D. Colon
Answer:C
Bile saltsare sodium and potassium salts o bileacids derived rom cholesterol byhepatocytes. A ter synthesis, the primary bileacidscholicand chenodeoxycholicacid areconjugated to either taurineor glycineand then secreted intothebiliarysystem. Approximately90 to 95%o these primarybile salts and acids are absorbed by active transport at the terminal ileum, while the remainder enter the colon and are converted to secondary bile acids (deoxycholic and lithocholic acids) and their associated salts by resident bacteria. Bile acids and salts reabsorbed in the terminal ileum are reabsorbed through the portalcirculation,whilethoselost in thestoolarereplaced by hepaticsynthesis.(SeeSchwartz10th ed.,p.1270.)
244
Right lobe Posterior segment structures Middle hepatic v. Anterior segment structures Gall bladder Portal v. Hepatic a. Falciform ligament Left lobe Medial segment structures Lateral segment structures Middle HV Right HV Left HV
IVC
FIG.31-6. Confluence ofthe three hepaticveins(HVs)and the inferiorvena cava (IVC).Note that the middle and left hepaticveins(HVs) drain into a common trunkbefore entering the IVC.a.= artery;v.= vein.(Adapted with permission from Cameron JL,ed.AtlasofSurgery.Vol.I, GallbladderandBiliaryTract,theLiver,PortosystemicShunts,thePancreas.Toronto:BCDecker;1990,p.153.)
5. Which o the ollowing compounds is not synthesized predominantlybytheliver?
A. Albumin
B. Factor VIII
C. Factor VII
D. Factor II
6. Te gold standard or identi ying liver lesions by imaging is
A. Intraoperativeultrasound
B. Computedtomography(C )withtriple-phasecontrast
C. Magnetic resonance imaging (MRI) with gadoxetatebased contrast
D. Positron emission tomography(PE )scan
7. Temost common causeo acuteliver ailure(ALF) in the United Statesis
A. Acuteviralhepatitis
B. Cardiogenicshock
C. Autoimmunehepatitis
D. Drug/toxin-induced (includingacetaminophen)
Answer:B
he liver is the largest gland in the body, and responsible or synthesis o the majority o plasma proteins. he liver produces approximately 10 g o albumin per day, and albumin measurement can there ore be used as a surrogate or liver synthetic unction. his must be interpreted with caution, as albumin levelscan bein luenced byahost o actorsunrelated to hepatic unction, and albumin’s longhal -li e (15–20 days) makes it a poor marker or acute hepatic dys unction. Most clotting actors are synthesized predominantly in the liver, except or actor VIII. Due to this act the prothrombin time (P ) and international normalized ratio (INR) may also be used asmarkerso hepaticsynthetic unction.However,these too should be interpreted with caution as other conditions, includingvitamin Kde iciencyandwar arin use,mayprolong apatient’sP /INR.(SeeSchwartz10th ed.,p.1271.)
Answer:A
he use o intraoperative ultrasound o the liver has rapidly expandedovertheyearswiththeincreasingnumberandcomplexityo hepatic resections being per ormed. It has the abilitytoprovidethesurgeon withreal-timeaccuratein ormation use ul or surgicalplanning.Intraoperativeultrasound isconsidered thegold standard or detectingliver lesions,and studies have shown that it can identi y 20% to 30% more lesions than other preoperative imaging modalities. Importantly, it has been shown to in luence surgical management in almost 50% o planned liver resections or malignancies. Applications or intraoperative ultrasound o the liver include tumor staging, visualization o intrahepatic vascular structures, and guidanceo resectionsplanebyassessment o therelationship o amasstothevessels.In addition,biopsyo lesionsandablation o tumors can be guided by intraoperative ultrasound. (SeeSchwartz10th ed.,p.1273.)
Answer:D
Acuteliver ailure(ALF) isde ined asdevelopment o hepatic encephalopathy within 26 weeks o severe liver injury in a patient with nohistoryo liver diseaseor portalhypertension. In developing countries, the most common etiology o ALF isviralin ections,includinghepatitisB,A,and E.In theWest (includingtheUnited States,Australia,United Kingdom,and most o Europe), 65% o ALF cases are related to drugs and toxins, especially acetaminophen. (See Schwartz 10th ed., pp.1275–1276.)
8. A patient presents with painless jaundice, and is ound to have cirrhosis. Tey have no history o alcohol abuse, but do note a history o diabetes mellitus and pseudogout
Tey also mention that multiple members o their amily havesu ered rom cirrhosis.What isthemost likelyetiology or their cirrhosis?
A. Alcoholabuse
B. Wilson disease
C. α1-antitrypsin defciency
D. Hemochromatosis
Answer:D
Chronic hepatitis C in ection is the most common cause o chronic liver disease in the United States. Other etiologies include alcohol abuse, nonalcoholic steatohepatitis (NASH), and autoimmune diseases (primarybiliarycirrhosis,primary sclerosingcholangitis,andautoimmunehepatitis).Hereditary hemochromatosis is the most common metabolic cause o cirrhosis, and should be suspected i a patient presents with skin hyperpigmentation, diabetes mellitus, pseudogout, cardiomyopathy,ora amilyhistoryo cirrhosis.(SeeSchwartz 10th ed.,p.1278.)
245
9. Which o the ollowing is not one o the physiologic changesnoted in patientswith cirrhosis?
A. Reduced restingenergyexpenditure
B. Reduced muscleand at stores
C. Increased cardiacoutput
D. Decreased systemicvascular resistance
Answer:A he clinical mani estations o cirrhosis are the result o numerous physiologic changes associated with a patient’s progressive liver ailure. Hypoalbuminemia results in inger clubbing, while spider angiomata and palmar erythema are thought to be caused byalterations in sex hormones. he physiologic basis or eminization o males (gynecomastia, loss o chest/axillaryhair, testicular atrophy) with cirrhosis is less well understood. Portal hypertension mani ests as caput medusa and varices. Cirrhotic patients su er rom chronic malnutrition,which maybeassociated with weakness,weight loss, and decreased at and muscle stores. Despite this act, patientswith cirrhosishaveelevated restingenergyexpenditure. Also noted are increased cardiac output and heart rate with decreased systemic vascular resistance and blood pressure. (SeeSchwartz10th ed.,p.1279.)
10. Clinicallysignifcantportalhypertension isevidentwhen the_______exceeds______mm Hg.
A. Wedged hepaticvenouspressure,10
B. Freehepaticvenouspressure,20
C. Hepaticvenouspressuregradient,10
D. Hepaticvenouspressuregradient,20
Answer:C
Portal hypertension occurs when the pressure in the portal system is increased due to actors that may be divided into three categories. Presinusoidal causes o portal hypertension include sinistral/extrahepatic (splenic vein thrombosis, splenomegaly, splenic atrioventricular istula) and intrahepatic (schistosomiasis, congenital hepatic ibrosis, idiopathic portal ibrosis, myeloproli erative disorder, sarcoid, gra tversus-host disease) etiologies. Sinusoidal portal hypertensionisaconsequenceo cirrhosiso anyetiology.Postsinusoidal hypertension can also be divided into intrahepatic (vascular occlusive disease) and posthepatic (Budd-Chiari, congestive heart ailure [CHF], IVC webs) etiologies. In evaluating patients with suspected portal hypertension, an enlarged portal vein on routine abdominal ultrasonography may suggest portal hypertension but this is not diagnostic. Doppler ultrasound allows identi ication o vascular occlusion and the direction o portal venous low. C and MRangiography are use ul or evaluating portal venous patencyand anatomy. he most accurate method or measuring portal hypertension is hepatic venography. his procedure introduces a balloon catheter directlyinto thehepaticvein where reehepatic venouspressure(FHVP)ismeasured. hehepaticvein isthen occluded by in lation o the balloon allowing measurement o thewedged hepaticvenouspressure(WHVP). hehepatic venous pressure gradient (HVPG) may then be calculated by subtracting the FHVP rom the WHVP (HVPG = WHVP–FHVP). Clinically signi icant portal hypertension is de ined as HVPG greater than 10 mm Hg. (See Schwartz 10th ed., pp.1280–1281.)
11. All o the ollowingtherapies are considered appropriate duringthemanagement o an acutevaricealhemorrhage EXCEP ?
A. Endoscopywith varicealband ligation
B. Short-term antibioticprophylaxis
C. Somatostatin analogues
D. Recombinant actor VIIa
Answer:D
Variceal bleeding is the leading cause o morbidity and mortality in those with portal venous hypertension. Approximately 30% o patients with compensated cirrhosis and 60% o those with decompensated cirrhosis will have varices, and one-third o these patients will experience a variceal bleed. heseepisodescarrya20to 30%risko mortality.Prevention o variceal bleeding may be accomplished through administration o nonselective β-blockers (eg, propranolol) and routineendoscopicsurveillanceand varicealband ligation.In the case o acute variceal bleeding, patients should be admitted
246
12.Acirrhotic patient is admitted with varicealbleeding.Te bleedingiscontrolledwithpharmacologictherapy,andthe patient recovers rom the acute episode. Assuming they receive no other therapies to treat their varices or their underlying cause, what is the likelihood that they will experiencearecurrent varicealbleed within 2years
A. 20%
B. 40%
C. 70%
D. 100%
13.Which o the ollowing INCORREC LYmatches a grading scale or patients with liver disease and one o its components?
A. Model or End-Stage Liver Disease (MELD): Serum creatinine
B. Child- urcotte-Pugh (C P):Bilirubin
C. C P:INR
D. MELD:Albumin
14. Which o the ollowing INCORREC LY pairs the C P class with overall risk o mortality ollowing an intraabdominaloperation?
A. ClassA:10%
B. ClassB:30%
C. ClassC:50%
D. ClassC:75%
to an ICU or resuscitation and management. While prompt resuscitation iscritical,administration o bothbloodproducts and crystalloid should be done with care. A target o hemoglobin o 8g/dLisappropriate,and administration o platelets and resh rozen plasma may be considered or patients with thrombocytopenia or severe coagulopathy. However, overresuscitation with both blood products and crystalloid solution hasbeen associatedwithincreasedrisko re-bleedingand morbidity.Useo recombinant actorVIIahasnotbeen shown to be better than standard therapy, and is not recommended Patients with cirrhosis who experience a variceal bleed are at high risk or developing bacterial in ection, including spontaneous bacterial peritonitis (~50% o in ectious complications), pneumonia, and urinary tract in ection. hese bacterial in ections not only carry their own risk o morbidity and mortality, but also are associated with increased risk o re-bleeding. For this reason short-term antibiotic prophylaxis(eg,ce triaxone) isrecommended or patientswith acute varicealbleeding.Management o thebleedingcan beaccomplished with vasoactive medications, including vasopressin and somatostatin analogues (eg, octreotide). hese therapies cause splanchnic vasoconstriction and slowthe lowo blood to the varices. hough vasopressin is the most potent vasoconstrictor, it is limited by its systemic e ects. hus, somatostatin analoguesarethepre erred agent.Further therapy or bleedingvaricesshould includeendoscopywith varicealband ligation.(SeeSchwartz10th ed.,pp.1281–1282.)
Answer:C he risk o re-bleed or patients is 70% over 2 years i they receive no urther treatment. Patients who recover rom an episode o variceal bleeding should be treated with ollow-up endoscopyand variceal band ligation. In appropriate patients, transjugular intrahepatic portosystemic shunt ( IPS) or orthotopic liver transplant should be considered. (SeeSchwartz10th ed.,p.1282.)
Answer:D
he Child- urcotte-Pugh (C P) was initially derived or use in predicting the risk o portocaval shunt procedures, and comprises ive components: bilirubin, albumin, INR, presence o encephalopathy, presence o ascites. he Model or End-Stage Liver Disease (MELD) score was developed as a model to predict mortality a ter IPS, but has been adapted and validated or use as the method o organ allocation or orthotopic liver transplantation (OL ) in the United States. It is a linear regression model based on the serum creatinine, totalbilirubin,and INR.(SeeSchwartz10th ed.,p.1280.)
Answer:C
he overall mortality or patients with cirrhosis undergoing intra-abdominal surgeryhas consistentlybeen shown to correlate with the C P classi ication. he estimated mortality is 10%, 30% and 75 to 80% or those with C P class A, B, and C cirrhosis, respectively. MELD score also predicts postoperative mortalityin cirrhotic patients, and has been shown to
247
15. What isthemost common complication ollowing IPS?
A. Encephalopathy
B. Hepaticischemia
C. In ection
D. Li e-threateninghemorrhage
correlate well with estimates based on C P classi ication. In general,thosepatientswith a MELD below10should beconsidered appropriate or surgery,whilethosewith scoresabove 15 should not be considered or elective procedures. (See Schwartz10th ed.,p.1280.)
Answer:A
IPS is a percutaneous procedure used or treatment o patients who have gastroesophageal varices in the setting o portalhypertension.It has largelyreplaced surgicalportosystemic shunts due to the act that it is both sa e and e ective whilealsoprovidingaminimallyinvasivealternativetomajor abdominal surgery. IPS unctions by creating an intrahepaticshunt between theportalandsystemiccirculation which causesareduction in theportalpressureand ultimatelyin the blood lowthrough varices.It is accomplished byendovascular access through the jugular vein to a hepatic vein radical and subsequent creation o a needle tract that connectsit to a branch o theportalvein.A ter dilation o thetract,ametallic stent is deployed to hold the new portosystemic connection open. Because this shunt reduces irst pass metabolism o the liver,themost common complication o IPSisencephalopathy which occurs in 25 to 30% o patients. Other complications such as hepatic ischemia, in ection, renal ailure, and hemorrhage may occur, but are rare. (See Schwartz 10th ed., p.1282.)
16. Initialmanagement o a pyogenicliver mayincludeallo the ollowingEXCEP ?
A. reatment o theunderlyingcause
B. Broad-spectrum intravenousantibiotics
C. Surgicaldrainageand/or resection
D. Percutaneousfne-needleaspiration and culture
Answer:A
Pyogenicabscessesarethemost common liver abscessesseen in the United States. hough traditionally a result o intraabdominal in ections such as appendicitis and diverticulitis, earlier diagnosis o these conditions in patients has reduced prevalence o these conditions as causes or pyogenic liver abscess. Other etiologies include impaired biliary drainage, subacutebacterialendocarditis,dentalwork,in ectedindwelling catheters, or direct extension rom abscesses related to in lammatoryboweldisease.Pyogenicliverabscessesaremost commonly seen in the right lobe o the liver, and Escherichia coli is the most commonlyisolated pathogen. Approximately 40%o abscessesarepolymicrobial,while20%o culturenegative. reatment o pyogenic liver abscesses include correction o the underlying cause and intravenous antibiotics or at least 8weeks,which ise ectivein approximately80to 90% o patients.Empiricantibioticcoverageshould includegramnegative and anaerobic organisms,with percutaneous aspiration and culture used to tailor long-term antibiotic therapy. Placement o a percutaneous drainage catheter may be considered,though it iso ten ine ectiveduetotheviscousnature o thecollection.Surgicaldrainageor resection isreserved or patients who ail nonoperative management. (See Schwartz 10th ed.,p.1284.)
17. Temost common benign hepaticlesion isthe
A. Hemangioma
B. Simplecyst
C. Adenoma
D. Bileduct hamartoma
Answer:B
While hemangiomas are the most common solid benign masses ound in the liver, the simple hepatic cyst is still the most common overall. Simple cysts have a prevalence o approximately 2.8 to 3.6%, and are more common in women by a ratio o 4:1. Cysts are generally ound incidentally during abdominal imaging, and small, asymptomatic cysts may
248
be managed conservatively. Large cysts may begin to cause abdominal pain, epigastric ullness, and early satiety. hese patients may be treated with percutaneous cyst aspiration and sclerotherapy which is e ective in approximately 90%o patients.For thosewho ailpercutaneoustreatment,or where percutaneoustreatmentisnotavailable,surgicalcyst enestration maybe considered. I surgical enestration is per ormed, the cyst wallshould be sent or pathologic analysis to exclude carcinoma.(SeeSchwartz10th ed.,p.1288.)
18. Which o the ollowing liver lesions carry a signifcant risko spontaneousrupture?
A. Hemangioma
B. Hepaticcyst
C. Adenoma
D. Bileduct hamartoma
19. Apatient presentswith results rom a contrast-enhanced C scan that describe a well-circumscribed lesion that demonstrates homogenous enhancement during arterial phase,isodensityon thevenousphase,and acentralscar. In general,what would betherecommended treatment?
A. Reassuranceand observation
B. Percutaneousradio requencyablation
C. Resection
D. ransarterialchemoembolization
20. What is the annual conversion rate to HCC or patients with cirrhosis?
A. Lessthan 1%
B. 1–2%
C. 2–6%
D. 6–10%
21. Patient’s eligible or the Mayo Clinical protocol to treat hilar cholangiocarcinomadoNO include
A. Patients with hilar cholangiocarcinoma and primary sclerosingcholangitis(PSC).
B. Patientswith unresectablecholangiocarcinoma.
C. Patientswith tumorslessthan 3cm.
D. Patientswhohavehad prior radiotherapy.
Answer:C
Hemangiomas are congenital vascular lesions that mayrange in size rom lessthan 1to25cm or greater. heyarepredominantly ound in women, and are generally asymptomatic. Large lesions may result in discom ort rom compression o nearbyorgans. hough hemangiomas are at risk or bleeding i they are biopsied, spontaneous rupture is rare. Adenomas, on the other hand, carry a signi icant risk or spontaneous rupture with intraperitoneal bleeding. For this reason, along with their potential or malignant degeneration,it isgenerally recommended that hepatic adenomas be resected once discovered.(SeeSchwartz10th ed.,pp.1290–1291.)
Answer:A
On contrast-enhanced imaging, a ocal nodular hyperplasia (FNH) can be recognized as a well-circumscribed mass that demonstrates enhancement on the arterial phase and isodensity on the venous phase. FNH also demonstrates a characteristic central scar. FNH are solid benign lesions, are similar toadenomas,andaremorecommon in women o childbearing age. Unlike adenomas, however, they are not prone to malignant degeneration or spontaneous rupture. For this reason, asymptomatic FNHs may be managed conservatively unless adenoma or HCC cannot be de initively excluded. Gadolinium-enhanced MRI mayallowbetter visualization o the ibrousseptaextending rom theFNH’scentralscar.While FNH and adenomas may appear similar on C or standard MRI, newMRI contrast agents, such as gadobenate dimeglumine (MultiHance), allow superior discrimination between thesetwolesions.(SeeSchwartz10th ed.,p.1291.)
Answer:C
HCCisthe i thmostcommon malignancyworldwide,andits risk actors include viral hepatitis, alcoholic cirrhosis, hemochromatosis, and NASH. Cirrhosis is present in 70 to 90%o patients who develop HCC, and the annual conversion rate rom cirrhosisis2to6%.(SeeSchwartz10th ed.,p.1291.)
Answer:D
Cholangiocarcinoma is an adenocarcinoma o the bile ducts, and represents the second most common primary liver malignancy. Cholangiocarcinoma may be intra- or extrahepatic, and the latter may be divided into proximal or distal. Proximal cholangiocarcinoma is also known as hilar cholangiocarcinoma or Klatskin tumor. he only curative treatment option or hilar cholangiocarcinoma is surgical resection, or which the reported 5-year survival rates range rom 25 to 40%. However, in the presence o primary sclerosing cholangitis (PSC, ~10% o patients with cholangiocarcinoma), the
249
22. Apatient undergoesroutinecholecystectomyand isincidentally ound to have gallbladder carcinoma that is staged as 1.Further treatment should include
A. No urther treatment.
B. Externalbeamradiation withsystemicchemotherapy.
C. Reoperation with central liver resection and hilar lymphadenectomy.
D. Reoperation with ormal lobectomy and bile duct resection.
23. Which o the ollowingisconsidered a primarydeterminant o suitability or resection when evaluatingapatient with hepaticcolorectalmetastases?
A. Number o metastatictumors
B. Sizeo metastatictumors
C. Predicted volumeo hepaticremnant
D. Prior therapy
24. Based on the standard Milan criteria, which o the ollowing patients with HCC would be eligible or transplantation?
A. One4.5-cm lesion in segment VI with invasion o the right portalvein.
B. Treelesionsconfned totheright lobe,with thelargest being2.5cm.
C. Asingle,5.5-cm lesion in segment II.
D. Tree lesions spread throughout the liver, with the largest being3.5cm.
results o surgical resection are poor due to associated liver dys unction and portal hypertension. For this reason, the Mayo Clinic protocol was developed to treat patients with hilar cholangiocarcinomaand PSC. histreatment comprises external beam radiation, 5-FU-based chemotherapy, and iridium-192brachytherapy ollowed byoperativestagingand OL in patients without metastatic disease. he 5-year survivalrate or patientscompletingthisprotocolis70%.Current eligibility criteria or this protocol include patients with hilar cholangiocarcinoma with PSC or patients with unresectable hilar cholangiocarcinoma who have not received prior radiotherapy.Furthermore,thepatient must havea primarytumor less than 3 cm in radial dimension and no evidence o intrahepatic or extrahepatic metastases. (See Schwartz 10th ed., pp.1291–1292.)
Answer:D
Gallbladder cancer is a rare and aggressive orm o biliary malignancy. In approximately one-third o cases it is diagnosed incidentally ollowing routine cholecystectomy. reatment or these patients is guided by stage o the tumor. In those patients with 1 tumors, no urther treatment is necessary. In patients with 2 or greater tumors, reoperation with central liver resection and hilar lymphadenectomy is recommended. he role or more radical resections is unclear. (See Schwartz10th ed.,p.1293.)
Answer:C
he liver is a common site or metastatic disease in patients with colorectal disease, and approximately 50 to 60% o patients diagnosed with colorectal cancer will develop liver metastases within their li etime. With the advent o more aggressive strategies or the management o metastatic colorectal cancer, including improved chemotherapeutic regimens and expanded use o metastasectomy, the 5-year survival or patients with isolated metastases to the liver may exceed 30%. Given these encouraging results, the paradigm or surgical evaluation and treatment o these patients has shi ted to primarily consider the health o the background liver and volumeo thehepaticremnant,and not tumor characteristics such as size and number. (See Schwartz 10th ed., pp.1293–1294.)
Answer:B
O L was irst attempted in the 1980s and 1990s, with initial seriesreporting5-year survivalrateso 20to 50%. hisled to the introduction o the Milan criteria which limited eligibility to patients with one tumor less than 5 cm or up to three tumors less than 3 cm and no evidence o gross intravascular or extrahepatic spread. Adoption o these guidelines resulted in signi icantimprovementin 5-year survival or patientswith HCCtreated with O L.(SeeSchwartz10th ed.,p.1295.)
250
25. Te only FDA-approved systemic chemotherapeutic agent or HCCis
A. Epirubicin
B. Cisplatin
C. 5-Fluorouracil
D. Sora enib
26. Which o the ollowingcorrectlypairstheBrisbane 2000 hepatic resection terminology with appropriate liver segments?
A. Right posterior sectionectomy:SegmentsIVand IV
B. Le hepatectomy:SegmentsI,II,III,and IV
C. Right Hepatectomy:SegmentsVI,VII,and VIII
D. Le lateralsectionectomy:SegmentsIIand III
OlderHepaticResectionTerminology
Righthepaticlobectomy
Lefthepaticlobectomy
Righthepatictrisegmentectomy
Lefthepatictrisegmentectomy
Leftlateralsegmentectomy
Rightposteriorlobectomy
Caudatelobectomy
Answer:D
hough systemicchemotherapyhasnot proven verye ective in the treatment o HCC, the multikinase inhibitor sora enib hasbeen approved or usespeci icallyin thesepatients.Based on results o the SHARP trial, the sora enib demonstrated a 3-month survivalbene itversusplacebo. hough theseresults are modest, it remains a treatment option or patients with advanced,unresectableHCC.(SeeSchwartz10thed.,p.1296.)
Answer:D
See able31-1.(SeeSchwartz10th ed., able31-8,p.1297.)
Brisbane2000HepaticResectionTerminology
Righthepatectomyorrighthemihepatectomy(V,VI,VII,VIII)
Lefthepatectomyorlefthemihepatectomy(II,III,IV)
Righttrisectionectomyorextendedrighthepatectomy(orhemihepatectomy,IV,V,VI,VII,VIII)
Lefttrisectionectomyorextendedlefthepatectomy(orhemihepatectomy,II,III,IV,V,VIII)
Leftlateralsectionectomyorbisegmentectomy(II,III)
Rightposteriorsectionectomy(VI,VII)
Caudatelobectomyorsegmentectomy(I)
Alternative“Sector”Terminology
Rightanteriorsectorectomy
Rightposteriorsectorectomyorrightlateralsectorectomy
Leftmedialsectorectomyorleftparamediansectorectomy(bisegmentectomy,III,IV)
Leftlateralsectorectomy(segmentectomy,II)
251
TABLE31–1 Brisbane 2000liverterminology
The Gallbladderand ExtrahepaticBiliarySystem
1. Te arterial supply o the common bile duct is derived rom
A. Tele hepaticartery
B. Teright hepaticartery
C. Tegastroduodenalartery
D. Teright hepaticand gastroduodenalarteries
E. Tele hepaticand gastroduodenalarteries
2. Anomalieso thehepaticarteryand cysticarteryarepresent in what percent o individuals
A. 15%
B. 25%
C. 35%
D. 50%
E. 75%
3. Tetreatment o choice or atypeIcholedochalcyst is
A. Observation.
B. Cystresection andprimaryre-anastomosiso thecommon bileduct.
C. Resection o the common bile duct, cholecystectomy, and hepatico-jejunostomy.
D. Resection o thecyst and choledocho-duodenostomy.
Answer:D
he majority o the blood low to the human common bile duct originates rom the right hepatic artery and gastroduodenal arteries, with major trunks running along the medial and lateral aspects o the common duct (o ten re erred to as the 3 o’clock and 9 o’clock positions). (See Schwartz 10th ed., p.1311.)
Answer:D
Variationsin theanatomyo thecysticand hepaticarteriesare exceedinglycommon, the “classical”anatomyonlyappearing in 50to60%o thepopulation. hecysticarteryisabranch o theright hepaticarteryin 90%o individuals. hemost common arterialanomalyo theportalarterialsystemisareplaced right hepatic arteryoriginating rom the superior mesenteric artery;thishappensin 20%o persons.(SeeSchwartz10thed., p.1312.)
Answer:C
Choledochal cysts are rare congenital cystic dilations o the extrahepatic and/or intrahepatic biliary tree. Females are a ected three to eight times more commonly than men. hough theyarecommonlydiagnosed in childhood,asmany asonehal o patientsarenot diagnosed untiladulthood. he most common presentations in adulthood are jaundice and cholangitis, and less than one-hal o patients present with the classic clinical triad o abdominal pain, jaundice, and a mass. Ultrasonography(US) or computed tomographic (C ) scanning will con irm the diagnosis, but a more de initive imaging technique such as endoscopic retrograde cholangiopancreatography(ERCP), percutaneous transhepatic cholangiography, or magnetic resonance cholangiopancreatography (MRCP) isrequired toassessthebiliaryanatomyand plan the appropriate surgical treatment. he risk o cancer development in these patients is up to 15%, and can largely be mitigated by excision o the biliary tree. ypes I, II, and IV cysts are treated with excision o the extrahepatic biliary tree with a Roux-en-Yhepaticojejunostomy. ype IV may also require a segmental liver resection.Sphincterotomyis recommended or typeIIIcysts.(SeeSchwartz10th ed.,p.1330.)
253 32
CHAPTER
4. Relaxation o thesphincter o Oddiin responsetoamealis largelyunder thecontrolo which hormone?
A. Gastrin
B. Cholecystokinin (CCK)
C. Motilin
D. Secretin
E. Ghrelin
5. What percentage o the bile acid pool is reabsorbed in the ileum through theenterohepaticcirculation?
A. 25%
B. 50%
C. 75%
D. 90%
E. 95%
6. Tesolubilityo cholesterolin bileisdetermined by
A. Cholesterol,calcium,bilirubin
B. Cholesterol,bilesalts,lecithin
C. Bilesalts,cholesterol,bilirubin
D. Calcium,cholesterol,bilesalts
Answer:B
hesphincter o Oddiisa complexstructurethat is unctionallyindependent romtheduodenalmusculatureandcreatesa high-pressurezonebetween thebileduct and theduodenum. he sphincter o Oddi is about 4 to 6 mm in length and has a basalrestingpressureo about 13mm Hgabovetheduodenal pressure.On manometry,thesphincter showsphasiccontractions with a requency o about 4/min and an amplitude o 12 to 140 mm Hg. he spontaneous motilityo the sphincter o Oddi is regulated by the interstitial cells o Cajal through intrinsic and extrinsic inputs rom hormones and neurons acting on the smooth muscle cells. Relaxation occurs with a rise in cholecystokinin (CCK), leading to diminished amplitudeo phasiccontractionsand reduced basalpressure,allowingincreased lowo bileinto theduodenum.During asting, thesphincter o Oddiactivityiscoordinated with theperiodic partial gallbladder emptying and an increase in bile lowthat occurs during phase II o the migrating myoelectric motor complexes.(SeeSchwartz10th ed.,p.1313.)
Answer:E
Bile is mainly composed o water, electrolytes, bile salts, proteins, lipids, and bile pigments. Sodium, potassium, calcium, and chlorine have the same concentration in bile as in plasma or extracellular luid. he pH o hepatic bile is usually neutral or slightly alkaline, but varies with diet; an increase in protein shi ts the bile to a more acidic pH. he primarybilesalts,cholateandchenodeoxycholate,aresynthesized in the liver rom cholesterol. hey are conjugated there with taurineand glycineand act within thebileasanions(bile acids) that arebalanced bysodium.Bilesaltsareexcreted into thebilebythehepatocyteand aid in thedigestion and absorption o atsin theintestines.In theintestines,about 80%o the conjugated bile acids are absorbed in the terminalileum. he remainder is de-hydroxylated (de-conjugated) bygut bacteria, orming secondary bile acids deoxycholate and lithocholate. heseareabsorbed in thecolon,transported to theliver,conjugated, and secreted into the bile. Eventually, about 95% o the bile acid pool is reabsorbed and returned via the portal venous system to the liver, the so-called enterohepatic circulation. Five percent is excreted in the stool, leaving the relativelysmallamount o bileacidstohavemaximum e ect.(See Schwartz10th ed.,p.1312.)
Answer:B
Purecholesterolstonesareuncommon and account or <10% o all stones. hey usually occur as single large stones with smooth sur aces. Most other cholesterol stones contain variable amounts o bile pigments and calcium, but are always >70% cholesterol by weight. hese stones are usually multiple,o variablesize,and maybehard and aceted or irregular, mulberry-shaped,and so t.Colorsrange rom whitish yellow and green to black. Most cholesterol stones are radiolucent; <10%are radiopaque. Whether pure or o mixed nature, the common primaryevent in the ormation o cholesterolstones issupersaturation o bilewithcholesterol. here ore,highbile cholesterol levels and cholesterol gallstones are considered one disease. Cholesterol is highly nonpolar and insoluble in water and bile. Cholesterol solubilitydepends on the relative
254
7. Acutecholecystitisisconsidered
A. A primary in ectious process with secondary in ammation.
B. Asterileprimaryin ammatoryprocess.
C. A primary in ammatory process with occasional bacterialcontamination.
D. Aprimaryautoimmuneprocess.
concentration o cholesterol,bilesalts,and lecithin (themain phospholipid in bile). Supersaturation almost always is causedbycholesterolhypersecretion rather than byareduced secretion o phospholipid or bilesalts.(SeeSchwartz10th ed., p.1318.)
Answer:C
Obstruction o the cystic duct by a gallstone is the initiating event that leads to gallbladder distention, in lammation, and edema o the gallbladder wall. Why in lammation develops onlyoccasionallywith cystic duct obstruction is unknown. It isprobablyrelated to theduration o obstruction o thecystic duct. Initially, acute cholecystitis is an in lammatoryprocess, probablymediated bythe mucosal toxin lysolecithin, a producto lecithin,aswellasbilesaltsandplatelet-activating actor. Increase in prostaglandin synthesis ampli ies the in lammatory response. Secondary bacterial contamination is documented in 15 to 30%o patients undergoingcholecystectomy or acute uncomplicated cholecystitis. In acute cholecystitis, the gallbladder wall becomes grossly thickened and reddish with subserosal hemorrhages. Pericholecystic luid o ten is present. he mucosa mayshowhyperemia and patchynecrosis.In severe cases,about 5to 10%,the in lammatoryprocess progressesand leadsto ischemia and necrosiso thegallbladder wall. More requently, the gallstone is dislodged and the in lammation resolves.(SeeSchwartz10th ed.,p.1320.)
8. A 54-year-old otherwise healthy woman presents to the emergencydepartment with abdominal pain, ever, chills, and con usion.Blood pressureis95/50,heart rate110,and temperature 39°C. Laboratory tests demonstrate a white blood cell count o 15,000, normal hematocrit and platelets,aswellasa direct bilirubin o 7.2.Initialmanagement should be
A. Emergency biliary decompression endoscopically or transhepatically.
B. Emergent cholecystectomy.
C. Intravenous(IV) uid resuscitation and antibiotics.
D. Observation and pain control.
E. Discharge home with oral antibiotics with planned cholecystectomyin thecomingweeks.
Answer:C
he initial treatment o patients with cholangitis includes intravenous (IV) antibiotics and luid resuscitation. hese patientsmayrequireintensivecareunit monitoringand vasopressor support. Most patients will respond to these measures. However, the obstructed bile duct must be drained as soon asthepatient hasbeen stabilized.About 15%o patients will not respond to antibiotics and luid resuscitation, and an emergency biliary decompression may be required. Biliary decompression may be accomplished endoscopically, via the percutaneous transhepatic route, or surgically. he selection o procedureshouldbebasedon thelevelandthenatureo the biliaryobstruction. Patients with choledocholithiasis or periampullary malignancies are best approached endoscopically, withsphincterotomyandstoneremoval,orbyplacemento an endoscopic biliarystent.In patientsin whom the obstruction ismoreproximalor perihilar,or when a stricture in a biliaryenteric anastomosis is the cause or the endoscopic route has ailed,percutaneoustranshepaticdrainageisused.When neither ERCP nor percutaneous transhepatic cholangiography (P C) is available, an emergent operation or decompression o the common bile duct with a tube maybe necessaryand li esaving. De initive operative therapy should be de erred until the cholangitis has been treated and the proper diagnosisestablished.Patientswith indwellingstentsand cholangitis usually require repeated imaging and exchange o the stent over aguidewire.(SeeSchwartz10th ed.,p.1323.)
255
9. Risk actors or acalculouscholecystitisinclude
A. Sepsis
B. Severeburns
C. Prolonged parenteralnutrition
D. Multipletrauma
E. Allo theabove
10. Appropriate management o a patient with cirrhosis secondarytosclerosingcholangitisincludes
A. Systemic immunosuppression with corticosteroids and calcineurin inhibitors
B. Anti-tumor necrosis actor ( NF) monoclonal antibodies(in iximab)
C. Consideration or transplantation
D. Ursodeoxycholicacid
E. Excision o the extrahepatic biliary tree (Kasai procedure)
Answer:E
Acute in lammation o the gallbladder can occur without gallstones.Acalculouscholecystitistypicallydevelopsin critically ill patients in the intensive care unit. Patients on parenteralnutrition with extensiveburns,sepsis,major operations, multipletrauma,or prolonged illnesswith multipleorgan system ailure are at risk or developing acalculous cholecystitis. he cause is unknown, but gallbladder distention with bile stasis and ischemia has been implicated as causative actors. Pathologic examination o the gallbladder wallrevealsedema o the serosa and muscular layers, with patchy thrombosis o arteriolesand venules.USisthediagnostictest o choice.Percutaneous USor C -guided cholecystostomy is typically the intervention o choice,astheyareo ten un it orsurgery.A ter recovery rom the systemic disease, cholecystectomy may be indicated.(SeeSchwartz10th ed.,pp.1327–1330.)
Answer:C
Sclerosing cholangitis (primary or secondary) is an uncommon disease characterized by in lammatory strictures involving the intrahepatic and extrahepatic biliary tree. It is a progressive disease which can lead to biliary cirrhosis. Medical therapy has long been attempted with immunosuppressants,antibiotics,steroids,and ursodeoxycholicacid,and has been disappointing. Surgical management with resection o the extrahepatic biliary tree and hepaticojejunostomy has produced reasonable results in patients with extrahepatic and bi urcation strictures,but without cirrhosisor signi icant hepatic ibrosis. In patients with sclerosing cholangitis and advancedliver disease,liver transplantation istheonlyoption. It o ers excellent results, with overall 5-year survival as high as 85%. Primary sclerosing cholangitis recurs in 10 to 20% o patients and may require retransplantation. (See Schwartz 10th ed.,p.1331.)
11. All o the ollowing patients should be re erred to or cholecystectomyEXCEP
A. A 45-year-old woman with recurrent bouts o biliary colicand documented gallstones.
B. A60-year-old man with biliarypancreatitis.
C. A 78-year-old man with choledocholithiasis and cholelithiasis.
D. A 30-year-old woman with acute cholecystitis in her second trimester o pregnancy.
12. Over a 10-year period, what percentage o patients with asymptomaticgallstoneswillremain symptom- ree?
A. 10%
B. 25%
C. 50%
D. 66%
E. 90%
Answer:C
Patients older than 70 years presenting with bile duct stones should have their ductal stones cleared endoscopically. Studies comparing surgery to endoscopic treatment have documentedlessmorbidityandmortality orendoscopictreatment in thisgroup o patients. heydonot need tobesubmitted or a cholecystectomy, as onlyabout 15%will become symptomatic rom their gallbladder stones, and such patients can be treated astheneed arisesbyacholecystectomy.(SeeSchwartz 10th ed.,p.1322.)
Answer:D
Gallstones in patients without biliary symptoms are commonlydiagnosed incidentallyon US,C scans,or abdominal radiography or at laparotomy. Several studies have examined the likelihood o developing biliary colic or developing signi icant complicationso gallstonedisease.Approximately3% o asymptomatic individuals become symptomatic per year (ie, develop biliary colic). Once symptomatic, patients tend to haverecurringboutso biliarycolic.Complicated gallstone disease develops in 3 to 5%o symptomatic patients per year. Over a 20-year period, about two-thirds o asymptomatic patients with gallstones remain symptom- ree. (See Schwartz 10th ed.,p.1317.)
256
13. All o the ollowing increase risk or the development o gallbladder cancer EXCEP
A. Femalegender
B. Historyo cholelithiasis
C. Historyo choledochalcysts
D. Smoking
E. Gallbladder polyps
14. Temost common typeo gallbladder cancer is
A. Oat cell
B. Adenocarcinoma
C. Adenosquamous
D. Anaplastic
E. Squamouscell
15. Te gallbladder lymphatics drain into which o the ollowingliver segments?
A. IIIand IV
B. Vand VI
C. IVand V
D. IIIand V
E. Iand IV
16. Adequatetreatment or agallbladder lesion involvingthe laminapropriao thegallbladder includes
A. Cholecystectomy ollowedbyadjuvantchemotherapy
B. Neoadjuvant chemoradiotherapy ollowed by surgicalresection.
C. Segmental liver resection and lymphadenectomy alone.
D. Cholecystectomyalone.
E. Extended right hepatectomy.
17. All o the ollowing are di erent morphological classifcationso bileduct adenocarcinomasEXCEP
A. Nodular
B. Scirrhous
C. Di uselyinfltrating
D. Serous
E. Papillary
Answer:D
Cholelithiasis is the most important risk actor or gallbladder carcinoma, and up to 95%o patients with carcinoma o the gallbladder have gallstones. However, the 20-year risk o developingcancer or patientswith gallstonesis<0.5% or the overallpopulation and 1.5% or high-risk groups. he pathogenesishasnot been de ined but isprobablyrelated tochronic in lammation. Larger stones (>3 cm) are associated with a 10- old increased risko cancer. herisko developingcancer o thegallbladder ishigher in patientswith symptomaticthan asymptomaticgallstones.
Polypoid lesions o the gallbladder are associated with increased risk o cancer, particularly in polyps >10 mm. he calci ied “porcelain”gallbladder isassociated with >20%incidenceo gallbladder carcinoma. hesegallbladdersshould be removed,even i thepatientsareasymptomatic.Patientswith choledochalcystshavean increased risk o developingcancer anywherein thebiliarytree,but theincidenceishighest in the gallbladder.(SeeSchwartz10th ed.,p.1334.)
Answer:B
Between 80 and 90%o the gallbladder tumors are adenocarcinomas. Squamous cell, adenosquamous, oat cell, and other anaplastic lesions occur rarely. he histologic subtypes o gallbladder adenocarcinomas include papillary, nodular, and tubular. Less than 10%are o the papillarytype, but these are associated with an overall better outcome, as they are most commonly diagnosed while localized to the gallbladder. (See Swartz10th ed.,p.1334.)
Answer:C
Lymphatic low rom the gallbladder drains irst to the cystic duct node (Calot), then the pericholedochal and hilar nodes, and inally the peripancreatic, duodenal, periportal, celiac, and superior mesenteric artery nodes. he gallbladder veins drain directlyinto theadjacent liver,usuallysegmentsIVand V, where tumor invasion is common. (See Schwartz 10th ed., p.1334.)
Answer:D
umors limited to the muscular layer o the gallbladder ( 1) are usually identi ied incidentally, a ter cholecystectomy or gallstonedisease. hereisnear universalagreement that simple cholecystectomy is an adequate treatment or 1 lesions and results in a near 100% overall 5-year survival rate. (See Schwartz10th ed.,p.1335.)
Answer:D
Over 95% o bile duct cancers are adenocarcinomas. Morphologically, they are divided into nodular (the most common type), scirrhous, di usely in iltrating, or papillary. (See Schwartz10th ed.,p.1335.)
257
18. Accordingto the Bismuth-Corlette classifcation system, perihilar cholangiocarcinomas extending into the right secondaryintrahepaticductsareclassifed as
A. ypeII
B. ypeIIIb
C. ypeIIIa
D. ypeIV
E. ypeI
19. Te best initial imaging test or evaluating or suspected cholangiocarcinomaincludes
A. Percutaneouscholangiography
B. Endoscopicretrogradecholangiopancreatography
C. Ultrasound
D. MRCP
E. Hepatobiliaryiminodiaceticacid (HIDA)scan
Answer:C
Perihilar cholangiocarcinomas, also re erred to as Klatskin tumors, are urther classi ied based on anatomic location by the Bismuth-Corlette classi ication. ype I tumors are conined to thecommon hepaticduct,but typeII tumorsinvolve the bi urcation without involvement o the secondary intrahepatic ducts. ype IIIa and IIIb tumors extend into the right and le t secondary intrahepatic ducts, respectively. ype IV tumors involve both the right and le t secondaryintrahepatic ducts.(SeeSchwartz10th ed.,pp.1335–1336.)
Answer:C
he initial tests are usually ultrasound or C scan. A perihilar tumor causesdilatation o theintrahepaticbiliarytree,but normal or collapsed gallbladder and extrahepatic bile ducts distal to the tumor. Distal bile duct cancer leads to dilatation o theextra- and intrahepaticbileductsaswellasthegallbladder.Ultrasound can establish thelevelo obstruction and rule outthepresenceo bileductstonesasthecauseo theobstructivejaundice.It isusuallydi icult to visualizethetumor itsel on ultrasound or on a standard C scan.Either ultrasound or spiral C can be used to determine portal vein patency. he biliary anatomy is de ined by cholangiography. P C de ines theproximalextent o thetumor,which isthemost important actor in determiningresectability.ERCP isused,particularly in theevaluation o distalbileduct tumors.For theevaluation o vascular involvement,celiacangiographymaybenecessary. With thenewer typeso MRI,asinglenoninvasivetest hasthe potentialo evaluatingthebiliaryanatomy,lymph nodes,and vascular involvement, as well as the tumor growth itsel . (See Schwartz10th ed.,p.1336.)
20. All o the ollowing examples are considered resectable lesionsEXCEP
A. Nodular typecholangiocarcinoma limited to thedistal common bileduct.
B. Lesionsrequiringresection o theportalvein aspart o thedissection.
C. Lesions involvingthe caudate lobe and positive portahepatisnodes.
D. Klatskin tumors requiring resection o segments IV and V.
E. Mid common bile duct lesion with positive celiac lymph nodes.
21. Patients with a history o choledochal cysts are at increased risko developingbiliarycancer
A. In thegallbladder alone.
B. Predominantly intrahepatic portions o the biliary tree.
C. In thedistalcommon bileduct.
D. At thesiteo thepreviouscyst.
E. Troughout thebiliarytree.
Answer:E
Patients should undergo surgical exploration i they have no signs o metastasis or locally unresectable disease. However, despite improvements in US, C scanning, and MRI, more than one-hal o patients who are explored are ound to have peritoneal implants, nodal or hepatic metastasis, or locally advanced disease that precludes resection. For these patients, surgical bypass or biliary decompression and cholecystectomy to prevent the occurrence o acute cholecystitis should beper ormed.(SeeSchwartz10th ed.,p.1336.)
Answer:E
Patientswithcholedochalcystshaveanincreasedrisko developingcancer anywherein thebiliarytree,but theincidenceis highest in the gallbladder. Sclerosing cholangitis, anomalous pancreaticobiliaryductjunction,andexposuretocarcinogens (azotoluene, nitrosamines) also are associated with cancer o thegallbladder.(SeeSchwartz10th ed.,p.1334.)
258
22. What percentage o bile duct injuries are identifed intraoperatively?
A. 85%
B. 5%
C. 25%
D. 50%
E. 10%
23. Tebest initialtest or asuspected postoperativebileleak includes
A. P C
B. ERCP
C. MRI
D. US
E. Plain flm X-ray
24. What is the best initial management or an intraoperatively identifed minor lateral injury to the common bile duct?
A. Placement o a -tubethrough thesiteo injuryin the duct.
B. Primaryoversewo theinjuryintraoperatively.
C. Resection o theinjured portion o theduct with endto-end anastomosis.
D. Intraoperativeplacement o endoscopicbiliarystent.
E. Resection o thebileduct and a Roux-en-Yhepaticojejunostomyreconstruction.
25. A er identifcation o a postoperative biliary stricture, what isthebest initialmanagement?
A. Operative resection o the involved biliary segment and reconstruction with an end-to-end Roux-en-Y hepaticojejunostomy.
B. ERCP with sphincterotomy and stenting o the pancreaticduct.
C. ranshepatic catheter placement or biliary decompression.
D. HIDAscan.
E. Operative placement o a tube at site o biliary stricture.
Answer:C
Onlyabout 25%o major bileductinjuries(common bileduct or hepaticduct) arerecognized at thetimeo operation.Most commonly,intraoperativebileleakage,recognition o thecorrect anatomy, and an abnormal cholangiogram lead to the diagnosiso abileductinjury.(SeeSchwartz10thed.,p.1332.)
Answer:D
Bileleak,most commonly rom thecysticduct stump,a transected aberrant right hepatic duct, or a lateral injury to the main bile duct, usually presents with pain, ever, and a mild elevation o liver unction tests. A C scan or an ultrasound will show either a collection (biloma) in the gallbladder area or ree luid (bile) in the peritoneum. (See Schwartz 10th ed., p.1332.)
Answer:A
Lateralinjurytothecommon bileductorthecommon hepatic duct,recognized at thetimeo surgery,isbest managed with a -tube placement.I the injuryis a small incision in the duct, the tubemaybeplaced through it asi it werea ormalcholedochotomy. In more extensive lateral injuries, the tube should beplaced through a separatecholedochotomyand the injuryclosed over the -tubeend to minimizetherisko subsequent stricture ormation.(SeeSchwartz10th ed.,p.1333.)
Answer:C
Patientswith bileduct stricture rom an injuryor asa sequela o previous repair usually present with either progressive elevation o liver unction tests or cholangitis. he initial management usually includes transhepatic biliary drainage catheter placement or decompression as well as or de ining the anatomy and the location and the extent o the damage. hese catheters will also serve as use ul technical aids during subsequent biliary enteric anastomosis. An anastomosis is per ormed between the duct proximal to the injury and a Roux loop o jejunum. Balloon dilatation o a stricture usually requires multiple attempts and rarely provides adequate long-term relie .Sel -expandingmetalor plasticstents,placed either percutaneously or endoscopically across the stricture, can providetemporarydrainageand,in thehigh-risk patient, permanentdrainageo thebiliarytree.(SeeSchwartz10th ed., p.1333.)
26. In the earlypostoperative period, what is the most common presentation o apatient with abiliaryinjury?
A. Fever
B. Abdominalpain
C. Steatorrhea
D. Elevated transaminases
E. Nausea
Answer:D
In the earlypostoperative period,patientspresent either with progressiveelevation o liver unction testsduetoan occluded or a stenosed bile duct, or with a bile leak rom an injured duct.(SeeSchwartz10th ed.,p.1333.)
259
1. Tepercentageo patientswho willhavean occurrenceo areplaced right hepaticarteryis
A. 1–2%
B. 5–10%
C. 15–20%
D. 20–25%
2. Temost common complication o chronicpancreatitisis
A. Pseudocysts
B. Duct stricturesand/or stones
C. Pancreaticnecrosis
D. Duodenalobstruction
3. Insulinomas associated with the multiple endocrine neoplasia(MEN)1syndrome
A. Donot usuallyrequireresection
B. Aresporadicin nature
C. Havealower rateo recurrence
D. Aremorelikelytobemalignant
4. A pancreatic cystic neoplasms that is <3 cm, has atypical cellspresent,and hasasolid component
A. Requiresarepeat C scan in 3to6months
B. Requiresarepeat C scan in 1year
C. Requirescontinued observation
D. Requiresresection
Answer:C
In 15to20%o patients,therighthepaticarterywillarise rom the superior mesenteric artery and travel upward toward the liver along the posterior aspect o the head o the pancreas (re erred to as a replaced right hepaticartery). It is important to look or this variation on preoperative computed tomographic (C ) scans and in the operating room (OR) so the replaced hepatic artery is recognized and injury is avoided. (SeeSchwartz10th ed.,p.1345.)
Answer:A
Achroniccollection o pancreatic luid surrounded byanonepithelialized wallo granulation tissueand ibrosisisre erred to asa pseudocyst.Pseudocystsoccur in up to 10%o patients with acute pancreatitis, and in 20 to 38% o patients with chronic pancreatitis, and thus, they comprise the most common complication o chronicpancreatitis.(SeeSchwartz10th ed.,pp.1375–1376.)
Answer:B
Approximately90%o insulinomasaresporadic,and 10%are associated with the multiple endocrine neoplasia (MEN)1 syndrome. Insulinomas associated with the MEN1syndrome aremorelikelytobemulti ocalandhaveahigherrateo recurrence.(SeeSchwartz10th ed.,p.1391.)
Answer:D
SeeFigure33-1.(SeeSchwartz10thed.,Figure33-75,p.1409.)
261 33
Pancreas
CHAPTER
Detailed clinical Hx and pancreas protocol CT
Symptomatic, or MCN/IPMN >3 cm Resect Further evaluation
Asymptomatic MCN/IPMN <3 cm, serous cystadenoma, branch duct IPMN
• Mucin present
• CEA>200 ng/mL
• Atypical cells present
• Solid component
• Dilated main pancreatic duct
FIG.33-1. Algorithm ormanagement o pancreaticcysticneoplasms.CEA= carcinoembryonicantigen;CT= computed tomography; ERCP= endoscopic retrograde cholangiopancreatography;EUS= endoscopicultrasound;FNA= ine-needle aspiration;Hx= history;IPMN= intraductalpapillarymucinousneoplasm o the pancreas;MCN= mucinouscysticneoplasm;mo = month(s);MRCP= magnetic resonance cholangiopancreatography;y= year(s).
5. Accordingto Ranson criteria a 67-year-old emale patient suspected o acute pancreatitis presenting to the OR with sudden onset o severeabdominalpain,a serum aspartate transaminase(AS ) >250U/dL,awhiteblood cell(WBC) >16,000/mm3, and a blood glucose >200 mg/dL would receiveadiseaseclassifcation o
A. Predicted severe
B. Predicted mild,uncomplicated
C. Predicted moderate
D. Amortalityo 10%
Answer:A
See able33-1.(SeeSchwartz10th ed, able33-7,p.1356.)
262
Repeat CT in 1 year Repeat CT q 6–12 months <1 cm <1 cm ≤2 cm 1–3 cm ↑interval if stable × 2 years No Yes Repeat CT q 3–6 months Resect 2–3 cm EUS-FNA MRCP/ERCP
6. Which o the ollowing is the most common presenting symptom in patientswith asomatostatinoma?
A. Cholelithiasis
B. Constipation
C. Hypoglycemia
D. Hypocalcemia
7. Te
etiologyassociated with chroniccalcifcpancreatitisis
A. Hyperparathyroidism
B. Hyperlipidemia
C. Alcoholabuse
D. Allo theabove
TABLE33-1 Ranson’sprognosticsignso pancreatitis
CriteriaorAcutePancreatitisNotduetoGallstones
Atadmission Duringtheinitial48h
Age>55y Hematocritall>10points
WBC>16,000/mm3 BUNelevation>5mg/dL
Bloodglucose>200mg/dL Serumcalcium<8mg/dL
SerumLDH>350IU/L ArterialPO2 <60mmHg
SerumAST>250U/dL Basedecit>4mEq/L
Estimatedfuidsequestration>6L
CriteriaorAcuteGallstonePancreatitis
Atadmission Duringtheinitial48h
Age>70y Hematocritall>10points
WBC>18,000/mm3 BUNelevation>2mg/dL
Bloodglucose>220mg/dL Serumcalcium<8mg/dL
SerumLDH>400IU/L Basedecit>5mEq/L
SerumAST>250U/dL Estimatedfuidsequestration>4L
AST= aspartate transaminase;BUN= blood urea nitrogen;LDH= lactate dehydrogenase;PO2 = partialpressure o oxygen;WBC= white blood cellcount.
Note:Fewerthan 3positive criteria predict mild,uncomplicated disease whereasmore than 6positive criteria predict severe disease with a mortalityrisko 50%.
Sources:Data rom Ranson JHC.Etiologicaland prognostic actorsin human acute pancreatitis:Areview.AmJGastroenterol77:633,1982,and From Ranson JH,Rikind KM, RosesDF,et al.Prognosticsignsand the role o operative management in acute pancreatitis.SurgGynecolObstet 139:69,1974.
Answer:A
Because somatostatin inhibits pancreatic and biliary secretions,patientswith asomatostatinomapresentwith gallstones due to bile stasis, diabetes due to inhibition o insulin secretion, and steatorrhea due to inhibition o pancreatic exocrine secretion andbilesecretion.Mostsomatostatinomasoriginate in the proximal pancreas or the pancreatoduodenal groove, with the ampulla and periampullary area as the most common site(60%). hemostcommon presentationsareabdominal pain (25%), jaundice (25%), and cholelithiasis (19%). his rare type o pancreatic endocrine tumor is diagnosed by con irming elevated serum somatostatin levels, which are usually above 10 ng/mL. Although most reported cases o somatostatinoma involve metastatic disease, an attempt at complete excision o the tumor and cholecystectomy is warranted in it patients.(SeeSchwartz10th ed.,p.1393.)
Answer:D
his type is the largest subgroup in the classi ication scheme proposed bySinger and Chari,and includespatientswith calci icpancreatitiso most etiologies( able33-2).Although the majority o patients with calci ic pancreatitis have a history o alcohol abuse, stone ormation and parenchymal calci ication can develop in a variety o etiologic subgroups; hereditary pancreatitis and tropical pancreatitis are particularly noteworthy or the ormation o stone disease. he clinician should there ore avoid the assumption that calci ic pancreatitis con irms the diagnosis o alcohol abuse. (See Schwartz 10th ed., able33-11,p.1365.)
263
TABLE33-2 Classi cation o chronicpancreatitisbased on etiologiccauses
with
8. In patients undergoing endoscopic retrograde cholangiopancreatography (ERCP) or diagnosis and staging o chronic pancreatitis, the population most at risk o developingprocedure-induced pancreatitisis
A. Onewith calculusdisease
B. Onewith intraductallesions
C. Onewith sphincter o Oddidys unction
D. Onewithahighpercentageo parenchymalcalcifcation
Answer:C
Endoscopic retrograde cholangiopancreatography (ERCP) is considered to be the gold standard or the diagnosis and staging o chronic pancreatitis. It also serves as a vehicle that enables other diagnostic and therapeutic maneuvers, such as biopsyor brushing or cytology, or the use o stents to relieve obstruction or drain a pseudocyst (Fig. 33-2). Un ortunately, ERCPalsocarriesarisko procedure-inducedpancreatitisthat occurs in approximately 5%o patients. Patients at increased risk include those with sphincter o Oddi dys unction and those with a previous history o post-ERCP pancreatitis. Post-ERCP pancreatitis occurs a ter uncomplicated procedures,as wellas a ter those that require prolonged manipulation. Severe pancreatitis and deaths have occurred a ter ERCP, it should be reserved or patients in whom the diagnosis is unclear despitetheuseo other imagingmethods,or in whom adiagnosticor therapeuticmaneuver isspeci icallyindicated. (SeeSchwartz10th ed.,Figure33-29,pp.1370–1371.)
264
ChronicCalcic Pancreatitis ChronicObstructive Pancreatitis ChronicInfammatory Pancreatitis ChronicAutoimmune Pancreatitis Asymptomatic PancreaticFibrosis Alcohol Pancreatictumors Unknown Associatedwithautoimmune disorders(e.g.,primarysclerosing cholangitis) Chronicalcoholic Hereditary Ductalstricture Endemicinasymptomatic residentsintropical climates Tropical Gallstone-ortrauma-induced orpancreasdivisum Hyperlipidemia Sjögrensyndrome Hypercalcemia Primarybiliarycirrhosis Drug-induced Idiopathic
Source:Reproduced
permission rom SingerMV,ChariST.Classi cation o chronicpancreatitis,in BegerHGet al,eds.ThePancreas.London:Blackwell-Science,1998,p.665. CopyrightWileyBlackwell.
FIG.33-2. Pancreaticduct stenting.At endoscopicretrograde cholangiopancreatography,a stent isplaced in the proximal pancreaticduct to relieve obstruction and reduce symptomso pain.Pancreatic duct stentsare le t in place oronlya limited time to avoid urtherin lammation.
9. reatment o a 1-cm gastrinoma in the wall o the duodenum isbest accomplished by
A. Enucleation
B. Full-thicknessresection
C. Duodenectomy
D. Whippleprocedure
10. Te ERCP fnding that is virtually diagnostic o intraductalpapillarymucinousneoplasms(IPMNs)is
A. Afsh-eyelesion
B. Calcifcation
C. Beaded or chain-o -lakesappearanceo theduct
D. Cyststhat resembleserouscystadenomas
Answer:B
Fi ty percent o gastrinomas metastasize to lymph nodes or the liver,and arethere oreconsidered malignant.Patientswhomeet criteria or operability should undergo exploration or possible removal o the tumor. Although the tumors are submucosal, a ull-thicknessexcisiono theduodenalwallisper ormedi aduodenal gastrinoma is ound. All lymph nodes in Passaro triangle are excised or pathologic analysis. I the gastrinoma is ound in the pancreas and does not involve the main pancreatic duct, it is enucleated. Pancreatic resection is justi ied or solitary gastrinomas with no metastases. A highly selective vagotomy can beper ormed i unresectablediseaseisidenti ied or i thegastrinomacannotbelocalized. hismayreducetheamounto expensive proton pump inhibitors required. In cases in which hepatic metastasesareidenti ied,resection isjusti iedi theprimarygastrinomaiscontrolled and themetastasescan besa elyand completely removed. Debulking or incomplete removal o multiple hepatic metastases is probably not help ul, especially in the settingo MEN1. heapplication o newmodalitiessuch asradiorequency ablation seems reasonable, but data to support this approach arelimited.Postoperatively,patientsare ollowed with astingserum gastrin levels,secretin stimulation tests,octreotide scans,andC scans.Inpatients oundtohaveinoperabledisease, chemotherapywith streptozocin,doxorubicin,and 5- luorouracil (5-FU) is used. Other approaches such as somatostatin analogues, inter eron, and chemoembolization also have been used ingastrinomawithsomesuccess.(SeeSchwartz10thed.,p.1392.)
Answer:A Intraductal papillary mucinous neoplasms (IPMNs) usually occur within thehead o thepancreasand arisewithin thepancreatic ducts. he ductal epithelium orms a papillary projection into the duct, and mucin production causes intraluminal cystic dilation o the pancreatic ducts (Fig. 33-3). Imaging studiesdemonstratedi usedilationo thepancreaticduct,and the pancreatic parenchyma is o ten atrophic due to chronic duct obstruction. However, classic eatures o chronic pancreatitis, such as calci ication and a beaded appearance o the duct, are not present. At ERCP, mucin can be seen extruding rom theampullao Vater,aso-calledfish-eyelesion,thatisvirtuallydiagnostic o IPMN (Fig. 33-4). (See Schwartz 10th ed., Figures33-77and33-8,pp.1411–1412.)
265
FIG.33-3. Intraductalpapillarymucinousneoplasm histology. Papillaryprojectionso ductalepithelium resemble villous morphologyand contain mucin- illed vesicles.(From Asiyanbola B, Andersen DK.IPMN.EditorialUpdate.AccessSurgery,McGraw-Hill, 2008,with permission.CopyrightThe McGraw-HillCompanies,Inc.)
266 A B C
FIG.33-4. Intraductalpapillarymucinousneoplasm (IPMN).A. Exampleso “ish-eye de ormity”o IPMN.Mucin isseen extruding rom the ampulla.B. Mucin coming rom pancreaticduct when necko pancreasistransected during Whipple procedure (le t).Intraoperative pancreaticductoscopyto assessthe pancreatictail(right).C.Viewso pancreatic duct during ductoscopy;normal(le t)and IPMN(right).
11. Pain rom chronicpancreatitiscan becaused by
A. Ductalhypertension
B. Parenchymaldisease
C. Obstructivepancreatopathy
D. Allo theabove
12. A patient undergoing the Frey procedure to relieve pain rom obstructive pancreatopathy is ound to have 85% parenchymal fbrosis. Te percentage o pain relie the patient islikelytoexperienceis
A. 50%
B. 10%
C. 100%
D. 60%
Answer:D
Pain rom chronic pancreatitis has been ascribed to three possible etiologies. Ductal hypertension, due to strictures or stones,maypredisposeto pain that isinitiated or exacerbated by eating. Chronic pain without exacerbation may be related to parenchymaldisease or retroperitonealin lammation with persistent neural involvement. Acute exacerbations o pain in the setting o chronic pain may be due to acute increases in duct pressureor recurrent episodeso acutein lammation in thesettingo chronicparenchymaldisease.Nealon and Matin havedescribed thesevariouspain syndromesasbeingpredictive o the response to various surgical procedures. Pain that is ound in association with ductal hypertension is most readily relieved by pancreatic duct decompression, through endoscopicstentingor surgicaldecompression.(SeeSchwartz 10th ed.,p.1371.)
Answer:C
he surgical relie o pain due to obstructive pancreatopathy maybe dependent on thedegreeo underlying ibrosisrather than thepresenceo ductalobstruction,per se,accordingto a recent study rom Johns Hopkins byCooper et al. hirty- ive patients with chronic pain associated with evidence o duct obstruction weretreated with localresection o thepancreatic head and longitudinal pancreatico-jejunostomy (LR-LPJ), or Frey procedure, and the degree o pain resolution a ter surgery was compared to the degree o underlying parenchymal ibrosis. A ter a ollow-up that averaged 22 months, patients with more than 80% ibrosis had 100% pain relie , whereas only 60% patients with less than 10% ibrosis experienced substantial or complete pain relie (Fig. 33-5). hese indings suggest that minimal ibrosis,or “minimalchangechronicpancreatitis,”mayproduce chronic pain due to extra-pancreatic or “peri-pancreatic”in lammatoryevents which are not ameliorated bydecompression.(SeeSchwartz10th ed.,Figure33-31, pp.1371–1372.)
FIG.33-5. Pain relie rom chronicpancreatitistreated with the Frey procedure correlateswith the degree o underlying ibrosis.Percent o patientswith pain relie orthose with mild orminimal ibrosis (MIF,n=13),intermediate ibrosis(INF,n=7),and severe orextensive ibrosis(SEF,n=14).P<0.05 orMIFversusSEFbychi-square analysis. (Reproduced with permission rom CooperM,MakaryMA,NgY, et al.Extent o pancreatic ibrosisasa determinant o symptom resolution a terthe Freyprocedure:Aclinicopathologic analysis. JGastrointestSurg17:682-687;2013.)
267
80 75 70 65 60 MIF INF SEF
13. Te only therapy shown to prevent the progression o chronicpancreatitisis
A. Pancreaticduct decompression
B. Major resection
C. ransduodenalsphincteroplasty
D. Roux-en-Ypancreaticojejunostomy
Answer:A hetraditionalapproachtosurgicaltreatmento chronicpancreatitis and its complications has maintained that surgery shouldbeconsidered onlywhen themedicaltherapyo symptomshas ailed.Nealon and hompson published alandmark study in 1993, however, that showed that the progression o chronicobstructivepancreatitiscouldbedelayedorprevented bypancreaticduct decompression.No other therapyhasbeen shown to prevent theprogression o chronicpancreatitis,and this studydemonstrated the role o surgeryin the earlymanagement o the disease ( able 33-3). Small-duct disease or “minimal change chronic pancreatitis” are causes or uncertaintyover thechoiceo operation,however.Major resections have a high complication rate, both early and late, in chronic alcoholic pancreatitis, and lesser procedures o ten result in symptomatic recurrence. So the choice o operation and the timingo surgeryare based on each patient’s pancreatic anatomy,thelikelihood (or lack thereo ) that urther medicaland endoscopictherapywillhalt thesymptomso thedisease,and the chance that a good result will be obtained with the lowest risk o morbidity and mortality. Finally, preparation or surgeryshould include restoration o protein-caloric homeostasis, abstinence rom alcohol and tobacco, and a detailed review o the risks and likelyoutcomes to establish a bond o trust and commitment between the patient and the surgeon. (SeeSchwartz10th ed., able33-20,p.1382.)
TABLE33-3 Efect o surgicaldrainage on progression o chronicpancreatitis
TreatmentGroup 24-MonthEvaluation
Operated(n=47) Mildtomoderate48(87%);severe6(13%)
Nonoperated(n=36) Mildtomoderate8(22%);severe28(78%)
Eighty-three patientswith chronicpancreatitiswere evaluated byexocrine,endocrine, nutritional,and endoscopicretrograde cholangiopancreatographystudies,and allhad mild to moderate disease and dilated pancreaticducts.APuestow-type duct decompression procedure wasper ormed in 47patients,and allsubjectswere restaged bythe same methods24monthslater.
Source:Reproduced with permission rom NealonWH,Thompson JC.Progressive losso pancreatic unction in chronicpancreatitisisdelayed bymain pancreaticduct decompression.Alongitudinalprospective analysiso the modi ed puestowprocedure.Ann Surg 217:458,discussion 466,1993.
268
14. Te part o the pancreas resected in order to ensure success ul resolution o pain long-term or patients with chronicpancreatitisis
A. Tehead
B. Tebody
C. Teneck
D. Tetail
15. In pylorus-preserving resections o the pancreas, the techniquewith thelowest rateo pancreaticleakageis
A. Stent
B. Glue
C. Octreotide
D. Allo theabove
Answer:A he common element o these variations on the theme o LR-LPJ remains the excavation or “coring out” o the central portion o the pancreatic head. It remains uncertain, however, whether and to what degree the dichotomy needs to be extended into the body and tail. he logical conclusion o all o these e orts is that the head o the pancreas is the nidus o thechronicin lammatoryprocessin chronicpancreatitis,and that removal o the central portion o the head o the gland is the keyto the success ul resolution o pain long-term. (See Schwartz10th ed.,pp.1342and 1387.)
Answer:D
hepreservation o thepylorushasseveraltheoreticaladvantages, including prevention o re lux o pancreaticobiliary secretions into the stomach, decreased incidence o marginal ulceration, normal gastric acid secretion and hormone release, and improved gastric unction. Patients with pyloruspreserving resections have appeared to regain weight better than historiccontrolsin some studies.Return o gastricemptying in the immediate postoperative period maytake longer a ter the pylorus-preservingoperation, and it is controversial whether there is any signi icant improvement in long-term qualityo li ewith pyloricpreservation.
echniques or the pancreaticojejunostomy include endto-side or end-to-end and duct-to-mucosa sutures or invagination (Fig. 33-6). Pancreaticogastrostomy has also been investigated.
Some surgeons use stents, glue to seal the anastomosis, or octreotide to decrease pancreatic secretions. No matter what combination o these techniques is used, the pancreatic leakage rate is always about 10%. here ore, the choice o techniques depends more on the surgeon’s personal experience. (SeeSchwartz10th ed.,Figure33-73,p.1403.)
269
270 A B C E D
FIG.33-6. Techniques orpancreaticojejunostomy.Ato D. Duct-to-mucosa,end-to-side.E. Intraoperative photographso end-to-side pancreaticojejunostomy.Fto J. End-to-end invagination.Kto O. End-to-side invagination.
271 F G H J I
FIG.33-6. (Continued)
272 K L N O M
FIG.33-6. (Continued)
16. Te prognosis actor that does NO decrease survival ratesin patientswith gastrinomasis
A. Liver metastases
B. Absenceo MEN1
C. Lymph nodemetastases
D. Primarytumor located outsideo Passarotriangle
Answer:C
Fi ty percent o gastrinomas metastasize to lymph nodes or the liver, and are there ore considered malignant. Patients who meet criteria or operabilityshould undergo exploration or possible removal o the tumor. Although the tumors are submucosal, a ull-thickness excision o the duodenal wall is per ormed i a duodenal gastrinoma is ound. All lymph nodes in Passaro triangle are excised or pathologic analysis. I thegastrinomais ound in thepancreasand doesnot involve the main pancreatic duct, it is enucleated. Pancreatic resection is justi ied or solitary gastrinomas with no metastases. A highly selective vagotomy can be per ormed i unresectable diseaseisidenti ied or i the gastrinoma cannot be localized. his mayreduce the amount o expensive proton pump inhibitors required. In cases in which hepatic metastases are identi ied, resection is justi ied i the primary gastrinoma is controlled and the metastases can be sa ely and completely removed. Debulking or incomplete removal o multiple hepatic metastases is probably not help ul, especially in the setting o MEN1. he application o new modalities such as radio requencyablation seemsreasonable,butdatatosupport this approach are limited. Postoperatively, patients are ollowed with asting serum gastrin levels, secretin stimulation tests,octreotidescans,andC scans.In patients oundtohave inoperable disease, chemotherapy with streptozocin, doxorubicin, and 5- luorouracil (5-FU) is used. Other approaches such assomatostatin analogues,inter eron,and chemoembolization alsohavebeen used in gastrinomawith somesuccess. Un ortunately,abiochemicalcureisachieved in onlyabout one-third o the patients operated on or Zollinger-Ellison syndrome (ZES). Despite the lack o success, long-term survival rates are good, even in patients with liver metastases. he15-year survivalrate or patientswithout liver metastases is about 80%, while the 5-year survival rate or patients with liver metastases is 20 to 50%. Pancreatic tumors are usually larger than tumors arising in the duodenum, and more o ten have lymph node metastases. In gastrinomas, liver metastases decrease survival rates, but lymph node metastases do not. he best results are seen a ter complete excision o small sporadic tumors originating in the duodenum. Large tumors associated with liver metastases, located outside o Passaro triangle, have the worst prognosis. (See Schwartz 10th ed., p. 1392.)
273
1. Which o the ollowingisNO alocation whereaccessory spleenscan be ound?
A. Gastrocolicligament
B. Gerota’s ascia
C. Largebowelmesentery
D. Broad ligament
2. Which o the ollowingsplenic ligaments is NO an avascular plane?
A. Gastrosplenic
B. Splenocolic
C. Phrenosplenic
D. Splenorenal
Answer:B
he most common anomaly o splenic embryology is the accessoryspleen. Present in up to 20%o the population, one ormoreaccessoryspleen(s)mayoccurinupto30%o patients with hematologic disease. Over 80%o accessory spleens are ound in theregion o thesplenichilum and vascular pedicle. Other locations or accessory spleens in descending order o requency are: the gastrocolic ligament, the tail o the pancreas,thegreater omentum,thegreater curveo thestomach, thesplenocolicligament,thesmallandlargebowelmesentery, thele t broad ligament in women,and thele t spermaticcord in men.(SeeSchwartz10th ed.,p.1424.)
Answer:A
O particular clinical relevance, the spleen is suspended in position byseveralligamentsand peritoneal oldstothecolon (splenocolic ligament);the stomach (gastrosplenic ligament); thediaphragm(phrenosplenicligament);thekidney,theadrenal gland, and the tail o the pancreas (splenorenal ligament) (Fig. 34-1). Whereas the gastrosplenic ligament contains the shortgastricvessels,theremainingligamentsareusuallyavascular, with rare exceptions, such as in a patient with portal hypertension. he relationship o the pancreas to the spleen also has important clinical implications. In cadaveric anatomic series, the tail o the pancreas has been demonstrated to lie within 1 cm o the splenic hilum 75% o the time and to actually abut the spleen in 30%o patients. (See Schwartz 10th ed.,Figure34-2,p.1425.)
275 34
Spleen
CHAPTER
3. Allo the ollowingare unctionso thespleen EXCEP
A. Clearance o damaged or aged red blood cells (RBCs) rom theblood.
B. Extramedullary site or hematopoiesis and recycling iron.
C. Initiation o adaptiveimmuneresponse rom ltration o lymph.
D. Clearanceo encapsulatedbacteria romthebloodstream.
Answer:C
he spleen has both ast and slow circulation o blood. It is duringslowcirculation that blood travels through the reticular spacesandspleniccordswhereitisexposedtocontactwith splenic macrophages which remove senescent blood cells. hrough this process the spleen is also able to remove erythrocyte inclusions such as Heinz bodies without lysing the cells. hroughthereticuloendothelialsystem thespleen clears encapsulated bacteria such as pneumococcus and Haemophilus influenzae which are poorly opsonized rom the hepatic reticuloendothelial system. In addition to these unctions the spleen servesasan extramedullarysite or hematopoiesisand playsa unctionalrolein therecyclingo iron.Whilethewhite pulp o thespleen isimportant in theinitiation o theadaptive immuneresponse,materialisdelivered to thespleen through theblood and not thelymph.(SeeSchwartz10th ed.,p.1427.)
4. Whicho the ollowingproteinsisnotalteredin hereditary spherocytosis(HS)?
A. Pyruvatekinase
B. Spectrin
C. Ankyrin
D. Band 3protein
Answer:A
heunderlyingabnormalityin hereditaryspherocytosis(HS) isan inheriteddys unction orde iciencyin oneo theerythrocytemembraneproteins(spectrin,ankyrin,band 3protein,or protein 4.2),which resultsin destabilization o themembrane lipidbilayer. hisdestabilization allowsareleaseo lipids rom the membrane, causing a reduction in membrane sur ace area and a lack o de ormability, leading to sequestration and destruction o thespherocyticerythrocytesin thespleen.
Although less common than glucose-6-phosphate dehydrogenase (G6PD) de iciency overall, pyruvate kinase de iciency is the most common RBC enzyme de iciency to cause congenitalchronichemolyticanemia.(SeeSchwartz10th ed., p.1429.)
276
Gastrosplenic ligament
Lessersac
Greateromentum
Splenocolic ligament
Sustentaculum lienis
Phrenicocolic ligament
FIG.34-1. Suspensoryligamentsofthe spleen.(Data from Poulin EC,Thibault C.The anatomicalbasisforlaparoscopic splenectomy. CanJSurg 36:484;1993.)
5. Splenectomyisindicatedasatreatmentin whicho the ollowingconditions?
A. Cold-antibodyautoimmunehemolyticanemia(AIHA)
B. Hodgkin’sdisease
C. G6PDde ciency
D. Abscesseso thespleen
6. Te disproportionately high rate o overwhelming postsplenectomy in ection (OPSI) in thalassemia patients is thought to be due to an immune de ciency. Which o the ollowingstrategieshasbeen shown toreducemortality?
A. Partialsplenectomy.
B. Prophylacticantibiotictherapy.
C. Delayingsplenectomyuntila er 2yearso age.
D. rans usion to maintain a hemoglobin (HGB) o >9mg/dL.
Answer:D
Autoimmunehemolyticanemias(AIHA)arecharacterizedby destruction o RBCs due to autoantibodies against RBC antigens.AIHAisdivided intowarm and cold categoriesbased on thetemperatureat which theautoantibodiesexert their e ect. In cold-agglutinin disease severe symptoms are uncommon and splenectomy is almost never indicated. Warm-agglutinin disease presents with mild jaundice as well as symptoms andsignso anemiawith one-thirdtoone-hal o patientspresentingwithsplenomegaly.Initialtreatmentiswithcorticosteroidswith splenectomybeingsecond-linetherapywith ailure o steroids. Although splenectomy has a 60 to 80% response rate recurrence iscommon.Hodgkin’s disease is a disorder o the lymphoid system characterized by the presence o ReedSternberg cells. Most patients present with lympadenopathy above the diaphragm with adenopathy below the diaphragm rare on presentation. Adenopathy below the diaphragm can arisewithdiseaseprogression andthespleen iso ten an occult site o spread although splenomegaly is uncommon. While splenectomyisper ormed or surgicalstagingin certain cases including clinical suspicion o lymphoma without evidence o peripheraldiseaseor restaging or suspicion o ailurea ter chemotherapy, staging laparotomy is less commonly perormed in the current era o minimally invasive surgery and advanced imaging techniques. G6PD de iciency is the most common RBCenzymede iciencyand can becharacterized by chronic hemolytic anemia, acute intermittent hemolytic episodes, or no hemolysis depending on the variant. reatment or G6PD de iciency involves avoidance o drugs known to precipitate hemolysis. reatment or splenic abscess involves initiation o broad-spectrum antibioticswith tailoringo antibiotictherapyoncecultureresultsbecomeavailableaswellas splenectomy. While splenectomy is the procedure o choice percutaneousor open drainageareoptions or patientsunable totoleratesplenectomy.(SeeSchwartz10th ed.,p.1431.)
Answer:A
he increase in in ectious complications associated with splenectomy in thalassemia patients is thought to be due to a coexisting immune de iciency that is caused by iron overload.Iron overload isassociated with both thalassemiaaswell asthetrans usionsthat accompanytreatment or thalassemia. Some investigators have tried partial splenectomy with some success in reducingmortalityassociated with splenectomyin these patients. In addition, splenectomy should be delayed until the patient is older than 4 years unless absolutelynecessary. While trans usion to maintain a hemoglobin (HGB) o >9 mg/dLis part o the treatment or thalassemia it does not reduce in ectious complications associated with splenectomy in these patients. here is little evidence supporting e icacy o prophylactic antibiotics in asplenic patients in preventing in ectious complications associated with splenectomy. (See Schwartz10th ed.,p.1432.)
277
7. A 30-year-old woman presents to her primary care provider with complaints o bleeding gums while brushing her teeth as well as menorrhagia and several episodes o epistaxis within the past month. She has been previously healthy with no prior medical problems or surgeries. Examination reveals petechiae and ecchymosis over the lower extremities. Laboratory results show white blood cell (WBC) count 7000/mm3, HGB 14 g/dL, hematocrit (HC ) 42%, and platelet count 28,000/mm3 with numerous megakaryocytes on peripheral smear. First-line therapy or thiscondition would be
A. Oralprednisone
B. Intravenous(IV)immunoglobulin
C. Rituximab
D. Splenectomy
8. Temost common physical ndingin apatient with hairy cellleukemia(HCL)is
A. Massivesplenomegaly
B. Shortnesso breath
C. Abdominalpain
D. Joint pain
9. Which o the ollowingisan indication or splenectomyin apatient with chronicmyelogenousleukemia(CML)?
A. Failureo chemotherapytodecreasesplenomegaly
B. Sequestration requiringtrans usion
C. Symptomaticrelie o earlysatiety
D. Presenceo bcr genemutation
Answer:A Idiopathic thrombocytopenic purpura (I P) is an autoimmune disorder characterized by a low platelet count and mucocutaneousand petechialbleeding. heusual irst lineo therapy or I Pisoralprednisonewith most responsesoccurring within the irst 3 weeks a ter initiating therapy. Intravenous(IV) immunoglobulin isgiven or internalbleedingwith platelet counts<5000/mm3,when extensivepurpuraexists,or to increase platelets preoperativelyand is thought to work by impairing clearance o immunoglobulin G-coated platelets by competing or binding to tissue macrophage receptors Both rituximab and thrombopoietin-receptor antagonists are second-line treatment options. Splenectomy is an option or re ractoryI Pandcan provideapermanentresponseinabout 75to85%o patients.(SeeSchwartz10th ed.,pp.1432–1433.)
Answer:A Hairy cell leukemia (HCL) is an uncommon blood disorder, representing only 2% o all adult leukemias. HCL is characterized bysplenomegaly, pancytopenia, and large numbers o abnormal lymphocytes in the bone marrow. hese lymphocytescontain irregular hair-likecytoplasmicprojectionsidenti iable on the peripheral smear. Most patients seek medical attention because o symptoms related to anemia, neutropenia, thrombocytopenia, or splenomegaly. he most common physical inding is splenomegaly, which occurs in 80% o patientswith HCLand thespleen iso ten palpable5cm below the costal margin. Many patients with HCL have ew symptoms and require no speci ic therapy. reatment is indicated or those with moderate to severe symptoms related to cytopenias,such asrepeated in ectionsor bleedingepisodes,or to splenomegaly, such as pain or earlysatiety. Splenectomydoes notcorrecttheunderlyingdisorder,butdoesreturncellcounts tonormalin 40to70%o patientsandalleviatespain andearly satiety.Newer chemotherapeuticagents(thepurineanalogues 2ʹ-deoxyco ormycin [2ʹ-DCF] and 2-chlorodeoxyadenosine [2-CdA]) are able to induce durable complete remission in most patients.(SeeSchwartz10th ed.,p.1434.)
Answer:C
Chronic myelogenous leukemia (CML) is a disorder o the primitive pluripotent stem cells in the bone marrow, resulting in a signi icant increase in erythroid, megakaryotic, and pluripotent progenitors in the peripheral blood smear. he genetic hallmark is a transposition between the bcr gene on chromosome 9 and the abl gene on chromosome 22. CML accounts or 7 to 15% o all leukemias, with an incidence o 1.5in 100,000in theUnited States.CMLis requentlyasymptomatic in the chronic phase, but symptomatic patients o ten present with the gradual onset o atigue, anorexia, sweating, and le t upper quadrant pain and early satiety secondary to splenomegaly. Enlargement o the spleen is ound in roughly one-hal o patients with CML. Splenectomy is indicated to easepain and earlysatiety.(SeeSchwartz10th ed.,p.1435.)
278
10. Which o the ollowing is an indication or splenectomy in polycythemiavera(PV)?
A. Failureo aspirin topreventthromboticcomplications
B. Frequent need or phlebotomy
C. Symptomsrelated tosplenomegaly
D. Prevention o progression tomyeloid metaplasia
Answer:C
Polycythemia vera (PV) is a clonal, chronic, progressive myeloproli erative disorder characterized by an increase in RBC mass, requently accompanied by leukocytosis, thrombocytosis, and splenomegaly. Patients a ected by PV typicallyenjoyprolonged survivalcompared to othersa ected by hematologic malignancies,but remain at risk or trans ormation to myelo ibrosis or acute myeloid leukemia (AML). he disease is rare, with an annual incidence o 5 to 17 cases per million population. Although the diagnosis may be discovered by routine screening laboratory tests in asymptomatic individuals,a ected patientsmaypresent with anynumber o nonspeci iccomplaints,includingheadache,dizziness,weakness, pruritus, visual disturbances, excessive sweating, joint symptoms, and weight loss. Physical indings include ruddy cyanosis,conjunctivalplethora,hepatomegaly,splenomegaly, and hypertension. he diagnosis is established byan elevated RBC mass (>25%o mean predicted value), thrombocytosis, leukocytosis, normal arterial oxygen saturation in the presenceo increased RBC mass,splenomegaly,lowserum erythropoietin (EPO) stores, and bone marrow hypercellularity. reatment should be tailored to the risk status o the patient andranges rom phlebotomyandaspirin tochemotherapeutic agents.As in essentialthrombocythemia (E ) splenectomyis not help ul in the early stages o disease and is best reserved or late-stagepatientsin whom myeloid metaplasia hasdeveloped and splenomegaly-related symptoms are severe. (See Schwartz10th ed.,pp.1435–1436.)
11. Which o the ollowing is the most common etiology o spleniccystsworldwide?
A. Bacterialin ection
B. rauma
C. Parasiticin ection
D. Congenitalanomaly
12. Which o the ollowingisan indication or surgicaltreatment o asplenicaneurysm?
A. Pregnancy
B. Size>1.5cm
C. Historyo thrombocytopenia
D. Historyo neutropenia
Answer:C
Splenic cysts are rare lesions. he most common etiology or splenic cysts worldwide is parasitic in estation, particularly echinococcal. Symptomatic parasitic cysts are best treated with splenectomy, though selected cases may be amenable to percutaneous aspiration, instillation o protoscolicidal agent, and reaspiration. Nonparasitic cysts most commonly result rom trauma and are called pseudocysts; however, dermoid, epidermoid, and epithelial cysts have been reported as well. he treatment o nonparasitic cysts depends on whether or not they produce symptoms. Asymptomatic nonparasitic cysts may be observed with close ultrasound ollow-up to exclude signi icant expansion. Patients should be advised o the risk o cyst rupture with even minor abdominal trauma i they elect nonoperative management or large cysts. Small symptomatic nonparasitic cysts may be excised with splenic preservation, and large symptomatic nonparasitic cysts may beunroo ed.Botho theseoperationsmaybeper ormedlaparoscopically.(SeeSchwartz10th ed.,pp.1436–1437.)
Answer:A
Although rare, splenic artery aneurysm (SAA) is the most common visceral artery aneurysm. Women are our times more likely to be a ected than men. he aneurysm usually arisesin themiddleto distalportion o thesplenicartery. he risk o rupture is between 3%and 9%;however, once rupture occurs, mortality is substantial (35–50%). According to a recent series,mortalityissigni icantlyhigher in patientswith underlying portal hypertension (>50%) than in those without it (17%). SAA is particularlyworrisome when discovered
279
13. A 45-year-old man presents to the emergency department with emesis o bright red blood.Laboratoryresults include HGB10g/dL,HC 30%, platelets 300,000/mm3 , internationalnormalized ratio(INR)1.0,aspartatetransaminase (AS ) 30 U/L, alanine transaminase (AL ) 45U/L,and albumin 4.0g/dL.A er appropriate resuscitation he undergoes esophagogastroduodenoscopy (EGD) which is notable or gastric varices. What is the appropriatetreatment or hiscondition?
A. ransjugular intrahepaticportosystemicshunt
B. Varicealband ligation
C. Splenorenalshunt
D. Splenectomy
14. Which o the ollowingisNO part o thetriad seen with Feltysyndrome?
A. Rheumatoid arthritis(RA)
B. Splenomegaly
C. Neutropenia
D. Trombocytopenia
during pregnancy, as rupture imparts a high risk o mortality to both mother (70%) and etus (95%). Most patients are asymptomatic and seek medical attention based on an incidentalradiographic inding.About 20%o patientswith SAA have symptoms o le t upper quadrant pain. Indications or treatment include presence o symptoms, pregnancy, intention to become pregnant, and pseudoaneurysms associated with in lammatoryprocesses.For asymptomaticpatients,size greater than 2cm constitutesan indication or surgery.Aneurysm resection or ligation alone is acceptable or amenable lesions in the mid-splenic artery, but distal lesions in close proximity to the splenic hilum should be treated with concomitant splenectomy.An excellent prognosis ollowselective treatment. Splenic arteryembolization has been used to treat SAA, but pain ul splenic in arction and abscess may ollow. (SeeSchwartz10th ed.,p.1438.)
Answer:D
While portal hypertension is most commonly a result o cirrhosis it can result rom other causes such as splenic vein thrombosis. Patients with splenic vein thrombosis can present with bleeding rom gastric varices in the setting o normal liver unction test results. hese patients also o ten have ahistoryo pancreaticdisease.Portalhypertension secondary tosplenicvein thrombosisispotentiallycurablewith splenectomy.(SeeSchwartz10th ed.,p.1438.)
Answer:D
he triad o rheumatoid arthritis (RA), splenomegaly, and neutropenia is called Felty syndrome. It exists in approximately 3% o all patients with RA, two-thirds o which are women. Immune complexes coat the sur ace o WBCs, leading to their sequestration and clearance in the spleen with subsequent neutropenia. his neutropenia (<2000/mm3) increasestherisk or recurrent in ectionsand o ten drivesthe decision or splenectomy. he size o the spleen is variable, rom nonpalpable in 5to 10%o patients,to massive enlargement in others. he spleen in Felty syndrome is our times heavier than normal. Corticosteroids, hematopoietic growth actors, methotrexate, and splenectomy have all been used to treat the neutropenia o Felty syndrome. Responses to splenectomyhavebeen excellent,with over 80%o patientsshowing a durable increase in WBC count. More than one-hal o patientswho had in ectionsprior to surgerydid not have any in ections a ter splenectomy. Besides symptomatic neutropenia, other indications or splenectomy include trans usiondependent anemia and pro ound thrombocytopenia. (See Schwartz10th ed.,p.1438.)
280
15. Which o the ollowing is the most e ective prevention strategyagainst OPSI?
A. Vaccination 2weeksa er splenectomy
B. Vaccination 2weeksbe oresplenectomy
C. Dailyantibioticprophylaxis
D. Carrying a reserve supply o antibiotics or seladministration
Answer:B
Asplenicpatientshavean increased susceptibilityto in ection or the remainder o their lives and although the overall li etimerisk o OPSI islowtheconsequencescan bedevastating. Patients undergoing splenectomy or hematologic or malignant indications have a greater risk o OPSI than patients undergoing splenectomy or trauma or iatrogenic injury, and OPSIismorecommon in children than adults.Providersneed to have a high index o suspicion when evaluating asplenic patients or possiblein ection.Patient education and vaccinations against encapsulated pathogens is the mainstay o preventativetherapy.Patientsshould bevaccinated 2weeksprior to elective splenectomyin order to optimize antigen recognition and processing. I splenectomyis per ormed emergently vaccinations are given postoperatively with an attempt to delayadministration or 2weekstoavoid thetransient immunosuppression associated with surgery. hereislittleevidence supporting e icacy o prophylactic antibiotics in asplenic patients and vaccination remains the most e ective prevention strategy.(SeeSchwartz10th ed.,p.1439.)
16. All o the ollowingare true regardinglaparoscopic splenectomyEXCEP
A. It isassociated with shorter hospitalstays.
B. It is associated with increased intraoperative blood loss.
C. It isassociated with decreased morbidity.
D. Patients are positioned in the right lateral decubitus position or the45°right lateraldecubitusposition.
Answer:B
Laparoscopicsplenectomyhasbecomethe avored procedure versus open splenectomy or elective splenectomy over the past two decadesand isnowconsidered thegold standard or elective splenectomy in patients with normal-sized spleens. With experienced surgeonslaparoscopicsplenectomyisassociated with decreased intraoperative blood loss, shorter hospital length o stay, and lower morbidityrates as compared to open splenectomy. Laparoscopic splenectomy is o ten perormed with thepatient in theright lateraldecubitusposition, patients are sometimes placed in a 45°right lateral decubitus position to acilitateeasier access or concomitant procedures such aslaparoscopiccholecystectomy.(SeeSchwartz10th ed., p.1440.)
17. What is the most common complication ollowing open splenectomy?
A. Pancreatitis
B. Le lower lobeatelectasis
C. Pleurale usion
D. Wound in ection
Answer:B
Complications ollowing splenectomy can be divided into pulmonary, hemorrhagic, in ectious, pancreatic, and thromboembolic. Pulmonary complications include le t lower lobe atelectasis, pleural e usion, and pneumonia with le t lower lobeatelectasisbeingthemost common complication overall. Hemorrhagic complications include intraoperative hemorrhage,postoperativehemorrhage,and subphrenichematoma. In ectious complications include subphrenic abscess and wound in ection.Placement o a drain in thele t upper quadrant can be associated with postoperative subphrenic abscess and isthere orenot routinelyrecommended.Pancreaticcomplications include pancreatitis, pseudocyst ormation, and pancreatic istula and o ten result rom intraoperativetrauma tothepancreasduringdissectiono thesplenichilum. hromboembolic complications include deep vein thrombosis and portalvein thrombosis.(SeeSchwartz10th ed.,p.1444.)
281
18. Which o the ollowing patients is at highest risk or OPSI?
A. A30-year old whounderwent splenectomy or I P.
B. A25-year old who underwent splenectomy or iatrogenicbleedinga er atotalcolectomy.
C. A3-year old who underwent splenectomy or hereditaryspherocytosis.
D. A 4-year old who underwent splenectomy due to bleedinga er amotor vehiclecrash.
Answer:C
While the all li etime risk o OPSI is low (ranging rom <1–5%) the consequences are serious. he reason or splenectomy is the single most in luential determinant o OPSI risk. here is evidence that those who undergo splenectomy or hematologicdiseaseare ar moresusceptibleto OPSI than patients who undergo splenectomy or trauma or iatrogenic reasons.When takingageintoconsideration children whoare 5yearso ageor younger and adultswho are50yearso ageor older seemtobeatan elevatedrisk. heintervalsincesplenectomyalso seemsto bea actor with thegreatest riskoccurring in the irst 2 years a ter splenectomy, however, it is important to remember that cases o OPSI can occur decades later and asplenic patients remain at li elong risk. (See Schwartz 10th ed.,p.1445.)
19. Which o the ollowing asplenic patients should receive prophylacticantibiotictherapytoprotect against OPSI?
A. A35-year-old man undergoingatooth extraction.
B. A4-year-oldchildwhorecentlyunderwentsplenectomy.
C. A 15-year-old boy who underwent splenectomy at age13.
D. Tereislittleevidencesupportingefcacyo prophylacticantibiotics.
Answer:D
Antibiotic therapyin asplenic patients alls into three categories:deliberatetherapy or establishedor presumedin ections, prophylaxis in anticipation o invasive procedures, and generalprophylaxis. hereislittleevidencesupportinge icacyo prophylacticantibioticsin anticipation o invasiveprocedures or e icacyo general prophylaxis and guidelines are not uniorm. Common recommendations include daily antibiotics until 5 years o age or at least 5 years a ter splenectomy with some advocating continuing antibiotics until young adulthood, however, there is little evidence supporting e icacy. It isunlikelythat randomized controlled trials on this issue will beper ormed duetothelowincidenceo overwhelmingpostsplenectomyin ection aswellasitsseriousconsequences.(See Schwartz10th ed.,p.1445.)
282
CHAPTER
AbdominalWall,Omentum, Mesentery,and Retroperitoneum
1. A cutaneous malignancy o the anterior abdominal wall
2inchesabovetheumbilicuswilldrain towhich lymphatic basin?
A. Umbilical
B. Axillary
C. Retroperitoneal
D. Inguinal
2. Teappropriatetreatment o rectusabdominisdiastasisis
A. Observation
B. Resection and primaryrepair
C. Mesh overlay
D. Lateralcomponent separation
3. Persistence o the vitelline duct can lead to which o the ollowing?
A. Colonicdiverticulum
B. Urachalcyst
C. Umbilicalcord hernia
D. Omphalomesentericduct cyst
4. Teusualpresentation o arectussheath hematomais
A. Unexplained anemia
B. Abdominalwallbulge
C. Sudden abdominalpain
D. Inabilitytostand erect
Answer:B
he lymphatic drainage o the anterior abdominal wall is principallyto the axillarynodal basin and the inguinal nodal basin. he area o demarcation is roughly the arcuate line (semilunar line o Douglas) at the level o the anterior iliac spine.(SeeSchwartz10th ed.,p.1450.)
Answer:A
Rectus abdominis diastasis (or diastasis recti) is a separation o the two rectus abdominis muscular pillars. his results in a bulge o the abdominal wall that is sometimes mistaken or a ventral hernia despite the act that the midline aponeurosis is intact and no hernia de ect is present. Computed tomography (C ) scanning can provide an accurate measure o the distance between the rectus pillars and will di erentiate rectus diastasis rom a true ventral hernia. Surgical correction has been described or cosmetic reasons but is unnecessary and risks the ormation o a true postoperative hernia. (See Schwartz10th ed.,p.1453.)
Answer:D
During the third trimester o pregnancy, the vitelline duct regresses.Persistenceo thevitellineduct remnant on theileal border results in a Meckel diverticulum. Complete ailure o the vitelline duct to regress results in a vitelline duct istula which is associated with drainage o small intestinal contents rom the umbilicus. I both the intestinal and umbilical ends o the vitelline duct regress into ibrous cords, a central vitelline duct (omphalomesenteric duct) cyst may occur. (See Schwartz10th ed.,p.1453.)
Answer:C
Hemorrhage rom the network o collateralizing vessels within the rectus sheath and muscles can result in a rectus sheath hematoma.Although ahistoryo trauma maybepresent, a rectus sheath hematoma can ollowvigorous coughing, sneezing, or extreme exertion. It typically occurs in elderly patients or those on anticoagulant therapy. Patients usually reportthesudden onseto unilateralabdominalpain andhave localized tenderness which is not accompanied by peritoneal signs.(SeeSchwartz10th ed.,p.1453.)
283 35
5. A 40-year-old woman who underwent total abdominal colectomy or amilial adenomatous polyposis (FAP)
5 years ago presents with a gradually expanding painless
4 cm mass o the anterior abdominal wall. A biopsy is returned as “desmoid tumor with no sign o malignancy.”
Tecorrect management is
A. Observation
B. Acourseo doxorubicin,dacarbazine,or carboplatin
C. Enucleation
D. Widelocalexcision
6. Repair o anew5-cm midlinepostoperativeventralhernia in an otherwisehealthypatient isbest accomplished with
A. Primarysuturerepair
B. Repair with syntheticmesh
C. Repair with transposition o rectusmuscle
D. Lateralcomponent separation with mesh overlay
7. Which o the ollowing statements regarding umbilical herniasistrue?
A. Umbilicalherniasarepresent in 10%o allnewborns.
B. Umbilical hernias should be repaired as soon as they arediagnosed.
C. Adults with small, nonincarcerated umbilical hernias should undergorepair.
D. Umbilical hernias are associated with disseminated carcinomatosis.
8. Spigelian herniasusuallyoccur
A. On thelateralborder o therectusabdominismuscle
B. In thelineaalba
C. In themedialwallo theinguinalcanal
D. In theposterior costovertebralangle
9. Laparoscopicrepair o incisionalherniasisassociatedwith which o the ollowing?
A. Reduced hospitalcost
B. Reduced recurrencerate
C. Reduced wound in ection rate
D. Reduced seroma ormation
10. Which o the ollowingstatementsabout omentalin arction aretrue?
A. Patientsusuallypresent with ever and lassitude.
B. Most casesarediagnosed on imagingstudies.
C. Most casesdonot requiresurgery.
D. Surgicalresection isindicated in allcases.
Answer:D
Desmoid tumorso theabdominalwallare ibrousneoplasms that occur sporadicallyor in the setting o amilial adenomatous polyposis (FAP). he condition can result in mortality due to aggressive local growth, so radical excision with con irmation o tumor- ree margins o resection is required. Medicaltreatment with an antineoplasticagent such asdoxorubicin, dacarbazine, or carboplatin can produce remission buttheprognosiso advanceddesmoidsispoor.(SeeSchwartz 10th ed.,p.1454.)
Answer:B
Postincisional hernias have an unacceptably high incidence o recurrence a ter primary suture repair. here ore, a mesh repair, per ormed either with an open surgical approach or laparoscopic approach, is pre erred. Muscle transposition procedures are usuallyunnecessaryin relativelysmallde ects that are not recurrent or related to another abdominal wall de ect.(SeeSchwartz10th ed.,p.1455.)
Answer:A
Umbilicalherniasarepresent in 10%o allnewborns,and are morecommon in prematurein ants.Most congenitalumbilical hernias close spontaneously by 5 years o age, so repair should be delayed until examination shows persistence o the hernia be ore the child enters school. Adults with small, uncomplicated, unincarcerated umbilical hernias can be ollowed untilsymptomsoccur.(SeeSchwartz10th ed.,p.1455.)
Answer:A
Spigelian hernias can occur anywhere along the length o the Spigelian lineor zone—an aponeuroticband o variablewidth at the lateral border o the rectus abdominis. he most common location o these rare hernias is at the level o the arcuate line. hese hernias are not always apparent on physical examination, and may cause local pain or incarceration. (See Schwartz10th ed.,p.1455.)
Answer:C
A recent Cochrane database review concluded that shortterm recurrence rates did not di er signi icantly and laparoscopic repairs were associated with higher in-hospital costs despitegenerallyshorter lengthso stay. hemajor bene it or laparoscopicrepairscomparedwithopen repairswasaconsistentlylower risk o wound in ections.(See Schwartz 10th ed., p.1456.)
Answer:C
Interruption o the blood supply to the omentum is a rare cause o symptoms o an acute abdomen. Depending on the location o the in racted tissue, symptoms may mimic acute appendicitis,acutecholecystitis,acutediverticulitis,or per orated ulcer. he diagnosis is usuallyin erred rom abdominal C scan which shows a localized in lammatory-appearing masso theomentum.Surgicalresection can hasten recovery, but clinically stable patients can be managed conservatively. (SeeSchwartz10th ed.,p.1457.)
284
11. A60-year-old woman presentswith abdominalpain,and a C scan revealsan omentalmass.Temost likelydiagnosisis
A. Desmoid tumor
B. Liposarcomao theomentum
C. Omentalin arction
D. Metastaticcarcinoma
12. Failureo xation o thesmallintestinalandright colonic mesenteryduringgestation can result in
A. Chronicconstipation
B. Intestinalmalrotation
C. Umbilicalhernia
D. Intussusception
13. Which o the ollowingstatementsabout sclerosingmesenteritisisFALSE?
A. It isalwaysassociated with difuseabdominalpain.
B. It can appear asamasson C scan.
C. It can improveor resolvewithout surgicaltherapy.
D. It can bemistaken or primaryor metastatictumor.
Answer:D
Primarytumorso theomentum arerare,asisomentalin arction. Metastatic tumors o the omentum are common, with metastatic ovarian carcinoma having the highest preponderance o omental involvement. Cancers o any portion o the gastrointestinal tract, as well as melanoma, uterine, and renal cancer, can metastasize to the omentum. (See Schwartz 10th ed.,p.1457.)
Answer:B
During etaldevelopment,a terthemidgutrotatesandreturns to the abdominal cavity, the mesentery o the duodenum, small intestine, and proximal colon become ixed to the retroperitoneum.Failureo ixation resultsin a spectrum o disorders associated with intestinal malrotation. (See Schwartz 10th ed.,p.1458.)
Answer:A
heetiologyo sclerosingmesenteritisisunknown but itscardinal eature is increased tissue densitywithin the mesentery. his can be associated with a discreet non-neoplastic mass or it can be more di use involving large swaths o thickened mesentery without well-de ined borders. Manycases are discovered incidentally on C scans per ormed or unrelated reasons. he process is sel -limited and may demonstrate regression on ollow-up imaging studies. (See Schwartz 10th ed.,p.1458.)
14. Teprimarytreatment o retroperitoneal brosisis
A. Corticosteroids
B. Cyclosporine
C. Radiation therapy
D. Surgicalresection
Answer:A
Oncemalignancy,drug-induced disease,and in ectiousetiologies are ruled out, corticosteroids are the mainstay o medical therapy. Surgical intervention is reserved or ureterolysis or ureteralstenting, or endovascular interventions or ileocavalobstruction.(SeeSchwartz10th ed.,p.1462.)
285
1. All o the ollowing are true about so tissue sarcoma EXCEP
A. Most common siteistrunkand retroperitoneum.
B. Tere are more than 11,000 new diagnoses o so tissuesarcomaannuallyin theUnited States.
C. Most so tissue sarcoma-specifc deaths are due to uncontrolled pulmonarymetastases.
D. Te overall 5-year survival rate or all stages o so tissuesarcomaapproximates50to60%.
2. Which o the ollowing is NO associated with the development o sarcoma?
A. Radiation exposure
B. Herbicideexposure
C. Chroniclymphedema
D. Historyo trauma
3. Internationale Contre le Cancer (AJCC/UICC) sarcoma stagingsystem?
A. umor size
B. Number o mitoses per high-powered microscopic feld
C. Lymph nodemetastaticstatus
D. Retroperitonealsarcomanomograms
Soft Tissue Sarcomas
Answer:A
Incidence rates are declining or most cancer sites, but they are increasingamongboth men and women or melanoma o the skin, cancers o the liver and thyroid. Incidence rates are decreasing or all our major cancer sitesexcept or breastcancer in women.(SeeSchwartz10th ed.,p.1465.)
Answer:D
External radiation therapy is a rare but well-established risk actor or so t tissue sarcoma that may be associated with radiation-induced mutations o the p53 gene. Exposure to herbicides, such as phenoxyacetic acids and to wood preservatives containing chlorophenols, has been linked to an increased risk o so t tissue sarcoma. In 1948, Stewart and reves irst described theassociation between chroniclymphedema a ter axillary dissection and subsequent lymphangiosarcoma. Although patients with sarcoma o ten report a historyo trauma,nocausalrelationship hasbeen established. More o ten, a minor injury calls attention to a pre-existing tumor.(SeeSchwartz10th ed.,p.1466.)
Answer:D
heseventhedition o theAmerican JointCommitteeon Cancer (AJCC) stagingsystem or so t tissuesarcomasisbased on histologic grade o aggressiveness, tumor size and depth, and the presence o nodal or distant metastases. Histologic grade is the most important prognostic actor or patients with so t tissue sarcoma. he eatures that de ine grade are cellularity, di erentiation (good, moderate, or poor/anaplastic), pleomorphism, necrosis (absent, <50%, or ≥50%), and number o mitoses per high-power ield (<10, 10–19, or ≥20). (See Schwartz10th ed.,p.1470.)
287 36
CHAPTER
4. All o the ollowing are known molecular pathogenic eventsin sarcomaEXCEP
A. Chromosomaltranslocations
B. Oncogeneamplifcation
C. Complexgenomicrearrangements
D. Epigeneticsuppression
5. For a 2G3NOMO sarcoma (stage II),treatment typically consistso
A. Surgeryalone
B. Surgeryand radiotherapy
C. Surgery,radiotherapy,pre-surgicalchemotherapy
D. Surgery,radiotherapy,pre-andpostsurgicalchemotherapy
Answer:D
In general, sarcomas resulting rom identi iable molecular events tend to occur in younger patients with histology suggestingaclear lineo di erentiation. heidenti iablemolecular events include point mutations, translocations causing overexpression o an autocrine grow actor, and oncogenic usion transcription actor producing a cellular environment prone to malignant trans ormation. In contrast, sarcomas without identi iablegeneticchangesor expression pro ilesignatures tend to occur in older patients and exhibit pleomorphic cytology and p53 dys unction. (See Schwartz 10th ed., pp.1466–1467.)
Answer:B
Recommendations or themanagement o so t tissuemasses
1. So t tissue tumors that are enlarging or greater than 3 cm should be evaluated with radiologic imaging (ultrasonography or computed tomography [C ]), and a tissue diagnosisshould bemadeusingcoreneedlebiopsy.
2. Once a sarcoma diagnosis is established, obtain imaging (magneticresonanceimaging or extremitylesionsand C or other anatomic locations) and evaluate or metastatic disease with chest C or intermediate- or high-grade (grade2or 3)or large( 2)tumors.
3. A wide local excision with 1- to 2-cm margins is adequate therapy or low-gradelesionsand 1tumors.
4. Radiation therapy plays a critical role in the management o large( 2),intermediate- or high-gradetumors.
5. Patientswith locallyadvanced high-gradesarcomasor distant metastasesshould beevaluated or chemotherapy.
6. Anaggressivesurgicalapproachshouldbetakeninthetreatment o patientswith an isolated localrecurrenceor resectabledistantmetastases.(SeeSchwartz10thed.,p.1472.)
6. Which o the ollowingistrueabout desmoid tumors?
A. Local recurrence is observed in up to one-third o patients regardless o microscopic margin o resection status.
B. A policy o watch ul waiting or desmoids has been validated in prospectiveclinicaltrials.
C. Analogousto other nonmetastasizingtumors,chemotherapy has no role in the treatment o desmoid tumors.
D. Due to their propensity or local invasion, radiotherapy,when used,must begiven at adoseo 75Gy.
Answer:A
he primarytherapy or desmoid tumors has long been considered surgical resection with wide local excision to achieve negative margins. However, local recurrence occurs in up to one-third o patients independently o the quality o surgical margins. Moreover, some authors advocate the possibility to observe patients at presentation, limiting surgery to those whoprogressor ailmedicaltherapies.Radiation therapymay be e ective in patients with unresectable tumors or as adjuvant therapy ollowingsurgery or recurrent diseasealthough long-term side e ects and the risk o radiation-induced sarcomashouldalwaysbeconsidered.When used,adoseo 50to 54Gyisusuallyrecommended.Systemictreatment isanother option, when surgery is not indicated, although usually reserved or patients with tumor-associated symptoms who have not responded to other interventions. Combinations o methotrexate and vinblastine have been shown to have activity,ashavesingle-agent pegylated liposomaldoxorubicin and sora enib.(SeeSchwartz10th ed.,p.1485.)
288
1. Te incidence o inguinal hernias in men has a bimodal distribution,which peaks
A. Be orethesecond year o li eand a er age50.
B. Be orethe rst year o li eand a er age40.
C. Be oretheeighth year o li eand a er age40.
D. Be orethe h year o li eand a er age50.
CHAPTER
InguinalHernias
Answer:B
Approximately 75% o abdominal wall hernias occur in the groin. he li etime risk o inguinal hernia is 27%in men and 3%in women.O inguinalhernia repairs,90%areper ormed in men and 10%in women. heincidenceo inguinalhernias in men hasa bimodaldistribution,with peaksbe ore the irst year o li e and a ter age 40. Abramson demonstrated the age dependence o inguinal hernias in 1978. hose between ages 25and 34yearshad ali etimeprevalencerateo 15%,whereas those aged 75 years and over had a rate o 47%( able 37-1). Approximately 70% o emoral hernia repairs are per ormed in women; however, inguinal hernias are ive times more common than emoral hernias. he most common subtype o groin hernia in men and women is the indirect inguinal hernia.(SeeSchwartz10th ed.,p.1495.)
Answer:C
Epidemiologic studies have identi ied risk actors that may predispose to a hernia. Microscopic examination o skin o inguinalherniapatientsdemonstrated signi icantlydecreased ratios o type I to type III collagen. ypeIII collagen doesnot contribute to wound tensile strength as signi icantlyas type I collagen.Additionalanalysesrevealed disaggregated collagen tractswith decreased collagen iber densityin herniapatients’ skin. Collagen disorders, such as Ehlers-Danlos syndrome, are also associated with an increased incidence o hernia ormation ( able 37-2). Recent studies have ound an association between concentrations o extracellular matrix elements and hernia ormation.Although asigni icant amount o work remains to elucidate the biologic nature o hernias, current evidence suggests they have a multi actorial etiology with both environmentaland hereditaryin luences.(See Schwartz 10th ed., able37-4,p.1502.)
289 37
TABLE37-1 Inguinalhernia prevalence byage Age(y) 25–34 35–44 45–54 55–64 65–74 75+ Currentprevalence(%) 12 15 20 26 29 34 Lietimeprevalence(%) 15 19 28 34 40 47 Current = repaired herniasexcluded;lietime = repaired herniasincluded.
2. Te two types o collagen ound to exist in a decreased ratioo theskin o inguinalherniapatientsare
A. ypesIand II
B. ypesIIand III
C. ypesIand III
D. ypesIIIand VI
3. According to the Nyhus classi cation system that categorizes hernia de ects by location, size, and type, type IIIC represents
A. Indirect hernia;internal abdominal ringnormal;typicallyin in ants,children,smalladults
B. Direct hernia;sizeisnot taken intoaccount
C. Recurrenthernia;modi ersA-Daresometimesadded, which correspond to indirect, direct, emoral, and mixed.
D. Femoralhernia
TABLE37-2 Connective tissue disordersassociated with groin herniation
Osteogenesisimperecta
Cutislaxa(congenitalelastolysis)
Ehlers-Danlossyndrome
Hurler-Huntersyndrome
Maransyndrome
Congenitalhipdislocationinchildren
Polycystickidneydisease
α1-Antitrypsindeciency
Williamssyndrome
Androgeninsensitivitysyndrome
Robinowsyndrome
Serpentine bulasyndrome
Alportsyndrome
TelHashomercamptodactylysyndrome
Lerichesyndrome
Testiculareminizationsyndrome
Rokitansky-Mayer-Kustersyndrome
Goldenharsyndrome
Morrissyndrome
Gerhardtsyndrome
Menkessyndrome
Kawasakidisease
Pannenstielsyndrome
Beckwith-Wiedemannsyndrome
Rubinstein-Taybisyndrome
Alopecia-photophobiasyndrome
Answer:D
See able37-3.(SeeSchwartz10th ed., able37-2,p.1498.)
TABLE37-3 Nyhusclassifcation system
TypeI Indirecthernia;internalabdominalringnormal;typicallyin inants,children,smalladults
TypeII Indirecthernia;internalringenlargedwithout impingementonthefoorotheinguinalcanal;does notextendtothescrotum
TypeIIIA Directhernia;sizeisnottakenintoaccount
TypeIIIB Indirectherniathathasenlargedenoughtoencroach upontheposterioringuinalwall;indirectslidingor scrotalherniasareusuallyplacedinthiscategory becausetheyarecommonlyassociatedwithextension tothedirectspace;alsoincludespantaloonhernias
TypeIIIC Femoralhernia
TypeIV Recurrenthernia;modiersA–Daresometimesadded, whichcorrespondtoindirect,direct,emoral,andmixed, respectively
290
4. axis
A. Should beper ormed when strangulation issuspected.
B. Re ers to the method o securingmesh to the inguinal ligament.
C. Should not berepeated morethan vetimes.
D. Should be used or incarcerated hernias without sequelaeor strangulation.
Answer:D
Incarceration occurs when hernia contents ail to reduce; however, a minimally symptomatic, chronically incarcerated hernia may also be treated nonoperatively. axis should be attempted or incarcerated hernias without sequelae o strangulation,and theoption o surgicalrepair should bediscussed prior to the maneuver. o per orm taxis, analgesics and light sedatives are administered, and the patient is placed in the rendelenburgposition. heherniasaciselongatedwith both hands, and the contents are compressed in a milking ashion toeasetheir reduction intotheabdomen.
he indication or emergent inguinal hernia repair is impendingcompromiseo intestinalcontents.Assuch,strangulation o hernia contents is a surgical emergency. Clinical signs that indicate strangulation include ever, leukocytosis, and hemodynamic instability. he hernia bulge is usually warm and tender, and the overlying skin may be erythematousor discolored.Symptomso bowelobstruction in patients withslidingor incarceratedinguinalherniasmayalsoindicate strangulation. axis should not be per ormed when strangulation is suspected, as reduction o potentially gangrenous tissue into the abdomen may result in an intra-abdominal catastrophe. Preoperatively, the patient should receive luid resuscitation, nasogastric decompression, and prophylactic intravenousantibiotics.(SeeSchwartz10th ed.,p.1505.)
5. Aherniasacthat extendsintothescrotum may
A. Requireextensivedissection and reduction
B. Requiredivision within theinguinalcanal
C. Requireamputation o thesac
D. Requirethesactobeinverted intothepreperitoneum
Answer:B
In cases where the viability o sac contents is in question, the sac should be incised, and hernia contents should be evaluated or signs o ischemia. he de ect should be enlarged to augment blood low to the sac contents. Viable contents may be reduced into the peritoneal cavity, while nonviable contents should be resected, and synthetic prostheses should be avoided in the repair. In elective cases, the sac maybe amputated at the internal inguinal ring or inverted into the preperitoneum. Both methods are e ective; however, patients undergoingsacexcision had signi icantlyincreased postoperativepain in aprospectivetrial.Dissection o adenselyadherent sac may result in injury to cord structures and should be avoided; however, sac ligation at the internal inguinal ring is necessary in these cases. A hernia sac that extends into the scrotum may require division within the inguinal canal, as extensive dissection and reduction risks injury to the pampini orm plexus,resultingin testicular atrophyand orchitis.
At this point, the inguinal canal is reconstructed, either with native tissue or with prostheses. he ollowing sections describethemost commonlyper ormed typeso tissue-based and prosthetic-based reconstructions.(SeeSchwartz10th ed., p.1505.)
6. Tetechniqueindicated or emoralherniasin caseswhere prostheticmaterialiscontraindicated is
A. TeBassinirepair
B. TeShouldicerepair
C. TeMcVayrepair
D. Lichtenstein tension- reerepair
Answer:C
he McVay repair addresses both inguinal and emoral ring de ects. histechniqueisindicated or emoralherniasand in cases where the use o prosthetic material is contraindicated (Fig. 37-1). Once the spermatic cord has been isolated, an incision in the transversalis ascia permits entryinto the preperitoneal space. he upper lap is mobilized by gentle blunt dissection o underlying tissue. Cooper ligament is bluntly dissected to expose its sur ace. A2- to 4-cm relaxingincision
291
7. General anesthesia induction resulting in reduction o an incarcerated or strangulated inguinal hernia during laparoscopicrepair
A. Requiresnoaction
B. Requiresabdominalexploration or nonviabletissue
C. Requiresimmediateconversion toan open repair
D. Requires proceeding with a totally extraperitoneal ( EP)repair
is made in the anterior rectus sheath vertically rom the pubic tubercle. his incision is essential to reduce tension on the repair; however, it may result in increased postoperative pain and higher risk o ventral abdominal herniation. Using either interruptedorcontinuoussuture,thesuperiortransversalis lap isthen astenedtoCooperligament,andtherepairiscontinued laterally along Cooper ligament to occlude the emoral ring. Lateralto the emoralring,atransition stitch isplaced,a ixing thetransversalis asciatotheinguinalligament. hetransversalisisthen sutured totheinguinalligament laterallytotheinternalring.(SeeSchwartz10thed.,Figure37-17,pp.1507–1508.)
8. TemedicalissueNO associated with hernia recurrence is
A. Malnutrition
B. Steroid use
C. Smoking
D. Alcoholuse
Answer:B
Laparoscopic inguinal hernia repairs rein orce the abdominal wall via a posterior approach. Principal laparoscopic methods include the transabdominal preperitoneal ( APP) repair, the totally extraperitoneal ( EP) repair, and the less commonly per ormed intraperitoneal onlay mesh (IPOM) repair. Although laparoscopic repairs in experienced hands arerelativelyexpedient,theynecessitatetheadministration o general anesthesia and its inherent risks. Any patient with a contraindication to the use o general anesthesia should not undergo laparoscopic hernia repair. Occasionally, general anesthesia induction may result in reduction o an incarcerated or strangulated inguinal hernia. I the surgeon suspects thismighthaveoccurred,theabdomen shouldbeexplored or nonviabletissueeither via laparoscopyor upon conversion to an open laparotomy.(SeeSchwartz10th ed.,p.1509.)
Answer:D
When a patient develops pain, bulging, or a mass at the site o an inguinal hernia repair, clinical entities such as seroma, persistent cord lipoma,and hernia recurrence should beconsidered. Common medical issues associated with recurrence include malnutrition, immunosuppression, diabetes, steroid use, and smoking. echnical causes o recurrence include improper mesh size, tissue ischemia, in ection, and tension in thereconstruction.A ocused physicalexamination should be per ormed. As with primaryhernias, ultrasound, (C ), or
292
Cooperligament
FIG.37-1. McVayCooperligament repair.
9. Nociceptivepain is
A. Te result o ligamentous or muscular trauma and in ammation.
B. Teresult o direct nervedamageor entrapment.
C. Te result o pain conveyed through aferent autonomicpain bers.
D. Characterized asacute.
10. Te triangle o pain is bordered by all o the ollowing EXCEP
A. Iliopubictract
B. Ductusde erens
C. Gonadalvessels
D. Re ected peritoneum
magnetic resonance imaging (MRI) can elucidate ambiguous physical indings. When a recurrent hernia is discovered and warrantsreoperation,an approach through avirgin plane acilitates its dissection and exposure. Extensive dissection o the scarred ield and mesh mayresult in injuryto cord structures, viscera, large blood vessels, and nerves. A ter an initial anterior approach, the posterior laparoscopic approach will usually be easier and more e ective than another anterior dissection. Conversely, ailed preperitoneal repairs should be approached usingan open anterior repair.(SeeSchwartz10th ed.,p.1514.)
Answer:A
Pain a ter inguinal hernia repair is classi ied into acute or chronic mani estations o three mechanisms: nociceptive (somatic),neuropathic,and visceralpain.Nociceptive pain is themost common o thethree.Becauseit isusuallyaresult o ligamentousor muscular trauma and in lammation,nociceptive pain is reproduced with abdominal muscle contraction. reatment consists o rest, nonsteroidal anti-in lammatory drugs(NSAIDs),andreassurance,asitresolvesspontaneously in most cases. Neuropathic pain occurs as a result o direct nervedamageor entrapment.It maypresent earlyor late,and itmani estsasalocalized,sharp,burning,ortearingsensation. It mayrespond to pharmacologic therapyand to localsteroid or anesthetic injections when indicated. Visceral pain re ers to pain conveyed through a erent autonomic pain ibers. It is usually poorly localized and may occur during ejaculation as a result o sympathetic plexus injury. (See Schwartz 10th ed.,p.1514.)
Answer:B
hepreperitonealanatomyseen in laparoscopicherniarepair led to characterization o important anatomic areas o interest,known asthetriangleofdoom,thetriangleofpain,and the circle of death (Fig. 37-2). he triangle o doom is bordered mediallybythe vasde erensand laterallybythe vesselso the spermatic cord. he contents o the space include the externaliliacvessels,deep circum lexiliacvein, emoralnerve,and genitalbrancho thegenito emoralnerve. hetriangleo pain isaregion bordered bytheiliopubictract and gonadalvessels, and it encompasses the lateral emoral cutaneous, emoral branch o thegenito emoral,and emoralnerves. hecircleo death isa vascular continuation ormed bythecommon iliac, internal iliac, obturator, in erior epigastric, and external iliac vessels.(SeeSchwartz10thed., able37-9,pp.1499and1501.)
293
Medial border: ductus deferens
Iliac artery
Iliac vein
Posterior border: peritoneal edge
Inferolateral border: iliopubic tract Deep circumflex iliac a. &v.
Lateral border: gonadal vessels
Genital branch of genitofemoral nerve
Superomedial border: gonadal vessels
11. Temostcommon causeo urinaryretention a er hernia repair is
A. Generalanesthesia
B. Narcoticanalgesia
C. Pain
D. Perioperativebladder distention
Lateral border: reflected peritoneum
Lat. femoral cutaneous n.
Ant. femoral cutaneous n. or other variable branches
Femoral br. of genitofemoral n.
Femoral n.
Answer:A
he most common cause o urinary retention a ter hernia repair is general anesthesia, which is routine in laparoscopic hernia repairs. Among 880 patients undergoing inguinal hernia repair with local anesthesia only 0.2% developed urinaryretention,whereastherateo urinaryretention was13% among 200 patients undergoing repair with general or spinal anesthesia.Other risk actors or postoperativeurinaryretention include pain, narcotic analgesia, and perioperative bladder distention. Initial treatment o urinary retention requires decompression o the bladder with short-term catheterization. Patients will generally require an overnight admission
294
FIG.37-2. Bordersand contentso the A.triangle o doom and B.triangle o pain.a.= artery;Ant.= anterior;br.= branch;Lat.= lateral;n.= nerve;v.= vein.(Modiied with permission rom Colborn GL,SkandalakisJE.Laparoscopic cadavericanatomyo the inguinalarea.ProblGen Surg12:13;1995.)
A B
Deep ring
12. Te outcome ound more commonly with APP repair compared with EPrepair is
A. Length o stay
B. imetorecovery
C. Risko intra-abdominalinjuries
D. Higher short-term recurrencerates
13. Teratioo inguinalherniasto emoralherniasis
A. 7:3
B. 5:1
C. 8:2
D. 10:1
14. Tehigh incidenceo inguinalherniasin preterm babies ismost o en dueto
A. Failureo theperitoneum toclose
B. Familialhistory
C. Femalegender
D. Developmentaldysplasiao thehip
and trial o normal voiding be ore discharge. Failure to void normallyrequiresreinsertion o thecatheter or up to a week. Chronic requirement o a urinary catheter is rare, although older patients may require prolonged catheterization. (See Schwartz10th ed.,p.1515.)
Answer:C
Althoughcontroversypersistsregardingtheutilityo EPversus APP,reviewstodate indnosigni icantdi erencesin operative duration, length o stay, time to recovery, or short-term recurrenceratebetweenthetwoapproaches.In APPrepair,therisko intra-abdominalinjuryishigherthanin EPrepair. his inding prompted the International Endohernia Society (IEHS) to recommend that APP should onlybeattempted bysurgeonswith su icient experience. A Cochrane systematic review ound that rateso port-siteherniasand visceralinjurieswerehigher or the APP technique, whereas EP maybe associated with a higher rate o conversion to an alternative approach; however, neither indingwassu icientlycompellingtorecommendonetechnique overtheother.(SeeSchwartz10thed.,p.1517.)
Answer:B
Approximately 75% o abdominal wall hernias occur in the groin. heli etimerisko inguinalherniais27%in men and 3% inwomen.O inguinalherniarepairs,90%areper ormedinmen and 10%in women. he incidence o inguinal hernias in men hasabimodaldistribution,with peaksbe orethe irstyear o age anda terage40.Abramson demonstratedtheagedependenceo inguinal hernias in 1978. hose between 25 and 34 years had a li etimeprevalencerateo 15%,whereasthoseaged 75yearsand overhadarateo 47%( able37-1).Approximately70%o emoral hernia repairs are per ormed in women; however, inguinal herniasare ive timesmorecommon than emoralhernias. he mostcommon subtypeo groin herniain men andwomen isthe indirectinguinalhernia.(SeeSchwartz10thed.,p.1495.)
Answer:A
Inguinal hernias may be congenital or acquired. Most adult inguinal hernias are considered acquired de ects in the abdominal wall although collagen studies have demonstrated aheritablepredisposition.Anumbero studieshaveattempted to delineate the precise causes o inguinal hernia ormation; however, the best-characterized risk actor is weakness in the abdominalwallmusculature( able37-4).Congenitalhernias, which make up the majorityo pediatric hernias, can be considered an impedance o normal development, rather than an acquired weakness. Duringthe normal course o development,thetestesdescend rom theintra-abdominalspaceinto the scrotum in the third trimester. heir descent is preceded by the gubernaculum and a diverticulum o peritoneum, which protrudes through the inguinalcanaland becomes the processusvaginalis.Between 36and 40weekso gestation,the processusvaginalisclosesand eliminatestheperitonealopening at the internal inguinal ring. Failure o the peritoneum to close results in a patent processus vaginalis (PPV), hence the high incidenceo indirect inguinalherniasin preterm babies.
Children with congenital indirect inguinal hernias will presentwithaPPV;however,apatentprocessusdoesnotnecessarilyindicatean inguinalhernia (Fig.37-3).In a studyo nearly
295
600 adults undergoing general laparoscopy, bilateral inspection revealed that 12% had PPV. None o these patients had clinicallysigni icant symptomso agroin hernia.In agroup o 300patientsundergoingunilaterallaparoscopicinguinalherniarepair,12%were ound to havea contralateralPPV,which was associated with a our old 5-year incidence o inguinal hernia. (See Schwartz 10th ed., able 37-3 and Figure 37-10, pp.1500and 1502.)
TABLE37-4 Presumed causeso groin herniation
Coughing
Chronicobstructivepulmonarydisease
Obesity
Straining
Constipation
Prostatism
Pregnancy
Birthweight<1500g
Familyhistoryoahernia
Valsalva’smaneuver
Ascites
Uprightposition
Congenitalconnectivetissuedisorders
Deectivecollagensynthesis
Previousrightlowerquadrantincision
Arterialaneurysms
Cigarettesmoking
Heavyliting
Physicalexertion
296
A B C D
FIG.37-3. Varying degreeso closure o the processusvaginalis (PV).A.Closed PV.B.Minimallypatent PV.C.Moderatelypatent PV. D.Scrotalhernia.
15. Injurytothelateral emoralcutaneousnerveresultsin
A. Inguinodynia
B. Osteitispubis
C. Meralgiaparesthetica
D. Nerveentrapment
16. Te hernia repair method associated with the lowest recurrencerateisthe
A. Lichtenstein tension- reerepair
B. Open electivetissue-based repair
C. Bassinitechnique
D. Shouldicerepair
Answer:C
Other chronic pain syndromes include local nerve entrapment, meralgia paresthetica, and osteitis pubis. At greatest risk o entrapment are the ilioinguinal and iliohypogastric nerves in anterior repairs and the genito emoral and lateral emoral cutaneous nerves in laparoscopic repairs. Clinical mani estationso nerveentrapment mimicacuteneuropathic pain, and they occur with a dermatomal distribution. Injury to thelateral emoralcutaneousnerveresultsin meralgiaparesthetica,a condition characterized bypersistent paresthesias o thelateralthigh.Initialtreatment o nerveentrapment consists o rest, ice, NSAIDs, physical therapy, and possible local corticosteroid and anesthetic injection.Osteitis pubis ischaracterized byin lammation o thepubicsymphysisand usually presents as medial groin or symphyseal pain that is reproduced by thigh adduction. Avoiding the pubic periosteum when placing sutures and tacks reduces the risk o developingosteitispubis.C scan or MRIexcludesherniarecurrence, and bone scan is con irmatory or the diagnosis. Initial treatment isidenticalto that o nerveentrapment;however,i pain remains intractable, orthopedic surgery consultation should be sought or possible bone resection and curettage. Irrespective o treatment, the condition o ten takes 6 months to resolve.(SeeSchwartz10th ed.,p.1515.)
Answer:A
he incidence o recurrence is the most-cited measure o postoperative outcome ollowing inguinal hernia repair. In evaluating the various available techniques, other salient signi iers o outcome include complication rates, operative duration, hospital stay, and quality o li e. he ollowing section summarizes the evidence-based outcomes o the various approachestoinguinalherniarepair.
Among tissue repairs, the Shouldice operation is the most commonly per ormed technique, and it is most requently executedatspecializedcenters.A2012meta-analysis rom the Cochrane Database demonstrated signi icantlylower rates o hernia recurrence (odds ratio [OR] 0.62, con idence interval [CI] 0.45–0.85) in patients undergoing Shouldice operations when compared with other open tissue-based methods. In experienced hands, the overall recurrence rate or the Shouldice repair is about 1%. Although it is an elegant procedure, its meticulous nature requires signi icant technical expertise toachieve avorableoutcomes,and it isassociated with longer operative duration and longer hospital stay. One study ound therecurrencerate or Shouldicerepairsdecreased rom 9.4to 2.5%a tersurgeonsper ormedtherepairsixtimes.Compared with mesh repairs, the Shouldice technique resulted in signi icantlyhigher rates o recurrence (OR3.65, CI 1.79–7.47); however, it is the most e ective tissue-based repair when mesh isunavailableor contraindicated.
Hernia recurrence is drastically reduced as a result o the Lichtenstein tension- ree repair. Compared with open elective tissue-based repairs, mesh repair is associated with ewer recurrences (OR 0.37, CI 0.26–0.51) and with shorter hospital stay and aster return to usual activities. In a multiinstitutionalseries,3019inguinalherniaswererepaired using the Lichtenstein technique, with an overall recurrence rate o 0.2%. Among other tension- ree repairs, the Lichtenstein
297
17. Aslidinghernia
A. Hasan abnormallyhigh recurrenceratea er repair
B. Can involvethebladder
C. Ismorecommon in theright groin
D. Occursalmost exclusivelyin women
techniqueremainsthemost commonlyper ormed procedure worldwide. Meta-analysis demonstrates no signi icant di erencesin outcomesbetween theLichtenstein and theplugand patch techniques; however, intra-abdominal plug migration and erosion into contiguous structures occurs in approximately 6% o cases. he Stoppa technique results in longer operative duration than the Lichtenstein technique. Nevertheless, postoperative acute pain, chronic pain, and recurrence rates are similar between the two methods.Perhaps the most compelling advantage o the Lichtenstein technique is that nonexpert surgeons rapidly achieve similar outcomes to their expert counterparts. Guidelines issued bythe European Hernia Societyrecommend the Lichtenstein repair or adults with either unilateral or bilateral inguinal hernias as the preerred open technique.(SeeSchwartz10th ed.,p.1516.)
Answer:B
Inguinal hernias may compress adjacent nerves, leading to generalized pressure, localized sharp pain, and re erred pain. Pressure or heaviness in the groin is a common complaint, especiallyat the conclusion o thedayor ollowingprolonged activity. Sharp pain tends to indicate an impinged nerve and may not be related to the extent o physical activity per ormed by the patient. Neurogenic pain may be re erred to the scrotum, testicle, or inner thigh. Questions should be directed to elicit and characterize extrainguinal symptoms. A change in bowel habits or urinary symptoms may indicate a slidingherniaconsistingo intestinalcontentsor involvement o the bladder within the hernia sac. (See Schwartz 10th ed., p.1503.)
298
Thyroid,Parathyroid,and Adrenal
1. Which surgeon was awarded the Nobel Prize in Physiology or Medicine or his work on the “physiology, pathology,and surgeryo thethyroid gland?”
A. TeodoreBillroth
B. EmilKocher
C. John Hunter
D. HarveyCushing
2. What congenitalanomalyarises rom the ormation o the thyroid gland?
A. Tethyroid isthmus
B. Tecricothyroid arch
C. Athyroglossalduct cyst
D. An endobronchialcyst
3. Te arterial supplyo the thyroid arises rom which o the ollowingvessels?
A. Teaorta
B. Teexternalcarotid arteries
C. Tethyrocervicaltrunk
D. Allo theabove
4. In what location, relative to the in erior thyroid artery (I A),istherecurrent laryngealnerve(RLN) ound?
A. Medialor posterior totheI A
B. Lateralor anterior totheI A
C. Passingbetween thebrancheso theI A
D. Allo theabove
Answer:B
he Nobel Prize in Physiology or Medicine was awarded to EmilKocher in 1909.In addition to hisresearch on thephysiology o the thyroid, Kocher’s operative methods greatly reduced the mortality risk o thyroidectomy. he Kocher clamp was designed to prevent hemorrhage rom the hypervascular gland duringthyroidectomy.(SeeSchwartz10th ed., p.1521.)
Answer:C
he medial thyroid anlage descends rom the base o the tongue through a channel called the thyroglossal duct at week 3 to 4 o gestation. he duct normallycloses a ter its descent, but may remain patent and is susceptible to secondary in ection and dilatation, re erred to as a thyroglossal duct cyst. Removal is accomplished with the Sistrunk operation which also removes the central portion o the hyoid bone. (See Schwartz10th ed.,p.1521.)
Answer:D
he superior thyroid arteries arise rom the external carotid arteries, and the in erior thyroid arteries arise rom the thyrocervicaltrunkshortlya ter their origin rom thesubclavian arteries.Athyroid ima arteryarisesdirectly rom theaorta or innominatearteryin 1to 4%o cases.(SeeSchwartz10th ed., p.1523.)
Answer:D
he RLN courses within the tracheoesophageal groove a ter emerging rom the vagus nerve at the level o the aortic arch. Asit ascendsin theneck,therecurrent laryngealnerve(RNL) maybranch,and maypass anterior, posterior, or interdigitate with branches o the I A. he location o the RLN must be con irmed be ore the I A is divided. (See Schwartz 10th ed., p.1524.)
299 38
CHAPTER
5. Although injuryto the RLN results in hoarseness (unilateralinjury) or airwayobstruction (bilateralinjury),injury to the superior laryngeal nerve (SLN) results in a more subtleinjury,a ectingtheabilityto
A. Speakloudlyor singhigh notes.
B. Cough.
C. Feelsensation in theanterior neck.
D. Grimace.
6. Tyroid hormones( 3and 4) haveregulatoryrolesin all o the ollowingEXCEP
A. Te hypoxia and hypercapnia drives o the respiratory center in thebrain.
B. Gastrointestinalmotility.
C. Tespeed o musclecontraction and relaxation.
D. Visualacuityin low-light conditions(“night vision”).
Answer:A
heexternalbranch o thesuperior laryngealnerve(SLN)lies on the in erior pharyngeal constrictor muscle and descends alongside the superior thyroid vessels be ore innervating the cricothyroid muscle. here ore the superior pole vessels should not be ligated en masse, but should be individually divided low on the thyroid gland. Injury to the SLN leads to inability to tense the ipsilateral vocal cord, and impairs the abilityto“hithighnotes”whilesinging,orprojectingthevoice loudly.(SeeSchwartz10th ed.,p.1524.)
Answer:D
hyroid hormones are responsible or maintaining the normal hypoxic and hypercapnic drive in the respiratory center o thebrain,and regulategastrointestinalmotilitywhich leads to diarrhea in hyperthyroidism and constipation in hypothyroidism. hey also regulate bone and protein turnover and the speed o muscle contraction and regulation, hepatic gluconeogenesis, cholesterol synthesis, and intestinal glucose absorption.(SeeSchwartz10th ed.,p.1528.)
7. In North America, hyperthyroidism is most o en caused by
A. oxicmultinodular goiter
B. Di usetoxicgoiter (Gravesdisease)
C. Tyroid cancer
D. Tyroid stimulating hormone-secreting pituitary adenoma
8. Subtotal or total thyroidectomy is pre erred or the treatment o Gravesdisease
A. When radioactiveiodinetherapyiscontraindicated.
B. When thegoiter islargeor airwayobstruction appears imminent.
C. In patients with demonstrated poor compliance with anti-thyroid medications.
D. Allo theabove.
9. What istherecommended courseo action when fneneedle aspiration biopsy (FNAB) o a thyroid nodule is “ ollicular neoplasm?”
A. Repeat FNAB
B. Lobectomy
C. Lobectomyand isthmusectomy
D. otalthyroidectomy
10. Which diseases are associated with germline mutations in theRETtyrosinekinasereceptor gene?
A. Multipleendocrineneoplasiatype2A(MEN2A)
B. Multipleendocrineneoplasiatype2B(MEN2B)
C. Hirschsprungdisease
D. Allo theabove
Answer:B Graves disease, named a ter Robert Graves, the Irish physician who described the disorder in three patients in 1835, is the most common cause o hyperthyroidism in North America, and accounts or 60 to 80% o cases. (See Schwartz 10th ed.,p.1531.)
Answer:D
Subtotalor totalthyroidectomyisnowpre erred over subtotal thyroidectomydue to a lower recurrence rate. Surgeryis preerred over medicaltherapy(radioactiveiodine) in childbearing women who desire to have children in the near uture, in noncompliant patients, or when airway obstruction appears likely.(SeeSchwartz10th ed.,p.1533.)
Answer:B
Follicular neoplasms o the thyroid are less aggressive than papillary neoplasms, and a ine needle aspiration biopsy (FNAB) may be unable to di erentiate between a ollicular adenoma and a ollicular carcinoma. For this reason unilaterallobectomyisrecommended or thisFNABdiagnosis.(See Schwartz10th ed.,p.1539.)
Answer:D
Mutations in the extracellular domain o the RET tyrosine kinase receptor are associated with multiple endocrine neoplasia type 2A (MEN2A), amilial medullary thyroid cancer (FM C), and Hirschsprung disease. Mutations in the intracellular domain are associated with MEN2B, FM C, and Hirschsprungdisease.(SeeSchwartz10th ed.,p.1540.)
300
11. Children exposed to theChernobyldisaster in 1986subsequentlydemonstrated an increased incidenceo which thyroid cancer?
A. Papillarythyroid cancer (P C)
B. Follicular thyroid cancer (F C)
C. Medullarythyroid cancer (M C)
D. Anaplasticthyroid cancer (A C)
12. Te recommended treatment or an otherwise healthy 50-year-old man with a 2-cm P C in the le lobe diagnosed byFNABis
A. Le lobectomy
B. Le lobectomyand isthmusectomy
C. otalle lobectomyand subtotalright lobectomy
D. Excisionalbiopsywith rozen section analysis
13. An adolescent patient with a thyroid mass undergoes FNABwhichreturnsasMC .Whatotherdiseasesshould bescreened or be oretreatment isundertaken?
A. Hyperparathyroidism
B. Pheochromocytoma
C. Mucocutaneousganglioneuromas
D. Allo theabove
14. An asymptomaticchild with a normalphysicalexamination is ound to harbor a mutation in codon 918 o the RET tyrosine kinase receptor, compatible with MEN2B. Ultrasound o theneckisunremarkableand serum calcitonin levelsarenormal.What courseisindicated?
A. Repeat examination and ultrasound yearly
B. Planned thyroidectomyin 3to5years
C. otalthyroidectomy
D. otalthyroidectomywith bilateralneckdissection
15. Postoperative complications within 24 hours o thyroid surgeryinclude
A. Hypocalcemia
B. Dyspnea
C. Dystonia
D. Allo theabove
16. A patient with primary hyperparathyroidism undergoes neck exploration where our small, normal appearing glands are ound. What are the possible locations o an additional,supernumerarygland?
A. In thethyroid gland
B. In thethymus
C. In thetracheoesophagealgroove
D. Allo theabove
Answer:A Papillarythyroidcancer accounts or 80%o allthyroidmalignanciesandisthepredominantthyroidcancer in children and individualsexposed to externalradiation.(See Schwartz10th ed.,p.1542.)
Answer:C
otal thyroidectomy or total lobectomy on the a ected side with subtotallobectomyon thenona ected sideistherecommended treatment o choice or uni ocal P C greater than 1 cm in diameter. De initive operation can be per ormed without rozen section when the diagnosis is unequivocal on FNAB.(SeeSchwartz10th ed.,p.1543.)
Answer:D
MC can be spontaneous (in 75%) or amilial (in 25%) in MEN2. MEN2A is associated with pheochromocytoma and hyperparathyroidism, whereas MEN2B is associated with pheochromocytoma,Mar anoid habitus,and mucocutaneous ganglioneuromas.(SeeSchwartz10th ed.,p.1550.)
Answer:C
Children with mutations at codon 634 o the RET tyrosine kinase receptor gene (MEN2A) are advised to undergo thyroidectomy be ore age 5, whereas children with mutations at codon 918 (MEN2B) should undergo thyroidectomy be ore age 1. I ultrasound o the neck is normal and calcitonin levels are normal, a ormal neck dissection can be avoided. (See Schwartz10th ed.,p.1550.)
Answer:D
Inadvertent injury (ischemia) or removal o the parathyroid glands can cause acute neuromuscular excitability due to hypocalcemia. An expandinghematoma in the neck maynot cause bleeding rom the wound, but can compress the membranousportion o thetracheaandcausedyspnea.Nerveinjuries can cause vocal cord paralysis or impaired speech. (See Schwartz10th ed.,p.1556.)
Answer:D
Supernumerary parathyroid glands occur in 7 to 13% o people, and maybe located in the thymus (most commonly), within the parenchyma o the thyroid gland, or in the tracheoesophagealgroove,themediastinum,or elsewherein the neck.(SeeSchwartz10th ed.,p.1557.)
301
17. A70-year-old woman with earlydementia but otherwise good physical health has an elevated parathyroid hormone (P H) leveland a sestamibi scan which localizes a single ocus o increased activity to the le lower neck. An ultrasound confrms an enlarged gland in the same area. What treatment is likely to provide the best outcome?
A. Bilateralneckexploration under generalanesthesia.
B. Unilateral, “mini-incision” parathyroidectomyunder localanesthesia.
C. Minimally invasive videoscopic parathyroidectomy rom ale axillaryapproachundergeneralanesthesia.
D. Percutaneous alcohol ablation with ultrasound guidanceunder localanesthesia.
18. Intraoperative, rapid P H assays provide guidance that all hyper unctioning glands have been removed during parathyroidectomy. What criterion is used to indicate satis actory resolution o the hyperparathyroidism duringtheprocedure?
A. Greater than 50% allin P H levelwithin 10minutes o removalo parathyroid tissue.
B. Greater than 25% allin P H levelwithin 30minutes o removalo parathyroid tissue.
C. Greater than 75% allin P H levelwithin 10minutes o removalo parathyroid tissue.
D. Greater than 90% allin P H levelwithin 30minutes o removalo parathyroid tissue.
19. Apatientwithpersistentulcer diseaseisdiagnosed with a gastrinoma. Serum chemistry studies indicate hypercalcemia,and an elevated P H levelisdocumented.What is theindicated courseo treatment?
A. Administration o mithramycin 25mg/kg/day or 4to 5daystolower calcium levels.
B. Administration o octreotide100mg ID to suppress gastrin secretion.
C. Abdominalexploration orremovalo thegastrinoma.
D. Neck exploration or removal o the parathyroid adenoma.
20. Which o the ollowing fndings is suggestive o a parathyroid carcinoma?
A. Anelevatedserumcalciumlevelgreaterthan14mg/dL.
B. An elevated P H levelgreater than fvetimesnormal.
C. Apalpablemassin theneck.
D. Allo theabove.
Answer:B
Localization studiessuch assestamibi scanshavebeen shown to allow more limited operations, including those utilizing “mini-incisions” under local anesthesia, or patients who are not good risks or general anesthesia. Improved cosmesis, shorter lengthso stay,andreducedcomplicationsarebene its rom thisapproach.(SeeSchwartz10th ed.,p.1564.)
Answer:A
When the P H level alls by greater than 50% within 10 minutes a ter removal o parathyroid tissue, the cause o the hyperparathyroidism is likely to have been removed, and theoperation can bestopped.(SeeSchwartz10thed.,p.1566.)
Answer:D
In patients with MEN1, hyperparathyroidism should be corrected be ore treatment o the gastrinoma because resolution o hypercalcemia may allow gastrin levels to all to normal. (SeeSchwartz10th ed.,p.1567.)
Answer:A
Parathyroid carcinoma occurs in about 1% o patients with primary hyperparathyroidism, or in about 1000 patients per year in the United States. Findings include an elevated serum calcium level greater than 14 mg/dL, and elevated P H level greater than ive times normal, and a palpable mass in the neck, but none o these may be present. Complete surgical removal is the most e ective therapy with radical neck dissection i lymphadenopathy is present; however, one-third o patients have metastatic disease when irst diagnosed.(See Schwartz10th ed.,p.1570.)
302
21. A50-year-old,healthy-appearingman undergoesevaluation o persistent hypertension.Serum chemistriesreveal hypokalemia(lessthan 3.2mmol/L) and imagingstudies reveal a unilateral adrenal mass. What is the likely diagnosis?
A. Secondaryhypercortisolism (Cushingdisease)
B. Primaryhypercortisolism (Cushingsyndrome)
C. Hyperaldosteronism (Conn syndrome)
D. Pheochromocytoma
22. A35-year-old woman undergoesan evaluation or in ertility. She has gained almost 100 lb in the past year, is hypertensive, and is borderline diabetic. She also complains o easy bruising. Her serum chemistries are normal with the exception o an elevated glucose. Imaging studiesrevealaunilateraladrenalmass.What isthelikely diagnosis?
A. Secondaryhypercortisolism (Cushingdisease)
B. Primaryhypercortisolism (Cushingsyndrome)
C. Hyperaldosteronism (Conn syndrome)
D. Pheochromocytoma
23. All o the ollowing imaging techniques are use ul to localizeapheochromocytomaEXCEP
A. Computed tomography(C )scan
B. Magneticresonanceimaging(MRI)scan
C. Metaiodobenzylguanidine(MIBG)scan
D. Octreotidescan
24. Pheochromocytomascan secreteexcessamountso allo the ollowingEXCEP
A. Dopa(L-dihydroxyphenylalanine)
B. Dopamine
C. Norepinephrine
D. Epinephrine
25. Tepreoperativepreparation o apatient with pheochromocytomashould includeallo the ollowingEXCEP
A. An alpha-adrenergicblocker such asphentolamine.
B. Abeta-adrenergicblocker such aspropranolol.
C. Intravenoushydration toavoid volumedepletion.
D. Systemicsteroidstoavoid adrenalinsufciency.
Answer:C
Primary aldosteronism, or Conn syndrome, is seen in about 1% o hypertensive patients. It is more common in middleaged individual and is usually associated with a single adenoma o the adrenal cortex. he hypertension is usually re ractory to medical treatment, and is classically associated with hypokalemia,but maybeseen in normokalemicindividuals.(SeeSchwartz10th ed.,p.1578.)
Answer:B
Cushing syndrome re ers to any cause o hypercortisolism caused by either an adrenal source or exogenous administration o steroids. Cushing disease re ers only to an adrenocorticotropic hormone (AC H)-secreting adenoma o the pituitarygland.Cushingsyndromedueto an isolated adrenal adenoma is ar less common than hypercortisolism due to a pituitaryadenoma,but adrenalectomyiscurative or primary adrenaltumorsor or adrenalhyperplasiathat persistsdespite e orts to resect a pituitary tumor. (See Schwartz 10th ed., p.1580.)
Answer:D
Pheochromocytomasaresolid tumorswhich appear on computed tomography (C ) scan as so t tissue masses. hey are detected with 85to 95%accuracy,but it isimportant to avoid intravenous-contrast enhancement when a pheochromocytoma is suspected;intravenous contrast can provoke a hypertensivecrisisdueto releaseo catecholamines.MRisuse ulto identi ypheochromocytomas,both becausetheyidenti yso t tissue masses, but also because this tumor tends to enhance on 2-weighted images. Radio-labeled metaiodobenzylguanidine (MIBG) is taken up avidlybythe pheochromocytoma because its structure is similar to norepinephrine. here ore theMIBGscan can localizean occult tumor.Octreotidescans are not used or pheochromocytoma as the tumor does not overexpress somatostatin receptors. (See Schwartz 10th ed., p.1586.)
Answer:A
Extra-adrenal pheochromocytomas (also known as paragangliomas) secrete norepinephrine, because these sites lack the enzyme (phenylethanolamine N-methyltrans erase) which converts norepinephrine to epinephrine. Adrenal pheochromocytomas secrete both epinephrine and norepinephrine as well as dopamine. Some pheochromocytomas secrete only dopamine,and patientswith thesetumorsmaybenormotensive.(SeeSchwartz10th ed.,p.1586.)
Answer:D
he preoperative preparation o a patient with a catecholamine-secreting tumor includes alpha-hypertension, a beta-adrenergicblocker to prevent tachycardia,and volume replacement to avoid hypotension due to alpha- and betablockers. Steroids are not needed to prevent adrenal insu iciency.(SeeSchwartz10th ed.,p.1586.)
303
26. A ollow-up C scan in a 60-year-old patient with previous nephrolithiasis reveals a 1.5-cm hypovascular round lesion with clear margins in the right adrenal gland. Te patient is not hypertensive, hyperglycemic, or hypokalemic.Urinarycatecholmetabolitesarewithin normallimits, and serum cortisol and AC H levels are normal. Which courseisadvisable?
A. Repeat C scan and chemicaltestsannually.
B. PercutaneousFNAB.
C. Adrenal venous sampling or cortisol, renin, and angiotensin.
D. Laparoscopicadrenalectomy.
27. Advantages o laparoscopic adrenalectomy compared with open adrenalectomy include all o the ollowing EXCEP
A. Decreased incidenceo wound in ection
B. Decreased length o hospitalstay
C. Decreased operativetime
D. Decreased narcoticanalgesicuse
28. In patientswhoundergobilateraladrenalectomyin treatment o Cushingdiseasea er ailed attemptsat resection o an AC H-secreting pituitary adenoma, the subsequent development o Nelson syndrome is associated with allo the ollowingEXCEP
A. Hyperpigmentation
B. Diminished visualfelds
C. Losso hearing
D. Headaches
Answer:A headrenal“incidentaloma”isan increasinglycommon indingwith theubiquitoususeo C scanning,with an incidence o 0.4 to 4.4%. A varietyo benign and malignant lesions can account or these indings,andadistanthistoryo malignancy elsewhere should raise the possibility o metastatic disease. Primarymalignancyo theadrenalglandisrare,andthe unctioningtumorsareexcludedbyscreeningtests or cortisoland catecholamine excess. In the absence o symptoms associated with adrenallydisease,annual ollow-up o these lesions with imaging and chemical tests seems prudent. (See Schwartz 10th ed.,p.1589.)
Answer:C
Laparoscopic (videoscopic) approaches to adrenalectomy have been shown to be advantageous or several outcomes including wound complications, analgesic use, and length o hospitalstay. hese advantagesarein balanceto adverseconsiderations including length o operative time and cost. (See Schwartz10th ed.,p.1591.)
Answer:C
Nelson syndrome describes symptoms due to the progressive enlargement o a persistent AC H-secreting pituitary ossa tumor. hese symptoms include hyperpigmentation, visual ield loss,headaches,and extraocular musclepalsies.Inter erencewith theol actorynerveisnot part o thesyndrome.(See Schwartz10th ed.,p.1594.)
304
1. Operativemanagement o a newborn with thechest X-ray shown in Fig.39-1should occur
A. Immediatelya er birth
B. Within 24hours
C. Within 72hours
D. Noneo theabove
CHAPTER
PediatricSurgery
Answer:D
he diagnosis o congenital diaphragmatic hernia (CDH) is madebychest X-ray,with thevast majorityo in antsdeveloping immediate respiratory distress and pulmonary hypertension. CDH care has improved considerably through e ective use o improved methods o ventilation and timely cannulation or extracorporeal membrane oxygenation. In the past, correction o the hernia was believed to be a surgical emergency, and patients underwent surgery shortly a ter birth. It is now accepted that the presence o persistent pulmonary hypertension that results in right-to-le t shunting across the patent oramenovaleortheductusarteriosusandthedegreeo pulmonaryhypoplasiaaretheleadingcauseso cardiorespiratory insu iciency. Current management there ore is directed toward managingthe pulmonaryhypertension,which is usually seen within 7 to 10 days, but in some in ants, may take up to several weeks. (See Schwartz 10th ed., Figure 39-3, pp.1604–1605.)
305
39
FIG.39-1. ChestX-rayshowingale tcongenitaldiaphragmatichernia.
2. Which o the ollowing is most consistent with pyloric stenosis?
A. Na140Cl110K4.2HCO3 26
B. Na142Cl90K5.2HCO3 39
C. Na140Cl95K4.0HCO3 18
D. Na139Cl85K3.2HCO3 36
3. An in ant presents to the emergency room with bilious emesisand irritability.Physicalexamination isnotable or abdominal tenderness and erythema o the abdominal wall. Abdominal X-ray demonstrates dilated proximal bowel with air- uid levels. What is the most appropriate next step in management,a er resuscitation?
A. Upper gastrointestinalseries
B. Barium enema
C. Gastrostomy
D. Laparotomy
4. Whicho the ollowingstatementsregardingHirschsprung diseaseisFALSE?
A. Constipation and abdominal distention are classic symptoms.
B. Approximately20%o casesarediagnosed beyond the newborn period.
C. Te underlying pathology is characterized by an absenceo ganglion cellsin Auerbach plexus.
D. Decompressive ostomy should involve distal, nondilated bowel.
Answer:D
In ants with hypertrophic pyloric stenosis (HPS) develop a hypochloremic, hypokalemic metabolic alkalosis. he urine pH levelis high initially, but eventuallydrops because hydrogen ions are pre erentially exchanged or sodium ions in the distal tubule o the kidney as the hypochloremia becomes severe (paradoxical aciduria). he diagnosis o pyloric stenosisusuallycan bemadeon physicalexamination bypalpation o thetypical“olive”in theright upper quadrant and thepresence o visible gastric waves on the abdomen.When the olive cannot be palpated, ultrasound (US) can diagnose the condition accurately in 95% o patients. Criteria or US diagnosis includea channellength o over 16mm and pyloricthickness over 4mm.(SeeSchwartz10th ed.,pp.1613–1614.)
Answer:D
he cardinal symptom o intestinal obstruction in the newborn is bilious emesis. Prompt recognition and treatment o neonatalintestinalobstruction can trulybeli esaving.Bilious vomiting is usually the irst sign o volvulus, and all in ants with bilious vomiting must be evaluated rapidly to ensure that they do not have intestinal malrotation with volvulus. hechild with irritabilityand biliousemesisshould raiseparticular suspicions or this diagnosis. I le t untreated, vascular compromise o the midgut initially causes bloody stools, but eventuallyresultsin circulatorycollapse.Additionalclues to the presence o advanced ischemia o the intestine include erythemaand edemao theabdominalwall,which progresses to shock and death. It must be re-emphasized that the index o suspicion or thiscondition must behigh,sinceabdominal signs are minimal in the early stages. Abdominal ilms show a paucity o gas throughout the intestine with a ew scattered air- luid levels. When these indings are present, the patient should undergo immediate luid resuscitation to ensure adequate per usion and urine output ollowed byprompt exploratorylaparotomy.(SeeSchwartz10th ed.,p.1617.)
Answer:D
Hirschsprung disease is characterized by the absence o ganglion cells in Auerbach plexus and hypertrophy o associated nervetrunks.It isthought toresult rom ade ect in themigration o neural crest cells, which migrate rom cephalad to caudad.In children whodonotrespondtononoperativetreatment,adecompressivestomaisrequired.Itisimportanttoensure that thisstoma isplaced in ganglion-containingbowel,which must be con irmed by rozen section at the time o stoma creation. he hypertrophied, dilated portion o the intestine in Hirschsprung disease contains normal ganglion cells, and it is in the narrow segment o the colon distal to the dilated portion that ganglion cellsareabsent.(SeeSchwartz10th ed., p.1625.)
306
5. Temost common orm o esophagealatresia(EA)is
A. PureEA(no stula)
B. Pure EF(noatresia)
C. EAwith distaltracheoesophageal stula
D. EAwith proximaltracheoesophageal stula
Answer:C
he ive major varieties o esophageal atresia (EA) and tracheoesophageal istula( EF)areshown in Fig.39-2. hemost commonlyseen varietyisEAwith distal EF(typeC),which occurs in approximately 85%o the cases in most series. he next most requent typeispureEA(typeA),occurringin 8to 10% o patients, ollowed by EF without EA (type E). his occurs in 8%o cases and is also re erred to as an H-type fistula,based on theanatomicsimilaritytothat letter (Fig.39-3). EA with istula between both proximal and distal ends o the esophagus and trachea (type D) is seen in approximately 2% o cases, and type B, EA with EF between proximal esophagusand trachea,is seen in approximately1%o allcases.(See Schwartz10th ed.,Figure39-8and 39-9,pp.1608–1609.)
6. Te predicted 4-year survival rate o a child with a Wilms tumor that iscon ned to onekidneyand isgrosslyexcised is
A. 24%
B. 38%
C. 68%
D. 97%
Answer:D
Following nephroureterectomy or Wilms tumor, the need or chemotherapy and/or radiation therapy is determined by thehistologyo thetumor and theclinicalstageo thepatient ( able 39-1). Essentially, patients who have disease con ined to one kidney that is completely excised surgically receive a short course o chemotherapy and can expect a 97% 4-year survival,with tumor relapserarea ter that time.Patientswith
307
A B C D E
FIG.39-2. The ive varietieso esophagealatresia and tracheoesophageal istula.A. Isolated esophagealatresia.B. Esophagealatresia with tracheoesophageal istula between proximalsegment o esophagusand trachea.C. Esophagealatresia with tracheoesophageal istula between distalesophagusand trachea.D. Esophagealatresia with istula between both proximaland distalendso esophagusand trachea. E.Tracheoesophageal istula without esophagealatresia (H-type istula).
FIG.39-3. Barium esophagram showing H-type tracheoesophageal istula (arrow).
7. A premature in ant boy has been started on enteral eeds shortly a er birth, but develops eeding intolerance 2 weeks postnatally. He displays abdominal tenderness, distention,and bloodystools.An abdominalradiograph is obtained and is shown in Fig. 39-4. What should be the next step in management?
A. Nasogastric decompression, parenteral nutrition,broadspectrumantibiotics.
B. Laparotomy, excision o the afected bowel with ostomy ormation.
C. Laparotomy, reduction o the volvulus, division o adhesions,appendectomy.
D. Water-solublecontrast enema.
more advanced disease or with un avorable histology receive moreintensivechemotherapyand radiation.Even in stageIV, curerateso 80%areachieved. hesurvivalratesareworsein the small percentage o patients considered to have un avorablehistology.(SeeSchwartz10th ed., able39-3,p.1639.)
TABLE39-1 Staging ofWilmstumor
StageI:Tumorlimitedtothekidneyandcompletelyexcised
StageII:Tumorthatextendsbeyondthekidneybutiscompletely excised.Noresidualtumorisapparentatorbeyondthemarginso excision.Thetumorwasbiopsied,ortherewaslocalspillageotumor connedtothe ank.
StageIII:Residualtumorconnedtotheabdomen.Lymphnodesinthe renalhilus,theperiaorticchains,orbeyondcontaintumor.Difuse peritonealcontaminationbythetumor,suchasbyspillageotumor beyondthe ankbeoreorduringsurgeryorbytumorgrowththathas penetratedthroughtheperitonealsurace.Implantsareoundonthe peritonealsuraces.Tumorextendsbeyondthesurgicalmarginseither microscopicallyorgrossly.Tumorisnotcompletelyresectablebecause olocalinltrationintovitalstructures.
StageIV:Hematogenousmetastases.
StageV:Bilateralrenalinvolvement.
Source:Reproduced with permission rom D’Angio GJ,BreslowN,Beckwith JB,et al. Treatment o Wilms’tumor.Resultso theThird NationalWilms’TumorStudy.Cancer 64(2):349-360;1989.Copyright ©1989American CancerSociety.
Answer:A he radiograph demonstrates pneumatosis intestinalis, in conjunction with the clinical scenario described, describes Bell stage II necrotizing enterocolitis (NEC). In all in ants suspected o having NEC, eedings are discontinued, a nasogastric tube is placed, total parenteral nutrition ( PN) is started, and broad-spectrum parenteral antibiotics are given. he in ant is resuscitated, and inotropes are administered to maintain per usion as needed. Intubation and mechanical ventilation may be required to maintain oxygenation. Subsequent treatment may be in luenced by the particular stage o NEC that is present. Patients with Bell stage II disease merit closeobservation.Serialphysicalexaminationsareper ormed looking or the development o di use peritonitis, a ixed mass, progressive abdominal wall cellulitis, or systemic sepsis.I in ants ailto improve a ter severaldayso treatment or i abdominal radiographs show a ixed intestinal loop, consideration should be given to exploratory laparotomy. (See Schwartz10th ed.,Figure39-19,pp.1619–1620.)
308
8. An in ant girl is ound to have persistent jaundice a er birth. A metabolic screen is normal, ultrasound demonstrates an absent gallbladder, and a technetium-99m iminodiaceticacidscanshowsradionuclidethatisconcentrated in theliver butnotexcretedintotheintestine.Whicho the ollowingistrue?
A. Tiscondition isusuallymanaged nonoperatively.
B. Surgeryshould beper ormed within 60dayso li e.
C. Cystoenterostomyprovidesadequatebiliarydrainage.
D. Most o thesepatientswillnot requiretransplantation.
Answer:B
hispatient hasbiliaryatresia,ararediseasecharacterized by ibroproli erative obliteration o the biliary tree, which progresses toward hepatic ibrosis, cirrhosis, and end-stage liver ailure. Surgical treatment is the irst-line therapy, consisting o creation o a hepatoportoenterostomy (Kasai procedure). Numerous studies suggest that the likelihood o surgical successisinverselyrelated to theageat thetimeo portoenterostomy.In antstreated prior to 60days o li e are more likelyto achieve success ul and long-term biliary drainage than older in ants. Although the outlook is less avorable or patients a ter the 12th week, it is reasonable to proceed with surgery even beyond this time point, as the alternative is certain liver ailure.Approximatelyone-thirdo patientsremainsymptomree a ter portoenterostomy; the remainder requires liver transplantation due to progressive liver ailure. Independent risk actorsthat predict ailureo theprocedureincludebridgingliver ibrosisat thetimeo surgeryand postoperativecholangiticepisodes.(SeeSchwartz10th ed.,pp.1628–1629.)
9. Teleadingcauseo death or children older than 1year o ageis
A. Malignancy
B. In ection
C. Injury
D. Congenitalanomalies
Answer:C
Injuryistheleadingcauseo death amongchildren older than 1year.In act,trauma accounts or almost hal o allpediatric deaths, more than cancer, congenital anomalies, pneumonia, heart disease, homicide, and meningitis combined. Motor vehicle collisions are the leading cause o death in people age 1to19years, ollowed byhomicideor suicide(predominantly with irearms) and drowning. Unintentional injuries account or 65%o all injury-related deaths in children younger than 19 years. Each year, approximately 20,000 children and teenagers die as a result o injury in the United States. For every childwhodies rom an injury,itiscalculatedthat40othersare hospitalized and 1120 are treated in emergencydepartments. An estimated 50,000 children acquire permanent disabilities each year,most o which are the result o head injuries. hus, the problem o pediatric trauma continues to be one o the major threats to the health and well-being o children. (See Schwartz10th ed.,p.1642.)
309
FIG.39-4. Abdominalradiograph o in ant with necrotizing enterocolitis.Arrowspoint to area o pneumatosisintestinalis.
10. A “double bubble” on an abdominal radiograph in an in ant ischaracteristico
A. Duodenalatresia
B. Jejunalatresia
C. Meconium ileus
D. Pyloricstenosis
11. Tede ect in gastroschisisis
A. othele o theumbilicus
B. otheright o theumbilicus
C. Trough theumbilicus
D. Superior totheumbilicus
Answer:A Whenever the diagnosis o duodenal obstruction is entertained, malrotation and midgut volvulus must be excluded. Other causes o duodenal obstruction include duodenal atresia, duodenal web, stenosis, annular pancreas, or duodenal duplication cyst. he classic inding on abdominal radiography is the “double bubble” sign, which represents the dilated stomach and duodenum (Fig. 39-5). In association with the appropriate clinical picture, this inding is su icient to conirm the diagnosis o duodenal obstruction. (See Schwartz 10th ed.,Figure39-13,p.1615.)
Answer:B
Gastroschisis represents a congenital anomaly characterized by a de ect in the anterior abdominal wall through which the intestinal contents reely protrude. Unlike omphalocele, there is no overlying sac and the size o the de ect is usually <4 cm. he abdominal wall de ect is located at the junction o the umbilicus and normal skin and is almost always to the right o the umbilicus (Fig. 39-6), whereas or omphalocele theumbilicalcordinsertsintothesac. heumbilicusbecomes partly detached, allowing ree communication with the abdominalcavity. heappearanceo thebowelprovidessome in ormation with respect to the in utero timing o the de ect. he intestine may be normal in appearance, suggesting that the rupture occurred relatively late during the pregnancy. More commonly, however, the intestine is thick, edematous, discolored, and covered with exudate, implying a more longstandingprocess.(SeeSchwartz10thed.,Figure39-31,p.1633.)
310
FIG.39-5. AbdominalX-rayshowing“double bubble”sign in a newborn in ant with duodenalatresia.The two“bubbles”are numbered.
12. Teinitialtreatment or apureEA(no stula)is
A. Repair o theEA.
B. Repair o theEAwith placement o agastrostomy.
C. Repair o the EA, Nissen undoplication, and placement o agastrostomy.
D. Gastrostomyalone.
13. Te most common branchial cle stula originates rom the
A. First branchialcle
B. Second branchialcle
C. Tird branchialcle
D. Fourth branchialcle
Answer:D
Primary EA (type A) represents a challenging problem, particularly i the upper and lower ends are too ar apart or an anastomosis to be created. Under these circumstances, treatment strategies include placement o a gastrostomy tube and per orming serial bougienage to increase the length o the upper pouch. his occasionally allows or primary anastomosis to be per ormed. Occasionally, when the two ends cannot be brought sa ely together, esophageal replacement is required,usingeither agastricpull-up,reversegastrictube,or colon interposition.(SeeSchwartz10th ed.,p.1612.)
Answer:B
Paired branchial cle ts and arches develop early in the ourth gestational week. he irst cle t and the irst, second, third, and ourth pouchesgiverisetoadult organs. heembryologic communication between the pharynx and the external surace maypersist as a istula. A istula is seen most commonly with the second branchial cle t, which normally disappears, and extends rom the anterior border o the sternocleidomastoid muscle superiorly, inward through the bi urcation o the carotid artery, and enters the posterolateral pharynx just belowthetonsillar ossa.In contrast,athirdbranchialcle t istula passesposterior to the carotid bi urcation. he branchial cle t remnantsmaycontain smallpieceso cartilageand cysts, but internal istulas are rare. A second branchial cle t sinus is suspected when clear luid isnoted draining rom theexternal opening o the tract at the anterior border o the lower third o thesternomastoid muscle.Rarely,branchialcle t anomalies occur in association with biliary atresia and congenital cardiacanomalies,an association that isre erred toasGoldenhar complex.(SeeSchwartz10th ed.,p.1602.)
311
FIG.39-6. Gastroschisisin a newborn.Note the location o the umbilicalcord,and the edematous,thickened bowel.
14. Which o the ollowing is characteristic o congenital lobar emphysema?
A. It results rom dilatation o the airways associated with chronicsuppurativedisease.
B. Itsarterialbloodsupplycharacteristicallycomesdirectly rom theaorta.
C. It most commonly involves the upper lobes o the lungs.
D. Resection is usually per ormed due to risk o malignant degeneration.
15. An in antisre erredtoyourclinic orevaluation o aneck mass. On physical examination, the patient has a le lateralneck masswith hishead rotated to theoppositeside. Which o the ollowingisthemost likelydiagnosis?
A. Tyroglossalduct cyst
B. orticollis
C. Tymiccyst
D. Lymphaticmal ormation
Answer:C
Congenital lobar emphysema is a condition mani ested during the irst ew months o li e as a progressive hyperexpansion o one or more lobes o the lung. It usuallyoccurs in the upper lobes o the lung(le t greater than right), and is caused by intrinsic bronchial obstruction rom poor bronchial cartilage development or extrinsic compression. reatment is resection o the a ected lobe, which can be sa ely per ormed using either an open or thoracoscopic approach. he other choices describe bronchiectasis, pulmonary sequestration, and congenitalpulmonaryairwaymal ormation,respectively. (SeeSchwartz10th ed.,pp.1605–1607.)
Answer:B
he management o neck masses in children is determined by their location and the length o time that they have been present. Neck lesions are ound either in the midline or lateralcompartments.Midlinemassesincludethyroglossalduct remnants, thyroid masses, thymic cysts, or dermoid cysts. Lateral lesions include branchial cle t remnants, lymphatic mal ormations (previously known as cystic hygroma), vascular mal ormations, salivary gland tumors, torticollis, and lipoblastoma. he presence o a lateral neck mass in in ancy in association with rotation o the head toward the opposite side o the mass indicates the presence o congenital torticollis. hislesion results rom ibrosiso thesternocleidomastoid muscle, and is histologically characterized by the deposition o collagen and ibroblasts around atrophied muscle cells. In the vast majority o cases, physical therapy based on passive stretchingo the a ected muscle is o bene it. Rarely, surgical transection o thesternocleidomastoid maybeindicated.(See Schwartz10th ed.,pp.1601–1603.)
16. A9-month-old boypresentswith two episodeso vomiting as well as episodes o colicky pain. His abdominal examination is notable or upper abdominal tenderness, and hisstoolsareguaiac-positive.USdemonstratesatarget sign, and he is then taken or an air-contrast enema. What is the likelihood that he will need operative intervention?
A. 25%
B. 50%
C. 75%
D. 90%
Answer:A
Patientswith intussusception should beassessed or the presence o peritonitis and or the severity o systemic illness. In the absence o peritonitis, the child should undergo radiographicreduction. heairenemaisdiagnosticandmayalsobe curative,and it isthepre erred method o diagnosisand treatment o intussusception.Air isintroduced with amanometer, and the pressure that is administered is care ully monitored. Under most instances, this should not exceed 120 mm Hg. Success ulreduction ismarked by reere luxo air intomultipleloopso smallbowelandsymptomaticimprovementasthe in ant suddenlybecomespain- ree.Unlessboth o thesesigns are observed, it cannot be assumed that the intussusception isreduced.I reduction isunsuccess uland thein ant remains stable,thein antshouldbebroughtbacktotheradiologysuite or a repeat attempt at reduction a ter a ewhours. his strategy has improved the success rate o nonoperative reduction in many centers. In addition, hydrostatic reduction with barium maybeuse uli pneumaticreduction isunsuccess ul. he overallsuccessrateo radiographicreduction variesbased on the experience o the center and is typically between 60 and 90%.(SeeSchwartz10th ed.,p.1622.)
312
17. A newborn is ound to have a blind rectal pouch on examination. Which o the ollowing are not associated with thiscondition?
A. EA
B. Cardiacde ects
C. Omphalocele
D. ethered spinalcord
18. Which typeo choledochalcyst arises rom theintraduodenalportion o thecommon bileduct?
A. ypeI
B. ypeII
C. ypeIII
D. ypeIV
19. Which o the ollowing is not characteristic o prunebellysyndrome?
A. Bilateralintra-abdominaltestes
B. Dilatation o theureters
C. Dilatation o theurinarybladder
D. Mal ormed renalparenchyma
Answer:C he patient is described to have an imper orate anus, the embryologic basis o which involves ailure o descent o the urorectal septum, resulting in a blind rectal pouch that o ten has a istulous tract. Approximately 60% o patients have an associated mal ormation, the most common o which is a urinary tract de ect (approximately 50% o patients). Skeletal de ects are also seen, with the sacrum most commonly involved. Spinal cord anomalies, especially tethered cord, are common,particularlyin children with high lesions.Gastrointestinalanomaliesoccur,most commonlyEA.Cardiacanomalies may be noted, and occasionally patients present with a constellation o de ects as part o the VAC ERRL syndrome (vertebral anomalies, anorectal anomalies, cardiac de ects, tracheoesophageal istula, renal anomalies, and radial limb hyperplasia).(SeeSchwartz10th ed.,p.1627.)
Answer:C
Choledochal cysts re er to a spectrum o congenital biliary tract disorders, with a cyst wall composed o ibrous tissue anddevoido mucosallining. ypeIcystsarecharacterizedby usi orm dilatation o the bile duct, type II by isolated diverticula protruding rom the wall o the common bile duct, and type III cysts arise rom the intraduodenal portion o the common bile duct (also known as choledochoceles). ype IVA consists o multiple dilatations o the intrahepatic and extrahepatic bile ducts, whereas ype IVBinvolves onlythe extrahepatic bile ducts. Lastly, type V (Caroli disease) cysts are all intrahepatic,and usuallyconsist o multipledilatations.Cysts can lead to abdominal pain, cholangitis, pancreatitis, as well asabiliarytract malignancy.(SeeSchwartz10th ed.,p.1630.)
Answer:D
Prune-belly syndrome re ers to a disorder that is characterized by extremely lax lower abdominal musculature, dilated urinary tract including the bladder, and bilateral undescended testes. heterm prune-bellysyndromeappropriately describes the wrinkled appearance o the anterior abdominal wall that characterizes these patients. he incidence is signi icantly higher in men, and patients mani est a variety o comorbidities, the most signi icant o which is pulmonary hypoplasia, which is not survivable in the most severe cases. Skeletal abnormalities include dislocation or dysplasia o the hip and pectus excavatum. Approximately 80% o these patientswillhave some degreeo vesicoureteralre lux,which can predispose to urinarytract in ection.Despite the marked dilatation o theurinarytract,most children with prune-belly syndrome have adequate renal parenchyma or growth and development.(SeeSchwartz10th ed.,p.1634.)
313
20. Undescended testesareusuallyrepaired bywhat age?
A. 6months
B. 1year o age
C. 2yearso age
D. 4yearso age
Answer:C
Men with bilateral undescended testicles are o ten in ertile. When the testicle is not within the scrotum, it is subjected to ahigher temperature,resultingin decreased spermatogenesis. Mengel and coworkers studied 515 undescended testicles by histology and demonstrated a decreasing presence o spermatogonia a ter 2 years o age. Despite orchidopexy, the incidence o in ertilityis approximately two times higher in men with unilateralorchidopexycompared with men with normal testicular descent.Consequently,it isnowrecommended that the undescended testicle be surgically repositioned by 1 year o age.(SeeSchwartz10th ed.,p.1636.)
314
1. Which o the ollowing is INCORREC concerning the anatomyo thepenis?
A. Te paired corpora spongiosum unctions as one compartment duetovascular interconnections.
B. Priapism is de ned as a persistent erection o more than 4hours’duration.
C. Te corpora cavernosum are enclosed by the tunica albuginea.
D. Te corpora cavernosum and spongiosum are surrounded byBucks ascia.
2. Which o the ollowingistrueabout bladder cancer?
A. For patients with bladder cancer invading into the bladder muscle ( 2 lesion) immediate radiotherapy ollowed bysurgeryo ersthebest chanceo cure.
B. Patientswith limited lymph node involvement maybe cured bysurgeryalone.
C. Continent neobladders have yet to be success ullyutilized in patientsundergoingcystectomy.
D. Intravesicalchemotherapyprior to surgeryisroutinely used or bladder cancers invading into the bladder muscle( 2lesion).
Answer:A
he corpora cavernosum are the paired, cylinder-like structures that are the main erectile bodies o the penis. he corpora cavernosum consist o a tough outer layer called the tunica albuginea and spongy,sinusoidaltissue insidethat ills with blood to result in erection. hetwo corporacavernosum havenumerousvascular interconnections,sothey unction as one compartment. Surrounding all three bodies o the penis are the outer dartos ascia and the inner Buck ascia. (See Schwartz10th ed.,p.1652.)
Answer:B
For patients who have disease invading into bladder muscle ( 2), immediate (within 3 months o diagnosis) cystectomy with extended lymph nodedissection o ersthebest chanceo survival. Patients with limited lymph node involvement may be cured with surgery alone. Patients have multiple reconstructive options, including continent and noncontinent urinary diversions. he orthotopic neobladder has emerged as a popular urinary diversion or patients without urethral involvement. his diversion type involves the detubularization o a segment o bowel, typically distal ileum, which is then re ashioned into a pouch that is anastomosed to the proximalurethra (neobladder) or to theskin (continent cutaneous diversion). Patients with non–muscle-invasive bladder cancer (con ined to the bladder mucosa or submucosa) can be managed with transurethral resection alone and adjuvant intravesical(instilled into the bladder) chemotherapy/immunotherapy.(SeeSchwartz10th ed.,pp.1653–1654.)
3. Which o the ollowing is true concerning testicular cancer?
A. Most common malignancy in men between 15 and 35years.
B. Most commonlypresentsasapain ulenlargingmass.
C. Initial workup includes chest, abdominal, and brain imaging.
D. Most common siteo metastasesistothelungs.
Answer:A
esticular cancer is the most common solid malignancy in men between 15 and 35 years. Chest and abdominal imagingmust be per ormed to evaluate or evidence o metastasis. hemost common siteo spread istheretroperitoneallymph nodes extending rom the common iliac vessels to the renal vessels.(SeeSchwartz10th ed.,p.1654.)
315 40
Urology
CHAPTER
4. Which o the ollowing statements about kidneycancer is FALSE?
A. Lesionsareusuallysolid but can becystic.
B. Maybesporadicor hereditary.
C. Surgical debulking can improve survival in patients whopresent with metastaticdisease.
D. Patients are not curable (and there ore should not be operated on) i tumor thrombus extends proximally intothevenacava.
5. Which o the ollowingisFALSEconcerningcarcinomao theprostate?
A. Annualdigitalrectalexamination and serum prostatespeci cantigen (PSA)determinationsarerecommended beginningat age55.
B. Lungmetastasisislesscommon than bonemetastasis.
C. Radical prostatectomy is associated with a 5% incidenceo permanent urinaryincontinence.
D. Onceprostatecancer hasspread,it isnolonger curable butcanbecontainedbyloweringserumtestosteroneand/ orbyadministrationo androgenreceptorblockers.
6. Concerning ureteric trauma, which o the ollowing is true?
A. Retrograde pyelogram is the most sensitive test to detect ureteralinjury.
B. Bladder mobilization isnotintegraltorepair o ureteric injury.
C. Kidneymobilization isnot integraltorepair o ureteric injury.
D. Useo uretericstentsisnot use ulin predictingpostrepair strictures.
Answer:D
Renal tumors are usually solid, but they also can be cystic. Most cases o renal cell carcinoma (RCC) are sporadic, but manyhereditary orms have been described. Up to 20 to 30% o patientsmaypresent with metastaticdisease,in which case, surgicaldebulkingcan improvesurvival,asshown in randomized controlled trials.Up to 10%o RCCinvadesthelumen o the renal vein or vena cava. he degree o venous extension directly impacts the surgical approach. Patients with thrombus below the level o the liver can be managed with crossclampingaboveand belowthethrombusand extraction rom a cavotomy at the insertion o the renal vein. (See Schwartz 10th ed.,pp.1655–1656.)
Answer:C
heAmerican UrologicalAssociation hasadvised or screening or men 55 to 69 years o age. he most common site o spread o prostatecancer isthepelviclymph nodesand bone. Radical prostatectomy is associated with early incontinence and erectile dys unction (ED) (depending on nerve sparing). Incontinence improves signi icantly with time, with <1%o men in experienced hands su eringsevere long-term problems with urinary control. (See Schwartz 10th ed., pp. 1657–1658.)
Answer:A
A retrograde pyelogram is the most sensitive test or ureteral injury. Partial injuries can be primarilyrepaired, although all devitalized tissue must be debrided to avoid delayed tissue breakdown and urinoma ormation.Ureteralstentsshould be placed in this situation to acilitate healing without stricture. Midurethral level injuries can be treated with a ureteroureterostomyi aspatulated,tension- reerepair can beachieved. For longer de ects, the bladder can be mobilized and brought up to the psoas muscle (psoas hitch). For additional length, a tubularized lap o bladder (Boari lap) can be created and anastomosed to the remainingureter.(See Schwartz10th ed., pp.1659–1660.)
7. reatment o acuteurinaryretention mayincludeallo the ollowingEXCEP
A. Coudecatheterization.
B. Fluid replacement using1:1mLo D5W.
C. Placement o asuprapubicdrainagetube.
D. Continuousbladder irrigation i hematuriaisthecause o retention.
Answer:B reatment should include placement o a urethral catheter as quickly as possible. However, benign prostatic hyperplasia (BPH) or urethral strictures o ten make the placement o a catheter di icult. For men with BPH, a coude (French or curved) catheter is help ul in negotiating past the angulation in the prostatic urethra. Placement o a suprapubic tube approximatelytwo ingerbreadths above the pubic symphysis maybeneeded.I hematuriaisthecauseo retention,continuous bladder irrigation o ten is necessary to prevent clot ormation. Fluid replacement typically is 0.5 mLo 0.45 normal saline or every1mLo urineoutput above200mLin 1hour, although sodium and potassium supplementation requirements depend on the electrolyte status o the patient. (See Schwartz10th ed.,pp.1661–1662.)
316
8. Regarding testicular torsion, which o the ollowing is FALSE?
A. Undescended testicleisarisk actor.
B. Decreased blood fow relative to contralateral testicle demonstrablebyultrasound.
C. esticular salvage decreases to <5% i surgery is delayed >6hours.
D. Surgicalexploration shouldinclude xation o thecontralateraltesticle.
9. Allo the ollowingaretrueconcerningpriapism EXCEP
A. Priapismisde nedasapersistenterection or >4hours unrelated tosexualstimulation.
B. Etiologic actors include sickle cell disease, malignancy,totalparenteralnutrition,penilesha ractures.
C. Low fow priapism can be con rmed with a penile blood gasdetermination.
D. reatment mayrequireinjection o phenylephrine.
10. Which o the ollowingistrueconcerningBPH?
A. Consequences o BPH include gross hematuria, chronic in ection, bladder calculi, urinary retention, and paraphimosis.
B. Medicaltreatment includesalphablockers.
C. ransurethral resection can lead to transurethral resection syndrome.
D. Electrovaporization o theprostateshould beavoided becauseo thedanger o closed-spacethermalinjury.
Answer:C
Risk actors or torsion include undescended testis, testicular tumor, and a “bell-clapper” de ormity—poor gubernacular ixation o the testicles to the scrotal wall. he diagnosisismadebyclinicalhistoryand examination,but can besupported bya Doppler ultrasound,which typicallyshows decreased intratesticular blood low relative to the contralateraltestis.Immediatesurgicalexploration can salvagean ischemictestis.Morethan 80%o testescan besalvaged i surgery is per ormed within 6 hours; this rate decreases to <20% as time lapses beyond 12 hours. At the time o surgery, the contralateral testes must also be explored and ixed to the dartos ascia due to the possibility that the same anatomic de ect allowingtorsion existson thecontralateralside.(SeeSchwartz 10th ed.,pp.1662–1663.)
Answer:B
Priapism is a persistent erection or more than 4 hours unrelated tosexualstimulation.Risk actorsincludesicklecelldisease or trait, malignancy, medications, cocaine abuse, certain antidepressants, and total parenteral nutrition. Low- lowpriapism can be con irmed with a penile blood gas o the cavernosalbodiesdemonstratinghypoxic,acidoticblood.Injection o phenylephrine (up to 200 mgin 20 mLnormalsaline) into the corporal bodies may be required. (See Schwartz 10th ed., p.1663.)
Answer:A
Besides voiding symptoms, consequences o BPH include gross hematuria, in ections due to incomplete emptying, bladder calculi, and urinary retention. Over time, incomplete emptying may lead to chronic bladder overdistention that can result in a de- unctionalized bladder. Medical treatment o BPH is usually the irst step. Alpha blockers act on α-receptorsin thesmooth muscleo theprostateand decrease itstone. ransurethralresection o theprostateisthemainstay o endoscopic surgical BPH treatment. It is extremely e ectiveat improving lowand decreasingresidualurine.Complications are rare but include incontinence and excessive luid absorption o the hypotonic irrigating solution used during resection,resultingin thetransurethralresection syndrome.It isdueto hyponatremia and luid overload,and although rare, can result in death. Mental status changes and pulmonary edema are managed bydiuresisand sodium supplementation with hypertonic saline in severe cases. Because o these rare, but potentially dangerous side e ects, laser or electrovaporization o theprostatehasgrown popular.It isassociated with verylimited luid absorption, and saline can be used because there isno monopolar electrocautery.(See Schwartz10th ed., p.1665.)
317
11. Which o the ollowingistrueconcerningurolithiasis?
A. Maya ectupto20%o thepopulationoverthecourse o ali etime.
B. Most common typeiscalcium oxalatelithiasis.
C. Most patients will bene t rom a chronic urinary acidi cation program.
D. Study o choice or diagnosis is contrast computed tomography(C )scanning.
Answer:B
Urolithiasis,or urinarycalculusdisease,maya ect up to 10% o the population over the course o a li etime. Calculi are crystallineaggregateso oneor morecomponents,most commonly calcium oxalate. Computed tomography (C ) scans willdemonstrateallcalculiexcept thosecomposed o crystalline-excreted indinavir,an antiretroviralmedication.For this reason,noncontrastC scanshavebecomethestudyo choice to evaluate or urolithiasis.Patientswith recurrent stoneswill bene it rom examination o stone composition and 24-hour urine metabolic workup to determine the underlying etiology. Better hydration is use ul or all etiologies. Additionally, most patients will bene it rom alkalization o the urine (eg,potassium citrate).(SeeSchwartz10th ed.,p.1666.)
318
1. Concerning human papillomavirus (HPV) vaccination, which o the ollowingisFALSE?
A. wo HPVvaccineshavebeen developed and approved bytheU.S.Food and DrugAdministration (FDA).
B. Both vaccinesgeneratehigh concentrationso neutralizingantibodiestoHPVL1protein.
C. Prospective randomized clinical trials have demonstrated that vaccination prevents nearly 100%o HPV subtype-speci cprecancerouscellchanges.
D. HPV immunizations prevents 90% o all cervical cancers.
CHAPTER
2. Concerning uterine leiomyomas, all o the ollowing are trueEXCEP
A. Most common pelvictumor.
B. Hasaracialpredilection.
C. Classi ed accordingtoanatomiclocation.
D. Rarelynecessitateshysterectomy.
Answer:D
wo human papillomavirus (HPV) vaccines have been developed and approved by the U.S. Food and Drug Administration (FDA). Vaccination generates high concentrations o neutralizing antibodies to HPV L1 protein, the antigen in both vaccines. Several randomized clinical trials involving approximately 35,000 young women have shown that both Gardasil and Cervarix prevent nearly 100% o the HPV subtype-speci ic precancerous cervical cell changes or up to 4yearsa ter vaccination amongwomen who werenot in ected at thetimeo vaccination.Cervicalcancer screeningcontinues toplayan important rolein detection and treatment o cervical intraepithelial neoplasia (CIN) II/III and prevention o cervicalcancer in thesehigh-riskpatients.Cervicalcancer screening continues to be o great importance since HPV immunization willnot prevent approximately25to 30%o cervicalcancersin HPV-naïve women and does not protect against the development o cancer in women already in ected with carcinogenic HPVtypes.(SeeSchwartz10thed.,pp.1681–1682.)
Answer:D
Leiomyomas, also known colloquially as fibroids, are the most common emale pelvic tumor and occur in response to growth o the uterine smooth muscle cells (myometrium). heyarecommon in thereproductiveyears,and byage50,at least 60%o whiteand up to 80%o blackwomen are(or have been) a ected. Leiomyomas are described according to their anatomic location (Fig. 41-1) as intramural, subserosal, submucosal, pedunculated, cervical, and rarelyectopic. Most are asymptomatic; however, abnormal uterine bleeding caused by leiomyomas is the most common indication or hysterectomyin theUnited States.(SeeSchwartz10th ed.,Figure41-9, pp.1683–1684.)
319 41
Gynecology
3.
A. Morecommon in in ertilewomen.
B. Etiologyisin ammation-induced.
C. Can involvethethoraciccavity.
D. Can causeincreasein serum CA-125.
4. Pregnancy-related surgical conditions include all o the ollowingEXCEP
A. rauma-related hypovolemia may be compounded by pregnancy-induced decreases in systemic vascular resistance.
B. Gastric motilityis decreased, leadingto increased risk o aspiration.
C. Increased likelihood o thromboemboliceventsdueto increase in several coagulation actors induced by pregnancy.
D. Fetal autoregulation o blood pressure duringall three trimesterso pregnancy.
5. ypical indications or cesarean deliveryinclude all o the ollowingEXCEP
A. Questionable etalstatus
B. Breech presentation
C. Cephalopelvicdisproportion
D. Maternalcoagulopathy
Answer:B
Endometriosis is especially prevalent in patients su ering rom chronic pelvic pain (80%) and in ertility (20–50%). he pathophysiology o endometriosis is poorly understood; etiologic theories explaining dissemination o endometrial glands include retrograde menstruation, lymphatic and vascular spread o endometrialglands,and coelomicmetaplasia. Endometriosis commonly involves the ovaries, pelvic peritoneal sur aces, and uterosacral ligaments. Other possible sites include the rectovaginal septum, sigmoid colon, intraperitoneal organs, retroperitoneal space, ureters, incisional scars, umbilicus, and even the thoracic cavity. Endometriosis can also cause increases in serum cancer antigen 125 (CA-125). (SeeSchwartz10th ed.,pp.1689–1690.)
Answer:D
rauma-related hypovolemia may be compounded by pregnancy-induced decreasesin systemicvascular resistance. Gastric motility is decreased, increasing the risk o aspiration. Several coagulation actors are also increased in pregnancy, increasing the likelihood or thromboembolic events. It should also be recognized that the etus will be impacted signi icantlybymaternal hypotension, as blood maybe shunted away rom the uterus. Only the third-trimester etus has any ability to autoregulate in the context o decreased uterine blood low and oxygen delivery. (See Schwartz 10th ed., p.1691.)
Answer:D
ypical indications or cesarean delivery include nonreassuring etal status, breech or other malpresentations, triplet and higher order gestations, cephalopelvic disproportion, ailure to progress, placenta previa, and active genital herpes. (See Schwartz10th ed.,p.1693.)
320
Which o the ollowing concerning endometriosis is FALSE?
Subserous Pedunculated Submucous Prolapsed Intercavitary Intramural
FIG.41-1. Typesofuterine myomas.
6. Pelvic oordys unctionincludeallo the ollowingEXCEP
A. Urinaryincontinence
B. Pelvicorgan prolapse
C. Fecalincontinence
D. Dyspareunia
7. All o the ollowing are true concerning stress incontinenceEXCEP
A. Can beduetolacko urethrovaginalsupport.
B. Can beduetointrinsicsphincter de ciency.
C. Goal o surgical repair is to create a partial urethral obstruction.
D. Urethral reimplantation is sometimes necessary i other approaches ail.
Answer:D
Pelvic loor disorders can be categorized, rom an urogynecologic perspective, into three main topics: emale urinary incontinence and voidingdys unction,pelvicorgan prolapse, and disorderso de ecation.(SeeSchwartz10th ed.,p.1694.)
Answer:D
Stress incontinence is believed to be caused by lack o urethrovaginal support (urethral hypermobility) or intrinsic sphincter de iciency (ISD). ISD is a term applied to a subset o stress-incontinent patients who have particularly severe symptoms, including urine leakage with minimal exertion. his condition is o ten recognized clinically as the low pressure or “drainpipe” urethra. he urethral sphincter mechanisminthesepatientsisseverelydamaged,limitingcoaptation o the urethra. Standard surgical procedures used to correct stress incontinence share a common eature: partial urethral obstruction that achieves urethral closure under stress. (See Schwartz10th ed.,p.1695.)
8. Concerningvulvar carcinoma,allo the ollowingaretrue EXCEP
A. Etiologymaybedueto an HPV-dependent pathwayo carcinogenesis.
B. Approximately50%aresquamouslesions.
C. Hematogenousdissemination israre.
D. Staging and primary surgical treatment are typically per ormed asasingleprocedure.
9. Which o the ollowing is true concerning endometrial carcinoma?
A. Tird most common gynecologicalmalignancy.
B. Equally requent in menopausal and postmenopausal women.
C. Risk actorsincludeobesity,smoking.
D. Use o combination oral contraception pills has a protectiveefect.
Answer:B
Evidence supports an HPV-dependent pathway o carcinogenesiswith risks actorssimilar to vulvar intraepithelialneoplasia (VIN) in the majority o cases. Vulvar carcinomas are squamous in 90% o cases. Spread o vulvar carcinoma is by direct local extension and via lymphatic microembolization. Hematogenous spread is uncommon. Staging and primary surgical treatment are typically pre ormed as a single procedure and tailored to the individual patient. (See Schwartz 10th ed.,p.1696.)
Answer:C
Endometrial cancer is the most common gynecologic malignancyand ourth most common cancer in women. It is most common in menopausalwomen in the i th decade o li e;up to 15 to 25%o cases occur prior to menopause, and 1 to 5% occur be ore age 40. Risk actors or the most common type o endometrialcancer includeincreased exposureto estrogen without adequate opposition by progesterone, either endogenous(obesity,chronicanovulation) or exogenous (hormone replacement). Additional risk actors include diabetes, Lynch II syndrome(hereditarynonpolyposiscolorectalcancer),and prolonged use o tamoxi en. amoxi en is a mixed agonist/ antagonist ligand or theestrogen receptor.It isan agonisticin theuterusand an antagonisticto thebreast and ovary.Protective actors or endometrialcancerincludesmokinganduseo combination oral contraceptive pills. (See Schwartz 10th ed., pp.1698–1699.)
10. Which o the ollowing is FALSE concerning epithelial ovarian cancer (EOC)risk actors?
A. Risk actorsincludeearlymenarche.
B. Risk actorsincludelatemenopause.
C. Risk actorsincludeprevioushysterectomy.
D. Risk actorsincludenulliparity.
Answer:C
Risk actors or development o epithelial ovarian cancer (EOC) include events that appear to increase the number o li etimeovulations(eg,earlymenarche,latemenopause,nulliparity),whereaseventsthatdecreasethenumbero ovulations decrease risk (eg, pregnancy, breast eeding, oral contraceptives).Additionally,ahistoryo tuballigation or hysterectomy alsodecreasesEOCrisk.(SeeSchwartz10th ed.,p.1701.)
321
11. Te objectives o surgery in EOC include which o the ollowing?
A. Establishinghistologicaldiagnosis
B. Surgicalstaging
C. Surgicalcytoreduction
D. A,B,and C
Answer:D he objectives o surgery in EOC are three old. he irst is to make the histologic diagnosis. he second is to assess the extent o diseasethrough completesurgicalstaging. hethird objective is (complete when easible) surgical cytoreduction or debulking.(SeeSchwartz10th ed.,p.1701.)
322
1. Electromyography (EMG) and nerve conduction studies (NCS)areuse ul or assessingthe unction o
A. Peripheralnerves
B. Bilateralcarotid arteries
C. Intracranialliquids
D. Spinalnerves
2. Te lesion that can cause mass efect and rapidly kill the patient is
A. In erior ossalesions
B. Posterior ossalesions
C. Progressiveobtundation
D. Bradycardiallesions
CHAPTER
Answer:A
Electromyography and nerve conduction studies (EMG/ NCS) are use ul or assessing the unction o peripheral nerves. EMG records muscle activity in response to a proximal stimulation o the motor nerve. NCSrecord the velocity and amplitude o the nerve action potential. EMG/NCStypically is per ormed approximately 3 to 4 weeks a ter an acute injury,asnervesdistalto theinjurycontinueto transmit electricalimpulsesnormallyuntildegeneration o thedistalnerve progresses.(SeeSchwartz10th ed.,p.1712.)
Answer:B
heposterior ossa(brain stem and cerebellum) requiresspecial consideration because the volume o the posterior ossa within thecranialvault issmall.Posterior ossalesionssuch as tumors, hemorrhage, or stroke can cause mass e ect that can rapidly kill the patient in two ways. Occlusion o the ourth ventricle can lead to acute obstructive hydrocephalus, raised intracranial pressure (ICP), herniation, and eventuallydeath. his mass e ect can also lead directly to brain stem compression (Fig. 42-1). Symptoms o brain stem compression includehypertension,agitation,and progressiveobtundation, ollowed rapidly by brain death. A patient exhibiting any o these symptoms needs an emergent neurosurgical evaluation or possibleventriculostomyor suboccipitalcraniectomy (removalo thebonecoveringthecerebellum). hissituation is especially critical, as expeditious decompression can lead to signi icant unctional recovery. (See Schwartz 10th ed., Figure42-5,pp.1714–1715.)
323 42
Neurosurgery
3. According to the Glasgow Coma Scale (GCS), a patient with ahead injuryscoreo 5isclassi ed as
A. Mild
B. Moderate
C. Severe
D. Seizure
4. In regard to the halo test, a positive indicator or cerebrospinal uid (CFS) when tinged with blood will show the ollowingwhen dropped on an absorbent tissue
A. A single ring with a darker center spot containing blood componentssurrounded byalight haloo CFS.
B. A double ring with a darker center spot containing blood componentssurrounded byalight haloo CFS.
C. Asingleringwith a lighter center spot containingCFS surrounded byadarker haloo blood components.
D. Adoubleringwith alighter center spotcontainingCFS surrounded byadarker haloo blood components.
FIG.42-1.
seen asa hypodense area in the right cerebellarhemisphere (arrowhead)on head computed tomography(CT)in a patient with rapidlyprogressing obtundation 2daysa terthe initialonset o symptoms.Swelling o the in arcted tissue causesposterior ossa masse ect.The ourth ventricle is obliterated and not visible,and the brain stem isbeing compressed.
Answer:D
raumaticbrain injury( BI)can beclassi ied asmild,moderate,or severe.For patientswith ahistoryo head trauma,classi ication is as ollows: severe head injury i the GCSscore is 3 to 8, moderate head injury i the Glasgow Coma Scale (GCS)scoreis9to12,andmild headinjuryi theGCSscoreis 13 to 15. Many patients present to emergency rooms and trauma bays with a history o BI. A triage system must be used to maximize resource utilization while minimizing the chance o missing occult or progressing injuries. (See Schwartz10th ed.,p.1718.)
Answer:B
Copious clear drainage rom the nose or ear makes the diagnosis o cerebrospinal luid (CSF) leakage obvious. O ten, however, the drainage maybe discolored with blood or small in volumei somedrainsintothethroat. hehalotestcan help di erentiate. Allowa drop o the luid to all on an absorbent sur ace such as a acial tissue. I blood is mixed with CSF, the drop will orm a double ring, with a darker center spot containingblood components surrounded bya light halo o CSF. I this test is indeterminate, the luid can be sent or beta-2 trans errin testing, a carbohydrate- ree iso orm o transerrin exclusively ound in the CSF. (See Schwartz 10th ed., pp.1716–1717.)
324
Maturing cerebellarstroke
5. Neurapraxiaisde ned as
A. Tedisruption o axonsand myelin.
B. Tedisruption o axonsand endoneurialtubes.
C. Tetemporary ailureo nerve unction withoutphysicalaxonaldisruption.
D. Te temporary ailure o nerve unction with physical axonaldisruption.
6. Apatientwhowithdraws rom pain,ismumblinginappropriatewords,and openshiseyestopain hasaGCSscoreo
A. 3
B. 6
C. 9
D. 12
Answer:C
Neurapraxia isde ined asthetemporary ailureo nerve unction without physical axonal disruption. Axon degeneration does not occur. Return o normal axonal unction occurs over hours to months, o ten in the 2- to 4-week range. (See Schwartz10th ed.,p.1026.)
Answer:C
See able42-1.(SeeSchwartz10th ed., able42-2,p.1712.)
aAdd the three scoresto obtain the GlasgowComa Scale (GCS)score,which can range rom 3to 15.Add“T”a terthe GCSi intubated and no verbalscore ispossible.Forthesme patients,the GCScan range rom 3Tto 10T.
7. Temost common malignant tumor o thebrain is
A. Ependymoma
B. Astrocytoma
C. Ganglioglioma
D. eratoma
Answer:B
Astrocytomaisthemostcommon primarycentralnervoussystem (CNS) neoplasm. heterm glioma o ten isused to re er to astrocytomas speci ically, excluding other glial tumors. Astrocytomasaregraded rom I to IV.GradesI and II arere erred to as low-grade astrocytoma, grade III as anaplastic astrocytoma, andgradeIVasglioblastomamultiforme(GBM).Prognosisvariessigni icantlybetweengradesI/II,III,andIV,butnotbetween I and II. Median survival is 8 years a ter diagnosis with a lowgrade tumor, 2 to 3 years with an anaplastic astrocytoma, and roughly 1 year with a GBM. GBMs account or almost twothirds o all astrocytomas,anaplastic astrocytomas account or two-thirdso therest,and low-gradeastrocytomastheremainder.Figure42-2demonstratesthetypicalappearanceo aGBM. (SeeSchwartz10thed.,Figure42-20,pp.1733–1734.)
325
MotorResponse VerbalResponse Eye-OpeningResponse Obeyscommands 6 Oriented 5 Opensspontaneously 4 Localizestopain 5 Conused 4 Openstospeech 3 Withdrawsrompain 4 Inappropriatewords 3 Openstopain 2 Flexorposturing 3 Unintelligiblesounds 2 Noeyeopening 1 Extensorposturing 2 Nosounds 1 Nomovement 1
TABLE42-1 The Glasgow Coma Scale scorea
8. A 25-year-old man is seen in the emergency department afer he struck his head against the windshield in an automobile accident. He opens his eyes and withdraws his arm duringpain ulstimulationo hishand.Herespondsverbally toquestionswithinappropriatewords.HisGCSscoreis
A. 6
B. 9
C. 12
D. 15
9. Te most common level o cervical radiculopathy rom cervicaldischerniation is
A. C4–C5
B. C5–C6
C. C6–C7
D. C7– 1
Answer:B
he initial assessment o the trauma patient includes the primary survey, resuscitation, secondary survey, and de initive care.Neurosurgicalevaluation beginsduringtheprimarysurveywith thedetermination o theGCSscore(usuallyre erred to simplyas the GCS) or the patient. he GCSis determined by adding the scores o the best responses o the patient in each o three categories. he motor score ranges rom 1 to 6, verbal rom 1 to 5, and eyes rom 1 to 4. he GCSthere ore ranges rom 3 to 15, as detailed in able 42-1. racheal intubation or severe acial or eye swelling can impede verbal and eyeresponses.In thesecircumstances,thepatient isgiven the scoreo 1with amodi ier,such asverbal“1 ”where = tube. (SeeSchwartz10th ed.,p.1715.)
Answer:C
he cervical nerve roots exit the central canal above the pedicle o the same-numbered vertebra and at the level o the higher adjacent intervertebraldisc.For example,theC6nerve root passes above the C6 pedicle at the level o the C5–C6 discs. he cervical nerve roots maybe compressed acutelyby disc herniation, or chronically by hypertrophic degenerative changes o the discs, acets, and ligaments. able 42-2 summarizes the e ects o various disc herniations. Most patients with acute disc herniations. Most patients with acute disc herniations will improve without surgery, nonsteroidal antiin lammatorydrugs (NSAIDs), or cervical traction may help alleviate symptoms. Patients whose symptoms do not resolve
326
A B
FIG.42-2.
A. PostcontrastT1-weighted axialmagneticresonance imaging (MRI)demonstrating a ring-enhancing lesion in the anteromedial right temporallobe with centralnecrosis(darkarea)consistent with glioblastoma multiorme.B.T2-weighted axialMRIwith extensive bright signalsigniying peritumoraledema seen with glioblastoma multiormes.
or whohavesigni icantweaknessshouldundergodecompressivesurgery. hetwo main options or nerveroot decompression are anterior cervical discectomyand usion (ACDF) and posterior cervical oraminotomy (keyhole oraminotomy). ACDFallowsmoredirect accesstoand removalo thepathology (anterior to the nerve root). However, the procedure requires usion because discectomy causes a collapse o the interbody space and instability will likely occur. Fig. 42-3 demonstrates a C6–C7 ACDF with the typical interposed gra t and plating system. Keyhole oraminotomy allows or decompression without requiring usion,but it islesse ective or removingcentrallylocated canalpathology.(SeeSchwartz 10th ed., able42-6and Figure42-27,pp.1740–1741.)
Source:Adapted with permission rom Greenberg MS.HandbookofNeurosurgery,7th ed.NewYork:Thieme;2010,Table 18-18,p.461.
FIG.42-3. A. Anteroposteriorcervicalspine X-rayshowing the position o an anteriorcervicalplate used orstabilization a terC6–C7 discectomy.Patient presented with right tricepsweaknessand dysesthesiasin the right i th digit.Magnetic resonance imaging (MRI)revealed a right paracentralC6–C7herniated disc compressing the exiting C7nerve root.B. Lateralcervicalspine X-rayo the same patient clearly demonstratesthe position o the plate and screws.The allogra t bone spacerplaced in the drilled-out discspace isalso apparent.
327
Level Frequency(%) RootInjured Refex Weakness Numbness C4–C5 2 C5 — Deltoid Shoulder C5–C6 19 C6 Biceps Bicepsbrachii Thumb C6–C7 69 C7 Triceps Wristextensors(wristdrop) Secondandthirddigits C7–T1 10 C8 — Handintrinsics Fourthandfthdigits
TABLE42-2 Cervicaldischerniationsand symptomsbylevel
B A
10. A 35-year-old mother o two children, 5 and 6 years, has had amenorrhea and galactorrhea or thepast 12months. Her serum prolactin level is elevated, and radiographs o herskullshowan“emptysella.”Temostlikelydiagnosisis
A. Menopause
B. Pregnancy
C. Pituitarytumor
D. Sheehan syndrome
Answer:C
Pituitary adenomas arise rom the anterior pituitary gland (adenohypophysis). umors <1 cm diameter are considered microadenomas; larger tumors are macroadenomas. Pituitarytumorsmaybe unctional(ie,secreteendocrinologically activecompoundsat pathologiclevels) or non unctional(ie, secrete nothing or inactive compounds). Functional tumors are o ten diagnosed when quite small, due to endocrine dys unction. he most common endocrine syndromes are Cushing disease, due to adrenocorticotropic hormone secretion, Forbes-Albright syndrome, due to prolactin secretion, and acromegaly, due to growth hormone secretion.Non unctionaltumorsaretypicallydiagnosedaslarger lesions causing mass e ects such as visual ield de icits due to compression o the optic chiasm or panhypopituitarism due to compression o the gland. Figure 42-4 demonstrates a large pituitary adenoma. Hemorrhage into a pituitary tumor causes abrupt symptoms o headache, visual disturbance, decreased mental status, and endocrine dys unction. his is known as pituitary apoplexy. (See Schwartz 10th ed., Figure42-23,pp.1735–1736.)
328
FIG.42-4. PostcontrastT1-weighted sagittalmagneticresonance imaging (MRI)demonstrating a large sellar/suprasellarlesion (arrowheads)involving the third ventricle superiorlyand abutting the midbrain and ponsposteriorly.The patient presented with progressive visual ield and acuityloss.Pathologyand laboratory workrevealed a non unctioning pituitaryadenoma.
11. Te most common intradural extramedullary tumors in children are
A. Ependymomas
B. Astrocytomas
C. Dermoids
D. eratomas
12. Diabetics, intravenous (IV) drug abusers, and dialysis patients have an increased risk o contracting the CNS in ection
A. Pyogenicvertebralosteomyelitis
B. Subduralempyema
C. Discitis
D. Epiduralabscess
Answer:B Astrocytomasarethemostcommonintramedullarytumorsin children, although theyalso occur in adults. heymayoccur at all levels, although more o ten in the cervical cord. he tumor may inter ere with the CSF-containing central canal o the spinal cord, leading to a dilated central canal, re erred to as syringomyelia (syrinx). Spinal astrocytomas are usually low grade, but complete excision is rarely possible due to the nonencapsulated, in iltrative nature o the tumor. As a result, patients with astrocytomas are worse overall than patients with ependymomas.(SeeSchwartz10th ed.,pp.1738–1739.)
Answer:A
Pyogenic vertebral osteomyelitis is a destructive bacterial in ection o the vertebrae, usually o the vertebral body. Vertebral osteomyelitis requently results rom hematogenous spread o distant disease, but may occur as an extension o adjacent disease,such aspsoasabscessor perinephricabscess. Staphylococcus aureus and Enterobacter spp. are the most requent etiologic organisms. Patients usually present with ever and back pain. Diabetics, IV drug abusers, and dialysis patients have increased incidence o vertebral osteomyelitis. Epiduralextension maylead tocompression o thespinalcord or nerverootswith resultant neurologicde icit.Osteomyelitis presentsa lyticpictureon imagingand must bedistinguished rom neoplastic disease. Adjacent intervertebral disc involvementoccurs requentlywithpyogenicosteomyelitis,butrarely with neoplasia. Plain ilms and computed tomography (C ) helpassesstheextento bonydestruction or de ormitysuchas kyphosis.Magneticresonanceimaging(MRI) showsadjacent so t tissue or epidural disease. Most cases can be treated success ully with antibiotics alone, although the organism must be isolated to steer antibiotic choice. Blood cultures may be positive. Surgical intervention may be required or debridement when antibiotics alone ail, or or stabilization and usioninthesettingo instabilityandde ormity.(SeeSchwartz 10th ed.,p.1745.)
329
OrthopedicSurgery
1. Long bone ractures can be described as tting any o the ollowingrecognized typesEXCEPT
A. Convoluted
B. Transverse
C. Oblique
D. Spiral
2. Goals o racture reduction include all o the ollowing EXCEPT
A. Restorelength
B. Restoremarrowintegrity
C. Restorerotation
D. Restoreangulation
3. Which o the ollowing is true concerning compartment syndrome?
A. Duetodecreased intracompartmentalpressure
B. Typi ed byhyperesthesia
C. Can be assessed by needles placed into a ected compartment
D. Pain relieved bypassivemusclestretching
4. Shoulder dislocations are requentlyassociated with all o the ollowingEXCEPT
A. Injuriestolabrum
B. Humeralhead ractures
C. Seizures
D. Axillaryvasculaturedisruptions
Answer:A
Musculoskeletal injuries resulting rom trauma include ractures o bones, damage to joints, and injuries to so t tissues. Long bone ractures can be described as transverse, oblique, spiral, segmental, or comminuted. (See Schwartz 10th ed., p.1756.)
Answer:B
Reduction isper ormed with axialtraction and reversalo the mechanism o injury in order to restore length, rotation, and angulation.(SeeSchwartz10th ed.,p.1757.)
Answer:C
Compartment syndrome is an orthopedic emergency caused by signi icant swelling within a compartment o an injured extremity that jeopardizes blood low to the limb. Increased pressure within the compartment compromises per usion to musclesandcan causeischemiaornecrosis.Patientscomplain o pain and numbness, and passive stretch o muscles within the compartment causes severe pain. While the diagnosis is based on clinical examination, pressures can be measured with needlesplaced intothecompartment,which isnecessary in unconsciouspatientswho willnot showtheseexamination indings.(SeeSchwartz10th ed.,p.1757.)
Answer:D
The shoulder is one o the most commonly dislocated joints and most dislocations are anterior. They are o ten associated with injuriesto the labrum (Bankart lesion),impression ractureso thehumeralhead (Hill-Sachslesion),and rotator cu tears. Posterior dislocations are associated with seizures or electricshock.Adequateradiographsarerequired todiagnose a shoulder dislocation, with the axillary view being the most critical.(SeeSchwartz10th ed.,p.1759.)
331 43
CHAPTER
5. Which o the ollowingis NOT a component o the elbow dislocation “TerribleTriad”?
A. Elbowdislocation
B. Radialhead racture
C. Coronoid racture
D. Radialnervedamage
6. Each o the ollowing is associated with pelvic racture EXCEPT
A. Promptoperativeintervention or pubicrami ractures.
B. Li e-threateninghemorrhage.
C. Associated genitourinary(GU)injury.
D. Displacement associated with two or more racturesin thepelvicring.
Answer:D
Asevereinjury,known astheTerribleTriad,includesan elbow dislocation, a radial head racture, and a coronoid racture. These are unstable injuries and require repair o the torn lateral collateral ligament (LCL), ixation or replacement o the radial head, and possible ixation o the coronoid depending on the size o the racture ragment. (See Schwartz 10th ed., p.1759.)
Answer:A Pelvic ractures are indicative o high energy trauma and are associatedwithhead,chest,abdominal,andurogenitalinjuries. Hemorrhage rom pelvic trauma can be li e-threatening and patients can present with hemodynamic instability, requiring signi icant luid resuscitation, and blood trans usions. The bleedingthatoccursiso ten duetoinjurytothevenousplexus in the posterior pelvis, though it can also be due to a large vesselinjurysuch asaglutealartery.Other associated injuries are bladder and urethral injuries that mani est with bleeding rom the urethral meatus or blood in the catheter and need to be assessed with a retrograde urethrogram. The pelvis is a ringstructuremadeup o thesacrum and thetwoinnominate bonesthat areheld together bystrongligaments.Becauseit is a ring, displacement can onlyoccur i the ringis disrupted in two places.Displaced sacral racturesand iliacwing ractures are treated with screws or plates, while pubic rami ractures can usually be managed nonoperatively. (See Schwartz 10th ed.,p.1760.)
7. Which o the ollowingisassociated with hip racture?
A. Morecommon in men than women.
B. Mortalityratein rst year afer hip ractureis25%.
C. Usuallymanaged nonoperatively.
D. Traction with bed rest rather than earlymobilization is thechie therapeuticgoal.
Answer:B
Hip racturesarean extremelycommon injuryseen in orthopedicsand areassociated with signi icant morbidityand mortality. They most o ten occur in elderly patients a ter ground level alls, are much more common in women than men, and occur morecommonlyin patientswith osteoporosis.Patients who su er hip ractures are at increased risk or many complications,includingdeepvein thrombosis,pulmonaryembolism, pneumonia, deconditioning, pressure sores, and even death, as the mortality rate in the irst year ollowing a hip racture is around 25%. One o the most important reasons or per ormingsurgeryisto prevent thesecomplications,and getting patients out o bed and walking as soon as possible diminishes their risk. There ore, surgeryis almost always the treatment o choice or hip ractures. (See Schwartz 10th ed., pp.1760–1761.)
8. Chronicunremittingbackpain suggestsallo the ollowing possibilitiesEXCEPT
A. In ection
B. Malignancy(primary)
C. Spinalcord in arction
D. Metastaticdisease
9. Scoliosiscurvesareclassi ed asanyo the ollowingpossibilitiesEXCEPT
A. Congenital
B. Covascular
C. Neurological
D. Myogenic
Answer:C
Back pain occurs in the majority o adults but is usually sellimited resolving in 1 to 2 weeks. Chronic unremitting back pain suggeststhepossibilityo in ection,malignancy,or metastaticdisease.(SeeSchwartz10th ed.,p.1771.)
Answer:B
Scoliotic curves are classi ied as congenital, degenerative, metabolic (mucopolysaccharidoses), neurogenic (cerebral palsy),andmyogeniccurves(musculardystrophy).Idiopathic scoliosisisthemostcommon orm,andrepresentsaspectrum o geneticdisease.(SeeSchwartz10th ed.,p.1771.)
332
10. Surgical management o arthritis includes all o the ollowingEXCEPT
A. Arthroplasty
B. Osteotomy
C. Arthrodesis
D. Arthrolysis
Answer:D
A ull description o surgical options can be ound in section “Surgical Management o Arthritis.” (See Schwartz 10th ed., pp.1772–1773.)
333
Surgeryofthe Hand and Wrist
1. For vascular injuriesto thehand requiringtourniquet,the maximum time the tourniquet should be applied to prevent tissuenecrosisis
A. 1hour
B. 2hours
C. 3hours
D. 4hours
2. Anesthetic agents with epinephrine should NO be used in
A. Te ngertip
B. Tehand
C. Tewrist
D. Te orearm
3. Most nondisplaced ractures do NO require surgical treatment EXCEP
A. Toseo thelunateboneo thewrist.
B. Toseo thecapitateboneo thewrist.
C. Toseo thescaphoid boneo thewrist.
D. Allnondisplaced racturesrequiresurgicaltreatment.
Answer:B
Initial treatment or an actively bleeding wound should be direct local pressure or not less than 10 continuous minutes. I this is unsuccess ul,an upper extremitytourniquet in lated to100mm Hgabovethesystolicpressureshould beused.One should keep thistourniquet timeto lessthan 2hoursto avoid tissue necrosis. Once bleeding is controlled well enough to evaluate the wound, it may be cautiously explored to evaluate or bleedingpoints.Onemust beverycautiousi attempting to ligate these to ensure that adjacent structures, such as nerves,arenot included in theligature.(SeeSchwartz10th ed., p.1799.)
Answer:A
A commonly held axiom is that epinephrine is unacceptable to be used in the hand. Several recent large series have dispelled this myth. Epinephrine should not be used in the ingertip and not in concentrations higher than 1:100,000 (ie, what is present in commerciallyavailable local anesthetic with epinephrine).Beyond that, its use is acceptable and may be use ul in an emergencyroom (ER) where tourniquet control may not be available. Also, because most ER procedures are done under pure local anesthesia, many patients will not tolerate the discom ort o the tourniquet beyond 30 minutes. Epinephrine will provide hemostasis and also prolong the e ect o thelocalanesthetic.(SeeSchwartz10th ed.,p.1796.)
Answer:C
Most nondisplaced ractures do not require surgical treatment. he scaphoid bone o the wrist is a notable exception tothisrule.Duetopeculiaritiesin itsvascular supply,particularly vulnerable at its proximal end, nondisplaced scaphoid ractures can ail to unite in up to 20%o patients even with appropriate immobilization. Recent developments in hardware and surgical technique have allowed stabilization o the racture with minimal surgical exposure. One prospective randomized series o scaphoid wrist ractures demonstrated shorteningo timeto union byup to 6weeksin thesurgically treated group,but no di erencein rateo union.Surgerymay beuse ulin theyounger,moreactivepatient who would bene it romanearlierreturnto ullactivity.(SeeSchwartz10thed., pp.1795–1796.)
335 44
CHAPTER
4. A patient shown to have wasting at the interdigital web spaces,experiencesnumbnesso thering nger andexhibitsWartenbergsign on physicalexamination most likelyis su ering rom
A. Cubitaltunnelsyndrome
B. Carpaltunnelsyndrome
C. Pronator syndrome
D. Anterior interosseousnervesyndrome
Answer:A he ulnar nerve also innervates the dorsal sur ace o the small inger and ulnar side o the ring inger, so numbness in these areas can be explained by cubital tunnel syndrome. he patient may also report weakness in grip due to e ects on the lexor digitorum pro undus (FDP) tendons to the ring andsmall ingersandtheintrinsichandmuscles.Patientswith advanced diseasemaycomplain o inabilityto ullyextend the ringand small inger interphalangeal(IP)joints.
Physical examination or cubital tunnel syndrome begins withinspection.Look orwastinginthehypothenareminence and the interdigital web spaces. When the hand rests lat on the table, the small inger may rest in abduction with respect to the other ingers;this is called Wartenberg sign. inel sign is o ten present at the cubital tunnel. Elbow lexion test will o ten bepositive.Grip strength and inger abduction strength should be compared to the una ected side. Froment sign can be tested by placing a sheet o paper between the thumb and index ingerandinstructingthepatienttoholdon tothepaper while the examiner pulls it awaywithout lexingthe inger or thumb(thisteststhestrength o theadductor pollicisand irst dorsalinterosseousmuscles).I thepatientmust lextheindex inger and/or thumb (FDP-index and lexor pollicis longus [FPL], both median nerve supplied) to maintain traction on the paper, this is a positive response. (See Schwartz 10th ed., p.1806.)
5. Temostcommon primarymalignanttumoro thehandis
A. Melanoma
B. Basalcellcarcinoma
C. Squamouscellcarcinoma(SCC)
D. Epithelioid sarcoma
6. Necrotizingin ections
A. O en present with pain out o proportion to ndings.
B. O en havedischargepresent.
C. Debridement should begin ollowing con rmation by wayo radiograph ndings.
D. Oralantibioticsshould begin immediately.
Answer:C
Squamouscellcarcinoma(SCC)isthemostcommon primary malignant tumor o the hand, accounting or 75 to 90%o all malignancies o the hand. Eleven percent o all cutaneous SCC occurs in the hand. It is the most common malignancy o the nail bed. Risk actors include sun exposure, radiation exposure, chronic ulcers, immunosuppression, xeroderma pigmentosa, and actinic keratosis. (See Schwartz 10th ed., p.1817.)
Answer:A
Bacteria spread along the ascial layer, resulting in the death o so t tissues,which isin part dueto theextensiveblood vessel thrombosis that occurs. An inciting event is not always identi ied. Immunocompromised patients and those who abuse drugs or alcohol are at greater risk, with intravenous drug users having the highest increased risk. he in ection can by mono- or polymicrobial, with group A β-hemolytic Streptococcus being the most common pathogen, ollowed by α-hemolytic Streptococcus, Staphylococcus aureus, and anaerobes. Prompt clinical diagnosis and treatment are the most important actors or salvaging limbs and saving li e. Patients will present with pain out o proportion with indings. Appearance o skin may range rom normal to erythematousor maroon with edema,induration,and blistering.Crepitusmayoccur i agas- ormingorganism isinvolved. “Dirtydishwater luid”maybeencountered asa scant grayish luid,but o ten thereislittleto no discharge. heremaybeno appreciable leukocytosis. he in ection can progress rapidly and can lead to septic shock and disseminated intravascular coagulation. Radiographs mayreveal gas ormation, but they must not delay emergent debridement once the diagnosis
336
7. Temajorityo acutecaseso in ectionsfexor tenosynovitis(F S)aredueto
A. Systemiclupuserythematosus
B. Chronicinfammation asaresult o diabetes
C. Rheumatoid arthritis(RA)
D. Purulent in ection
8. Temost common so tissuetumor o thewrist is
A. Mucouscyst
B. Ganglion cyst
C. Lipoma
D. Schwannoma
9. Allhandin ectionsEXCEP the ollowingrequiresurgical management
A. Paronychia
B. Felon
C. Cellulitis
D. Osteomyelitis
issuspected.Intravenous antibioticsshould be started immediately to cover gram-positive, gram-negative, and anaerobic bacteria.Patientswillrequire multiple debridements,and the spread o in ection is normallywider than expected based on initial assessment. Necrotizing myositis, or myonecrosis, is usuallycaused byClostridium perfringensdue to heavilycontaminated wounds.Unlikenecrotizing asciitis,muscleisuniversallyinvolvedand oundtobenecrotic. reatmentincludes emergent debridement o allnecrotictissuealongwith empiricalintravenousantibiotics.(SeeSchwartz10th ed.,p.1811.)
Answer:D
Flexor tenosynovitis (F S) is a severe pathophysiologic state causing disruption o normal lexor tendon unction in the hand. A variety o etiologies are responsible or this process. Most acute cases o F S are due to purulent in ection. F S also can occur secondary to chronic in lammation as a resulto diabetes,rheumatoidarthritis,crystallinedeposition, overuse syndromes, amyloidosis, psoriatic arthritis, systemic lupuserythematosus,and sarcoidosis.(SeeSchwartz10th ed., pp.1811–1812.)
Answer:B
Ganglion cyst is the most common so t tissue tumor o the hand and wrist,comprising50to 70%o allso t tissuetumors in this region. hey can occur at any age but are most common in thesecond to ourth decadeswith aslight predilection toward emales.(SeeSchwartz10th ed.,p.1815.)
Answer:C
All hand in ections other than cellulitis will require surgical management. Clinical examination, particularly noting the area o greatest tenderness and/or in lammation, is the single most use ul diagnostic tool to localize any purulence requiring drainage. Speci ic recommendations or di erentiating among the possible locations o hand in ection are included in thediagnosticalgorithm shown in Fig.44-1.(SeeSchwartz 10th ed.,Figure44-23,p.1814.)
337
10. Which o the ollowing patient groups has a 1000- old increased risk o developing squamous cell carcinoma (SCC)?
A. ransplant patientson immunosuppression
B. Patientswith xerodermapigmentosa
C. Patientswith actinickeratosis
D. Patientsexposed toinorganicarsenic
Answer:B
SCC is the most common primary malignant tumor o the hand, accounting or 75 to 90% o all malignancies o the hand.Eleven percent o allcutaneousSCCoccursin thehand. It is the most common malignancy o the nail bed. Risk actorsincludesun exposure,radiation exposure,chroniculcers, immunosuppression, xeroderma pigmentosa, and actinic keratosis. Marjolin’s ulcers represent malignant degeneration o old burn or traumatic wounds into an SCC and are a more aggressive type. ransplant patients on immunosuppression have a our old increased risk and patients with xeroderma pigmentosa have a 1000- old increased risk o developing an SCC. hey o ten develop as small, irm nodules or plaques with indistinct margins and sur ace irregularities ranging rom smooth to verruci orm or ulcerated (Fig. 44-2). hey are locally invasive, with 2 to 5% lymph node involvement. Metastasis rates o up to 20%have been reported in radiation or burn wounds. Standard treatment is excision with 0.5- to 1.0-cm margins. Other treatment options include curettage and electrodessication, cryotherapy, and radiotherapy. (See Schwartz10th ed.,Figure44-27,p.1817.)
338
Nondiagnostic Fracture Foreignbody Cellulitis admit,IVAbx serialexam Siteoffluctuance Entirefinger se Y No PyogenicFTS Kanavel’s signspresent MRIifno fluctuance Subcutaneous abscess Thenar abscess Midpalm abscess Hypothenar abscess Distal Lossof palmar concavity Radialto IFMC Ulnarto SFMC Webspace abscess Palm Painwith axialloading ofjoint Pyogenicvs crystalline arthritis Consider arthrocentesis Noimprovement in48hours Handinflammation PlainX-rays Partialfinger Dorsal Centered onjoint Between digits Localizedfluctuance
FIG.44-1. Diagnosticalgorithm.Diagnosticworkup fora patient with hand inflammation to evaluate forinfection.See text fordetails about particularinfectiousdiagnoses.Abx= antibiotics;exam = examination;FTS= flexortenosynovitis;IFMC= indexfingermetacarpal; IV= intravenous;MRI= magneticresonance imaging;SFMC= smallfingermetacarpal.
11. Which o the ollowingabout enchondromasistrue?
A. Havenever been reported in thetrapezoid.
B. Discovery is o en prompted by patients presenting with hand pain.
C. Arethemostcommonmalignantprimarybonetumors.
D. Temost common location isthemiddlephalanges.
Answer:A his is the most common primary benign bone tumor o the hand and wrist and iso cartilageorigin.Upto90%o allbone tumors in the hand and wrist are enchondromas, with 35 to 54% o all enchondromas occurring in the hand and wrist. hey are o ten ound incidentally on X-rays taken or other reasons(eg,hand trauma). heyareusuallysolitaryand avor the diaphysis o small tubular bones and are most common in the second and third decades o li e. he most common location is in the proximal phalanges, ollowed by the metacarpals and then middle phalanges. Enchondroma has never been reported in thetrapezoid.Presentation isusuallyasymptomatic, but pain may occur i there is a pathologic racture or impending racture. he etiology is believed to be rom a ragment o cartilage rom thecentralphysis.Histologyshows well-di erentiated hyaline cartilage with lamellar bone and calci ication. wo variants o enchondroma include Ollier disease (multiple enchondromatosis) and Ma ucci syndrome (multiple enchondromatosis associated with multiple so t tissue hemangiomas). Malignant trans ormation is very rare in the solitary orm, but there is a 25%incidence by age 40 in Ollier patients and a 100% li e-time incidence in Maucci patients. When malignant trans ormation does occur, it is almost uni ormlya chondrosarcoma with pain and rapid growth. Diagnosis is usually made based on history, physical examination, and X-rays. here is a well-de ined, multilobulated central lucency in the metaphysis or diaphysis that can expand causing cortical thinning or sometimes, thickening (Fig.44-3).Furtherimagingisseldom needed,butaC would bethestudyo choice.(SeeSchwartz10th ed.,Figure44-29B, p.1819.)
339
FIG.44-2. Squamouscellcarcinoma involving the nailfold and nail bed.Note the wart-like and ulcerated appearance.
12. Proper handling o a traumatically amputated digit or limbincludeswhich o the ollowing?
A. Placedryin awaterproo bag.
B. Immersein an antisepticsolution.
C. Prep and wrapped in moistened gauze.
D. Placeon dryice.
Answer:C
In preparation or replantation,theamputatedpartand proximal stump should be appropriately treated. he amputated part should be wrapped in moistened gauze and placed in a sealed plastic bag. his bag should then be placed in an ice water bath. Do not use dry ice, and do not allow the part to contact icedirectly; rostbitecan occur in theamputated part, which will decrease its chance o survival a ter replantation. Bleeding should be controlled in the proximal stump by as minimala meansnecessary,and thestump should bedressed with a nonadherent gauze and bulky dressing. (See Schwartz 10th ed.,p.1800.)
340
FIG.44-3. Enchondroma.X-rayofthe phalanxdemonstratesawelldefined centrallucency.Surrounding cortexmaythin orthicken. Thinning ofthe cortexcontributesto riskofpathologic fracture.
Plasticand Reconstructive Surgery
1. All o the ollowing are true about split-thickness skin gra sEXCEP
A. Degree o contraction is dependent on amount o dermisin gra .
B. High reliabilityo take.
C. Healing with abnormal pigmentation more common in thin than thickgra s.
D. Meshing gra s improve their ultimate cosmetic appearance.
2. Which o the ollowingde nitionsisINCORREC ?
A. Flap composition: Description o the tissue componentswithin thefap.
B. Flap contiguity: Te position o a fap relative to its recipient bed.
C. Pedicle: Bridge o tissue that remains between a fap and itssource;blood vesselsthat nourish afap.
D. Free fap: Flaps that are completely detached rom the bodyprior to their reimplantation with microvascular anastomoses.
Answer:D
Manyo thecharacteristicso asplit-thicknessgra t aredetermined by the amount o dermis present. Less dermis translatesintolessprimarycontraction (thedegreetowhich agra t shrinks in sur ace area a ter harvesting and be ore gra ting), more secondary contraction (the degree to which a gra t shrinks during healing), and better chance o gra t survival. hin split gra ts have low primary contraction, high secondary contraction, and high reliability o gra t take, o ten even in imper ect recipient beds. hin gra ts,however,tend to heal with abnormal pigmentation and poor durability compared with thick split gra ts and ull-thickness gra ts. Split gra ts maybemeshedtoexpandthesur aceareathatcan becovered. (SeeSchwartz10th ed.,p.1832.)
Answer:B
he composition o a lap describes its tissue components. he contiguity o a lap describes its position related to its source.Distant lapsaretrans erred rom adi erent anatomic region to the de ect. heymayremain attached to the source anatomicregion (pedicled laps) or maybetrans erred as ree laps by microsurgery. hese are completely detached rom the body, and their blood supply is reinstated by microvascular anastomoses to recipient vesselsclose to the de ect. he term pediclewas originallyused to describe a bridge o tissue that remains between a lap and its source, similar to how a peninsula remains attached to its mainland. However, as knowledgeo lap blood supplyand (micro)vascular anatomy hasimproved over theyears,theterm pediclehasincreasingly becomereserved or describingtheblood vesselsthat nourish the lap.(SeeSchwartz10th ed.,p.1833.)
3. Factors infuencing the development o cle lip/palate includeallthe ollowingEXCEP
A. Increased parentalage.
B. Vitamin Ade ciencyduringpregnancy.
C. In ectionsduringpregnancy.
D. Smokingduringpregnancy.
Answer:B
he cause o oro acial cle ting is elt to be multi actorial. Factors that likely increase the incidence o cle ting include increased parental age, drug use and in ections during pregnancy, and smoking during pregnancy. (See Schwartz 10th ed.,pp.1840–1841.)
341 45
CHAPTER
4. Principles o reconstructive surgery include all o the ollowingEXCEP
A. Adequate restoration o lost anatomic components without residualde cits.
B. Uncomplicated and timelywound healing.
C. Individualization o speci c reconstructive technique tospeci cpatient de cit.
D. Compromiseo extent o tumor resection i needed or speci creconstructivesurgicaloutcome.
Answer:D
he reconstructive surgeon aims to restore lost anatomic components adequately. Residual de icits, seemingly inconsequential, may progress to psychological morbidity, societal malacceptance, and social withdrawal. Uncomplicated and timelywoundhealingisimportanttoallowadjuvanttherapies when indicated and smooth discharge to home and occupation. Each de ect can be addressed by a number o methods, but the technique must be decided or each individual patient.Although amorecomplexreconstruction might o er improved outcomes, it may bring an increased risk o complications. Some patients maythere ore bene it rom use o a simpler method with more acceptable anesthetic and operative risk rather than a gold-standard reconstruction. (See Schwartz10th ed.,p.1862.)
342
Anesthesia forthe SurgicalPatient
1. Malignant hyperthermia(MH)can developa er receiving generalanesthesia. riggeringagentsinclude,butareNO limited to
A. Halothane,iso urane
B. Des urane
C. Electro urane
D. Hypo urane,osteo urane
2. Postoperative nausea and vomiting (PONV) typically occursin what percentageo surgicalcases?
A. 10–15%
B. 90%
C. 20–30%
D. None
3. Te intentional dilution o blood volume o en is re erred toas
A. Acutenormovolemichemodilution (ANH)anemia
B. Acutehypovolemicnormodilution (AHN)anemia
C. Hypercoagulablehemodilution (HH)anemia
D. Noneo theabove
Answer:A
Malignant hyperthermia(MH)isahereditary,li e-threatening, hypermetabolic acute disorder, developing during or a ter receiving general anesthesia. he clinical incidence o MH is about 1:12,000 in children and 1:40,000 in adults. A genetic predisposition and one or more triggering agents are necessaryto evoke MH. riggeringagents include all volatile anesthetics (eg, halothane, en lurane, iso lurane, sevo lurane, and des lurane) and the depolarizing muscle relaxant succinylcholine. Volatile anesthetics and/or succinylcholine cause a rise in the myoplasmic calcium concentration in susceptible patients, resulting in persistent muscle contraction. (See Schwartz10th ed.,p.1918.)
Answer:C
Postoperative nausea and vomiting (PONV) typically occurs in 20 to 30%o surgical cases, with considerable variation in requency reported between studies (range, 8–92%). PONV is generally considered a transient, unpleasant event carrying little long-term morbidity;however, aspiration o emesis, gastricbleeding,and wound hematomasmayoccur with protractedor vigorousretchingor vomiting. roublesomePONV can prolongrecoveryroom stayand hospitalization and isone o the most common causes o hospital admission ollowing ambulatorysurgery.(SeeSchwartz10th ed.,p.1915.)
Answer:A
he intentional dilution o blood volume o ten is re erred to as acute normovolemichemodilution (ANH) anemia. ANH is a technique in which whole blood is removed rom a patient, while the circulating blood volume is maintained with acellular luid. Blood is collected via central lines with simultaneous in usion o crystalloid or colloid solutions. Collected bloodisrein useda ter major bloodlosshasceased,or sooner, i indicated. Blood units are rein used in the reverse order o collection. Under conditions o ANH, the increased plasma compartment becomes an important source o O2, which is delivered tothetissues.(SeeSchwartz10th ed.,p.1914.)
343 46
CHAPTER
4. Hydroxyethyl starch (HES), gelatin, and albumin are severaltypeso
A. Crystalloids
B. Coagulates
C. Antigens
D. Colloids
5. O theoptionsbelow,which isastageo apatient undergoinginhalation induction?
A. Con usion
B. Aggression
C. Excitement
D. Noneo theabove
6. Regionalanesthesiamaybeuse ulin patientswith
A. Advanced liver disease
B. Renaldisease
C. Diabetesmellitus
D. Colorectalcancer
7. P3 o the American Society o Anesthesiologists physical statusclassifcation system iswhich o the ollow
A. Anormalhealthypatient.
B. A moribund patient who is not expected to survive without operation.
C. Apatientwithseveresystemicdiseasethatisaconstant threat toli e.
D. Apatient with severesystemicdisease.
Answer:D
Several types o colloids are available, but three are most commonly used—hydroxyethyl starch (HES), gelatin, and albumin. he HES preparations di er rom one another according to their concentration, molecular weight, and extent o hydroxy- ethylation or substitution, with resultant varying physiochemical properties. HES solutions most o ten are described according to their weight-averaged mean molecular weight in kilodaltons(kDa):high molecular weight (450 kDa), middle molecular weight (200 kDa, 270 kDa), and low molecular weight (130 kDa, 70 kDa). HES 450 kDa solutions are available in a normal saline solution (HES450/NS) and in a lactated, balanced salt solution (HES 450/BS).Although allo thesecolloidsareusedin Europe,gelatins are not available in the United States, and the only HES preparationsapprovedbytheU.S.FoodandDrugAdministrationarethe6%highmolecularweight(450kDa) ormulations. (SeeSchwartz10th ed.,p.1911.)
Answer:C
Patients undergoing inhalation induction progress through three stages: (a) awake, (b) excitement, and (c) surgical level o anesthesia. Adult patients are not good candidates or this type o induction, as the smell o the inhalation agent is unpleasant and the excitement stage can last or several minutes, which may cause hypertension, tachycardia, laryngospasm,vomiting,and aspiration.Children,however,progress through stage 2 quickly and are highly motivated or inhalation induction asan alternativeto theintravenous(IV) route. he bene it o postinduction IV cannulation is the avoidance o manypresurgicalanxieties,and inhalation induction isthe most common technique or pediatric surgery.(See Schwartz 10th ed.,p.1909.)
Answer:A
Regional anesthesia may be use ul in patients with advanced liver disease, assuming coagulation status is acceptable. When general anesthesia is selected, administration o modest doses o volatile anesthetics with or without nitrous oxide or entanyl o ten is recommended. Selection o nondepolarizingmusclerelaxantsshould consider clearancemechanisms or these drugs. For example, patients with hepatic cirrhosis maybe hypersensitive to mivacurium because o the lowered plasma cholinesterase activity. Per usion to the liver is maintained by administering luids (guided by illing pressures) and maintainingadequate systemic pressure and cardiac output.(SeeSchwartz10th ed.,p.1907.)
Answer:D
American Society o Anesthesiologists physical status classiication system
P1:Anormalhealthypatient.
P2:Apatient with mild systemicdisease.
P3:Apatient with severesystemicdisease.
P4: A patient with severe systemic disease that is a constant threat toli e.
344
8. Duringapreoperativephysicalexamination,specifcareas toinvestigate or therespiratorysystem include
A. Consciousness;neurocognition;peripheralsensory
B. Cervicalspinemobility;visualizeuvula;artifcialteeth; thyromentaldistance
C. Blood pressure;standingand sitting, bilateral;peripheral pulses; heart auscultation; heart rate; murmur; rhythm
D. Auscultation o lungs;wheezes;rales
CentralNervousSystem CardiovascularSystem
Consciousness; neurocognition;peripheral sensory
Bloodpressure;standingandsitting, bilateral;peripheralpulses;heart auscultation;heartrate;murmur;rhythm
P5:A moribund patient who is not expected to survive without theoperation.
P6: A declared brain-dead patient whose organs are being removed or donor purposes.(SeeSchwartz10th ed.,p.1905.)
Answer:D
he physical examination is targeted primarily at the central nervous system (CNS), cardiovascular system, lungs, and upper airway. Speci ic areas to investigate are given in able46-1.(SeeSchwartz10th ed., able46-5,p.1904.)
RespiratorySystem OralAirway
Auscultationolungs; wheezes;rales
Cervicalspinemobility;visualizeuvula; artifcialteeth;thyromentaldistance
345
TABLE46-1 Preoperative physicalexamination
SurgicalConsiderationsin the Elderly
1. Physiologicaging,or “senescence,”isde ned asdecreased unctional reserve o critical organ systems. It is generally believed tooccur at what age?
A. 55years
B. 70years
C. 90years
D. Node ned age
2. Frailty is a major risk actor or postoperative complications.Whicho the ollowingmeasuresdoesNO contributeto railty?
A. Impaired cognition
B. Oneor more allsin past 6months
C. Osteoporosis
D. Hematocrit <35%
3. What is the most common indication or surgical intervention in theelderly?
A. Coronarydisease
B. Appendicealdisease
C. Longbone racture
D. Biliarytract disease
4. What is the leading cause o postoperative morbidity and mortalityin theelderly?
A. Congestiveheart ailure
B. Cerebrovascular accident
C. Pneumonia
D. Renal ailure
5. Elderly patients with acute peritonitis may NO present with typical symptoms o acute abdominal pain, ever, or leukocytosisduetoan impaired immuneresponse.Ahigh indexo suspicion isneededastheinitialclinicaldiagnosis in elderly patients with acute appendicitis is correct in what percent o cases?
A. Lessthan 50%
B. 70–80%
C. 90–95%
D. 100%
Answer:B
hecut o or thede inition o senescencehasincreased rom 55 years in the late 1960s to 70 years currently. his is a generalde inition aseach patient shouldbeassessed or their own capacity to respond to the stress o surgery. (See Schwartz 10th ed.,p.1923.)
Answer:C
Frailty, disability(inabilityto per orm more than one activity o daily living), and comorbidities such as anemia and hypoalbuminemiaaretheprimaryconditionswhich impact on the outcomeo surgicalproceduresin theelderly.Each condition should be assessed preoperatively. (See Schwartz 10th ed., p.1926.)
Answer:D
Biliarytract diseaseduetopigmented or cholesterolstone ormation and acute cholecystitis are common problems in the elderly and most likely to lead to surgical intervention. he mortality risk o emergency cholecystectomy is our times that o elective cholecystectomy in the elderly. (See Schwartz 10th ed.,p.1924.)
Answer:A
Impaired cardiac reserve means that elderly patients cannot increase their ejection raction as demand increases, but are dependent on ventricular illingand increased stroke volume tomeet theneed or increased output.Intravascular volumeis there orecriticallyimportant,but luid overload ispoorlytoleratedandcan result in congestiveheart ailure.(SeeSchwartz 10th ed.,pp.1926–1927.)
Answer:A
In elderly patients with acute appendicitis or acute cholecystitis, one-third lack symptoms o abdominal pain, one-third are a ebrile, and one-third have a normal white blood cell count. here ore an “unimpressive” abdominal examination isirrelevant in theevaluation o theelderlypatient whosetolerance or ood hassuddenlychanged.(SeeSchwartz10th ed., p.1926.)
347 47
CHAPTER
6. Determination o an elderlypatient’snutritionalstatusand reversal o malnutrition is important to prevent all o the ollowingcomplicationsEXCEP
A. Poor wound healing
B. Increased nosocomialin ections
C. Delayed unctionalrecovery
D. Adversedrugreactions
7. In elderly patients undergoing heart valve replacement, bioprosthetic valves are pre erred over synthetic valves because
A. Tere is less need or anticoagulation which is hazardousin theelderly.
B. Teoperativetimeisshorter which reducestherisk o pulmonarycomplications.
C. Synthetic valves have a higher incidence o manu acturingde ects.
D. Te extent o hemolysis is less with bioprosthetic valves.
8. Cancer treatment in elderlypatientsis
A. Alwayslesssuccess ul.
B. Complicated by the act that clinical trials usually do not includeelderlysubjects.
C. Recommended equallyasin younger patients.
D. Doesnot changeoverallli eexpectancy.
9. Avoidance o surgical resection o breast cancer in an elderlypatient isunacceptablebecause
A. Mortality rates o breast surgery are very low in the elderly.
B. Medical therapy (eg, tamoxi en) instead o surgery is equivalent tosurgicaloutcomes.
C. O decreased li eexpectancy.
D. Screeningmammographyislessuse ulin theelderly.
10. What is the likelihood that an asymptomatic, 2 cm, solitarypulmonarynoduleseen on chest X-rayin a 70-yearold smoker isamalignancy?
A. 25%
B. 50%
C. 70%
D. 90%
Answer:D
Poor nutritional status in elderly patients is common. It is estimated that 9 to 15% o patients older than 65 years are malnourished in the outpatient setting, and this increases to 12 to 50%in the acute hospital setting, and 25 to 60%in the chronicallyinstitutionalized.A ormalnutritionalassessment and preoperative nutritional repletion, i needed, can signi icantly reduce the incidence o postoperative complications. he incidence o postoperative complications is increased in patients with an albumin level <3.5 g/L. (See Schwartz 10th ed.,p.1929.)
Answer:A
Prolonged anticoagulation is more hazardous in the elderly where therisk o allsisincreased.Even a all rom thestanding position can result in a atal intracranial bleed in an anticoagulated patient.(SeeSchwartz10th ed.,p.1031.)
Answer:B
he requency o re errals or surgical treatment o equivalent stage cancer is decreased in the elderly or virtually all tumors. Despite this, survival a ter surgery is nearly equivalent or same stage malignancy as in younger patients. he data derived rom clinical trials o adjuvant and neoadjuvant therapyarelesshelp ul or decision makingin elderlypatients because elderly subjects are usually not included in clinical studies.(SeeSchwartz10th ed.,p.1931.)
Answer:A
Elderlypatientshavea higher incidence o breast cancer than younger patients,and their mortalityrisk rom breast surgery is less than 1%. Medical therapyis less e ective than surgical removal o the primarytumor,and screeningmammography reducescancer-related mortalityin patientsup to age75.(See Schwartz10th ed.,p.1932.)
Answer:C
Lung cancer is the leading cause o cancer-related death in patients older than 70 years. he peak incidence is between 75 and 79 years. Non–small-cell lung cancer accounts or roughly 80% o all cases. In cases o resectable primary lung cancer, resection remains the treatment o choice, with an expected survival o almost 3 years. (See Schwartz 10th ed., pp.1933–1934.)
348
11. In elderlytraumapatients,injuries rom allsaccounts or about 20% o etiologies. Te management o these patients is complicated by all o the ollowing actors EXCEP
A. Concurrent medication use includinganticoagulants and betablockers.
B. Coexistingdementiaand cerebrovascular disease.
C. Underlyingcardiacarrhythmias.
D. Lack o advanced directives to guide clinical-care decisions.
12. Minimally invasive approaches to surgical treatment have been shown to be associated with reduced morbidityandmortalityin elderlypatients orallo the ollowing EXCEP
A. Repair o abdominalaorticaneurysm.
B. Removalo thegallbladder.
C. Colon resection.
D. Parathyroidectomy.
13.Te goals o palliative surgery include all o the ollowing EXCEP
A. Relie o symptoms.
B. Preservation o qualityo li e.
C. Eradication (cure)o thedisease.
D. Prevention o complicationso disease.
14. Tegoalso ethicaldecision makingin theelderlypatient includeallo the ollowingEXCEP
A. Acknowledgement o medical utility.
B. Clari y and ollow patients’ wishes regarding likesustainingtherapies.
C. Respect patient’s autonomy to decide the course o therapy.
D. Abidebythewisheso the amilyi theyconfict with thoseo thepatient.
15. Tecombined recommendationso theAmerican College o SurgeonsNationalSurgicalQualityImprovement Program (ACS-NSQIP)andtheAmerican GeriatricsSociety (AGS) have provide best practice guidelines or the preoperative evaluation o elderly surgical patients. Tese recommendations are required standards or training programsin which o the ollowingspecialties?
A. Generalsurgeryresidencytraining
B. Cardiothoracicsurgicalresidencytraining
C. Vascular surgicalresidencytraining
D. Allo theabove
Answer:D
Elderly trauma victims are more likely to have underlying organ-speci ic limitations and medication use which complicates resuscitation and hemostasis. hey are also more likely to be undertriaged and treated at non-level 1 trauma centers. (SeeSchwartz10th ed.,p.1935.)
Answer:D
Minimally invasive approaches to surgical treatment have dramatically decreased mortality and morbidity rates in elderly subjects or aortic aneurysm repair, cholecystectomy, and colorectal surgery. Laparoscopic techniques or endovascular methods have enabled the surgical correction o multipleabdominalconditionswith ar lessmorbidityin theelderly than open surgical techniques. Neck surgery is usually well tolerated byelderlypatients.(SeeSchwartz10th ed.,p.1936.)
Answer:C
Palliative surgery is directed at preserving quality o li e, avoiding devastating complications o disease, and relie o symptoms. It is not intended to alter the course o a disease processbut isconsidered when surgicaltreatment can extend ameaning ulexistence or thepatient.(SeeSchwartz10th ed., p.1938.)
Answer:D
he primacy o the patient and his/her decisions regarding their own care is o greatest importance in the ethical care o the elderly surgical patient. A surrogate decision maker may beappointed bythepatient,but that individualshould decide medical decisions based on what they believe would be the choiceo thepatient.(SeeSchwartz10th ed.,p.1930.)
Answer:D
he combined recommendations o American College o Surgeons National Surgical Quality Improvement Program (ACS-NSQIP) and the American Geriatrics Society (AGS) have been endorsed by all training programs under the aegis o the American College o Surgeons (ACS). (See Schwartz 10th ed.,p.1929.)
349
Ethics,Palliative Care,and Care at the End ofLife
1. Biomedical ethics is a system o analysis and deliberation which is intended to direct physicians and surgeons to moral“goodness”in patient care.It includesconsideration o allo the ollowingEXCEP
A. Autonomy: Te patient’s right to decide or himsel / hersel what carewillbeprovided.
B. Bene cence: Te concept that proposed treatments willbene t thepatient.
C. Nonmale cence: Te avoidance o treatments which mayharm thepatient.
D. Equipoise: Te lack o a pre erence or one treatment over another.
2. Livingwillsaredocumentswhich aremeant toguidedecision makingwhen
A. Tepatient’s amilycannot becontacted.
B. Te patient is rendered incompetent or unresponsive byan illnessjudged tobeterminal.
C. Multipleattemptsat resuscitation have ailed.
D. Te patient’s amily disagrees with the course o treatment.
3. “In ormed consent”impliesallo the ollowingEXCEP
A. Te patient has been provided with the pertinent detailso his/her diagnosis,prognosis,and theoptions or and riskso treatment.
B. Te in ormation has been provided accordingto what areasonableperson would beexpected tounderstand.
C. Te discussion o the options, risks, and possible hazardshasbeen documented.
D. Tere are witnesses to the discussion who also understand thediscussion.
Answer:D
he patient and the doctor decide together what treatment is in the best interest o the patient, and share the bene its and theburdenso thisjoint decision making. hephysician’srole is to clari ythe indications, risks, and bene its o the possible treatmentcourses;thepatient’sroleistodecidewhatcourseto take.(SeeSchwartz10th ed.,p.1941.)
Answer:B
Living wills are intended to guide decisions by physicians, amilymembers, and/or surrogate decision makers when the patient himsel /hersel isunableto render an opinion and the condition or disease is judged to be terminal or “hopeless.”
(SeeSchwartz10th ed.,p.1942.)
Answer:D
he “reasonable person” standard or in ormed consent has precedent in a 1972 court case which rejected the notion that “simple consent” or treatment was su icient. he court decided that the acts o diagnosis, treatment options, and risks that a “reasonable person” would want to know should beincluded in thediscussion to obtain consent or treatment. hereisno requirement or witnessdocumentation when the patientisbelievedtobecompetent tohandletheir own a airs.
(SeeSchwartz10th ed.,p.1942.)
351 48
CHAPTER
4. When discussing possible surgical options with a patient, it isappropriatetodoallo the ollowingEXCEP
A. Document that the patient is capable o rendering in ormed consent.
B. Askthepatient toidenti yasurrogatehealth caredecision maker in theevent he/sheisincapableo deciding treatment choices.
C. Avoid discussing the “pain and sufering” aspects o a treatment plan.
D. Providean opportunity or thepatient to askquestions or todeliberatewith othersi thecondition permits
Answer:C he obligation o the surgeon is to ensure that the patient is competent and understands the discussion o treatment options. I the condition is not an emergency, the patient should be allowed to consult with others or to ask or time to consider their response. All aspects o the expected course and treatment options, including symptoms, side-e ects and complications, should be discussed. (See Schwartz 10th ed., p.1944.)
352
1. Patients in rural and developing locales o en have dicultyaccessing health care and undergoing surgical interventions.Interest in globalsurgeryhasgrown signi cantly in recent yearsduein part toallo the ollowingEXCEPT
A. Trained surgeonsmigratingtoareaso need.
B. Improved control o acute in ectious diseases previouslythecauseo signi cant morbidity.
C. Technology allowing improved access to health care in ormation and training.
D. Recognition o the cost-efectiveness o surgery as a publichealth intervention.
CHAPTER
GlobalSurgery
Answer:A
Although much o the world remains with limited access to surgery, this is changing. Patients who have been denied surgicalcarepreviouslydueto concernsabout cost and accessare no longer being ignored. Over time, it has become apparent that thepatientsin need o surgicalcarearethosewhohavethe greatestsocioeconomicimpacton society(ie,young,otherwise healthyand productivememberso society).O eringapotentialcureviasurgeryallowsthepatienttoreturn tonormal unction and contribute to societyin a meaning ul way;improving the in rastructure o the country. Many trained physicians, especially surgeons, who planned to return to areas o need ind that the lack o resources and in rastructure hamper their opportunity or development.There ore, ullytrainedsurgeons tend to remain in larger, more developed areas. While this is seen across the globe, migration o practitioners to economically and culturally avorable locales impacts Low and Middle IncomeCountries(LMICs)more,astheyhaveahigher burden o diseasedpatientswith ewerhealthcareworkersandasteeper gradient rom developed to developing areas o their country. (SeeSchwartz10thed.,pp.1960–1962and1970–1976.)
2. A24-year-old man in Tanzaniaistravelingwithout helmet on the back o a motorbike sideswipes a large truck. He is broughtin tothehospitaland oundtohaverib ractures,a emur racture,and atraumaticbrain injury.I onewanted to calculate the impact o this injuryon his li e, one could calculate a Disability Adjusted Li e Year (DALY) score. What componentsmakeup thisscore?
A. Yearso li elived.
B. Severity o illness and years o li e lived with that disability.
C. Patient quality o li e and years o li e lived with impaired qualityo li e.
D. Years o li e lost, compared to country average, and a weighted calculation o li eyearslived with disability.
Answer:D
The Disability Adjusted Li e Year (DALY) score is a measure o overall disease burden expressed as the number o years lost due to ill-health, disability, or earlydeath. The DALYhas become commonly used in public health in dealing with the Health Impact Assessment. DALYs deal with potential li e yearslost dueto prematuredeath or disability.Thiscombines morbidityand mortalityinto asinglemetric.Health liabilities are typically measured in Years o Li e Lost (YLL). However, YLLdoes not take into account disability, o ten expressed as Years Lived with Disability (YLD). DALYs are calculated by taking the sum o these two components. A valuation o the “severity”o thedisabilitymust bemadetoaccuratelyaccount or thedegreeo patient disability.
DALY=YLL+YLD
The DALY re lects chronic illness and amount o time a person remains a ected. One DALY is equal to 1 year o healthy li e lost. DALYcan be applied to a multitude o conditionsincludingphysical,psychiatric,and neurologicconditions.(SeeSchwartz10th ed.,pp.1973–1974.)
353 49
3. A er you establish an advanced surgical department in a oreign country, a 55-year-old woman presents with right upper quadrant pain o 3 months occurring a er every meal. An ultrasound shows a thickened gallbladder wall and pericholecysticfuid and adiagnosiso cholecystitisis made. Te patient undergoes a laparoscopic cholecystectomy. Te World Health Organization’s (WHO) 10 basic and essential objectives or sa e surgical practice ocus on allo the ollowingEXCEP
A. Preparing or li e-threateninglosso airwayor respiratory unction.
B. Minimizingsurgical-sitein ection.
C. Minimizing cost and length o hospital stay or the patient.
D. Communicatingcriticalpatient in ormation.
Answer:C
Surgeonsstrivetoconstantlyimprovemorbidityandmortality o theirpatients.In resource-limitedareasaccesstocertainpre andpostopinterventionsor monitoringmaybelimitedandin order tominimizecomplications,theWorldHealthOrganization (WHO) enacted the Sa e Surgery Saves Lives Initiative. hiscampaign targeted preventableinjuriesand was ound to have a 50% decrease in mortality. he 10 objectives or sa e surgery include: identi ying the correct patient and site, prevention o harm and pain, preparing or li e-threatening airwaycompromise,preparing or high-riskblood loss,avoiding allergic reactions or reactions to drugs, minimizing surgicalsite in ections, preventing retained sponges, identi ying all surgicalspecimens,e ectivehand-o so patientin ormation, and routine patient surveillance ( able 49-1). (See Schwartz 10th ed., able49-2,p.1968.)
TABLE49-1 Ten basicand essentialobjectivesforsafe surgery(WHOa)
1.Operateonthecorrectpatientatthecorrectsite
2.Usemethodknowntopreventharmromanestheticadministration, whileprotectingthepatientrompain
3.Recognizeande ectivelyprepareorlie-threateninglossoairway orrespiratoryunction
4.Recognizeande ectivelyprepareorriskohighbloodloss
5.Avoidinducinganyallergicoradversedrugreactionknowntobea signifcantriskorthepatient
6.Consistentlyusemethodknowntominimizeriskosurgicalsite inection
7.Preventinadvertentretentionoinstrumentsorspongesinsurgical wounds
8.Secureandaccuratelyidentiyallsurgicalspecimens
9.Eectivelycommunicateandexchangecriticalpatientinormation orthesaeconductotheoperation
10.Establishroutinesurveillanceosurgicalcapacity,volume,andresults aData romWHOGuidelines orSa e Surgery2009.http://whqlibdoc.who.int/publications/ 2009/9789241598552_eng.pd .
4. A 50-year-old man presents to a clinic in Cameroon with abdominalpain.On urther questioning,you learn that he lives 60miles rom the nearest hospitaland hasnot seen a physician in years. Te patient is counseled regarding screening colonoscopy, which he undergoes and multiple polyps are noted. Te patient is re erred to or surgery. Factors which must be considered that impact surgical servicesinclude
A. Socioeconomic status o the patient and cultural background.
B. Accessibilityo acilitiestothepatient.
C. Availabilityand traininglevelo supportingspecialties (anesthesia,nursing,etc).
D. Allo theabove.
Answer:D
hree actors, among many others, that must be considered when planning surgical care in low-resource settings are listed above. It is important or surgeons to understand and acknowledge their patients circumstances and customs while considering the physical and human resources o their environment. Sa e and e ective surgical care in low-resource setting requires care ul attention to several ields traditionally outsideo surgicaltraining—suchaslogistics,business,public policy,engineering,and publichealth.(SeeSchwartz10th ed., pp.1974–1975.)
354
5. A 55-year-old East A rican woman presents with a palpable mass in her right breast that has been present or severalyears.You areconcerned or cancer and knowher case atalityrate issigni cantlyhigher than a similar patient in North America.Why?
A. Later presentation tocare.
B. Limited accesstoscreening.
C. Lacko accesstosurgicalcare.
D. Allo theabove.
6. When considering surgical care in LMICs, “task shi ing” iso en talkedabout asawaytoprovidesurgicalcarewhile additional surgeons are trained and recruited. What does thisterm re er to?
A. Shi ingcare rom ruralareastoestablished centers.
B. raining nonsurgeon physicians and midlevel practitionerstoprovidebasicsurgicalcare.
C. Establishinga system that allows selected surgical diseasetobetreated medically.
D. Shi ingthetaskso surgicalcare rom western medical sta tolocalsta .
7. You areplanningto workabroad but acolleaguementions that “surgery is too expensive” in LMICs and you should ocusonprovidingmosquitonettinginstead.Youdisagree, which o the ollowing answers could help you support your case.
A. Surgical treatment is o en primary prevention or additional disease (eg, Cesarean sections help prevent obstetrical stulae).
B. A number o studies demonstrate that surgical care is belowthe$100/DALYthresholdset bytheWorldBank or cost-e ectivecare.
C. District hospital level surgical service can be provide or $11-33/DALYaverted.
D. Allo theabove.
Answer:D
Patients in LMICs bear a substantial proportion o the global burden o cancer morbidity and mortality. In the year 2030, theproportion o newcancer cases rom LMICsisexpected to be at least 70%with nearlytwo-thirds o globalcancer deaths in LMICs. Cancer mortality is inversely associated with the strength o a country’s health system, which is re lected in patient access to screening and treatment as well as the ease with which patient can access medical care. (See Schwartz 10th ed.,p.1958.)
Answer:B askshi tinginvolvestrainingnonsurgeon physicians,nurses, midwives, and advanced care practitioners to provide basic surgical or anesthetic care. his may be an inevitable act o li e in LMICs and the surgeon shortage is prodigious. ask shi ting in this way may allow services that were only available under ully trained surgeons to be more accessible and mitigate the signi icant surgeon’s shortage in many LMICs. Concerns about quality o care, supervision, and the impact on surgeon’spro essionaldevelopmentremain areaso debate. (SeeSchwartz10th ed.,pp.1960–1962.)
Answer:D
A common question raised to proponents o surgical care in LMICsisoneo cost.Anumber o independentstudiesaswell aswork bythe World Bank and WHO showthat,while nothingis ree,the burden o surgicaldisease is so huge in LMICs andthesurgicalneedsohighthateven relativelycost-e ective interventionscan cost-e ectivelyreducetheburden o human disease. Compared to other public health initiates, developing basic and emergency surgical care at a district hospital level is as cost-e ective, or even more so, than typical health programssuch asHIV-AIDStreatmentor measlesimmunization!Surgeons truly are the ront line o global public health. (SeeSchwartz10th ed.,pp.1972–1974.)
355
A-Aindex,46
Abdominalaorticaneurysm (AAA),159
Abdominalsurgery,21
Abdominalwall
anterior,lymphaticdrainageo ,283
cutaneousmalignancyo ,283
desmoid tumorso ,284
ABI (ankle-brachialindex),159
ABO blood typing,79
Abscess(es)
in Crohn disease,232
hepatic,treatment o ,35
pyogenic,35
pyogenicliver,management o ,248
“Abundancementality”model,4
Acalculouscholecystitis,256
Accelerated partialbreast irradiation (APBI),119
Accessoryspleen,275
Achalasia,202t
Acquired FXIII de ciency,18
Acquired heart disease,143–149
angina pectoris,143
atrial brillation (AF),144
cardiacmyxomas,146
cardiactumor,145,146
Ehlers-Danlossyndrome,145
holosystolicmurmur,143,144t
le ventricle(LV) aneurysms,144
mitralstenosis(MS),144
pericarditis,148
tricuspid stenosis,145
Acquired mitralstenosis(MS),144
Acquired platelet hemostaticde ect,18
Acquired tricuspid valve( V),145
ACS-NSQIP.seeAmerican Collegeo SurgeonsNationalSurgical QualityImprovement Program (ACS-NSQIP)
AC H.seeAdrenocorticotropin hormone(AC H)
Actinomycetesin ection,108
Actinomycosis,108
Activated clottingtime(AC ),148
Activated protein C,47
Acuteaorticdissection,154,155
descending,management o ,156
Acuteappendicitis,in elderly,347
Acutecholecystitis,255
Acutecoagulopathyo trauma (ACO ),47
Acutedescendingaorticdissection, management o ,156
Acuteembolicmesentericischemia,161
Acutehypercalcemia,14
Acuteliver ailure(ALF) causeso ,245 de ned,245
Acutemyeloid leukemia (AML),279
Acutemyocardial ailure,intra-aortic balloon pumpingin,30
Acutenormovolemichemodilution (ANH) anemia,343
Acutepancreatitis,222
Acuteperitonitis,in elderly,347
Acutephaseproteins,5
Acuterenal ailure,14
Acuteurinaryretention,316
Acutevaricealhemorrhage,246–247
Adenocarcinoma,126 apocrinegland,113 appendix,240 bileduct,257 diagnosiso ,127 duodenum,222 esophagus,197 predictive/prognostictumor marker or,127 rectal,228
Adenoid cysticcarcinomas,125–126
Adenomatouspolyps,234
Adenosinediphosphate(ADP),17
ADH.seeAntidiuretichormone (ADH);Atypicalductal hyperplasia(ADH)
Adjustablegastricband procedure, complicationso ,215
Adrenalectomy,laparoscopic,304
Adrenalincidentaloma,304
Adrenalinsu ciency,6
Adrenalpheochromocytomas,303
Adrenocorticotropichormone (AC H)-secretingadenoma, 303,304
Adrenocorticotropin hormone (AC H), unctions,6
Adults,ASD in,141
Age/aging
inguinalherniasprevalenceand, 289,289t
physiologicaging,347
Age-related calci cAS, 149
AGS.seeAmerican GeriatricsSociety (AGS)
Air embolism,42
Airwaypressures,90
AJCC (American Joint Committeeon Cancer),112
Albumin,245,344
uid resuscitation using,10 serum calcium levelsand,11 Aldosteronism,303
ALF.seeAcuteliver ailure(ALF)
ALH.seeAtypicallobular hyperplasia (ALH)
Alkalization,urolithiasisand,318 Alkalosis vs.acidosis,13 changein potassium and,11 Allergicreactions,21 Allergy(ies) to antimicrobialagents,36 penicillin,36
Alpha-adrenergicneurotransmitters,191
Alpha blockers,191,317
α-Fetoprotein (AFP),131
Alvimopan,83,221
Amaurosis ugax,162
Amenorrhea,327–328,328
American Collegeo Chest Physicians recommended duration o antithrombotictherapy or DV ,184,184t
American Collegeo SurgeonsNational SurgicalQualityImprovement Program (ACS-NSQIP),349
American GeriatricsSociety(AGS),349
American Joint Committeeon Cancer (AJCC),112,287
American Societyo Anesthesiologists classi cation system,344–345
American UrologicalAssociation,316
Amino acid,6
Amputated digit or limb,340
Amsterdam criteria,235
Analsphincter,225
Anaplasticastrocytoma,325
Anastomoticpseudoaneurysms,152
Anesthesia,343–345 classi cation system,344–345 preoperativephysicalexamination, 345,345t regional,344
Anestheticagents,withepinephrine,335
INDEX A
Note:Pagenumbers ollowed byt indicatetables;those ollowed by indicate gures.
Aneurysms.seealsospeci ctypes aortic,151 alse,151
usi orm,151
pseudoaneurysms,151,152 saccular,151 thoracic,151–157 true,151
Angina pectoris,143
Angioembolization,51
Angiography,172
Angiosarcomas,149
Anion gap (AG)
metabolicacidosiswith,9,10 normal,12
Ankle-brachialindex(ABI),159
Anterior abdominalwall,lymphatic drainageo ,283
Anthrax,inhalational,37
Antibiotictherapy or incisionalSSIs,39 or OPSI,282
Anticancer chemotherapyagents,71
Anticoagulation
cardiopulmonarybypass(CPB),148 laboratorytest to determinedegree o ,19 prolonged,in elderly,348
Antidiuretichormone(ADH) production,in injured patients,27 sodium depletion and,9
Antihemophilic actor ( actor VIII),22
Anti–HER-2/neu therapy,104
Anti-in ammatorycytokines,28
Antimetabolites,71
Antimicrobialagents,allergyto,36
Antire uxmechanism,human,190
Antire uxsurgery,193 candidates or,193–194 indications or,194 outcomes,196
preoperativetesting,194–195 valvecreated during,195
Antithrombotictherapy, or DV , recommended duration o , 184,184t
Aorticaneurysms,151 mortalityrates,155–156
Aorticarch aneurysm,mortalityrates, 155–156
Aorticdissection,154,156
anatomiccomplications,156,157t
ascending,157
descending,management o ,156 management o ,155 symptoms,156
tearing,156
Aorticinsu ciency(AI),145
age-related calci cASand,149 angiosarcomas,149 causeo isolated,148
destination therapy,149
Aorticvalvereplacement (AVR),145
Aorticvalvestenosis,135
Aortobi emoralbypass,161
Aortobi emoralbypassgra ing, postoperativecomplications, 172,172t
Aortoiliacdisease,169,169
Aortoiliacocclusivelesions, classi cation o ,169,170 ,170t APBI.seeAccelerated partialbreast irradiation (APBI)
APCgene,233
Apocrineglands,adenocarcinoma o , 113
Apoptosispathways,99,99 Appendectomy,237 indications,238 wound,37 Appendicealcarcinoid,238 Appendicitis,237 acute,in elderly,347 Alvarado score,239,240t imagingo ,237 management o ,238 pregnancyand,238 signsand symptoms,239 Appendix,237–240 adenocarcinoma o ,240 lymphoma o ,240 removalo ,239 Argininevasopressin,27 Aromataseinhibitors(AIs),117 Arrhythmia,144 duringlaparoscopy,93 Arterialdisease,159–178
Arterialinjuries,46,46t Arterialswitch operation,137 Arteritis
giant cell,176–177 thyroid gland,299 Arteryanatomicvariants,hepatic, 242,243
Arthritis in in ammatoryboweldisease,230 surgicalmanagement,333
Ascendingaorticdissection,157
Ascendingthoracicaneurysms,153 ASD.seeAtrialseptalde ect (ASD)
Aspergillus,130–131
Aspergillusfavus,130
Aspergillus umigatus,130
Aspergillusniger,131
Aspergillusterreus,131
Aspiration pneumonia,causeso ,129
Aspirin,206
Asthma,re uxassociated,193 Astrocytomas,325,325 ,328
AS RO guidelines,119
AsymptomaticASDs,135
Asymptomatichypocalcemia,12 Atheroscleroticdisease,168
Atrialarrhythmias,141
Atrial brillation (AF),144,174
Atrialseptalde ect (ASD),135 in adults,141
asymptomatic,135
Atypicalductalhyperplasia(ADH),117
Atypicallobular hyperplasia(ALH),117
Autoimmunehemolyticanemias (AIHA),277
Automobileaccident,treatmenta er,50
Autonomy,351
Axonotmesis,64
Azathioprine(AZA),73
Azathioprine,sideefectsand drug interactions,74t
Azul dine(sul asalazine),230
B
B- shunt,140 Backpain,332
Bacterialproli eration,in burns,60 Bacterialsepsis,thrombocytopenia and,22
Bankart lesion,331 Bankblood,23
Bariatricprocedures,215–217.seealso speci ctypes
nutritionalde cienciesa er,216
Baroreceptors,25–26
Barrett esophagus(BE),192 de ned,193 hallmarko ,193
Basalcellcarcinoma (BCC),109,110 diagnosiso ,113 super cial,113
Baxter ormula,57
BCC.seeBasalcellcarcinoma (BCC) Beck’striad,42
“Bell-clapper”de ormity,317
Bene cence,351
Benign prostatichyperplasia (BPH), 316,317
Betablockers,191
β-lactam,109
BicarbonateLoss,9
BidirectionalGlenn procedure,136
Bilateraladrenalectomy,304
Bile,254
cholesterolsolubilityin,254–255
Bileacids,220
Bileduct
biliarystricture,259 blood supply,253
injuries,259
postoperativepresentation,259 resectablelesions,258
Bileduct adenocarcinomas,257
Bileleak,259
Bilesalts,244
Biliaryatresia,309
Biliarytract disease,347
Biliopancreaticdiversion,215
Biliousemesis,in in ants,306
358
Biloma,a er cholecystectomy,83
Biomedicalethics,351
Biopsy(ies)
approachesto,71 coreneedlebiopsy,71 excisional,133
incisional,71,133 lymph node,complications,82 needle,133
sentinellymph nodebiopsy,119
Bismuth-Corletteclassi cation system, 258
Bladder calculi,BPH,317
Bladder cancer,315
Bladder injuries,53
Bladder muscle,diseaseso ,315
Bleeding dueto massivetrans usion,21–22 surgical,17–23
varicealbleeding,246–247
Bleedingpepticulcer,206,207
Blind rectalpouch,in in ants,313
Blood supply
bileduct,253
breast,116
cervicalesophagus,189 ap,341
to omentum,284
Blood ureanitrogen (BUN),11
elevation o ,14
Blunt abdominaltrauma,45,45t
Blunt carotid injuries,47
Boari ap,316
Bodymassindex(BMI),215
Bodyweight,water percentageo ,10–11
Bowelin arction,223
BPH.seeBenign prostatichyperplasia (BPH)
Brain.seealsoNeurosurgery
malignant tumor o ,325,325
Brain damage,electricalinjuryand,56
Branchialcle cysts,122
Branchialcle stula,311
Breast,115–119
anatomy,115
blood supply,116
changesin pregnancy,115
development be oreand during
pregnancy,116
gynecomastia,116
in ammatoryconditions,117
lesionswith malignant potential,117
Breast cancer
APBI,119
AS RO guidelines,119
breast LCIS,118
DCIS,118
diagnosiso ,118
drugsused in,117 in elderly,348
metastasis,118
duringpregnancy,119
risk actors,70,70t–71t,117 sentinellymph nodebiopsy,119 staging,118
Breast LCIS,118
Brisbane2000hepaticresection terminology,251,251t Buck ascia,315 Burns,55–60 bacterialproli eration in,60 electricalinjury,56–57 excision,59 ormula,57 ourth degreeburns,57 ullthickness(third-degree),57–58,60 hydro uoricacid skin burn,107 hydrogen cyanidetoxicityand,56 hypermetabolicresponsein,58 management,55 nutritionalrequirementso patients with,58 partialthickness,57,60 size,55 steam burn,59 super cial,57 surgicaltreatment,59 treatment,58 wounds,57
C-reactiveprotein (CRP),5 asmarker o injuryresponse,5 Calcineurin inhibitors,80 druginteractionsandsideefects,80t Calcium,totalserum calcium levels,11 Calcium chloride,11 Calcium gluconate,11 Calcium in usion,11 Calcium oxalate,318 Cancer(s).seealso umor(s);speci c types anticancer chemotherapyagents,71 biopsy,approachesto,71 breastcancer,risk actors,70,70t–71t cellcycle,phaseso ,69 chemoprevention,72 elderlypatients,348 hallmarks,69 head and neck,incidenceo ,121 hereditary,70 incidencerates,69 morbidityand mortality,in LMICs, 355
routinecheckup,71 skin,109–113 tumorigenictrans ormation o cells,69 UVexposureand,110 Cantlie’sline,241 Capacitivecoupling,93,94 Capriniscore,182–183 Carbohydrate,digestion in small intestine,220 Carbonicacid,13
Cardiaccatheterization,139
Cardiacdamage,electricalinjuryand,56
Cardiacmurmurs,143,144t
Cardiacmyxomas,146
Cardiacoutput
Fickequation or,89 noninvasivemonitoringmethods,89 thermodilution technique,88–89
Cardiactamponade,43 clinicalsignso ,30
Cardiactransplantation acuterejection,monitoring,78 diagnosisleadingto,79 donorsand recipientsmatching,76 orthotopic,76 patient evaluation or,77
Cardiactumor,145,146
Cardiogenicshock,25 causeo ,in trauma patient,42 characteristics,30 de ned,30 mortalityrates or,30
Cardiopulmonarybypass(CPB) anticoagulation,148 coagulation during,20
Cardiopulmonaryresuscitation (CPR),43
Carotid,blunt injuryto,47
Carotid bi urcation occlusivedisease, 170,171
Carotid bodytumors,163,164
Carotid coiling,167,167
Carotid endarterectomy,81
Carotid stenosis
ipsilateralstrokein,163 risk actors,164
Cartilage,healingo ,65
Cataracts,high-voltageelectricalburns and,56–57
Catecholamines,29
Catecholamine-secretingtumor,303
Cavotomy,316
2009CDC NHSN report,60
CDKinhibitors,104
Celecoxib,72
Celiotomy,46
Cellculture,102–103,103
Cellcycle,101,102 and itscontrolsystem,101,102 regulator o ,104
Celldeath.seealsoNecrosis
HMGB1releasea er,6
Cell-sur acereceptors,100
Cellular apoptosis,99,99
Cellulitis,337,338
nonpurulent,complicated,treatment o ,109
Centralnervoussystem (CNS) neoplasm,325
Centralvenouspressure(CVP),42,88
Cerebralper usion pressure(CPP),49,91
Cerebrospinal uid (CSF) leakage,324
Cervarix,319
359
C
Cervicalcancer screening,319
Cervicaldischerniation,326,327 ,327t
Cervicalintraepithelialneoplasia (CIN) II/III,319
Cervicalradiculopathy,326
Cesarean delivery,320
Chemodectoma,163
Chemoprevention,cancer,72
Chemoreceptors,26
Chemotherapy,315 anticancer agents,71
Chernobyldisaster in 1986,301
Chest walltumor/mass,129 excisionalbiopsyo ,133 laboratoryevaluation o ,129 percentageso malignancy,130
Child- urcotte-Pugh (C P),247 classi cation,247–248
Children.seealsoIn ants;Pediatric surgery causeo death,309 with congenitalheart disease,139 high pulmonaryblood ow,139 injuryin,309
intraduralextramedullarytumors,328 papillarythyroid cancer in,301 thyroidectomyin,301 undescended testiclesin,314 vomitingin,312 VSD in,139
Chloride,58
Cholangiocarcinoma,249–250 imagingo ,258 perihilar,258
Cholangitis,255
Cholecystectomy bilomaa er,83 candidates or,256 gallbladder cancer,250
Cholecystitis
acute,255 in diabeticpatient,14
Choledochalcysts,253,258,313
Choledochoceles,313
Cholelithiasis,257
Cholestasis,86
Cholesterol,solubilityin bile,254–255
Chondromas,129
Chroniccalci cpancreatitis,263,264t
ChronichepatitisC in ection,245
Chronicin ammatorydisease,113
Chroniclimb ischemia(CLI),175,175t
Chronicmyelogenousleukemia (CML),278
Chronicpancreatitis,261,264t long-term resolution rom pain,269 pain rom,267 pain relie rom,a er Frey procedure,267 therapyto prevent,268,268t
Chroniculcers,malignant trans ormation o ,65
Chronicvenousinsu ciency(CVI), 179–180,180 Chvosteksign,12 Chylothorax,133 Chylousascites,222 Circleo death,293,294 Circulatoryarrest,155 Cirrhosis,245,249 physiologicchangesin,246 portalhypertension,280 sclerosingcholangitisand,256 Clean/contaminated wounds,34 Clean wounds,34 Cle lip/palate,341 CLI.seeChroniclimb ischemia (CLI) Closed-loop obstruction,220 Clotting actors,17 CO2 pneumoperitoneum,94–95 gas-speci cefects,94 pressure-speci cefects,94,95 COA.seeCoarctation o theaorta(COA) Coagulation duringCPB,20 a er tissueinjury,22 Coagulopathy,trauma-related,18–19 Coarctation o theaorta(COA),136 recurrent,136 CochraneReview,184 Coercivestyle,o leadership,3 Colectomy,225 a er diverticulitis,232 Collagen
inguinalhernias,289 or wound healing,65 Collagen disorders,289 Colloid resuscitation,26 Colloids,344 Colloid solutions,26 Colocutaneous stulas,in diverticulitis, 233 Colon,225 Colon cancer amilyhistoryand,235 geneticmutationsin,233–234 treatment objectives,235 Colonoscopy,227,234 Colon resection laparoscopicsurgery,96 Colorectalcarcinoma,233 risko ,234 Colostomy,52 Colostrum,115
Colovesical stulas,in diverticulitis,233 Comatose/obtunded patients,54 Combined restrictiveand malabsorptiveprocedures,215 Command-and-controltechnique con ict resolution and,4 leadership and,3 Comminuted,longbone ractures,331 Communication efective,3 leadership and,3
Communication errors,causeso ,3 Comorbidity,347 Compartment syndrome,331
Complicated U Is,83
Computed tomographic(C ) scanning o thoracicaneurysms,154 Computed tomography(C )-guided aspiration in necrotizingpancreatitis,33 Computer-enhanced laparoscopic surgery,95
Concomitant cardiovascular disease (CVD),77
Con ict resolution techniques,4 “abundancementality”model,4 command-and-controltechnique,4 ault- ndingmodel,4 objectivityand,4 willingnessand,4
Congenitaldiaphragmatichernia (CDH),in in ants,305,305 Congenital actor XIII (FXIII) de ciency,18
Congenitalheart disease,135–141 aorticvalvestenosis,135 atrialseptalde ect,135 children with,139 coarctation o theaorta,136 cor triatriatum,138 in elderly,347 high pulmonaryblood ow,139 hypoplasticle heart syndrome,137 in antswith,139 patent ductusarteriosus,141 tricuspid atresia,136 truncusarteriosus,141 ventricular septalde ect,138–140
Congenitalhernias,295
Congenitallobar emphysema,312 Congenitallymphedema,186 Congestiveheart ailure,hypocalcemia and,10
Connectivetissuedisorders,with groin herniation,290t Conn syndrome,303
Contaminated wounds,34
Continent cutaneousdiversion,315
Continent urinarydiversions,315
Contrast angiography,172
Contrast-enhanced C scan,o hepatic lesions,249
Coreneedlebiopsy,71
Coronaryarterybypass,146 Coronoid racture,334 Corpora cavernosum,317
Corticosteroids or in ammatoryboweldisease, 230–231 or septicshock,31 sideefectsanddruginteractions,74t
Corticotropin-releasinghormone (CRH),6
360
Cortisol,6
Cor triatriatum,138
Coudecatheter,316
CPP.seeCerebralper usion pressure (CPP)
Cranialnerves,in swallowing mechanism,190
Craw ord classi cation scheme,156
CRH.seeCorticotropin-releasing hormone(CRH)
Cricothyroidotomy,41,41 disadvantageo ,41
Crohn disease,221,229,230 geneticde ect associated with,221 indications or surgery or,231 perianallesionsin,232 resection,221 risk actors,221 therapy or,230–231
Cross-clamping,316
CRP.seeC-reactiveprotein (CRP)
Cryopreserved gra s,176
C P.seeChild- urcotte-Pugh (C P)
Cubitaltunnelsyndrome,336
Cushingdisease,303,304,327
Cutaneouscysts,109
Cutaneousmalignancy,o abdominal wall,283
CVI (chronicvenousinsu ciency), 179–180,180
Cyclin-CDKcomplex,104
Cyclin-dependentkinase(CDK),98,104
Cyclosporine(CSA)
gra rejection prevention and,77 metabolism o ,80 sideefectsanddruginteractions,74t
Cystectomy,315
Cysticartery,anomalieso ,253
Cystichygroma,312
Cysticmedialdegeneration,152
Cysticmedialnecrosis,152
Cysts,hepatic,248–249
Cytokines anti-in ammatory,28 in ammatory,HMGB1released by,6 in SIRS,85
Dabigatran,19
Diagnosticperitoneallavage(DPL),45
Dailymaintenance uidsintake,12
DALY(DisabilityAdjusted Li eYear) score,353
Damage-associated molecular patterns (DAMPs),25
best-characterized,6
mitochondrial,in injury-mediated in ammatoryresponse,6
Damagecontrolsurgery(DCS),49
DAMPs.seeDamage-associated molecular patterns(DAMPs)
Darkskin patient,melanomain,111
Dartos ascia,315
DCIS.seeDistalcarcinoma in situ (DCIS)
DCS(damagecontrolsurgery),49
DDAVP,20
DE.seeDifuseesophagealspasm (DES)
Declaration o cardiacdeath (DCD),80
Deep vein thrombosis(DV ),82,181 antithrombotictherapy or, recommended duration o , 184,184t
De ecation,227 disorderso ,321
Dementia,P H levelsand,302 Democraticleadership,3
Denonvilliers ascia,226
Densegranulede ciency,20 Dermoid cysts,109
Descendingaorticdissection, management o ,156
Descendingthoracicaorticinjuries,50
Desmoid tumors,127,288 o abdominalwall,284
Destination therapy,149
Detubularization,315
Diabetesmellitus cholecystitiswith,14 a er gastricsleeveprocedureand RYGB,216 renal ailureand,77 tight glucosemanagement in,29 wound healingin,64
Diagnosticperitoneallavage,positive ndingon,45t
Diaphragmatichernia,196
Diarrhea,metabolicacidosiswith normalAGin,9
Diastasisrecti,283
Difuseesophagealspasm (DES), 201,202t
Dilution (excess reewater), hyponatremiadueto,9
Direct thermalinjury,55 Direct thrombin inhibitors(D Is),186 Dirtywounds,34
Disability,347
DisabilityAdjusted Li eYear (DALY) score,353
Disseminated intravascular coagulation (DIC),18
Distalaorticaneurysms,151
Distalcarcinoma in situ (DCIS),118
Distalemboli,sourceo ,174
Distalileum,315
Diuresis,BPH,317
Diversion colitis,227
Diverticulitis,227,232 colectomya er,232 colocutaneous stulas,233 colovesical stulas,233 Hincheystagingsystem,233
Donabedian model,81,81
“Doublebubble”sign,310,310
“Drainpipe”urethra,321
Drug-induced I P,19
D Is(direct thrombin inhibitors),186
Dumping,a er gastrectomy,213
Duodenalhematomas,48,52
Duodenalobstruction,in in ants, 310,310
Duodenalswitch,215 gastricsleeveprocedure,216
Duodenum
adenocarcinoma o ,222 gastrinoma in,265
DV .seeDeep vein thrombosis(DV )
Dyspnea,147
E2F,98
Earlyhigh-pitched murmur,144t
Earlysystolicmurmur,144t
ECG.seeElectrocardiogram (ECG)
ED.seeErectiledys unction (ED)
EDS.seeEhlers-Danlossyndrome(EDS)
Ehlers-Danlossyndrome(EDS), 62,145,152,289
Eisenmenger syndrome,139 Elbowdislocation,332
Elderly,347–349 acuteappendicitis,347 acuteperitonitis,347 breast cancer in,348 cancer treatment in,348 ethicaldecision makingin careo ,349 lungcancer,348 minimallyinvasiveapproaches,349 nutritionalstatus,348 palliativesurgeryin,349 poor nutritionalstatus,348 postoperativemorbidityand mortalityin,347 prolonged anticoagulation,348 recommendationso ACS-NSQIP and AGS,349 surgicalconsiderations,347–349 surgicalintervention,indication or, 347 trauma in,54,349
Electricalinjury,56
Electrocardiogram (ECG) changeswith hyperkalemia,9 changeswith hypocalcemia,10
Electrocardiographic(ECG) monitoring,in ICU,88
Electrolytemanagement,o surgical patient,9–15
Electromyography(EMG),323
Electrovaporization,317
Embolicmesentericischemia,161
Empyema,postoperative,86
Enchondromas,339,340
End-diastolicpressure(EDP),88
End-diastolicvolume(EDV),88
361
D
E
End ileostomy,228
End o li e,careat,351–352
End-systolicpressure-volumeline,88
End-tidalCO2 (PetCO2),91
Endoleaks,153,153t
Endometrialcancer,321
Endometriosis,320
Endoscopicretrograde cholangiopancreatography (ERCP),51,264,264 ,265
Endoscopicretrograde pancreatography(ERCP),52
Endotoxin-induced thrombocytopenia, 22
Enterocutaneous stula,221
Enterocytes,cellular turnover rateo , 219
Enterohepaticcirculation,220
Enzyme-linked receptors,100
EOC. see Epithelial ovarian cancer (EOC)
Eosinophilicgranulomas,132
Epidermalcysts,109
Epiduralanalgesia,221
Epinephrine,27 anestheticagentswith,335
Epithelial ovarian cancer (EOC), 320–322
Equipoise,351
ERCP (endoscopicretrograde cholangiopancreatography), 264,264 ,265
ERCP (endoscopicretrograde pancreatography),52
Erectiledys unction (ED),316
Erection,315
Erythromycin,83
efect on gastricemptying,204,204t
Esophagealatresia(EA),307,307
primaryEA(typeA),311
Esophagealcancer
adenocarcinoma,197
dysphagia and,199
leiomyoma,200
neoadjuvant chemoradiotherapy or, 199
radiation therapy,199
salvageesophagectomy or,198
squamouscellcarcinomaso ,197–198 stageo ,198
in United States,197
Esophagealhiatalhernia,196
Esophagectomy,198
candidate or,198–199
salvageesophagectomyand,199
Esophagus
Barrett esophagus(BE),192–193
cervicalportion o ,blood supplyto, 189
injuries,43,192,200
motilitydisorders,202t
mucosaldamage,causeso ,192
narrowingo ,189 nonperistalticcontractionso ,201 Estrogen,116
Ethicaldecision making,in careo elderly,349 Ethics,351–352 biomedical,351 Eukaryotes,transcription mechanisms in,97–98
Eukaryoticgeneexpression,controlo , 97,97 Ewingsarcomas,128,129 Excess reewater (dilution), hyponatremiadueto,9 Excision,59,66 Excisionalbiopsy,133 Exons,103–104
Extracellular uid volumede cit, symptomsand signs,13 Extracolonicdisease,in in ammatory boweldisease,230 Extracorporealmembraneoxygenation (ECMO),80
Extremities, vascular injuries o , 46, 46t
Extrinsicpathways,clotting actors,17 Extubation,82
F
Fine-needleaspiration biopsy(FNAB), 300,301
Fissures,in Crohn disease,232
5-Acetylsalicylicacid (5-ASA),230
Flaps,341
Flexor digitorum pro undus(FDP) tendons,336
Flexor tenosynovitis(F S),337
Fluid management,o surgicalpatient, 9–15
Fluid replacement,316
Fluid resuscitation,usingalbumin,10
FNA.seeFine-needleaspiration (FNA)
FOB ( ecaloccult blood test),227
Focalnodular hyperplasia (FNH),249
Focused abdominalsonography or trauma (FAS ) examination, 43,45
Follicle-stimulatinghormone(FSH),116
Follicular neoplasms,o thyroid gland, 300
Fontaineclassi cation system,172,173t
Forbes-Albright syndrome,327
Formylpeptides,6
Fourth-degreeburns,57
Fracturereduction,331
Fractures,longbone,331
Frailty,347
Failureo xation,smallintestine,285
Falseaneurysms,151
Falsediverticula,232
Familialadenomatouspolyposis(FAP), 70,222,284
FAS examination.seeFocused abdominalsonography or trauma(FAS ) examination
Fat content,pulmonary ailure ormulas,7
Fatigue,147
Fault- ndingmodel,4 FBN-1gene,62
FDA.seeU.S.Food and Drug Administration (FDA)
Fecaloccult blood test (FOB ),227 Feltysyndrome,280
Female dailymaintenance uidsintake,12 malignantpleuralefusions,131,132t
P H levelin dementia,302 totalbodywater ( BW),11
Femaleurinaryincontinence,321
Femoralhernias,inguinalherniasratio to,295
Fetus,cardiacabnormalities,138
Fibrinolysis,47
Fibroblasticphase,wound healing,62 Fibromuscular dysplasia (FMD), 164–165, 165
Fibrosis,180,180
Fickequation, or cardiacoutput,89
Fine-needleaspiration (FNA),71,127
Freyprocedure,267 pain relie rom chronicpancreatitis a er,267,267t
Frozen plasma,22
Fullthickness(third-degree) burns, 57–58,60
Fulminant colitis, in Crohn disease, 232
Fundoplication,195
Fusi orm aneurysms,151
G
Gadolinium,160,163
Galactorrhea,327–328,328
Gallbladder
acutein ammation o ,256 lesionso ,treatment or,257
lymphaticsdrainage,257
lymphaticsdrains,257
polypoid lesionso ,257
Gallbladder cancer,250,257 typeso ,257
Gallstones,256
Ganglion cyst,337
Gardasil,319
Gas-speci cefects,CO2 pneumoperitoneum,94
Gastrectomy,dumpinga er,213
Gastriccancer
MAL lymphoma,210,211 risko ,208,208t surgicalresection,209 typeIII gastriccarcinoids,211–212
Gastriccarcinoma surgery,15
362
Gastricemptying,erythromycin efect on,204,204t
Gastricparietalcells,204
Gastricsleeveprocedure,216 resolution o ,217
type2diabetesmellitusa er,216
Gastrin,191
Gastrin-stimulated acid secretion,203
Gastrinomas
in duodenum,265
hyperparathyroidism and,302 preoperativeimaging,205 survivalrates,273
Gastroesophagealjunction (GEJ), 195,200
Gastroesophagealre ux,190–191
Gastroesophagealre uxdisease (GERD),190,193–194,217
Gastrointestinalstromaltumors (GIS s),209,210 ,222
Gastrointestinal(GI) system
healingo ull-thicknessinjuries,63
hyperkalemia symptoms,9
Gastroschisis,310,311
GCSscore.seeGlasgowComa Scale (GCS) score
Gelatin,344
Generegulation,103–104
Genitourinaryinjury,53
Genome,genepresent in,98
GERD.seeGastroesophagealre ux disease(GERD)
Geriatricpatients.seeElderly
Germ celltumor,131,131t
Giant cellarteritis,176–177
GlasgowComa Scale(GCS) score, 43,44t,91
head injuryscore,324
trauma patient,326,326t
Glenn procedure,bidirectional,136
Glioblastomamulti orme(GBM),325
Glioma,325
Globalsurgery,353–355
Glomustumor,163
Glucocorticoids,6
Glucosemanagement,in criticallyill patient,29
Glucose-6-phosphatedehydrogenase (G6PD) de ciency,276
Glucosetransporter 2(GLU 2),220
Glucosetransporter 5(GLU 5),220
Glutamine,6
Goleman,Daniel,3
Gonadotropin-releasinghormone (GnRH),116
Gra patency,143
Gra rejection,prevention o , cyclosporineand,77
Gram-negativebacteria,129
Graves,Robert,300
Gravesdisease,300
Grayplatelet syndrome,20
Groin pseudoaneurysm, a ter angiography, 165–166
Gunshot wound,51 multipleintra-abdominalinjuries a er,51 treatment or,52–53
Gynecology,319–322
Gynecomastia,116
H-type stula,307 Hair removal,33
Hairycellleukemia(HCL),278
Hamartomas,benign calci cation in,126
Hand and wrist
amputated digit or limb,340 enchondromas,339,340 exor tenosynovitis(F S),337 in ections,surgicalmanagement, 337,338 necrotizingin ections,336–337 nondisplaced ractures,335 scaphoid bone,335 squamouscellcarcinomas(SCC), 336–337,338,339 tumor o wrist,337 vascular injuriesto,335 Wartenbergsign,336
Hands,spasmsin,hypocalcemiaand,12 Harris-Benedict equation,58 Hartmann procedure,228
Harvard BusinessReview,3 hCG(human chorionicgonadotropin), 131
HCV.seeHepatitisC virus(HCV)
Head and neck
branchialcle cysts,122 cancers,incidenceo ,121 disorderso ,121–123 oraltonguecarcinoma,122
Head injuries
GCSscore,324 hypertonicsalineusein,10 Healing.seealsoWound healing o cartilage,65 Health Impact Assessment,353
Helicobacter pyloriin ection,203–204
Heller myotomy,95
Hemangiomas,248–249
Hematoma,duodenalhematomas,48,52
Hematuria,316
Hemicolectomy,228,229
Hemodialysis,14
Hemolytictrans usion reaction,22
Hemoperitoneum,massive,43
Hemorrhage
larger arteriolesvasoconstrict in,26 pelvictrauma and,332 pituitarytumor,328 rectussheath,283
Hemorrhagicdisorder,22
Hemorrhagicshock,14–15
intravenouscolloidsolution usein,26
Hemostasis,17–23
congenital actor XIII (FXIII) de ciencyand,18 physiologiceventsin,17
Hemothorax,massive,43
Heparin,replacement in emergency surgery,19
Heparin-induced thrombocytopenia (HI ),18,84,185
Hepaticabscesses pyogenic,35 treatment,35
Hepaticartery anomalieso ,253 replaced right hepaticartery,261
Hepaticveins,244,244
HepatitisC,245 blood trans usion and risko ,21
HepatitisC virus(HCV),37
Hepatoportoenterostomy,309
HER-2/neu,104
Hereditarycancer,70
Hereditaryhemochromatosis,245
Hereditarynonpolyposiscolorectal cancer (HNPCC),235
Hereditaryspherocytosis(HS),276
Hermansky-Pudlaksyndrome,20
Hernia(s)
cervicaldischerniation,326,327,327t circleo death,293,294
congenital,295
congenitaldiaphragmatichernia,in in ants,305,305 diaphragmatic,196
esophagealhiatal,196 emoral,295
hiatal,194–195,196
incidenceo recurrence,297–298 incisional,284
inguinal,289–298
nociceptivepain,293
Nyhusclassi cation system,290,290t postincisional,284 recurrence,292–293 repair method,297–298 saccontents,291
sliding,298
spigelian,284
APP repair vs. EP repair,295
triangleo doom,293,294
triangleo pain,293,294
umbilical,284
urinaryretention a ter repair o , 294–295
Hiatalhernias,194–195,196
Hidradenitissuppurativa,113,117
High-mobilitygroup protein B1 (HMGB1),6
proin ammatorybiological responses,6
High-voltageinjuries,56
363
H
Hilar cholangiocarcinoma,249–250
Hill-Sachslesion,331
Hincheystagingsystem,233
Hip racture,332
Hirschsprungdisease,306
HI .seeHeparin-induced thrombocytopenia (HI )
HIV-in ected blood,HIVtransmission rom needlestickwith,36
HIVtransmission blood trans usion and,21 rom needlestick,36
HLAtyping,79
HMGB1.seeHigh-mobilitygroup protein B1(HMGB1)
Hollenhorst plaque,162–163
Holosystolic(pansystolic) murmer, 143,144t
Hormone-producingcells,219
Hormones
LEStoneand,191 peptidehormones,220 secretin-glucagon amilyo ,220 thyroid hormones( 3and 4),300
Horner syndrome,128
Host de enses,trans errin rolein,33
HPAaxis.seeHypothalamic-pituitaryadrenal(HPA) axis
HPV.seeHuman papillomavirus(HPV)
HPVL1protein,319
Human chorionicgonadotropin (hCG),131
Human error,trans usion reaction and, 22
Human genome,genepresent in,98
Human immunode ciencyvirus (HIV),36.seealsoHIV transmission
postexposureprophylaxis or,38
Human leukocyteantigen (HLA),73 unction o ,77
Human papillomavirus(HPV),121
Human papillomavirus(HPV) vaccination,319
Hydration,urolithiasisand,318
Hydro uoricacid skin burn,107
Hydrogen cyanidetoxicity,56–57
Hydroxyethylstarch (HES),344
Hyperacuterejection,73
Hyperbilirubinemia,86
Hyperchloremicacidosis,urinary [NH4 +] and,13
Hypercortisolism,303
Hyperglycemia,29
Hyperkalemia
ECGchangeswith,9
GI symptoms,9
myocardialefectso ,11
serum potassium levelsand,11 symptomso ,9
Hypernatremia, water requirement, 11
Hyperparathyroidism,301 causeo ,302 gastrinomaand,302 Hyperre exia,12 Hypertension,renal ailureand,77 Hypertensivelower esophageal sphincter,202t Hyperthyroidism,300 Hypertonicsaline,useso ,10 Hypertrophicpyloricstenosis(HPS), 306
Hypertrophicscars(H S),65 vs.keloids,65
Hypocalcemia asymptomatic,12 conditionsdueto,10 ECGchangeswith,10 spasmsin hand and,12 Hypochloremicacidosis,13 Hypokalemicacidosis,13 Hypomagnesemia,12 Hyponatremia,9
Hypoplasticle heart syndrome,137 postoperativemanagement,140 Hypothalamic-pituitary-adrenal(HPA) axis,6
Hypovolemicshock,25 characteristics,27,30 Hysterectomy,319
I
IABP (intra-aortic balloon pump), 148
ICP.seeIntracranialpressure(ICP) Idiopathicscoliosis,332 Idiopathicthrombocytopenicpurpura (I P),19,278
IdiopathicV E,181 IL-6.seeInterleukin (IL)-6 Ileal pouch-anal reconstruction, 228–229
Ileostomy,228 Ileum,219 Ileus,postoperative,83 Iliaclesions,169,170 ,170t Immunethrombocytopenia (I P),18 primary,19
Immunosuppressivedrugs,74t Immunotherapy,315 Impedancecardiography,89 Inadvertent injury(ischemia),o thyroid gland,301 Incarcerated hernias,taxis or,291 Incidentaloma,adrenal,304 Incisionalbiopsy,71,133 Incisionalhernias,284 IncisionalSSIs,34 therapy or,39 Indeterminatecolitis,230 Indomethacin,141 Inefectiveesophagealmotility disorders,202t
In ants.seealsoChildren;Pediatric surgery
biliaryatresia,309 biliousemesisin,306 blind rectalpouch,313 chest X-ray,305
congenitaldiaphragmaticherniain, 305,305 with congenitalheart disease,139 “doublebubble”sign,310,310 duodenalobstruction in,310,310 incidenceo inguinalherniasin, 295–296,296 ,296t neckmassin,312
operativemanagement o ,305 with patent ductusarteriosuswith moderaterespiratorydistress, 141
pneumatosisintestinalis,308 postoperativemanagement,140
In ection(s).seealsoSurgical in ections;Surgicalsite in ections(SSIs);speci ctypes Actinomycetes,108 de ned,33 hand,surgicalmanagement,337,338 Helicobacter pylori,203–204 intravascular catheter,36 necrotizing,336–337 diagnosiso ,35–36 surgical,33–39 wounds,controlling,84
In erior thyroid artery(I A),299
In erior vena cava(IVC),179,244,244 In ertility,evaluation or,303
In ammation
CRP asmarker o ,5 systemicresponse,5(seealso In ammatoryresponse)
In ammatoryboweldisease,229 arthritisin,230
erythema nodosum in,230 extracolonicdiseasein,230 pericholangitisin,230 pyodermagangrenosum in,230 therapy or,230–231
In ammatoryresponse conditionsrelated to,5 mitochondrialDAMPsin,6 In ormed consent,351 reasonableperson standard or,351 Inguinalhernias,289–298
bimodaldistribution,289 circleo death,293,294 collagen in,289
connectivetissuedisorderswith,290t incidencein babies,295–296, 296 ,296t
incidenceo recurrence,297–298
laparoscopicrepairs,292
lateral emoralcutaneousnerve,297 McVayrepair,291–292,292
364
nociceptivepain,293
Nyhusclassi cation system,290,290t
prevalencebyage,289,289t
ratio to emoralhernias,295
recurrence,292–293
repair method,297–298
risk actors,289
saccontents,291
slidinghernia,298
APP repair vs. EP repair,295 taxis,291
triangleo doom,293,294
triangleo pain,293,294
urinaryretention a er repair o , 294–295
Inhalationalanthrax,37
Inhalation induction,344
Inherited V E,182,182t
Injection sclerotherapy, or varicose veins,185
Injury-associated stress,6
Injury(ies).seealso rauma
ADH production and,27
bileduct,259
bladder injuries,53
blunt carotid injuries,47 in children,309
descendingthoracicaorticinjuries,50
direct thermalinjury,55
esophageal,43,192
to emoralcutaneousnerve,297
hydro uoricacid skin burn,107
lacticacid and,13
li e-threatening,43
multipleintra-abdominalinjuries
a er gunshot wound,51
neck,44
nerve,64
nonoperativemanagement,51
parenchymal,45
pulmonary,44–45
skin,107
superior laryngealnerveinjury,85
systemicresponseto,5–7
thermal,108
thyroid gland,301
o trachea,43
traumaticgenitourinaryinjury,53
traumaticspleen injury,51–52
vascular,46,46t
wounds(seeWound(s))
Injury-mediated systemic in ammatoryresponse eatures,5
HMGB1and,6
mitochondrialDAMPsin,6
Intermittent claudication,diferential diagnosiso ,173t
Internalanalsphincter,225
Internal stula,in Crohn disease,232
InternationalEndohernia Society (IEHS),295
Internationalnormalized ratio (INR), 245
Intestinalmalrotation,de ned,226
Intra-abdominaladhesions,220
Intra-abdominalinjuries,a er gunshot wound,51
Intra-aorticballoon pump (IABP),148
Intra-aorticballoon pumping,30
Intra-arterialblood pressure monitoringsystem,87
Intracranialpressure(ICP) monitoring,91 therapy or increased,49
Intraductalpapillarymucinous neoplasms(IPMNs),265, 265 –266
Intraduralextramedullarytumors,328 Intraoperativeultrasound,o liver,245 Intraperitonealonlaymesh (IPOM) repair,292
Intrathoracicstomach,196,197
Intravascular catheter in ections,36 Intrinsicpathway,clotting actors,17 Intrinsicplatelet de ect,20 Intrinsicrenal ailure,85 Intrinsicsphincter de ciency(ISD),321
Intrinsicstenosis,vein gra sand,176 Introns,103–104
Invasive mycoses, mortality due to, 130–131
Ipsilateralstroke,in carotidstenosis,163
Ischemia
acuteembolicmesenteric,161 CLI,175,175t mesenteric,168 nonocclusivemesenteric,162,162 spinalcord,151 tissue,108
Ischemiccardiomyopathy,146
Ischemicstrokes,161
Ischemiculcer vs.neuropathiculcer,175t ISD.seeIntrinsicsphincter de ciency (ISD)
Isolated limb in usion (ILI),112,112
Isolated limb per usion (ILP),112
IVC lters,placement o ,184
J
Insulinomas,MEN1syndromeand,261
Insulin resistance,29
Intentionaldilution,o blood volume, 343
Interleukin (IL)-6,5
Interleukin (IL)-10,28
Jaundice,245
Jejunum,219
Jugular venousoximetry,91–92
K
K-ras,233
Kaposisarcoma,111–112
Kasaiprocedure,309
Kayexalate,11
Keloids,65,66 vs.hypertrophicscars(H S),65 treatment o ,66
Klatskin tumor,249–250,258
Kocher,Emil,299
Kocher clamp,299 L
Lactate,31
Lacticacid,injuryand,13
Lacto errin,33
Langerhanscellhistiocytosis(LCH),132
Laparoscopiccholecystectomy,biloma a er,83
Laparoscopicsplenectomy,281
Laparoscopicsurgery
adrenalectomy,304 advantageso ,304 arrhythmiaduring,93 colon resection,96 computer-enhanced surgery, advantageso ,95 o inguinalhernias,292 in pregnancy,94
Larger arteriolesvasoconstrict,in hemorrhage,26
Lateralcollateralligament (LCL),332 LCH (Langerhanscellhistiocytosis), 132
LCIS.seeLobular carcinoma in situ (LCIS)
LCL.seeLateralcollateralligament (LCL)
Leadership,3–4 coercivestyleo ,3 command and control,3 command-and-controltechnique and,4 communication and,3 democratic,3 undamentalprinciples,3 mentorship,3 styles,3 timemanagement,3 vision,3 willingness,3
Le ventricle(LV) aneurysms,144 Leiomyomas,200,319
Lepidicpredominant adenocarcinoma (LPA),126
Lesions in Crohn disease,232 liver,imagingo ,245 posterior ossa,323,324
Leukocytosis,237
Leukopenia,73
Leukoplakia,o vocalcords,121
Lichtenstein tension- reerepair,297,298
Li e-threateninginjuries,43
Li-Fraumenisyndrome,70
Ligaments,o spleen,275,276
365
Liver,241–251
acuteliver ailure,245
acutevaricealhemorrhage,246–247
anatomy,241–242,241 –242
arteryanatomicvariants,242,243 bilesalts,244
chemotherapeuticagents,251 cholangiocarcinoma,249–250
chronichepatitisCin ection,245 alci orm ligaments,241–242, 241 –242
gradingscale or disease,247 HCC,249 hemangiomas,248–249
in erior vena cava(IVC),244,244 intraoperativeultrasound o ,245 lesions,248–249 lesions,imagingo ,245,249 metastaseso ,250
plasma proteinssynthesisin,245 portalhypertension,246 veins,244,244
Livingwills,351
LMICs.seeLowand MiddleIncome Countries(LMICs)
Lobectomy,128
Lobular carcinoma in situ (LCIS),117 breast,118
Localbrain tissueoxygen tension (PbtO2),monitoring,92
Longbone ractures,331
Lowand MiddleIncomeCountries (LMICs),353,355
cancer morbidityand mortalityin, 355
surgicalcarein,355 taskshi ing,355
Lower esophagealsphincter (LES),190 de cient,causeo ,192 hormones,191
Lower extremityocclusivedisease,176 diagnosiso ,172
Fontaineclassi cation system or, 172,173t
Lower legcompartment syndrome,159
Low-gradeastrocytoma,325
Low-grademucosa-associated lymphoid tissue(MAL ) lymphoma,210,211
Lowpressureurethra,321
Low-resourcesettings,surgicalcarein, 354
LPA(lepidicpredominant adenocarcinoma),126
Lungcancer
abscess,surgicaldrainageo ,129, 130t
in elderly,348
non–small-celllungcarcinoma,126, 128,348
squamouscellcarcinomao ,125–126
Luteinizinghormone(LH),116
Lymphaticdrainage anterior abdominalwall,283 gallbladder,257 Lymphedema,186 Lymphedemapraecox,186 Lymphedematarda,186
Lymph nodebiopsies,82 Lymphoma,131,131t,222
o appendix,240 Lynch II syndrome,321 Lynch syndrome,235
M
Ma enideacetate,58 Magnesium de ciency,15 Magnesium depletion,12 Magneticresonanceangiography (MRA),160,160 advantageso ,163 Magneticresonance cholangiopancreatography (MRCP),52
Major histocompatibilitycomplex (MHC),77
Malabsorptivebariatricoperation,215 Male
malignantpleuralefusions,131,132t totalbodywater ( BW),10–11 Malignant hyperthermia (MH),343 Malignant pleuralefusions,131,132t Malignant trans ormation,o chronic ulcers,65 Malignant tumor,o brain,325 ,3255 Mallory-Weisstears,200 Mar anoid habitus,301 Mar an syndrome,62,152,154 Marjolin ulcer,65
Massiveblood trans usion,bleeding dueto,21–22
Massivetrans usion protocols,47,48 Maternalhypotension,320 Matrixmetalloproteinases(MMPs),62 May-Turner syndrome,182 McVayrepair technique,291–292,292 Mechanicalmitralvalvereplacement, 146
Meckeldiverticulum,219 ectopictissuein,222 persistenceo vitellineduct,283 Mediastinalgranuloma,131,131t Mediastinalmass,signsand symptoms, 131,131t
Melanoma,110 characteristics,112 in darkskin patient,111 nodular,111 ocular,111 siteo distant metastasis or,110 subtypeo ,110 treatment,112
MELD (Model or End-StageLiver Disease) score,247
MEN1,302 MEN2A,301
MEN2syndrome,70
Ménétrier disease,213
Mentorship,leadership and,3 Meralgia paresthetica,297
6-Mercaptopurine,73 Merkelcellcarcinoma,111
Mesentericischemia,168
Mesentericvein thrombosis(MV ), 187
Messenger RNA(mRNA) size,detection method,100
Metabolicacidosis vs.alkalosis,13 hemorrhagicshockand,14–15 with increased AG,10 with normalAG,9 serum bicarbonateleveland,10 vomitingand,13
Metabolicsupport,systemicresponse to,5–7
Metaiodobenzylguanidine(MIBG) scan,303
Metastasis,315
Methicillin-resistant Staphylococcus aureus(MRSA),109
Metoclopramide,83
Metronidazole,230
MIA(minimallyinvasive adenocarcinoma),126 MIC2gene,128
Micronutrients, or wound healing, 64–65
Microorganisms,resident,39
Microvascular circulation,in cellular per usion regulation,26
Mid-diastolicmurmur,144t
Mid-systolicmurmur,144t
Mid to latesystolicmurmur,144t
Midurethrallevelinjuries,316
Migratoryright lower quadrant (RLQ) pain,237
Milan criteria,250
Minimallyinvasiveadenocarcinoma (MIA),126
Minimallyinvasivesurgery,93–96 Miscommunication,3
Mithramycin,14
Mitochondrial DAMPs, in injurymediated in lammatory response, 6
MitochondrialDNA(mtDNA),6
Mitralstenosis(MS),144
Model or End-StageLiver Disease (MELD) score,247
Modern con ictresolution techniques,4 Modi ed Norwood procedure,137 Molecular andgenomicsurgery,97–104 Mondor disease,117
Monoclonalantibody,104 Monopolar electrocautery,317
366
Morbidity cancer,in LMICs,355 postoperative,in elderly,347
Morbidityand mortality(M&M) con erences,3
Mortality cancer,in LMICs,355 postoperative,in elderly,347
Mortalityrates
aorticarch aneurysm,155–156 or cardiogenicshock,30
Motilin,191
MRA(magneticresonance angiography),160,160
MRCP (magneticresonance cholangiopancreatography),52
mtDNA. see Mitochondrial DNA (mtDNA)
Mucocutaneousganglioneuromas,301
Mucosalulcerations,229
Multipleendocrineneoplasia (MEN)1 syndrome,261
Multipleintra-abdominalinjuries,a er gunshot wound,51
Multipleorgan dys unction syndrome (MODS),85
MV .seeMesentericvein thrombosis (MV )
Mycophenolatemo etil(MMF),73 sideefectsanddruginteractions,74t
MYH gene,233–234
Myocardialcontusion,42
Myoglobinuria,56
Myometrium,319 N
1988NationalInstituteso Health
ConsensusReport,21
NationalNosocomialIn ections
Surveillance(NNIS) system,37
NEC (neuroendocrinecarcinoma),126
Neck.seeHead and neck
Neckinjuries,44
Neckmass in in ants,312 management o ,312
Necrosis.seealsoCelldeath
HMGB1releasea er,6
Necrotizingenterocolitis(NEC), 308,309
Necrotizingin ections,336–337 diagnosiso ,35–36
Necrotizingpancreatitis,34,38–39
Needlebiopsy,133
Nelson syndrome,304
Neoadjuvant chemoradiotherapy, or esophagealcancer,199
Neobladder,315
Neostigmine,83
Nerveconduction studies(NCS),323
Nerveinjuries,typeso ,64
Nervoussystem,electricalinjuryand,56
Neurapraxia,64,325
Neuroendocrinecarcinoma(NEC),126
Neurogenicshock,25,26 characteristics,30
Neuropathiculcer vs.ischemiculcer, 175t
Neurosurgery,323–329 astrocytomas,325,325 ,328 cervicaldischerniation,326,327,327t CSFleakage,324 electromyography(EMG),323 head injuryscore,324 malignant tumor o brain,325,325 nerveconduction studies(NCS),323 pituitaryadenomas,327–328,328 posterior ossa lesions,323,324 pyogenicvertebralosteomyelitis,328
Neurotmesis,64
Neutralizingagents,107
Neutropenia,280
1991NIH ConsensusCon erence,215 Nissen undoplication,95 Nitinolstents,160
NNIS(NationalNosocomialIn ections Surveillance) system,37 Nociceptivepain,a er inguinal hernias,293
NOD2gene,221
Nodular melanoma,111
Noncontinent urinarydiversions,315 Noninvasivemethods,cardiacoutput monitoring,89
Nonmale cence,351
Nonmetastaticin-transit disease,112 Nonocclusivemesentericischemia, 162,162
Nonoperativemanagement,solid organ injuries,51 Nonperistalticcontractions,o esophagus,201
Nonpurulent,complicated cellulitis, treatment o ,109
Non–small-celllungcarcinoma (NSCLC),126,128 in elderly,348
Nonspeci cmedialdegeneration,151
Nonsteroidalanti-in ammatorydrugs (NSAIDs),148,206,206t,293, 326
Norepinephrine,27
Normalsaline,10
excessiveadministration o ,14
North America,hyperthyroidism incidence,300
Northern blotting,100
Norwood procedure,135
Nosocomialin ection,83–84
Nosocomialpneumonia,38
NSABP Breast Cancer Prevention rial, 72
NSAIDs.seeNonsteroidalantiin ammatorydrugs(NSAIDs)
NSCLC.seeNon–small-celllung carcinoma (NSCLC)
Nutcracker esophagus,202t
Nutritionalde ciencies,bariatric proceduresand,216
Nutritionalrequirements
o elderly,348 o patientswith burns,58
Nyhusclassi cation system,290,290t O
Obesity.seealsoWeight loss
bariatricprocedures,215–217
BMI,215 surgicalmanagement o ,215–217
Objectivity,con ict resolution techniquesand,4
Oblique,longbone ractures,331
Obstructiverenal ailure,85
Obstructiveshock,25
Octreotide,213
Ocular melanoma,111
Omentalin arction,284
Omentum blood supplyto,284 tumorso ,285
Omphalocele,313
Oncogene,monoclonalantibodyto,104
Oncology.seeCancer
Open splenectomy,281
Oraltonguecarcinoma,122
Organ/spaceSSIs,34
Orthopedicsurgery,331–333
Orthotopiccardiactransplantation,76
Orthotopicliver transplant (OL ),247 Milan criteria,250
Orthotopicneobladder,315
Osmoticpressure,11 between plasmaand interstitial uid compartments,12 units,11
Osteosarcoma,129 o rib,133
Ostium primum de ects,135
Ostium secundum de ects,135
Overdamped system,87
Overwhelmingpostsplenectomy in ection (OPSI),277 antibiotictherapy,282 prevention strategy,281 risk actors,282
Oxandrolone,59
Oxygen delivery(DO2),87 strategies or increasing,90
P-2trial,117
p53gene,234
PaCO2,14 Pain rom chronicpancreatitis,267 a er inguinalhernias,293
367
P
Palliativecare,351–352
Palliativesurgery,in elderly,349
PAMPs(pathogen-associated molecular patterns),6
PAN (polyarteritisnodosa),177–178
Pancoast,Henry,128
Pancoast tumors,128
Pancreas,261–273
pseudocysts,261
pylorus-preservingresectionso , 269,270 –272
replaced right hepaticartery,261
Pancreaticcysticneoplasms,261,262
Pancreaticductdecompression,268,268t
Pancreaticojejunostomy,269,270 –272
Pancreatictrauma,management o ,52
Pancreatitis
acute,222 chronic(seeChronicpancreatitis)
chroniccalci c,263,264t
necrotizing,treatment,34,38–39
Ranson criteria,262,263t
Panel-reactiveantibody(PRA)assay,79
Papillarythyroidcancer,in children,301
Paragangliomas,163,303
Parathyroid carcinoma,302
Parathyroidectomy,302
Parathyroid glands removalo ,301,302 supernumerary,301
Parathyroid hormone(P H) level,in dementia,302
Parenchymalinjuries,45
Parkland ormula,57
Partial undoplication,195
Partial-thicknessburns,57,60
Partial-thicknessinjury,108
Partialurethralobstruction,321
Patent ductusarteriosus with moderaterespiratorydistressin in ants,141 treatment o ,141
Pathogen-associated molecular patterns(PAMPs),6,25
Patient sa ety,81–86
Pattern recognition receptors(PRRs),25
Peakairwaypressure(Ppeak),90
Pediatricsurgery,305–314.seealso Children;In ants
biliousemesisin newborn,306
blind rectalpouch in newborn,313
branchialcle stula,311
chest X-ray,305
choledochalcysts,313
congenitallobar emphysema,312
esophagealatresia (EA),307,307
gastroschisis,310,311
Hirschsprungdisease,306
neckmassin in ants,312 operativemanagement o newborn, 305
primaryEA(typeA),311
prune-bellysyndrome,313 pyloricstenosis,306 tracheoesophageal stula ( EF), 307,307
undescended testicles,314 Wilmstumor,307–308,308t Pedicle,341
Pelvicbleeding,in traumapatient,46 Pelvic oor disorders,321 Pelvic ractures,332 Pelvicorgan prolapse,321 Pelvicpacking,46 Penicillin allergy,36 Penis,anatomyo ,315 Pepticulcer disease(PUD) antrectomy or,207 bleeding,206,207 indications or surgeryin,208,208t
Peptidehormones,220
Percutaneoustransluminalangioplasty (P A),168
Perianallesions,in Crohn disease,232
Pericardialtamponade,42 Pericarditis,148
Pericholangitis,230
Perihilar cholangiocarcinomas,258
Peritonealadhesions,67
Peritonitis,acute,in elderly,347
Peyer patches,219
Phenylethanolamine
N-methyltrans erase,303 Pheochromocytoma,301,303 preoperativepreparation,303
Phlegmasiacerulea dolens,183,183
Physiologicaging,de ned,347 Physiologicmonitoring,o surgical patient,87–92
Physiologicre ux,190–191
Pituitaryadenomas,327–328,328
Pituitaryapoplexy,328
Pituitarygland,AC H-secreting adenoma,303,304
Plasmacytoma,129
Plasmaproteins,synthesisin liver,245 Plastic and reconstructive surgery, 341–342
Plateau airwaypressure(Pplateau),90
Platelet aggregation,17
Platelet de ect
acquired platelet hemostaticde ect, 18
intrinsic,20
Platelet-derived growth actor BB (PDGF-BB),67
Platelet actor-4(PF4),18
Platelets
adherencetoinjuredendothelium,17 impaired unction o ,18
Pleural uid analysis,133
Plicaecircularis,219 PMNs(polymorphonuclear neutrophils),61
Pneumatictourniquets,59
Pneumatosisintestinalis,308
Pneumonectomy,128
Pneumonia,84
aspiration pneumonia,129 community-acquired,38 diagnosiso ,38 nosocomial,38
ventilator-associated (seeVentilatorassociated pneumonia(VAP))
Pneumothorax,tension,41
Point o criticaloxygen delivery (DO2crit),87
Polyarteritisnodosa (PAN),177–178
Polycythemiavera(PV),279
Polymerasechain reaction (PCR),100
Polymorphonuclear neutrophils (PMNs),61
Polypectomy,234
Polypoid lesions,o gallbladder,257
Portalhypertension,246,280
Posterior ossalesions,323,324
Postexposureprophylaxis, or HIV,38
Postincisionalhernias,284
Postoperativehyponatremia,9
Postoperativeileus,83
Postoperativemorbidityand mortality, in elderly,347
Postoperativenausea and vomiting (PONV),343
PostoperativeU Is,35
Postrenal ailure,causeso ,85
Postrenaltransplantgra thrombosis,74
Potassium changein,alkalosisand,11 serum potassium levels, hyperkalemia and,11
Pouchitis,229
Predictive/prognostictumor marker, or adenocarcinoma,127
Pregnancy appendicitisand,238 breast cancer during,119 breast development be oreand during,116 changesin breast with,115 laparoscopicsurgeryin,94
Pregnancy-related surgicalconditions, 320
Preoperativephysicalexamination, 345,345t
Prerenal ailure,85
Presacral ascia,226
Pressure-speci cefects,CO2 pneumoperitoneum,94,95
Presystolicmurmur,144t
Priapism,315,317
Primaryaldosteronism,303
Primaryhemostasis,17
Primaryimmunethrombocytopenia (I P),19
Primarylymphedema,186
368
Primitiveneuroectodermaltumors (PNE s),128
Proaccelerin ( actor V),22
Progesterone,116
Prognostictumor marker, or adenocarcinoma,127
Prokaryotes,transcription o ,102
Prolactin,116
Proli erativephase,wound healing,61
Prolonged anticoagulation,in elderly, 348
Prolonged mechanicalventilation, nosocomialpneumoniaand,38
Prophylaxis,PEincidenceand,82
Prostatecancer,316
Protaminesul ate,19
Protein(s)
acutephaseproteins,5
osmoticpressurebetween plasma and interstitial uid compartmentsand,12 plasma,synthesisin liver,245
RNAsynthesis,101,101
Prothrombin time(P ),245
Protocol-driven ventilator weaning strategies,82
Proton pump inhibitor (PPI) therapy, 193
Provoked V E,181
Prune-bellysyndrome,313
Pseudoaneurysms,47,151 anastomotic,152
Pseudocysts,261,279
Pseudohyponatremia,9
Psoashitch,316
Pulmonaryarteryocclusion pressure (PAOP),88
Pulmonaryembolism (PE),82,181 incidence,prophylaxisand,82
Pulmonary ailure ormulas, at content,7
Pulmonaryinjuries,44–45
Pulmonarytractotomy,44
Pulsecontour analysis,89
Pulsepressurevariability(PPV),90
Pyloricstenosis,306
Pylorus-preservingresections,o pancreas,269,270 –272
Pyogenicabscesses,35
Pyogenicliver abscesses,35 management o ,248
Pyogenic vertebral osteomyelitis, 328–329
Quality,Donabedian model,81,81
Radialhead racture,332
Radiation enteritis,o smallbowel,222
Radiation therapy,199
esophagealcancer,199
Radicalprostatectomy,316
Ranson criteria,262,263t Rapid shallowbreathingindex,82 RCC.seeRenalcellcarcinoma (RCC)
Reconstructivesurgery,341–342 principleso ,342
Rectaladenocarcinoma,228
Rectosacral ascia(Waldeyer ascia),226
Rectum,225 anatomy,226
Rectusabdominisdiastasis,283 Rectussheath hematoma,283
Recurrent COA,136
Recurrent laryngeal nerve (RLN), 299–300
Recurrent periductalmastitis,117 Re eedingsyndrome,15
Re uxassociated asthma,193 Regionalanesthesia,344
Renalarteryocclusivelesions,166,166
Renalarteryrevascularization,168,168t Renalarterystenosis,163
Renalcellcarcinoma (RCC),316
Renal ailure causeso ,77,85 classi cation o ,85
Renaltransplantation contraindications,78 living,78,79 postoperativecomplications,74 screeningbe ore,79 surgicaltechniques,75,75 –76 survivala er,75
Renaltumors,316
Renovascular hypertension,168
Replaced right hepaticartery,261 Resident microorganisms,39 Rest pain,164
Restrictivebariatricoperation,215 Resuscitativethoracotomy(R ),43
Retrogradepyelogram,316 Retroperitoneal brosis,285
Retroperitoneallymph nodes,315 RETtyrosinekinasereceptor gene, 300,301
Rhabdomyomas,145,147
Rheumatoid arthritis(RA),280
Rib(s),osteosarcomao ,133
Rivaroxaban,19
RNAgelblot analysis,100 RNApolymerase,102 RNAsynthesis,101,101 Rogersscore,182–183
Roux-en-Ygastricbypass(RYGB),215 long-term bene ts,216 nutritionalde cienciesa er,216 postoperativecomplications,215 resolution o ,217 smallbowelobstruction a er,216 type2diabetesmellitusa er,216
RYGB.seeRoux-en-Ygastricbypass (RYGB)
S Saccontents,hernias,291
Saccular aneurysms,151
Sa eSurgerySavesLivesInitiative,354
Salivarygland,tumorso ,122
Salivarygland neoplasms,122
Salvageesophagectomy,199
Sano shunt,137,140
Sarcomas
AJCC/UICC stagingsystem,287 incidencerates,287–288 management o ,288 molecular pathogenicevents,288 risk actors,287 so tissue,287–288
2G3NOMO sarcoma,288
Scaphoid bone,o wrist,335
SCC.seeSquamouscellcarcinoma(SCC) 2010Scienti cRegistryo ransplant Recipients(SR R) annual report,74
Sclerosingadenosis,117
Sclerosingcholangitis,256
Sclerosingmesenteritis,285 Scolioticcurves,332
Scoringsystem,182–183
Secondarylymphedema,186
SecondaryV E,181
Secretin-glucagon amilyo hormones, 220
Segmental,longbone ractures,331 Senescence,de ned,347
Sentinellymph node(SLN) biopsy,119 Sepsis corticosteroids or,31 severe,treatment o ,38 SurvivingSepsisCampaign guidelines or,89
Septicshock,25,35 corticosteroids or,31 SurvivingSepsisCampaign guidelines or,89 treatment,29
Seroma,a er so -tissuebiopsy,82 Serotonin,17
Serum bicarbonate,metabolicacidosis and,10
Seven-transmembrane-G-protein–coupledreceptors(GPCRs),100 Severediarrhea,metabolicacidosis with normalAGin,9
Severesepsis,38
σ Factors(sigma actors),102 Shock,25–31.seealsospeci ctypes baroreceptorsand,25–26 categories,25 chemoreceptorsand,26 clinicalsignso ,28 bylargetension pneumothorax,25 Short bowelsyndrome,223
Short-chain attyacids,227
Shoulder dislocations,331
369
Q
R
Shouldiceoperation,297–298
Sigmoidectomy,228
Sigmoidoscopy,227
Silver sul adiazine,58
Singlenucleotidepolymorphisms,98
Sinusvenosusde ects,135
Sirolimus sideefect,80 sideefectsanddruginteractions,74t
SIRS(systemicin ammatoryresponse syndrome),33
Skin
BCC,109,110,113
cellulitis,109
hydro uoricacid skin burn,107 injuryto,107
melanoma, 110–112 (see also Melanoma)
SCC,110
thermalinjury,108
trichilemmalcysts,109
Skin tags,in Crohn disease,232
SLN (sentinellymph node) biopsy,119
Smallbowelobstruction causeo ,220
a er RYGB,216
therapyo ,221
Smallintestine,219–223
carbohydratedigestion,220 cellular turnover rateo enterocytes, 219
daily uid entersin,219
enterohepaticcirculation,220 ailureo xation,285
hormone-producingcells,219 peptidehormones,220 primarymalignancyo ,222
radiation enteritiso ,222
Smallvesselvasculitis,177,177t
Sodium chloride,10 high concentration o ,14
Sodium depletion,ADH and,9
Sodium-glucosecotransporter 1 (SGL 1),220
Sodium supplementation,BPH,317
So tissuesarcomas,287–288
AJCC/UICC stagingsystem,287
incidencerates,287
management o ,288
molecular pathogenicevents,288 risk actors,287
2G3NOMO sarcoma,288
Somatostatinoma,263
Sora enib,251
Spasms,in hand,12
Sphincter o Oddi,254
Spigelian hernias,284
Spinalastrocytomas,328
Spinalcord ischemia,151
Spleen,275–282
accessory,275
unctionso ,276
hereditaryspherocytosis(HS),276 ligaments,275,276 traumaticinjuryo ,51–52
Splenectomy,51,277 complicationsa er,281 laparoscopic,281
Splenic arteryaneurysm (SAA), 279–280
Spleniccysts,279 Splenomegaly,280 Splenorrhaphy,51
Split-thicknessskin gra s,341 SpontaneousV E,181 Spores,131
Squamouscellcarcinoma(SCC),110, 125–126,336 o esophagus,197–198 o hand,336–337,338,339 Square-root sign,147 SSIs.seeSurgicalsitein ections(SSIs)
STARtrial,117
Steam burn,59 Stenosis.seealsoCarotid stenosis intrinsic,176 severityo ,categories,162 Steroids,wound healingand,63 Stewart-Hamilton equation,88–89 Stomach,203–213.seealsoGastriccancer bleedingpepticulcer,206,207 consistentlylargest arteryto,203 gastrin-stimulated acid secretion, 203
Helicobacter pylori in ection, 203–204
Stomanecrosis,228
Stoolwet-mount and culture,227 Storagepooldisease,20 Stress.seealsoInjury-associated stress mtDNAand,6 Stressgastritis,83 Stressincontinence,321 Stroke,163
carotid bi urcation occlusivedisease in,170,171
Subcutaneousnodules,113 Subtotallobectomy,301
Subtotalthyroidectomy,300 Sul asalazine(Azul dine),230 Sul ur granules,in wound drainage, 108
Super cialBCC,113
Super cialburns,57
Super cialvein thrombophlebitis (SV ),184–185
Superior laryngealnerve(SLN),300
Superior laryngealnerveinjury,85
Superior mesentericartery(SMA), 242,243
Supernumeraryparathyroidglands,301
Suprapubictube,316 Surgicalbleeding,17–23 Surgicaldebulking,316
Surgicalin ections,33–39.seealso In ection(s) incisional,34 organ/space,34
Surgicalin ectiousdiseasetherapy,34 Surgicalsitein ections(SSIs),34 development o ,37 incisionalSSIs,34,39 organ/spaceSSIs,34
Surgicalwounds.seeWound(s) SurvivingSepsisCampaign guidelines,89 Suspensoryligaments, o spleen, 275, 276
SV (super cialvein thrombophlebitis),184–185
Swallowing,cranialnervesin,190
Syringomyelia(syrinx),328
Systemicin ammatoryresponse syndrome(SIRS),33 cytokinein,85 inclusion criteria or,85t mortalityassociated with,85t
Systemicresponse,to injuryand metabolicsupport,5–7
SystemicVentricleReconstruction (SVR) trial,140
Systolicand diastolicmurmur,144t
2G3NOMO sarcoma,288 acrolimus(FK506),74t amoxi en,72 endometrialcancer and,321
APP repair.see ransabdominal preperitoneal( APP) repair askshi ing,355 axis, or incarcerated hernias,291 earing,156
EG(thromboelastography),22 emporalarteritis,176–177 ension pneumothorax,42 diagnosiso ,41
EP repair.see otallyextraperitoneal ( EP) repair erribletriad,332 esticular cancer,315 esticular torsion,317 esticular tumor,317
etralogyo Fallot ( OF),137 age or correction o ,137
Talassemia,overwhelming postsplenectomyin ection in, 277
Termalinjury,108
Termodilution technique,88–89
Tird-degreeburns,57–58,60
Toracicaneurysms,151–157
ascending,153 causeo ,151 Craw ord classi cation scheme,156 imagingo ,154 typeso ,151
370
T
Toracicaorticinjuries,50
Toracicescharotomy,58
Toracotomy,50,141 indications or,50t
Trombocytopenia bacterialsepsisand,22
heparin-induced,84 heparin-induced (HI ),185
Tromboelastography( EG),22
Trombolytictherapy,174,174t
Tromboticthrombocytopenic purpura( P),20
Tymoma with myastheniagravis, 131,131t
Tyroglossalduct,299
Tyroglossalduct cyst,299
Tyroid cancer,papillary,in children, 301
Tyroidectomy in children,301 subtotal,300 total,300
Tyroid gland
arterialsupplyo ,299
congenitalanomaly,299
ollicular neoplasmso ,300
Gravesdisease,300 hyperthyroidism,300
inadvertent injury(ischemia),301 postoperativecomplicationsa er surgery,301
Tyroid hormones( 3and 4),300
Tyroid imaartery,299
Tyroid nodule,FNABo ,300 imemanagement,leadership and,3
IPS.see ransjugular intrahepatic portosystemicshunt ( IPS) issue actor ( F),17 issueinjury,coagulation a er,22 issueischemia,108 issuenecrosis,335 issuepressures,108
NF-α.see umor necrosis actoralpha ( NF-α)
ToErr IsHuman,3
ongue,tumorso ,122
orticollis,312
otalanomalouspulmonaryvenous connection ( APVC),136
otalbodysur acearea( BSA),55
otalbodywater ( BW),10–11
otalbodyweight,water percentageo , 10–11
otalcolectomy,228–229
otallobectomy,301
otallyextraperitoneal( EP) repair, 292,295
otalthyroidectomy,300,301
oupet undoplication,195
oxicepidermalnecrolysis( EN),113
oxic megacolon, in Crohn disease, 232
rachea,injurieso ,43 rachealstenosis,125 racheoesophageal stula ( EF), 307,307 racheostomy,43,84,123 ransabdominalpreperitoneal( APP) repair,292,295 ransAtlanticInter-SocietyConsensus ( ASC) group,169 classi cation o aortoiliacocclusive lesions,169,170 ,170t ranscranialDoppler ( CD) measurements,91 ranscription,o prokaryotes,102 ranscription mechanisms,in eukaryotes,97–98 ransesophagealechocardiogram,139 rans errin,roleo ,33 rans orminggrowth actor-beta ( GF-β),99 rans usion,17–23 bleedingdueto,21–22
HepatitisCriskand,21 HIV-1transmission and,21 rans usion reaction,22 rans usion-related acutelunginjury ( RALI),21 rans usion-related complication,21 rans usion-related deaths,21 ransient ischemicattack( IA),163 crescendo IAs,171 ransient monocular blindness, 162–163 ransjugular intrahepaticportosystemic shunt( IPS),247 complicationsa er,248 ranslation,101,101 ransmitter-gated ion channels,100 ransplantation,73–80.seealso Cardiactransplantation;Renal transplantation contraindications,76,78 gra rejection prevention, cyclosporine or,77 hyperacuterejection,73 immunologicrejection,antigens or, 77
immunosuppression,78 postoperativecomplications,74 screeningbe ore,79 survivala er,75 ransurethralresection,315,317 ransverse,longbone ractures,331 rastuzumab (Herceptin),104 rauma,41–54.seealsoInjury(ies) blunt abdominaltrauma,45,45t cardiogenicshockand,42,42 coagulopathyrelated to,18 cortisollevelsa er,6 cricothyroidotomy,41,41 GCSscore,326,326t in geriatricpatients,54
li e-threateninginjuries,43 management o pancreatic,52 pelvicbleeding,46 systemicin ammatoryresponse a er,5
rauma-related coagulopathy,18–19
rauma-related hypovolemia,320
raumatic brain injury ( BI), classi ication, 324
raumaticgenitourinaryinjury,53 raumaticshock,25
raumaticspleen injury,51–52
riangleo doom,293,294
riangleo pain,293,294
richilemmalcysts,109
ricuspid atresia,136
ricuspid stenosis,145
riglyceridelevel,pleural uid analysis and,133
rousseau sign,12
rueaneurysms,151
runcusarteriosus,141 uberculosis,130 umor marker, or adenocarcinoma, 127
umor necrosis actor-alpha( NF-α),28 umor(s).seealsoCancer(s)
cardiactumor,145,146
carotid body,163,164
chest wall,129
desmoid tumors,127
o abdominalwall,284
GIS s,209,210
intraduralextramedullary,328
malignant,o brain,325,325
o omentum,285
Pancoast tumors,128
o tongue,122
o wrist,337 umor suppressor gene,233
Tunica albuginea,315 ype2diabetesmellitus,a er gastric sleeveprocedureandRYGB,216 ypeBiliaclesions,169,170 ,170t ypeIII collagen,inguinalhernias,289 ype III gastric carcinoid tumors, 211–212
Ulcerativecolitis,229,230,231 eatures,236 indications or emergencysurgery, 231
Ulcers.seealsoWound(s)
malignant trans ormation o ,65 neuropathicvs.ischemic,175t venousulceration,181
Umbilicalhernias,284
Uncomplicated U Is,83
Underdamped system,87
Undescended testicles,317 in children,314
371
U
United States
ALFin,245
causeso renal ailurein,77 chronichepatitisCin ection in,245 esophagealcancer in,197 mortalitydueto invasivemycosesin, 130–131
Ureterictrauma,316
Uretero-ureterostomy,316
Urethralcatheter,316
Urethralhypermobility,321
Urethralstrictures,316
Urinary[NH4 +],hyperchloremic acidosisand,13
Urinaryretention BPH,317
a er herniarepair,294–295
Urinarytract in ections(U Is),83–84 classi cation o ,83 complicated,83 postoperative,35 uncomplicated type,83
Urolithiasis(urinarycalculusdisease), 318
Urology,315–318
U.S.Food and DrugAdministration (FDA),319
USInstituteo Medicine,3
Uterinemyomas,319,320
U Is.seeUrinarytract in ections (U Is)
Valsalva maneuver,227
VAP.seeVentilator-associated pneumonia(VAP)
Varicealbleeding,246–247
Varicoseveins,injection sclerotherapy or,185
Vascular injuries,46,46t
Vasculitis,177,177t
Vasodilatoryshock,25 characteristics,29
Vasogenicshock,25
Vasopressors,29
Vein gra s cryopreserved,176 intrinsicstenosisand,176
Veins anatomy,179
hepatic,244,244
Venousintima,179
Venousthromboembolism (V E),181
idiopathicV E,181 inherited,risk actors or,182,182t
provoked V E,181 riskand recommended thromboprophylaxis,183t scoringsystems,182–183 secondaryV E,181 spontaneousV E,181 Venousulceration,181 Venousvalves,179
Ventilation, prolonged mechanical, 38
Ventilator-associated pneumonia (VAP),82,84.seealso Pneumonia
Ventricular septalde ect (VSD),138 anatomyo ,140 in children,139 determinant o operability,140 owacross,139 spontaneousclosureo ,138 typeso ,141
VIN.seeVulvar intraepithelial neoplasia (VIN)
Virchow,Rudol ,181 Virchowtriad,181 Vision,leadership and,3
Vitamin Ade ciency,and wound healing,64
Vitellineduct,persistenceo ,283 Vocalcords,leukoplakiao ,121 Voidingdys unction,321 Voidingsymptoms,BPH,317 Vomiting in children,312 hypochloremic,hypokalemic metabolicacidosisand,13 von Willebrand disease,182 von Willebrand actor (vWF),17 VSD.seeVentricular septalde ect (VSD)
Vulvar carcinoma,321
Vulvar intraepithelialneoplasia (VIN), 321
W
Waldeyer ascia,226 Wallerian degeneration,64 Wartenbergsign,336 Water hypernatremiaand,11 total bodyweight percentage o , 10–11
Watermelon stomach,212,212 Weaningprotocol,82 Weight loss,216.seealsoObesity
Whole-organ transplantation,75 –76
Willingness con ict resolution techniquesand,4 leadership and,3
Wilmstumor,307–308,308t
World Bank,355
World Health Organization (WHO), 130,354,355 objectives or surgicalpractice, 354,354t
Sa eSurgerySavesLivesInitiative,354
Wounddrainage,sul ur granulesin,108
Wound healing,61–67
o cartilage,65
collagen or,65
diabetesmellitusand,64 broblasticphase,62 impaired,causeso ,64 longbone racture,63
micronutrients or,64–65
PMNsrolein,61 proli erativephase,61 steroidsefect on,63
vitamin Ade ciencyand,64 zinc or,65
Wound in ection,86 controllingpostoperative,84 Wound management techniques, thermalinjury,108
Wound(s),34.seealsospeci ctypes appendectomy,37 burn (seeBurns) classi cation,34
gunshot,51,52–53
malignant trans ormation,65 stages,108 surgicalmanagement,37 a er surgery,treatment or,39 tissueischemia,108
Wrist.seeHand and wrist
Wrong-sitesurgeries,causeso ,82–83
Xeroderma pigmentosa,338
YearsLived with Disability(YLD),353
Yearso Li eLost (YLL),353
Zenker diverticulum,201
Zinc, or wound healing,65
Zollinger-Ellison syndrome(ZES),204, 205
Zoneo coagulation,108
Zuska disease,117
372
V
X
Y
Z